You are on page 1of 280

CASE TITLE DOCTRINE FACTS-ISSUE/s-HELD

Substantive law is that part of the law which Nature of the case - Petition for Review on Certiorari under Rule 45 praying for (1) the nullification of the CA Decision in CA-
1.Bernabe vs. creates, defines and regulates rights, or GR CV No. 51919 and the October 1999 CA Resolution
Alejo 140500 21 which regulates the rights and duties which
January 2002- give rise to a cause of action; that part of Facts:
ABAD the law which courts are established to The late Fiscal Ernesto A. Bernabe allegedly had a son with his 23yr old secretary of , herein plaintiff-appellant Carolina Alejo.
administer; as opposed to adjective or The son was born on September 18, 1981 and was named Adrian Bernabe. Fiscal Bernabe died on August 13, 1993, while his
remedial law, which prescribes the method wife Rosalina died on December 3 of the same year, leaving Ernestina as the sole surviving heir.
of enforcing rights or obtains redress for
their invasion. On May 16, 1994, Carolina, in behalf of Adrian, filed the aforesaid complaint praying that Adrian be declared an
acknowledged illegitimate son of Fiscal Bernabe and as such he (Adrian) be given his share in Fiscal Bernabes estate,
Article 285 of the Civil Code is a which is now being held by Ernestina as the sole surviving heir.
substantive law, as it gives Adrian the right
to file his petition for recognition within 4 RTC - dismissed the complaint for recognition, ruling that under the provisions of the Family Code, the death of the putative
years from attaining majority age. father had barred the action. The court added that since the putative father had not acknowledged or recognized Adrian
Therefore, the Family Code cannot impair Bernabe in writing, the action for recognition should have been filed during the lifetime of the alleged father to give him the
or take Adrians right to file an action for opportunity to either affirm or deny the child's filiation.
recognition, because that right had already
vested prior to its enactment.
CA- ruled that in the interest of justice, Adrian should be allowed to prove that he was the illegitimate son of Fiscal Bernabe.
Because the boy was born in 1981, his rights are governed by Article 285 of the Civil Code, which allows an action for
recognition to be filed within four years after the child has attained the age of majority. The subsequent enactment of the
Family Code did not take away that right.
Hence, this appeal.

Petitioner’s contention: that respondent is barred from filing an action for recognition, because Article 285 of the Civil Code
has been supplanted by the provisions of the Family Code. She argues that the latter Code should be given retroactive effect,
since no vested right would be impaired.
Respondent’s contention: that the filing of an action for recognition is procedural in nature and that as a general rule, no
vested right may attach to [or] arise from procedural laws.

Issue/s:
1. Whether or not Adrians right to an action for recognition, which was granted by Article 285 of the Civil Code, had
already vested prior to the enactment of the Family Code. - Yes.
2. Whether or not respondent has a cause of action to file a case against petitioner, the legitimate daughter of the
putative father, for recognition and partition with accounting after the putative fathers death in the absence of any
written acknowledgment of paternity by the latter. - Yes
Held:
The Petition has no merit.

Bustos v. Lucero distinguished substantive from procedural law in these words:

Substantive law creates substantive rights and the two terms in this respect may be said to be synonymous. Substantive
rights is a term which includes those rights which one enjoys under the legal system prior to the disturbance of normal
relations. Substantive law is that part of the law which creates, defines and regulates rights, or which regulates the rights and
duties which give rise to a cause of action; that part of the law which courts are established to administer; as opposed to
adjective or remedial law, which prescribes the method of enforcing rights or obtains redress for their invasion

In Fabian v. Desierto, the Court laid down the test for determining whether a rule is procedural or substantive:

In determining whether a rule prescribed by the Supreme Court, for the practice and procedure of the lower courts, abridges,
enlarges, or modifies any substantive right, the test is whether the rule really regulates procedure, that is, the judicial process
for enforcing rights and duties recognized by substantive law and for justly administering remedy and redress for a disregard or
infraction of them. If the rule takes away a vested right, it is not procedural. If the rule creates a right such as the right to
appeal, it may be classified as a substantive matter; but if it operates as a means of implementing an existing right then the rule
deals merely with procedure.

Applying the foregoing jurisprudence, we hold that Article 285 of the Civil Code is a substantive law, as it gives Adrian the
right to file his petition for recognition within four years from attaining majority age. Therefore, the Family Code
cannot impair or take Adrians right to file an action for recognition, because that right had already vested prior to its
enactment.

Illegitimate children who were still minors at the time the Family Code took effect and whose putative parent died during their
minority are thus given the right to seek recognition (under Article 285 of the Civil Code) for a period of up to four years from
attaining majority age. This vested right was not impaired or taken away by the passage of the Family Code.

Indeed, our overriding consideration is to protect the vested rights of minors who could not have filed suit, on their own, during
the lifetime of their putative parents Born in 1981, Adrian was only seven years old when the Family Code took effect and only
twelve when his alleged father died in 1993. The minor must be given his day in court

Procedural rules are tools designed to Facts:


2. Sps. Bergonia facilitate the adjudication of cases. Courts
vs. CA 189151 and litigants alike are, thus, enjoined to - This is a petition for certiorari under Rule 65 of the Rules of Court filed by the spouses David Bergonia and Luzviminda
25 January 2012- abide strictly by the rules. And while the Castillo (petitioners) assailing the Resolutions issued by the Court of Appeals (CA) on May 18, 2009 and June 29, 2009 in CA-
1 2

DE GALA Court, in some instances, allows a G.R. CV No. 91665.


relaxation in the application of the rules,
this, we stress, was never intended to forge - Petitioners were the plaintiffs in Civil Case No. Br. 23-749-03 entitled "Spouses David Bergonia and Luzviminda Castillo v.
a bastion for erring litigants to violate the Amado Bravo, Jr." in the Regional Trial Court (RTC), Branch 23, Roxas, Isabela. On January 21, 2008, the RTC rendered a
rules with impunity. The liberality in the decision adverse to the petitioners. The petitioners consequently sought a reconsideration of the said decision but the same
interpretation and application of the rules was denied by the RTC in an Order dated April 25, 2008 which was received on May 6, 2008. On May 7, 2008, the petitioners
applies only in proper cases and under filed a Notice of Appeal
justifiable causes and circumstances. While
it is true that litigation is not a game of - In January 2009, the Law Firm of Lapeña & Associates filed with the CA its formal entry of appearance as counsel for
technicalities, it is equally true that every the petitioners, in view of the withdrawal of the former counsel, Atty. Panfilo Soriano. The substitution of lawyers was noted in
case must be prosecuted in accordance the Resolution dated January 20, 2009. In the same resolution, the CA further directed the appellants therein to remit the
4

with the prescribed procedure to insure an deficient amount of ₱20.00 within 5 days from notice. Thereafter, the CA issued a Resolution on January 30, 2009 requiring
orderly and speedy administration of the filing of the Appellant’s Brie f within 45 days from receipt.
justice. The instant case is no exception to
this rule. -On April 8, 2009, respondent Amado Bravo, Jr. (the defendant-appellee therein), filed a Motion to Dismiss Appeal
In the present case, we find no cogent dated April 2, 2009 stating that the petitioners failed to file their Appellant’s Brief within the 45-day period granted to
reason to exempt the petitioner from the them by the CA in the Resolution dated January 30, 2009. Citing Section 1 (e), Rule 50 of the Rules of Court,
effects of its failure to comply with the respondent prayed for the dismissal of the petitioners’ appeal.
Rules of Court. - In an Opposition/Comment promptly filed on April 8, 2009, the petitioners alleged that the Motion to Dismiss
6

filed by the respondent had no basis considering that they or their counsel did not receive any resolution from the CA
requiring them to file their Appellants’ Brief within 45 days.
- On May 18, 2009, the CA ruled that the appeal is considered abandoned and dismissed for failing to file the
appelant’s brief within the reglementary period.
- On May 25, 2009, the CA issued a Resolution which stated, among others, that the January 30, 2009 notice to
9

file a brief addressed to petitioners’ counsel was received by a certain Ruel de Tomas on February 5, 2009.
- On June 5, 2009, the petitioners filed a Compliance and Motion for Reconsideration praying that the dismissal
10

of their appeal be set aside in the interest of justice and equity. The petitioners claimed that their failure to file their brief
was due to the fact that they were never furnished a copy of the said January 30, 2009 Resolution of the CA directing
them to file their brief.
- Subsequently, in a Manifestation filed on June 16, 2009, the petitioners asserted that their counsel – the Law
Firm of Lapeña and Associates – has no employee in the name of Ruel de Tomas. However, they explained that Atty.
Torenio C. Cabacungan, Jr., an associate of the law firm personally knows a person named "Ruel" who sometimes
visits their office and who may have accidentally received the said January 30, 2009 Resolution of the CA. In such a
case, the same should not be considered officially served upon them as the latter was not connected with nor
authorized to perform any act for and in behalf of counsel.
- On June 29, 2009, the CA denied the motion for reconsideration.

Issue/s:
Whether or not the dismissal of the appeal due to the failure of the petitioners to file their appelants brief within the
reglementary period was proper

Held:
YES.
Here, there was no hint of whimsicality or gross and patent abuse of discretion on the part of the CA when it dismissed the
appeal of the petitioners for the failure of the latter to file their appellants’ brief.
Section 1 (e), Rule 50 of the Rules of Court succinctly provides that:
Section 1. Grounds for dismissal of appeal. – An appeal may be dismissed by the Court of Appeals, on its own motion or on
that of the appellee, on the following grounds:
xxxx
(e) Failure of the appellant to serve and file the required number of copies of his brief or memorandum within the time provided
by these Rules; x x x
In a long line of cases, this Court has held that the CA’s authority to dismiss an appeal for failure to file the appellant’s brief is a
matter of judicial discretion. Thus, a dismissal based on this ground is neither mandatory nor ministerial; the fundamentals of
justice and fairness must be observed, bearing in mind the background and web of circumstances surrounding the case.
Having in mind the peculiar circumstances of the instant case, we find that the petitioners’ excuse for their failure to file their
brief was flimsy and discreditable and, thus, the propriety of the dismissal of their appeal. Indeed, as aptly ruled by the CA, the
records of the case clearly showed that the petitioners, through their counsel, received the January 30, 2009 Resolution which
required them to file their appellants’ brief. Thus:
The records of this case are clear that the Resolution of 30 January 2009 requiring the [petitioners] to file the required brief was
received by a certain Ruel de Tomas for [petitioners’] counsel on 05 February 2009. Hence, mere denial by [petitioners’]
counsel of the receipt of his copy of the Resolution cannot be given weight in the absence of any proof that the said person is
neither an employee at his law office nor someone unknown to him. Likewise, it is highly implausible that any person in the
building where [petitioners’] counsel holds office would simply receive a correspondence delivered by a postman.
Verily, the petitioners were only able to offer their bare assertion that they and their counsel did not actually receive a copy of
the January 30, 2009 Resolution and that the person who apparently received the same was not in any way connected with
their counsel. There was no other credible evidence adduced by the petitioners which would persuade us to exculpate them
from the effects of their failure to file their brief.
The Court notes that, in concluding that the petitioners indeed received a copy of the January 30, 2009 Resolution, the CA was
guided by the Report of the Judicial Records Division of the CA and by the certification issued by the Postmaster of Quezon
City. Indubitably, the petitioners’ bare assertions could not overcome the presumption of regularity in the preparation of the
records of the Post Office and that of the CA.
Nonetheless, the petitioners cite a cacophony of cases decided by this Court which, in essence, declared that dismissal of an
appeal on purely technical ground is frowned upon and that, as much as possible, appeals ought to be decided on the merits in
the interest of justice and equity.
The petitioners' plea for the application of the principles of substantial justice in their favor deserves scant consideration. The
petitioners should be reminded that technical rules may be relaxed only for the furtherance of justice and to benefit the
deserving.While the petitioners adverted to several jurisprudential rulings of this Court which set aside procedural rules, it is
noted that there were underlying considerations in those cases which warranted a disregard of procedural technicalities to
favor substantial justice. Here, there exists no such consideration.
The petitioners ought to be reminded that the bare invocation of "the interest of substantial justice" is not a magic wand that will
automatically compel this Court to suspend procedural rules. Procedural rules are not to be belittled or dismissed simply
because their non-observance may have resulted in prejudice to a party's substantive rights. Like all rules, they are required to
be followed except only for the most persuasive of reasons when they may be relaxed to relieve a litigant of an injustice not
commensurate with the degree of his thoughtlessness in not complying with the procedure prescribed.
In Asian Spirit Airlines v. Spouses Bautista, this Court clarified that procedural rules are required to be followed except only for
the most persuasive of reasons when they may be relaxed to relieve a litigant of an injustice not commensurate with the
degree of his thoughtlessness in not complying with the procedure prescribed:
We agree with the petitioner’s contention that the rules of procedure may be relaxed for the most persuasive reasons. But as
this Court held in Galang v. Court of Appeals:
Procedural rules are not to be belittled or dismissed simply because their non-observance may have resulted in prejudice to a
party’s substantive rights. Like all rules, they are required to be followed except only for the most persuasive of reasons when
they may be relaxed to relieve a litigant of an injustice not commensurate with the degree of his thoughtlessness in not
complying with the procedure prescribed.

Facts: First Union borrowed from BPI the sums of P5 million and $123,218.32, evidenced by separate promissory notes. As
3.Bank of the Section 5, Rule 7 of the Rules of Court - security defendant Linda and her spouse executed a REM Agreement covering 2 condo units. Linda executed a Surety
Philippine Failure to comply with the foregoing Agreement agreeing to be solidarily liable with First Union for its obligations to BPI. Despite repeated demands, First Union
Islands vs. CA requirements shall not be curable by mere failed to pay BPI. Thereafter, BPI initiated with RTC of Pasig extra-judicial foreclosure proceedings against the 2 mortgaged
168313 06 amendment of the complaint or other condo units. The properties were sold at public auction where BPI was the highest bidder but First Union still owed BPI a
initiatory pleading but shall be cause for the balance of P4.7 million and the foreign currency loan obligation remained unpaid.
October 2010-
ESPIRITU dismissal of the case without prejudice,
unless otherwise provided, upon motion First Union’s and Linda’s continued failure to settle their debt prompted BPI to file, a complaint for collection of sum of money
and after hearing. with the RTC of Makati. The complaint’s verification and certificate of non-forum shopping were signed by Ma. Cristina Asis
(Asis) and Kristine Ong (Ong). However, no Secretary’s Certificate or Board Resolution was attached to evidence Asis’ and
Ong’s authority to file the complaint.

Thereafter, First Union and Linda filed a motion to dismiss on the ground that BPI violated Rule 7, Section 5 of the Rules of
Civil Procedure; BPI failed to attach to the complaint the necessary board resolution authorizing Asis and Ong to institute the
collection action against First Union and Linda.

Thus, BPI filed an "Opposition to the Motion to Dismiss," arguing that the verification and certificate of non-forum shopping
sufficiently established Asis’ and Ong’s authority to file the complaint and proof of their authority could be presented during the
trial. Further, BPI alleged that a complaint "can only be dismissed under Section 5, Rule 7 of the 1997 Rules of Civil Procedure
if there was no certification against forum shopping." The provision, according to BPI, "does not even require that the person
certifying should show proof of his authority to do so."
Instead of submitting a board resolution, BPI attached a "Special Power of Attorney" (SPA) dated executed by Zosimo
Kabigting (Zosimo), Vice-President of BPI. The SPA authorized Asis and Ong or any lawyer from the Benedicto Versoza
Gealogo and Burkley Law Offices to initiate any legal action against First Union and Linda.

First Union and Linda argued that "an initiatory pleading which does not contain a board resolution authorizing the person to
show proof of his authority is equally guilty of not satisfying the requirements in the Certification against Non-Forum Shopping.
It is as though no certification has been filed." Thereafter, the RTC granted First Union’s and Linda’s Motion to Dismiss and
also denied BPI’s Motion for Reconsideration.
The CA, however, found that BPI failed to comply with the procedural requirements on non-forum shopping. Citing Sec. 5, Rule
7 of the Rules of Court, the CA ruled that the requirement that a petition should sign the certificate of non-forum shopping
applies even to corporations since the Rules of Court do not distinguish between natural and civil persons.
Issue/s: WON there is substantial compliance on the submission of a verification and certificate of non-forum shopping

Held: The verification of a complaint and the attachment of a certificate of non-forum shopping are requirements that are basic,
necessary and mandatory for procedural orderliness. It has long been recognized that strict compliance with the Rules of Court
is indispensable for the prevention of needless delays and for the orderly and expeditious dispatch of judicial business.
Procedural rules are not to be disdained as mere technicalities that may be ignored at will to suit the convenience of a party.
BPI did not submit any proof of authority in the first instance because it did not believe that a board resolution evidencing such
authority was necessary. In fact, BPI merely attached to its opposition a special power of attorney issued by Mr. Kabigting, a
bank vice-president, granting Asis and Ong the authority to file the complaint. Thus, no direct authority to file a complaint was
initially ever given by BPI – the corporate entity in whose name and behalf the complaint was filed.
Nonetheless, we hold that compliance with the certification requirement on non-forum shopping should not be made subject to
a party’s afterthought, lest the policy of the law be undermined.

Substantive justice in favor of procedural FACTS:


4.CMTC technicalities with its dismissal of Petitioner instituted a Complaint for Unfair Competition and/or Copyright Infringement and Claim for Damages with Prayer for
International petitioner’s appeal for failure to file the Temporary Restraining Order and Writ of Preliminary Injunction against respondent before the Regional Trial Court of Makati
Marketing appellant’s brief on time without (trial court).On February 14, 2005, the trial court rendered a Decision dismissing the complaint filed by petitioner.
Corporation vs. considering at all whether or not petitioner’s After receiving a copy of the trial court’s Decision, petitioner seasonably filed a Notice of Appeal before the CA on March 4,
appeal deserved full consideration on the 2005.Thereafter, the appellate court issued a Notice to File the Appellant’s Brief on May 20, 2005, which was received by the
Bhagis merits. law office representing petitioner on May 30, 2005.
International However, despite said notice, petitioner failed to file its appellant’s brief timely. Hence, on August 19, 2005, the appellate court
Trading issued a Resolution dismissing the appeal filed by petitioner.
Corporation Upon receipt of the order of dismissal, petitioner filed its Motion for Reconsideration with Motion to Admit Appellant’s Brief,
which was filed forty-two (42) days late from the date of its expiration on July 15, 2005.
170488 10 On November 15, 2005, the appellate court denied petitioner’s Motion for Reconsideration with Motion to Admit Appellant’s
December 2012- Brief. It ruled that one of the grounds by which the Court of Appeals may, on its own motion or that of the appellee, dismiss the
NAKAGAWA appeal is the failure on the part of the appellant to serve and file the required number of copies of his brief within the time
prescribed by the Rules of Court, viz.:
For this Court to admit the appellant’s brief after such wanton disregard of the Rules would put a strain on the orderly
administration of justice.
Hence, this petition for review on certiorari.
ISSUE: WON the dismissal of petitioner’s appeal is proper for its failure to file the appellant’s brief within the reglementary
period.

HELD: No, appellate court erred in dismissing the appeal.


Procedural rules should be treated with utmost respect and due regard, since they are designed to facilitate the adjudication of
cases to remedy the worsening problem of delay in the resolution of rival claims and in the administration of justice. However,
we have recognized exceptions to the Rules, but only for the most compelling reasons where stubborn obedience to the Rules
would defeat rather than serve the ends of justice.

The same principle was highlighted in Philippine National Bank and Development Bank of the Philippines v. Philippine Milling
Company, Incorporated, et al. where the Court ruled that even if an appellant failed to file a motion for extension of time to file
his brief on or before the expiration of the reglementary period, the Court of Appeals does not necessarily lose jurisdiction to
hear and decide the appealed case, and that the Court of Appeals has discretion to dismiss or not to dismiss appellant’s
appeal, which discretion must be a sound one to be exercised in accordance with the tenets of justice and fair play having in
mind the circumstances obtaining in each case.
Ergo, where strong considerations of substantive justice are manifest in the petition, the strict application of the rules of
procedure may be relaxed, in the exercise of its equity jurisdiction. Thus, a rigid application of the rules of procedure will not be
entertained if it will obstruct rather than serve the broader interests of justice in the light of the prevailing circumstances in the
case under consideration.
In the instant case, it is apparent that there is a strong desire to file an appellant’s brief on petitioner’s part.When petitioner filed
its motion attaching therewith its appellant’s brief, there was a clear intention on the part of petitioner not to abandon his
appeal. As a matter of fact, were it not for its counsel’s act of inadvertently misplacing the Notice to File Brief in another file,
petitioner could have seasonably filed its appellant’s brief as its counsel had already prepared the same even way before the
receipt of the Notice to File Brief.
It bears stressing at this point then that the rule, which states that the mistakes of counsel binds the client, may not be strictly
followed where observance of it would result in outright deprivation of the client’s liberty or property, or where the interest of
justice so requires. In rendering justice, procedural infirmities take a backseat against substantive rights of litigants. Corollarily,
if the strict application of the rules would tend to frustrate rather than promote justice, this Court is not without power to exercise
its judicial discretion in relaxing the rules of procedure.Also, it must be stressed that petitioner had no participatory negligence
in the dismissal of its appeal.1âwphi1 Hence, the ensuing dismissal of its appeal was completely attributable to the gross
negligence of its counsel. For said reason, the Court is not averse to suspending its own rules in the pursuit of justice. Where
reckless or gross negligence of counsel deprives the client of due process of law, or when the interests of justice so require,
relief is accorded to the client who suffered by reason of the lawyer’s gross or palpable mistake or negligence.
"the extraordinary writ of certiorari is always
5. A.L. Ang available where there is no appeal or any FACTS:
Network, Inc. vs. other plain, speedy and adequate remedy Petitioner filed a complaint for sum of money under the Rule of Procedure for Small Claims Cases before the MTCC, seeking
Mondejar in the ordinary course of law." to collect from respondent the amount of ₱23,111.71 which represented her unpaid water bills for the period June 1, 2002 to
200804 22 September 30, 2005.
January 2014-
Due to respondent's failure, petitioner disconnected respondent's water line for not paying the adjusted water charges since
NATO March 2003 up to August 2005.

The MTCC rendered a Decision in favor of respondent, holding that she should be considered to have fully paid petitioner.

Aggrieved, petitioner filed a petition for certiorari under Rule 65 of the Rules of Court before the RTC, ascribing grave abuse of
discretion on the part of the MTCC in finding that it (petitioner) failed to establish with certainty respondent's obligation, and in
not ordering the latter to pay the full amount sought to be collected.

The RTC issued a Decision dismissing the petition for certiorari, finding that the said petition was only filed to circumvent the
non-appealable nature of small claims cases as provided under Section 23 of the Rule of Procedure on Small Claims Cases.
To this end, the RTC ruled that it cannot supplant the decision of the MTCC with another decision directing respondent to pay
petitioner a bigger sum than that which has been awarded. Petitioner moved for reconsideration but was denied.

ISSUE/S:

Whether or not the RTC erred in dismissing petitioner's recourse under Rule 65 of the Rules of Court assailing the propriety of
the MTCC Decision in the subject small claims case.

HELD: Yes.

Section 23 of the Rule of Procedure for Small Claims Cases states that:

SEC. 23. Decision. — After the hearing, the court shall render its decision on the same day, based on the facts established by
the evidence (Form 13-SCC). The decision shall immediately be entered by the Clerk of Court in the court docket for civil cases
and a copy thereof forthwith served on the parties.

The decision shall be final and unappealable.

Considering the final nature of a small claims case decision under the above-stated rule, the remedy of appeal is not allowed,
and the prevailing party may, thus, immediately move for its execution. Nevertheless, the proscription on appeals in small
claims cases, similar to other proceedings where appeal is not an available remedy, does not preclude the aggrieved party
from filing a petition for certiorari under Rule 65 of the Rules of Court. This general rule has been enunciated in the case of
Okada v. Security Pacific Assurance Corporation, wherein it was held that:

In a long line of cases, the Court has consistently ruled that "the extraordinary writ of certiorari is always available where there
is no appeal or any other plain, speedy and adequate remedy in the ordinary course of law." In Jaca v. Davao Lumber Co., the
Court ruled:
x x x Although Section 1, Rule 65 of the Rules of Court provides that the special civil action of certiorari may only be invoked
when "there is no appeal, nor any plain, speedy and adequate remedy in the course of law," this rule is not without exception.
The availability of the ordinary course of appeal does not constitute sufficient ground to prevent a party from making use of the
extraordinary remedy of certiorari where appeal is not an adequate remedy or equally beneficial, speedy and sufficient. It is the
inadequacy – not the mere absence – of all other legal remedies and the danger of failure of justice without the writ that usually
determines the propriety of certiorari.

This ruling was reiterated in Conti v. Court of Appeals:

Truly, an essential requisite for the availability of the extraordinary remedies under the Rules is an absence of an appeal nor
any "plain, speedy and adequate remedy" in the ordinary course of law, one which has been so defined as a "remedy which
(would) equally (be) beneficial, speedy and sufficient not merely a remedy which at some time in the future will bring about a
revival of the judgment x x x complained of in the certiorari proceeding, but a remedy which will promptly relieve the petitioner
from the injurious effects of that judgment and the acts of the inferior court or tribunal" concerned. x x x (Emphasis supplied)

In this relation, it may not be amiss to placate the RTC's apprehension that respondent's recourse before it (was only filed to
circumvent the non-appealable nature of [small claims cases], because it asks [the court] to supplant the decision of the lower
[c]ourt with another decision directing the private respondent to pay the petitioner a bigger sum than what has been awarded.
Verily, a petition for certiorari, unlike an appeal, is an original action designed to correct only errors of jurisdiction and not of
judgment. Owing to its nature, it is therefore incumbent upon petitioner to establish that jurisdictional errors tainted the MTCC
Decision. The RTC, in turn, could either grant or dismiss the petition based on an evaluation of whether or not the MTCC
gravely abused its discretion by capriciously, whimsically, or arbitrarily disregarding evidence that is material to the controversy.

In view of the foregoing, the Court thus finds that petitioner correctly availed of the remedy of certiorari to assail the propriety of
the MTCC Decision in the subject small claims case, contrary to the RTC's ruling.

Likewise, the Court finds that petitioner filed the said petition before the proper forum (i.e., the RTC). To be sure, the Court, the
Court of Appeals and the Regional Trial Courts have concurrent jurisdiction to issue a writ of certiorari. Such concurrence of
jurisdiction, however, does not give a party unbridled freedom to choose the venue of his action lest he ran afoul of the doctrine
of hierarchy of courts. Instead, a becoming regard for judicial hierarchy dictates that petitions for the issuance of writs of
certiorari against first level courts should be filed with the Regional Trial Court, and those against the latter, with the Court of
Appeals, before resort may be had before the Court. This procedure is also in consonance with Section 4, Rule 65 of the Rules
of Court.

Hence, considering that small claims cases are exclusively within the jurisdiction of the Metropolitan Trial Courts, Municipal
Trial Courts in Cities, Municipal Trial Courts, and Municipal Circuit Trial Courts, certiorari petitions assailing its dispositions
should be filed before their corresponding Regional Trial Courts. This petitioner complied with when it instituted its petition for
certiorari before the RTC which, as previously mentioned, has jurisdiction over the same. In fine, the RTC erred in dismissing
the said petition on the ground that it was an improper remedy, and, as such, RTC Case No. 11-13833 must be reinstated and
remanded thereto for its proper disposition.

WHEREFORE, the petition is GRANTED. The Decision dated November 23, 2011 and Resolution dated February 16, 2012 of
the Regional Trial Court of Bacolod City, Branch 45 are REVERSED and SET ASIDE. RTC Case No. 11-13833 is hereby
REINSTATED and the court a quo is ordered to resolve the same with dispatch.
Jurisdiction over the subject matter is Facts:
6. Medical Plaza determined by the allegations in the
Makati complaint. It is not affected by the pleas or Respondent Robert H. Cullen purchased from Meridien Land Holding, Inc. (MLHI) condominium Unit No. 1201 of the Medical
Condominium the theories set up by the defendant in an Plaza Makati. Said title was later cancelled covered by Condominium Certificate of Title No. 45808 of the Register of Deeds of
Corp vs. Cullen answer or a motion to dismiss. Makati. And Condominium Certificate of Title No. 64218 was issued in the name of respondent.
181416 11
November 2013- Jurisdiction should be determined by On September 19, 2002, petitioner, through its corporate secretary, demanded from respondent payment for alleged unpaid
considering both the relationship of the association dues and assessments. Respondent disputed this demand claiming that he had been religiously paying his dues
PEREZ
parties as well as the nature of the question shown by the fact that he was previously elected president and director of petitioner. Petitioner, on the other hand, claimed that
involved. respondent’s obligation was a carry-over of that of MLHI. Consequently, respondent was prevented from exercising his right to
The nature of the action is determined by vote and be voted for during the 2002 election of petitioner’s Board of Directors. Respondent thus clarified from MLHI the
the body rather than the title of the veracity of petitioner’s claim, but MLHI claimed, that the matter was already settled and paid to defendant [MPMCC].
complaint
This prompted respondent to demand from petitioner an explanation why he was considered a delinquent payer despite the
Dispute as to the validity of the settlement of the obligation. Petitioner failed to make such explanation. Hence, the Complaint for Damages was filed by
assessments is purely an intra-corporate respondent against petitioner and MLHI.
matter between petitioner and respondent
and is thus within the exclusive jurisdiction Petitioner and MLHI filed their separate motions to dismiss the complaint on the ground of lack of jurisdiction.
of the RTC sitting as a special commercial
court.
MLHI claims that it is the Housing and Land Use Regulatory Board (HLURB) which is vested with the exclusive jurisdiction to
hear and decide the case.
Petitioner (MPMCC), on the other hand, raises the following specific grounds for the dismissal of the complaint:

(1) estoppel as respondent himself approved the assessment when he was the president;
(2) lack of jurisdiction as the case involves an intra-corporate controversy;
(3) prematurity for failure of respondent to exhaust all intra-corporate remedies; and
(4) the case is already moot and academic, the obligation having been settled between petitioner and MLHI.

On September 9, 2005, the RTC rendered a Decision granting petitioner’s and MLHI’s motions to dismiss and, consequently,
dismissing respondent’s complaint.

The trial court agreed with MLHI that the action for specific performance filed by respondent clearly falls within the exclusive
jurisdiction of the HLURB. As to petitioner, the court held that the complaint states no cause of action, considering that
respondent’s obligation had already been settled by MLHI. It, likewise, ruled that the issues raised are intra-corporate between
the corporation and member.

On appeal, the CA reversed and set aside the trial court’s decision and remanded the case to the RTC for further proceedings.
Contrary to the RTC conclusion, the CA held that the controversy is an ordinary civil action for damages which falls within the
jurisdiction of regular courts. It explained that the case hinged on petitioner’s refusal to confirm MLHI’s claim that the subject
obligation had already been settled as early as 1998 causing damage to respondent. Petitioner’s and MLHI’s motions for
reconsideration had also been denied.
Issue/s:

Whether or not the controversy involve intra-corporate issues as would fall within the jurisdiction of the RTC sitting as a special
commercial court or an ordinary action for damages within the jurisdiction of regular courts.

Held:
The petition is meritorious.
It is a settled rule that jurisdiction over the subject matter is determined by the allegations in the complaint. It is not affected by
the pleas or the theories set up by the defendant in an answer or a motion to dismiss.

In Go v. Distinction Properties Development and Construction, Inc.:

Basic as a hornbook principle is that jurisdiction over the subject matter of a case is conferred by law and determined by the
allegations in the complaint which comprise a concise statement of the ultimate facts constituting the plaintiff’s cause of action.
The nature of an action, as well as which court or body has jurisdiction over it, is determined based on the allegations
contained in the complaint of the plaintiff, irrespective of whether or not the plaintiff is entitled to recover upon all or some of the
claims asserted therein. The averments in the complaint and the character of the relief sought are the ones to be consulted.
Once vested by the allegations in the complaint, jurisdiction also remains vested irrespective of whether or not the plaintiff is
entitled to recover upon all or some of the claims asserted therein.

In determining whether a dispute constitutes an intra-corporate controversy, the Court uses two tests, namely, the relationship
test and the nature of the controversy test.

An intra-corporate controversy is one which pertains to any of the following relationships: (1) between the corporation,
partnership or association and the public; (2) between the corporation, partnership or association and the State insofar as its
franchise, permit or license to operate is concerned; (3) between the corporation, partnership or association and its
stockholders, partners, members or officers; and (4) among the stockholders, partners or associates themselves. Thus, under
the relationship test, the existence of any of the above intra-corporate relations makes the case intra-corporate.

Under the nature of the controversy test, "the controversy must not only be rooted in the existence of an intra-corporate
relationship, but must as well pertain to the enforcement of the parties’ correlative rights and obligations under the Corporation
Code and the internal and intra-corporate regulatory rules of the corporation."

In other words, jurisdiction should be determined by considering both the relationship of the parties as well as the nature of the
question involved.

Applying the two tests, we find and so hold that the case involves intra-corporate controversy. It obviously arose from the intra-
corporate relations between the parties, and the questions involved pertain to their rights and obligations under the Corporation
Code and matters relating to the regulation of the corporation.
Admittedly, petitioner is a condominium corporation duly organized and existing under Philippine laws, charged with the
management of the Medical Plaza Makati. Respondent, on the other hand, is the registered owner of Unit No. 1201 and is thus
a stockholder/member of the condominium corporation. Clearly, there is an intra-corporate relationship between the
corporation and a stockholder/member.
The nature of the action is determined by the body rather than the title of the complaint.
Though denominated as an action for damages, an examination of the allegations made by respondent in his complaint shows
that the case principally dwells on the propriety of the assessment made by petitioner against respondent as well as the validity
of petitioner’s act in preventing respondent from participating in the election of the corporation’s Board of Directors.
Respondent contested the alleged unpaid dues and assessments demanded by petitioner.

The nature of an action involving any dispute as to the validity of the assessment of association dues has been settled by the
Court in Chateau de Baie Condominium Corporation v. Moreno. In that case, respondents therein filed a complaint for intra-
corporate dispute against the petitioner therein to question how it calculated the dues assessed against them, and to ask an
accounting of association dues. Petitioner, however, moved for the dismissal of the case on the ground of lack of jurisdiction
alleging that since the complaint was against the owner/developer of a condominium whose condominium project was
registered with and licensed by the HLURB, the latter has the exclusive jurisdiction. In sustaining the denial of the motion to
dismiss, the Court held that the dispute as to the validity of the assessments is purely an intra-corporate matter between
petitioner and respondent and is thus within the exclusive jurisdiction of the RTC sitting as a special commercial court.

More so in this case as respondent repeatedly questioned his characterization as a delinquent member and, consequently,
petitioner’s decision to bar him from exercising his rights to vote and be voted for. These issues are clearly corporate and the
demand for damages is just incidental. Being corporate in nature, the issues should be threshed out before the RTC sitting as
a special commercial court. The issues on damages can still be resolved in the same special commercial court just like a
regular RTC which is still competent to tackle civil law issues incidental to intra-corporate disputes filed before it.

Moreover, Presidential Decree No. 902-A enumerates the cases over which the Securities and Exchange Commission (SEC)
exercises exclusive jurisdiction:

b) Controversies arising out of intra-corporate or partnership relations, between and among stockholders, members or
associates; between any or all of them and the corporation, partnership or association of which they are stockholders,
members, or associates, respectively; and between such corporation, partnership or association and the State insofar as it
concerns their individual franchise or right to exist as such entity; and

c) Controversies in the election or appointment of directors, trustees, officers, or managers of such corporations, partnerships,
or associations.29
To be sure, this action partakes of the nature of an intra-corporate controversy, the jurisdiction over which pertains to the SEC.
Pursuant to Section 5.2 of Republic Act No. 8799, otherwise known as the Securities Regulation Code, the jurisdiction of the
SEC over all cases enumerated under Section 5 of Presidential Decree No. 902-A has been transferred to RTCs designated by
this Court as Special Commercial Courts. While the CA may be correct that the RTC has jurisdiction, the case should have
been filed not with the regular court but with the branch of the RTC designated as a special commercial court. Considering that
the RTC of Makati City, Branch 58 was not designated as a special commercial court, it was not vested with jurisdiction over
cases previously cognizable by the SEC. The CA, therefore, gravely erred in remanding the case to the RTC for further
proceedings.

Indeed, Republic Act (RA) No. 9904, or the Magna Carta for Homeowners and Homeowners’ Associations, empowers the
HLURB to hear and decide inter-association and/or intra-association controversies or conflicts concerning homeowners’
associations. However, we cannot apply the same in the present case as it involves a controversy between a condominium unit
owner and a condominium corporation. While the term association as defined in the law covers homeowners’ associations of
other residential real property which is broad enough to cover a condominium corporation, it does not seem to be the legislative
intent. A thorough review of the deliberations of the bicameral conference committee would show that the lawmakers did not
intend to extend the coverage of the law to such kind of association.

To be sure, RA 4726 or the Condominium Act was enacted to specifically govern a condominium. Said law sanctions the
creation of the condominium corporation which is especially formed for the purpose of holding title to the common area, in
which the holders of separate interests shall automatically be members or shareholders, to the exclusion of others, in
proportion to the appurtenant interest of their respective units. The rights and obligations of the condominium unit owners and
the condominium corporation are set forth in the above Act.

Clearly, condominium corporations are not covered by the amendment. Thus, the intra-corporate dispute between petitioner
and respondent is still within the jurisdiction of the RTC sitting as a special commercial court and not the HLURB.

The CTA has jurisdiction over a special civil Facts:


7. City of Manila action for certiorari assailing an Petitioner City of Manila, through its treasurer, petitioner Liberty Toledo, assessed taxes for the taxable period from January to
vs. Judge interlocutory order issued by the RTC in a December 2002 against private respondents SM Mart, Inc., SM Prime Holdings, Inc., Star Appliances Center, Supervalue, Inc.,
Grecia-Cuerdo, local tax case. Ace Hardware Philippines, Inc., Watsons Personal Care Stores Phils., Inc., Jollimart Philippines Corp., Surplus Marketing Corp.
et al 175723 04 and Signature Lines. In addition to the taxes purportedly due from private respondents, said assessment covered the local
business taxes. Because payment of the taxes assessed was a precondition for the issuance of their business permits, private
February 2014-
respondents were constrained to pay the P19,316,458.77 assessment under protest.
PURIFICACION
Private respondents filed [with the Regional Trial Court of Pasay City] the complaint denominated as one for "Refund or
Recovery of Illegally and/or Erroneously-Collected Local Business Tax, Prohibition with Prayer to Issue TRO and Writ of
Preliminary Injunction". RTC granted private respondents' application for a writ of preliminary injunction. Petitioners filed a
Motion for Reconsideration but the RTC denied it in its Order. Petitioners then filed a special civil action for certiorari with the
CA assailing the July 9, 2004 and October 15, 2004 Orders of the RTC. The CA dismissed petitioners' petition for certiorari
holding that it has no jurisdiction over the said petition. The CA ruled that since appellate jurisdiction over private respondents'
complaint for tax refund, which was filed with the RTC, is vested in the Court of Tax Appeals (CTA), pursuant to its expanded
jurisdiction under Republic Act No. 9282 (RA 9282), it follows that a petition for certiorari seeking nullification of an interlocutory
order issued in the said case should, likewise, be filed with the CTA.

Issue/s: Whether or not the CTA has jurisdiction over a special civil action for certiorari assailing an interlocutory order issued
by the RTC in a local tax case.

Held: Section 5 (1), Article VIII of the 1987 Constitution grants power to the Supreme Court, in the exercise of its original
jurisdiction, to issue writs of certiorari, prohibition and mandamus. With respect to the Court of Appeals, Section 9 (1) of Batas
Pambansa Blg. 129 (BP 129) gives the appellate court, also in the exercise of its original jurisdiction, the power to issue,
among others, a writ of certiorari, whether or not in aid of its appellate jurisdiction. As to Regional Trial Courts, the power to
issue a writ of certiorari, in the exercise of their original jurisdiction, is provided under Section 21 of BP 129.

The foregoing notwithstanding, while there is no express grant of such power, with respect to the CTA, Section 1, Article VIII of
the 1987 Constitution provides, nonetheless, that judicial power shall be vested in one Supreme Court and in such lower courts
as may be established by law and that judicial power includes the duty of the courts of justice to settle actual controversies
involving rights which are legally demandable and enforceable, and to determine whether or not there has been a grave abuse
of discretion amounting to lack or excess of jurisdiction on the part of any branch or instrumentality of the Government.

On the strength of the above constitutional provisions, it can be fairly interpreted that the power of the CTA includes that of
determining whether or not there has been grave abuse of discretion amounting to lack or excess of jurisdiction on the part of
the RTC in issuing an interlocutory order in cases falling within the exclusive appellate jurisdiction of the tax court. It, thus,
follows that the CTA, by constitutional mandate, is vested with jurisdiction to issue writs of certiorari in these cases.

For RTCs outside Metro Manila to claim Facts:


8. Cabrera vs. jurisdiction over the subject matter of a civil On Oct. 25, 1976, Atty. Gella, respondents’ father, executed a private document confirming that he has appointed Severino as
Francisco case, the demand in the complaint must administrator of all his real properties located in Antique. When Severino died in 1991, Araceli (Severino’s wife) and Arnel
172293 28 exceed Php 200,000.00. (Severino’s son), with the consent of the respondents, took over the administration of the properties. Respondents instructed
August 2013- them to look for buyers of the properties, allegedly promising them “a commission of five percent of the total purchase price of
the said properties as compensation for their long continued administration.”
SABIO
Petitioners introduced real estate broker and President of ESV Marketing and Development Corp., Erlinda, to the respondents
who agreed to have the properties developed by Erlinda’s company. However, a conflict arose when respondents appointed
Erlinda as the new administratrix of the properties and terminated Araceli and Arnel’s services.

Petitioners, through counsel, wrote respondents and demanded for their five percent commission and compensation to no
avail. Hence, on Sept. 03, 2001, they filed a Complaint of Agent’s Compensation, Commission, and Damages.

RTC RULING: Ruled in favor of respondents on the grounds of (1) lack of jurisdiction, (2) failure to state a cause of action, and
(3) lack of legal capacity of Araceli to sue in behalf of the other heirs of Severino.

Respondents argued that for RTCs outside of Metro Manila to take cognizance of a civil suit, the jurisdictional amount must
exceed Php 200,000, five percent of the total market value of the lot was only Php 177,506.60. The complaints state no cause
of action since petitioners’ right to any commission remained inchoate as the lot has not yet been sold and they merely
introduced a real estate broker to respondents. Lastly, petitioners have no legal capacity to sue on behalf of the heirs of
Severino as the verification of non-forum shopping attached to the Complaint only mentioned Araceli and Arnel as plaintiffs.

CA RULING: Found no error in the ruling of the RTC, ruled in favor of respondents.

Issue/s:
Whether or not the RTC has jurisdiction over the subject matter of the case.

Held:
NO. To determine whether the RTC has jurisdiction over petitioners’ Complaint, respondents correctly cited Sec. 19(8) of BP
129, which provides:

Sec. 19. Jurisdiction in Civil Cases - RTC shall exercise exclusive original jurisdiction:
(8) In all other cases in which the demand, exclusive of interests, damages of whatever kind, attorney’s fees, litigation
expenses, and costs of the value of the property exceed Php 100,000.00, or in such other cases in Metro Manila, where the
demand, exclusive of the above mentioned items exceeds Php 200,000.00.

NOTE: The jurisdictional amount of Php 100,000.00 for RTC’s outside Metro Manila was adjusted to Php 200,000.00 in
accordance with Sec. 5 of RA 7691.

The demand in their Complaint must therefore exceed Php 200,000 in order for it to fall under the jurisdiction of the RTC.

JURISDICTION DOES NOT DEPEND Facts:


9. Platinum UPON THE REGULARITY OF THE Civil Case No. 94-1634- (Platinum filed against PATC- Collection suit)
Tours and EXERCISE BY THE COURT OF THAT On April 27, 1994, petitioner Platinum Tours and Travel Inc. (Platinum) filed a complaint for a sum of money with damages
Travel POWER OR ON THE CORRECTNESS OF against Pan Asiatic Travel Corporation (PATC) and its president Nelida G. Galvez. Platinum sought to collect payment for
Incorporated vs. ITS DECISIONS.— Jurisdiction is the the airline tickets which PATC bought from it.
power and authority of the court to hear, try On October 24, 1994, the RTC of Makati City, Br. 62, rendered a judgment by default in favor of Platinum and ordered PATC
Panlilio 133365
and decide a case. In general, jurisdiction and Nelida G. Galvez to solidarily pay Platinum actual damages of P359,621.03 with legal interest, P50,000 attorney's fees and
16 September may either be over the nature of the action, cost of suit.
2013-SOMEROS over the subject matter, over the person of On February 10, 1995, a writ of execution was issued on motion of Platinum. Pursuant to the writ, Manila Polo Club Proprietary
the defendants or over the issues framed in Membership Certificate No. 2133 in the name of Nelida G. Galvez was levied upon and sold for P479,888.48 to a certain Ma.
the pleadings. Rosario Khoo.
Jurisdiction over the nature of the On June 2, 1995, private respondent Jose M. Panlilio filed a motion to intervene in Civil Case No. 94-1634(collection
action and subject matter is conferred suit). Panlilio claimed that, in October 1992, Galvez had executed in his favor a chattel mortgage over her shares of stock in
by law. It is determined by the allegations the Manila Polo Club to secure her P1 million loan and that Galvez had already delivered to him the stock certificates valued at
of the complaint, irrespective of whether or P5 million.
not the plaintiff is entitled to recover upon On June 9, 1995, the trial court denied Panlilio's motion for intervention.
all or some of the claims asserted therein. On January 29, 1996, the trial court declared the execution sale null and void due to irregularities in the conduct thereof.
Jurisdiction over the person of the Civil Case No. 96-365: (Panlilio filed against Galvez- Collection suit)
plaintiff is acquired from the time he On May 3, 1996, Panlilio filed against Galvez a collection case with application for a writ of preliminary attachment of the
files his complaint; disputed Manila Polo Club shares. The case was raffled to Br. 146 of the RTC of Makati City.
jurisdiction over the person of the In the meantime, Panlilio again attempted to intervene in Civil Case No. 94-1634 (Platinum against PATC- Collection suit),
defendant is acquired by his voluntary this time by incorporating in his complaint a motion to consolidate Civil Case No. 96-365 and Civil Case No. 94-1634.
appearance in court and his submission On June 13, 1996, Judge Salvador Tensuan of Br. 146 granted the motion for consolidation on condition that Judge
to its authority, or by the coercive power Roberto Diokno of Br. 62, who was trying Civil Case No. 94-1634, would not object thereto. Judge Diokno in his order, dated
of legal processes exerted over his July 23, 1996, allowed the consolidation and setting for hearing Panlilio's application for a writ of preliminary attachment.
person. Platinum, as plaintiff in Civil Case No. 94-1634, moved for reconsideration but was denied.
Since jurisdiction is the power to hear and On January 31, 1997, Platinum filed a petition for certiorari at the CA of the order consolidation.
determine a particular case, it does not CA annulled the assailed order but left it to Judge Diokno to decide whether to return Civil Case No. 96-365 to Judge
depend upon the regularity of the exercise Tensuan in Br. 146, or to keep it in his docket and decide it as a separate case.
by the court of that power or on the Platinum filed a motion for partial reconsideration of the decision of the CA, praying that Civil Case No. 96-365 be returned to
correctness of its decisions. Judge Tensuan Br. 146 or re-raffled to another RTC Makati. However, it was denied by CA.
Platinum argues that, when Judge Diokno's order allowing the consolidation of the two cases was annulled and set aside, RTC
Br. 62's basis for acquiring jurisdiction over Civil Case No. 96-365 was likewise extinguished.

Issue: WON RTC Br. 62 under Judge Diokno has jurisdiction to try Civil Case No. 96-365? (Panlilio against Galvez- Collection
suit)
Held: YES.

Jurisdiction is the power and authority of the court to hear, try and decide a case. In general, jurisdiction may either be
over the nature of the action, over the subject matter, over the person of the defendants or over the issues framed in the
pleadings.
Jurisdiction over the nature of the action and subject matter is conferred by law. It is determined by the allegations of the
complaint, irrespective of whether or not the plaintiff is entitled to recover upon all or some of the claims asserted therein.
Jurisdiction over the person of the plaintiff is acquired from the time he files his complaint; while jurisdiction over the person
of the defendant is acquired by his voluntary appearance in court and his submission to its authority, or by the coercive
power of legal processes exerted over his person.
Since jurisdiction is the power to hear and determine a particular case, it does not depend upon the regularity of the exercise
by the court of that power or on the correctness of its decisions.
In the case at bar, there is no doubt that Panlilio's collection case docketed as Civil Case No. 96-365 falls within the
jurisdiction of the RTC of Makati, Br. 62. The fact that the CA subsequently annulled Judge Diokno's order granting the
consolidation of Civil Case No. 96-365 and Civil Case No. 94-1634, did not affect the jurisdiction of the court which
issued the said order.
"Jurisdiction" should be distinguished from the "exercise of jurisdiction." Jurisdiction refers to the authority to decide a case, not
the orders or the decision rendered therein. Accordingly, where a court has jurisdiction over the person and the subject matter,
as in the instant case, the decision on all questions arising from the case is but an exercise of such jurisdiction. Any error that
the court may commit in the exercise of its jurisdiction is merely an error of judgment which does not affect its authority to
decide the case, much less divest the court of the jurisdiction over the case.
|

- The Regional Trial Court and not the Facts:


10. People vs. Sandiganbayan, has jurisdiction over On December 4, 1982, about 8:00 o'clock or 8:30 o'clock in the evening, Vicente Elisan and his elder brother Ronie Elisan,
Cawaling informations for murder committed by were drinking tuba at C & J-4 Kitchenette (a Restaurant) of co-accused Andres Fontamillas in Poblacion, San Jose,Romblon.
117970 28 July public officers, including a town mayor. As the two were about to go home, they saw the six (6) accused outside, that is, Mayor Cawaling, the four (4) policemen,
1998- TAN namely, Hilario Cajilo, Andres Fontamillas, Ernesto Tumbagahan and Ricardo delos Santos, and civilian Alex Batuigas, the
- Jurisdiction is determined by the mayor's brother-in-law as if waiting for them. The two ran, Ronie Elisan, the victim, ran towards the ricefields, he fell down and
allegations in the complaint or the six caught up to him, he was seen by Vicente, who was hiding, kneeling and raising both hands up as if surrendering when
information. In the absence of any he was flashlighted and shot dead.
allegation that the offense was committed
in relation to the office of appellants or was Accused-appellants Mayor Ulysses M. Cawaling and Policemen Ernesto Tumbagahan, Ricardo De los Santos and Hilario
necessarily connected with the discharge of Cajilo were convicted of murder by the Regional Trial Court of Romblon for the killing of Ronie Elisan. The killing was qualified
their functions, the regional trial court, not to murder because of the aggravating circumstances of abuse of superior strength and treachery. The trial court ruled that
the Sandiganbayan, has jurisdiction to hear there was a notorious inequality of forces between the victim and his assailants, as the latter were greater in number and
and decide the case. armed with guns. It further ruled that abuse of superior strength absorbed treachery.

Appellants Tumbagahan and Cajilo argue that the trial court erred when it assumed jurisdiction over the criminal case. They
insist that the Sandiganbayan, not the regular courts, had jurisdiction to try and hear the case against the appellants, as they
were public officers at the time of the killing which was allegedly committed by reason of or in relation to their office. The Court
ruled that the information filed against the appellants contains no allegation that appellants were public officers who committed
the crime in relation to their office.
Issue/s: W/N the Regional Trial Court has jurisdiction

Held:
Yes. The trial court has jurisdiction. Jurisdiction is determined by the allegations in the complaint or information. In the absence
of any allegation that the offense was committed in relation to the office of appellants or was necessarily connected with the
discharge of their functions, the regional trial court, not the Sandiganbayan, has jurisdiction to hear and decide the case.

Section 4-a-2 of PD 1606, as amended by PD 1861 states that before the Sandiganbayan may exercise exclusive and original
jurisdiction over a case: (a) the offense was committed by the accused public officer in relation to his office; and (b) the penalty
prescribed by law is higher than prision correccional or imprisonment for six (6) years, or higher than a fine of six thousand
pesos (P6,000). Sanchez vs. Demetriou, clarified that murder or homicide may be committed both by public offcers and by
private citizens, and that public office is not a constitutive element of said crime.

Furthermore, the Information filed against the appellants contains no allegation that appellants were public officers who
committed the crime in relation to their office. The charge was for murder, a felony punishable under Article 248 of the Revised
Penal Code. As clarified in Aguinaldo, et al. vs. Domagas, et al.,"[I]n the absence of such essential allegation, and since the
present case does not involve charges of violation of R.A. No. 3019 (the Anti-Graft etc. Act), the Sandiganbayan does not have
jurisdiction over the present case.

Settled is the rule that a tribunal, board, or Facts:


11. Magpale vs. officer exercising judicial functions acts Petitioner Magpale was a government employee: first in the Presidential Assistance on Community Development, then in the
Civil Service without jurisdiction if no authority has been Philippine Ports Authority (PPA) as an Arrastre Superintendent. He was later promoted as Port Manager of the Port
Commission conferred by law to hear and decide the Management Unit (PMU), General Santos City. In the same year, he was reassigned to PPA-PMU, Tacloban City where he
case. discharged the functions of the same position. Magpale, after the designation of Atty. William Enriquez as OIC of PPA-PMU,
97381 05
Tacloban City, was ordered to immediately report to the Assistant General Manager for Operation, PPA, Manila, which he did.
November 1992- In an Internal Control Department Report, the PMU-Tacloban Inventory Committee and the COA stated that Magpale failed to
TAÑADA The exercise of the CSC’s power to hear
appeals is qualified by and should be read account for PPA equipment, to liquidate cash advances, and was found to have incurred unauthorized absences. As a result,
together with the provisions Executive he was charged with Dishonesty, Pursuit of Private Business without permission as required by the CSC Rules and
Order 292 (Administrative Code of 1987), Regulations, Frequent and Unauthorized Absences, and Neglect of Duty. In 1989, a Decision was rendered by the DOTC
particularly Section 49 (of Chapter 3, Secretary, finding Magpale guilty of gross negligence and frequent and unauthorized absences. He was dismissed from
Subtitle 3, Book V) which prescribes the service. After Magpale’s motion for reconsideration was denied, he appealed to the Merit System and Protection Board
following requisites for the exercise of the (MSPB) of respondent CSC. The MSPB reversed the decision of the DOTC Secretary and ordered for Magpale’s
power of appeal, to wit: (a) the decision reinstatement. The PPA, through its General Manager Rogelio Dayan, then appealed to the Civil Service Field Office-PPA, and
must be appealable; (b) the appeal must be the latter endorsed the appeal to respondent CSC. Magpale filed with the MSPB a Motion for Implementation of the MSPB
made by the party adversely affected by decision. The MSPB then issued an Order for the immediate implementation of its Decision. The CSC later rendered a
the decision; (c) the appeal must be made Decision, reversing the MSPB decision. Thereafter, Magpale filed the instant case before the SC alleging that: 1) appeal by the
within fifteen days from receipt of the government from an adverse decision of the MSPB is improper; 2) the PPA General Manager did not have the legal personality
decision, unless a petition for the to appeal the MSPB decision; and, 3) assuming appeal was available, the same was filed out of time.
reconsideration is seasonably filed; and (d)
the notice of appeal must be filed with the Issue/s:
disciplining office, which shall forward the Whether or not the appeal of the PPA to the CSC was proper.
records of the case, together with the
notice of appeal to the appellate authority Held:
within fifteen days from filing of the notice
of appeal, with its comments, if any. No. While it is true that, under Sec. 12, par. 11, Chapter 3, Subtitle A, Book V of the Administrative Code of 1987, the CSC
Under Section 47 of the same Code, the does have the power to “hear and decide administrative cases instituted by or brought before it directly or on appeal, including
CSC shall decide on appeal all contested appointments, and review decisions and actions of its offices and of the agencies attached to it”, the exercise of the
administrative disciplinary cases involving power is qualified by and should be read together with the other sections of the same sub-title and book of Executive Order
the imposition of: (a) a penalty of 292 (Administrative Code of 1987), particularly Section 49 which prescribes the following requisites for the exercise of the
suspension for more than thirty days; (b) power of appeal, to wit: (a) the decision must be appealable; (b) the appeal must be made by the party adversely affected by
fine in an amount exceeding thirty days the decision; (c) the appeal must be made within fifteen days from receipt of the decision, unless a petition for the
salary; (c) demotion in rank or salary or reconsideration is seasonably filed; and (d) the notice of appeal must be filed with the disciplining office, which shall forward
transfer; or (d) removal or dismissal from the records of the case, together with the notice of appeal to the appellate authority within fifteen days from filing of the notice
office. of appeal, with its comments, if any.
Under Section 47 of the same Code, the CSC shall decide on appeal all administrative disciplinary cases involving the
imposition of: (a) a penalty of suspension for more than thirty days; (b) fine in an amount exceeding thirty days salary; (c)
demotion in rank or salary or transfer; or (d) removal or dismissal from office.
The decision of the MSPB did not involve dismissal or separation from office, rather it exonerated Magpale and ordered his
reinstatement. Consequently, the MSPB decision was not a proper subject of appeal to the CSC. Settled is the rule that a
tribunal, board, or officer exercising judicial functions acts without jurisdiction if no authority has been conferred by law to hear
and decide the case.

Under Section 1 of Presidential Decree No. Facts:


12. Sandoval vs. 957 the National Housing Authority (NHA) On August 20, 1987 private respondent filed a complaint in the Regional Trial Court (RTC) of Manila for the collection of unpaid
Cañeba 90503 was given the exclusive jurisdiction to hear installments regarding a subdivision lot, pursuant to a promissory note, plus interest. On January 29, 1988 the trial court
27 September and decide certain cases as follows: rendered a decision
1990- UBAY "SECTION 1. In the exercise of its function It appears that petitioner was declared in default so much so that after receiving the evidence of private respondent, the trial
to regulate the real estate trade and court rendered its decision. On September 28, 1988 the trial court issued an order directing the issuance of a writ of execution
business and in addition to its powers to enforce its decision that had become final and executory.
provided for in Presidential Decree No. On September 30, 1988 petitioner filed a motion to vacate judgment and to dismiss the complaint on the ground that the lower
957, the National Housing Authority shall court has no jurisdiction over the subject matter and that its decision is null and void. A motion for reconsideration of the writ of
have exclusive jurisdiction to hear and execution was also filed by petitioner. An opposition to both motions was filed by private respondent to which a reply was filed
decide cases of the following nature: by petitioner.
A. Unsound real estate business practices: On February 17, 1989 the trial court denied the motion to vacate the judgment on the ground that it is now beyond the
B. Claims involving refund and any other jurisdiction of the Court to do so. It directed the issuance of a writ of execution anew.
claims filed by subdivision lot or Hence the herein petition wherein it is alleged that the trial court committed a grave abuse of discretion.
condominium unit buyer against the project
owner, developer, dealer, broker or Issue/s:
salesman; and Whether or not the ordinary courts have jurisdiction over the collection of unpaid installments regarding a subdivision lot
C. Cases involving specific performance of
contractual and statutory obligations filed Held:
by buyers of subdivision lot or The petition is impressed with merit.
condominium unit against the owner, Under Section 1 of Presidential Decree No. 957 the National Housing Authority (NHA) was given the exclusive jurisdiction to
developer, dealer, broker or salesman. hear and decide certain cases as follows:
||| "SECTION 1. In the exercise of its function to regulate the real estate trade and business and in addition to its powers provided
for in Presidential Decree No. 957, the National Housing Authority shall have exclusive jurisdiction to hear and decide cases of
the following nature:
A. Unsound real estate business practices:
B. Claims involving refund and any other claims filed by subdivision lot or condominium unit buyer against the project owner,
developer, dealer, broker or salesman; and
C. Cases involving specific performance of contractual and statutory obligations filed by buyers of subdivision lot or
condominium unit against the owner, developer, dealer, broker or salesman.

The language of this section, particularly, the second portion thereof, leaves no room for doubt that exclusive jurisdiction over
the case between the petitioner and private respondent is vested not on the RTC but on the NHA. The NHA was re-named
Human Settlements Regulatory Commission and thereafter it was re-named as the Housing and Land Use Regulatory Board
(HLURB).

Undeniably the sum of money sought to be collected by private respondent from petitioner represented unpaid installments of a
subdivision lot which the petitioner purchased. Petitioner alleges that he suspended payments thereof because of the failure of
the developer to develop the subdivision pursuant to their agreement.
When as in this case the attention of the trial court is drawn to its lack of competence and authority to act on the case, certainly
the trial court has a duty to vacate the judgment by declaring the same to be null and void ab initio.
This is as it should be. Inasmuch as the questioned judgment is null and void, it is, as above observed, as if no decision had
been rendered by the trial court. It cannot become final and executory, much less can it be enforced by a writ of execution.
The trial court, rather than reiterating the issuance of a writ of execution in this case, which it did, should have recalled and
cancelled the writ of execution of the judgment.
WHEREFORE, the petition is GRANTED. The questioned decision of the trial court dated January 29, 1988 is hereby declared
null and void for lack of jurisdiction.

The Supreme Court is a court of last resort, Facts:


13. Quesada vs. and must so remain if it is to satisfactorily Respondent Teruel filed with the Office of the City Prosecutor in Mandaluyong City an affidavit-complaint against petitioner,
DOJ 150325 31 perform the functions assigned to it by the Camacho, Jr., and Corgado with the crime of estafa under Article 315 (2) and (3) of the Revised Penal Code, thereto which
August 2006- fundamental charter and immemorial alleged that on June 13, 1998 at Shangrila Plaza Hotel, Quesada, Camacho, and Corgado represented themselves to Teruel
ZAPANTA tradition. It cannot and should not be as the president, vice-president/treasurer, and managing director, of VSH Group Corporation; that they offered to him a
burdened with the task of dealing with telecommunication device called Star Consultant Equipment Package which provides the user easy access to the internet via
causes in the first instance. Its original television; that they assured him that after he pays the purchase price of P65,000.00, they will immediately deliver to him two
jurisdiction to issue the so-called units of the internet access device; that relying on their representations, he paid them P65,000.00 for the two units; and that
extraordinary writs should be exercised despite demands, they, did not deliver to him the units. This was opposed by petitioner who filed a counter-affidavit
only where absolutely necessary or where Thereafter, an Information for estafa was filed with the RTC upon the recommendation of Assistant City Prosecutor Esteban A.
serious and important reasons exist Tacla, Jr. after the latter’s issuance of a Resolution finding probable cause. In the meantime, petitioner filed with the
therefor. Hence, that jurisdiction should Department of Justice a Petition for Review challenging the Resolution of the Investigating Prosecutor, but was however,
generally be exercised relative to actions or dismissed.
proceedings before the Court of Appeals, Pending the criminal case at the RTC, petitioner filed with the Supreme Court a Petition for Certiorari alleging that the
or before constitutional or other tribunals, Secretary of Justice, in dismissing his Petition for Review, acted with grave abuse of discretion amounting to lack or excess of
bodies or agencies whose acts for some jurisdiction.
reason or another are not controllable by
the Court of Appeals. Where the issuance Issue/s: W/N the filing of the petition directly with the Supreme Court constitutes a violation of the rule on hierarchy of courts
of an extraordinary writ is also within the
competence of the Court of Appeals or a Held:
Regional Trial Court, it is in either of these Yes. A petition for certiorari under Rule 65 of the 1997 Rules of Civil Procedure, as amended, must be filed with the Court of
courts that the specific action for the writ’s Appeals whose decision may then be appealed to this Court by way of a petition for review on certiorari under Rule 45 of the
procurement must be presented. same Rules. A direct recourse to this Court is warranted only where there are special and compelling reasons specifically
alleged in the petition to justify such action.
The Supreme Court is a court of last resort, and must so remain if it is to satisfactorily perform the functions assigned to it by
the fundamental charter and immemorial tradition. It cannot and should not be burdened with the task of dealing with causes in
the first instance. Its original jurisdiction to issue the so-called extraordinary writs should be exercised only where absolutely
necessary or where serious and important reasons exist therefor. Hence, that jurisdiction should generally be exercised relative
to actions or proceedings before the Court of Appeals, or before constitutional or other tribunals, bodies or agencies whose
acts for some reason or another are not controllable by the Court of Appeals. Where the issuance of an extraordinary writ is
also within the competence of the Court of Appeals or a Regional Trial Court, it is in either of these courts that the specific
action for the writ’s procurement must be presented.
The hierarchy of courts is determinative of the venue of appeals, and should also serve as a general
determinant of the appropriate forum for petitions for the extraordinary writs. It is a policy that is necessary to prevent inordinate
demands upon the Court’s time and attention which are better devoted to those matters within its exclusive jurisdiction, and to
prevent further over-crowding of the Court’s docket.

Here, while the Pangkat was not Nature of the case: Petition for Review on Certiorari assailing the Decision of the CA and its Resolution
14. Zamora vs. constituted, however, the parties met nine
Heirs of Carmen (9) times at the Office of the Barangay Facts:
146195 18 Chairman for conciliation wherein not only Carmen Izquierdo and Pablo Zamora entered into a verbal stipulation whereby the former leased to the latter one of her
November 2004- the issue of water installation was apartment units located at 117-B General Luna Street, Caloocan City.
discussed but also petitioners' violation of
ABAD the lease contract. It is thus manifest that After the death of Carmen (lessor) in 1996 her attorney-in-fact, Anita Punzalan, representing the heirs, herein respondents,
there was substantial compliance with prepared a new contract of lease wherein the rental was increased from P3,000.00 to P3,600.00 per month. However,
the law which does not require strict petitioners refused to sign it.
adherence thereto.
Pablo (lessee) died. His wife, Avelina Zamora, and their children (two of whom have their own families), herein petitioners,
continued to reside in the apartment unit. However, they refused to pay the increased rental and persisted in operating a
photocopying business in the same apartment.

Meanwhile, petitioner Avelina Zamora applied with the MWSS for a water line installation in the premises. Since a written
consent from the owner is required for such installation, she requested respondents' attorney-in-fact to issue it. However, the
latter declined because petitioners refused to pay the new rental rate and violated the restrictions on the use of the premises by
using a portion thereof for photocopying business and allowing three families to reside therein.

This prompted petitioner Avelina Zamora to file with the Office of the Punong Barangay of Barangay 16, Sona 2, District
I, Lungsod ng Caloocan, a complaint against Anita Punzalan (respondents' attorney-in-fact)

During the barangay conciliation proceedings, petitioner Avelina Zamora declared that she refused to sign the new lease
contract because she is not agreeable with the conditions specified therein.

Despite several barangay conciliation sessions, the parties failed to settle their dispute amicably. Hence, the Barangay
Chairman issued a Certification to File Action.

Consequently, on October 2, 1997, respondents, represented by Anita Punzalan, filed with the MTC, Caloocan City, a
complaint for unlawful detainer and damages against petitioners.

Petitioners filed a motion to dismiss the complaint on the ground that the controversy was not referred to the
barangay for conciliation.
(1) they alleged that the barangay Certification to File Action "is fatally defective" because it pertains to another dispute,
(2) when the parties failed to reach an amicable settlement before the Lupong Tagapamayapa, the Punong Barangay (as
Lupon Chairman), did not constitute the Pangkat ng Tagapagkasundo before whom mediation or arbitration proceedings
should have been conducted, in violation of Section 410(b), Chapter 7 (Katarungang Pambarangay)

Respondents opposed the motion to dismiss, the same being prohibited under Section 19 of the 1991 Revised Rule on
Summary Procedure.

MTC - issued an Order denying petitioners' motion to dismiss and considering the case submitted for decision in view of their
failure to file their answer to the complaint.

Petitioners filed a motion for reconsideration, contending that a motion to dismiss the complaint on the ground of failure to refer
the complaint to the Lupon for conciliation is allowed under Section 19 of the 1991 Revised Rule on Summary Procedure.
MTC rendered a Judgment in favor of respondents and against petitioners

RTC - affirmed RTC. Subsequently, it denied petitioners’ motion for recon

CA - affirmed RTC. Thereafter, petitioners filed a motion for reconsideration but was denied by the CA. Hence, the instant
petition.

Issue/s:
Whether or not mandatory conciliation is required in the present case

Held:
The primordial objective of Presidential Decree No. 1508 (the Katarungang Pambarangay Law), now included under R.A. No.
7160 (the Local Government Code of 1991), is to reduce the number of court litigations and prevent the deterioration of the
quality of justice which has been brought about by the indiscriminate filing of cases in the courts. To attain this objective,
Section 412(a) of R.A. No. 7160 requires the parties to undergo a conciliation process before the Lupon Chairman or the
Pangkat as a precondition to filing a complaint in court.

In the case at bar, the Punong Barangay, as Chairman of the Lupong Tagapamayapa, conducted conciliation proceedings to
resolve the dispute between the parties herein. Contrary to petitioners' contention, the complaint does not only allege, as a
cause of action, the refusal of respondents' attorney-in-fact to give her consent to the installation of water facilities in the
premises, but also petitioners' violation of the terms of the lease, specifically their use of a portion therein for their photocopying
business and their failure to pay the increased rental.

We cannot sustain petitioners' contention that the Lupon conciliation alone, without the proceeding before the Pangkat ng
Tagapagkasundo, contravenes the law on Katarungang Pambarangay. Section 412(a) of R.A. No. 7160, quoted earlier, clearly
provides that, as a precondition to filing a complaint in court, the parties shall go through the conciliation process either before
the Lupon Chairman, or the Pangkat.

Here, while the Pangkat was not constituted, however, the parties met nine (9) times at the Office of the Barangay Chairman
for conciliation wherein not only the issue of water installation was discussed but also petitioners' violation of the lease contract.
It is thus manifest that there was substantial compliance with the law which does not require strict adherence thereto.

The efforts of the Barangay Chairman, Facts:


15. Lumbuan vs. however, proved futile as no agreement - Petitioner Milagros G. Lumbuan is the registered owner of Lot 19-A, Block 2844 with Transfer Certificate of Title No. 193264,
Ronquillo was reached. Although no pangkat was located in Gagalangin, Tondo, Manila.
155713 05 May formed, in our mind, there was substantial On February 20, 1995, she leased it to respondent Alfredo A. Ronquillo for a period of three years with a monthly rental of
2006- DE GALA compliance with the law. It is noteworthy P5,000. The parties also agreed that there will be a 10% annual increase in rent for the succeeding two years, i.e., 1996 and
that under the aforequoted provision, the 1997,4 and the leased premises will be used exclusively for the respondent’s fastfood business, unless any other use is given,
confrontation before the Lupon Chairman with the petitioner’s prior written consent.
or the pangkat is sufficient compliance with
the precondition for filing the case in While the respondent at the start operated a fastfood business, he later used the premises as residence without the petitioner’s
court.17 This is true notwithstanding the prior written consent. He also failed to pay the 10% annual increase in rent of P500/month starting 1996 and P1,000/month in
mandate of Section 410(b) of the same law 1997 to the present. Despite repeated verbal and written demands, the respondent refused to pay the arrears and vacate the
that the Barangay Chairman shall leased premises.
constitute a pangkat if he fails in his
mediation efforts. Section 410(b) should be On November 15, 1997, the petitioner referred the matter to the Barangay Chairman’s office but the parties failed to arrive at a
construed together with Section 412, as settlement. The Barangay Chairman then issued a Certificate to File Action.
well as the circumstances obtaining in and
peculiar to the case. On this score, it is
significant that the Barangay Chairman or The petitioner filed against the respondent an action for Unlawful Detainer, docketed as Civil Case No. 157922-CV. It was
Punong Barangay is herself the Chairman raffled to the Metropolitan Trial Court (MeTC) of Manila, Branch 6. On December 15, 1997, the respondent received the
of the Lupon under the Local Government summons and copy of the complaint. On December 24, 1997, he filed his Answer by mail. Before the MeTC could receive the
Code. respondent’s Answer, the petitioner filed a Motion for Summary Judgment dated January 7, 1998.

Acting upon this motion, the MeTC rendered a decision on January 15, 1998, ordering the respondent to vacate and surrender
possession of the leased premises; to pay the petitioner the amount of P46,000 as unpaid rentals with legal interest until fully
paid; and to pay the petitioner P5,000 as attorney’s fees plus cost of the suit.

The respondent then filed a Manifestation calling the attention of the MeTC to the fact that his Answer was filed on time and
praying that the decision be set aside. The MeTC denied the prayer, ruling that the Manifestation was in the nature of a motion
for reconsideration which is a prohibited pleading under the Rules on Summary Procedure.

On appeal the RtC set aside the METC decision directing the parties to go back to the Lupon Chairman or Punong Barangay
for further proceedings and to comply strictly with the condition that should parties fail to reach an amicable settlement, the
entire records of the case will be remanded to the METC for it to decide the case anew. Respondent sought consideration. But
the RTC denied the motion.

Thus, he sought relief from the Court of Appeals through a petition for review.10 On April 12, 2002, the appellate court
promulgated a decision, reversing the decision of the RTC and ordering the dismissal of the ejectment case. The appellate
court ruled that when a complaint is prematurely instituted, as when the mandatory mediation and conciliation in the barangay
level had not been complied with, the court should dismiss the case and not just remand the records to the court of origin so
that the parties may go through the prerequisite proceedings.

The petitioner filed a motion for reconsideration, which was denied by the appellate court. Hence, this present petition.

In the meantime, while this petition was pending before this Court, the parties went through barangay conciliation proceedings
as directed by the RTC of Manila, Branch 38. Again, they failed to arrive at an amicable settlement prompting the RTC to issue
an Order11 remanding the case to the MeTC of Manila, Branch 6, where the proceedings took place anew. On April 25, 2000,
the MeTC rendered a second decision, the dispositive portion of which reads:
WHEREFORE, premises considered, judgment on the merits is hereby rendered for the plaintiff as follows:
“ 1. Ordering defendant and all persons claiming right of possession under him to voluntarily vacate the property located at Lot
19-A Block 2844, Gagalangin, Tondo, Manila and surrender possession thereof to the plaintiff;
2. Ordering defendant to pay to plaintiff the amount of P387,512.00 as actual damages in the form of unpaid rentals and its
agreed increase up to January 2000 and to pay the amount of P6,500.00 a month thereafter until the same is actually vacated;
3. Ordering the defendant to pay to plaintiff the sum of P10,000.00 as and for attorney’s fees plus cost of the suit.
SO ORDERED.”

The respondent appealed the foregoing decision.1avvphil.net The case was raffled to RTC of Manila, Branch 22, and docketed
as Civil Case No. 00-98173. The RTC ruled in favor of the petitioner and dismissed the appeal. The respondent elevated the
case to the Court of Appeals, where it is now pending.

Issue/s:
Whether or not the parties complied with the mandatory mediation and conciliation proceedings in the Barangay level

Held:
YES.
With the parties’ subsequent meeting with the Lupon Chairman or Punong Barangay for further conciliation proceedings, the
procedural defect was cured. Nevertheless, if only to clear any lingering doubt why the Court of Appeals erred in dismissing the
complaint, we shall delve on the issue.
The petitioner alleges that the parties have gone through barangay conciliation proceedings to settle their dispute as shown by
the Certificate to File Action issued by the Lupon/Pangkat Secretary and attested by the Lupon/Pangkat Chairman. The
respondent, on the other hand, contends that whether there was defective compliance or no compliance at all with the required
conciliation, the case should have been dismissed.
The primordial objective of the Katarungang Pambarangay Rules, is to reduce the number of court litigations and prevent the
deterioration of the quality of justice which has been brought about by the indiscriminate filing of cases in the courts. To attain
this objective, Section 412(a) of Republic Act No. 7160 requires the parties to undergo a conciliation process before the Lupon
15

Chairman or the Pangkat as a precondition to filing a complaint in court, thus:


SECTION 412. Conciliation. – (a) Pre-condition to Filing of Complaint in Court. – No complaint, petition, action, or proceeding
involving any matter within the authority of the lupon shall be filed or instituted directly in court or any other government office
for adjudication, unless there has been a confrontation between the parties before the lupon chairman or the pangkat, and that
no conciliation or settlement has been reached as certified by the lupon secretary or pangkat secretary as attested to by the
lupon or pangkat chairman….
Certiorari under Rule 65 is a remedy that Facts: In 1991, private respondents Custodio, Rufo, Tomas and Honorio, all surnamed Balane, filed a complaint for "Recovery
16. Heirs of offers only a limited form of review. Its of Ownership and Possession, Removal of Construction and Damages'' against Bertuldo Hinog (Bertuldo) for a 1,399sqm land
Bertuldo Hinog principal function is to keep an inferior in Malayo Norte, Bohol and that they allowed Bertuldo to use a portion of the property for 10 years and construct thereon a
vs. Melicor tribunal within its jurisdiction. It can be small house of light materials with annual rental of P100 only. Bertuldo alleged ownership by virtue of a Deed of Absolute Sale.
140954 12 April invoked only for an error of jurisdiction or However, during the pendency of trial, Bertuldo died without completing his evidence. In 1998, Atty. Sulpicio Tinampay
with grave abuse of discretion which is withdrew as counsel for Bertuldo as his services were terminated by petitioner Bertuldo Hinog III. Atty. Veronico Petalcorin then
2005- ESPIRITU tantamount to lack or in excess of entered his appearance as new counsel for Bertuldo.
jurisdiction, not to be used for any other
purpose, such as to cure errors in Atty. Petalcorin filed a motion to remove the complaint from the record and nullify all court proceedings on the ground that
proceedings or to correct erroneous private respondents failed to specify in the complaint the amount of damages claimed to pay the correct docket fees; and that
conclusions of law or fact. non-payment of the correct docket fee is jurisdictional. Accordingly, upon payment of deficiency docket fee, private
respondents filed to reinstate the case to which the trial court granted.
The Supreme Court will not entertain direct
resort to it unless the redress desired The petitioners, upon prior leave of court, filed their supplemental pleading but the trial court denied it on the ground that the
cannot be obtained in the appropriate Deed of Absolute Sale is a new matter which was never mentioned in the original answer. Thereafter, petitioners argued that
courts, and exceptional and compelling the trial court having expunged the complaint and nullified all court proceedings, there is no valid case and the complaint
circumstances, such as cases of national should not be admitted for failure to pay the correct docket fees; that there should be no case to be reinstated and no case to
interest and of serious implications, justify proceed as there is no complaint filed.
the availment of the extraordinary remedy
of writ of certiorari, calling for the exercise After the submission of private respondents' opposition and petitioners' rejoinder, the trial court denied petitioners'
of its primary jurisdiction. manifestation/rejoinder. On August 25, 1999, petitioners filed a motion for reconsideration but the same was denied by the trial
court.

On November 24, 1999, petitioners filed before us the present Petition for Certiorari and prohibition. They allege that the public
respondent committed grave abuse of discretion in allowing the case to be reinstated after private respondents paid the docket
fee deficiency since the trial court had earlier removed the complaint from the record.

Issue/s: WON direct recourse to the Supreme Court for Petition for Certiorari and Prohibition is proper

Held: NO. The petitioners erred in directly filing the instant petition for it violates the established policy of strict observance of
the judicial hierarchy of courts. This Court's original jurisdiction to issue writs of certiorari is not exclusive. It is shared by this
Court with Regional Trial Courts and with the Court of Appeals. This concurrence of jurisdiction is not, however, to be taken as
according to parties seeking any of the writs an absolute, unrestrained freedom of choice of the court to which application
therefore will be directed. There is after all a hierarchy of courts. That hierarchy is determinative of the venue of appeals, and
also serves as a general determinant of the appropriate forum for petitions for the extraordinary writs.

A direct invocation of the Supreme Court's original jurisdiction to issue these writs should be allowed only when there are
special and important reasons therefore, clearly and specifically set out in the petition. This is an established policy. It is a
policy necessary to prevent inordinate demands upon the Court's time and attention which are better devoted to those matters
within its exclusive jurisdiction, and to prevent further over-crowding of the Court's docket.

The rationale for this rule is two-fold: (a) it would be an imposition upon the precious time of this Court; and (b) it would cause
an inevitable and resultant delay, intended or otherwise, in the adjudication of cases, which in some instances had to be
remanded or referred to the lower court as the proper forum under the rules of procedure, or as better equipped to resolve the
issues because this Court is not a trier of facts.

In this case, no special and important reason or exceptional and compelling circumstance analogous to any of the above cases
has been adduced by the petitioners to justify direct recourse to this Court. The present petition should have been initially filed
in the Court of Appeals in strict observance of the doctrine on the hierarchy of courts. Failure to do so is sufficient cause for the
dismissal of the petition at bar.

Filing fee must be assessed on the basis of FACTS: 1. The Magaspi case was an action for recovery of ownership and possession of a parcel of land with damages.While
17. Manchester the original complaint the present case is an action for torts and damages and specific performance with prayer for temporary restraining order, etc.
Dev’t Corp. vs.
Court of Appeal 2. In the Magaspi case, the prayer in the complaint seeks not only the annulment of title of the defendant to the property,
the declaration of ownership and delivery of possession thereof to plaintiffs but also asks for the payment of actual moral,
75919 07 May
exemplary damages and attorney's fees arising therefrom in the amounts specified therein. However, in the present case, the
4

1987- prayer is for the issuance of a writ of preliminary prohibitory injunction during the pendency of the action against the
NAKAGAWA defendants' announced forfeiture of the sum of P3 Million paid by the plaintiffs for the property in question, to attach such
property of defendants that maybe sufficient to satisfy any judgment that maybe rendered, and after hearing, to order
defendants to execute a contract of purchase and sale of the subject property and annul defendants' illegal forfeiture of the
money of plaintiff, ordering defendants jointly and severally to pay plaintiff actual, compensatory and exemplary damages as
well as 25% of said amounts as maybe proved during the trial as attorney's fees and declaring the tender of payment of the
purchase price of plaintiff valid and producing the effect of payment and to make the injunction permanent. The amount of
damages sought is not specified in the prayer although the body of the complaint alleges the total amount of over P78 Million
as damages suffered by plaintiff.

3. Upon the filing of the complaint there was an honest difference of opinion as to the nature of the action in the Magaspi case.
The complaint was considered as primarily an action for recovery of ownership and possession of a parcel of land. The
damages stated were treated as merely to the main cause of action. Thus, the docket fee of only P60.00 and P10.00 for the
sheriff's fee were paid.

In the present case there can be no such honest difference of opinion. As maybe gleaned from the allegations of the complaint
as well as the designation thereof, it is both an action for damages and specific performance. The docket fee paid upon filing of
complaint in the amount only of P410.00 by considering the action to be merely one for specific performance where the amount
involved is not capable of pecuniary estimation is obviously erroneous. Although the total amount of damages sought is not
stated in the prayer of the complaint yet it is spelled out in the body of the complaint totalling in the amount of P78,750,000.00
which should be the basis of assessment of the filing fee.

4. When this under-re assessment of the filing fee in this case was brought to the attention of this Court together with
similar other cases an investigation was immediately ordered by the Court. Meanwhile plaintiff through another counsel with
leave of court filed an amended complaint on September 12, 1985 for the inclusion of Philips Wire and Cable Corporation as
co-plaintiff and by emanating any mention of the amount of damages in the body of the complaint. The prayer in the original
complaint was maintained. After this Court issued an order on October 15, 1985 ordering the re- assessment of the docket fee
in the present case and other cases that were investigated, on November 12, 1985 the trial court directed plaintiffs to rectify the
amended complaint by stating the amounts which they are asking for. It was only then that plaintiffs specified the amount of
damages in the body of the complaint in the reduced amount of P10,000,000.00. Still no amount of damages were specified in
7
the prayer. Said amended complaint was admitted.

On the other hand, in the Magaspi case, the trial court ordered the plaintiffs to pay the amount of P3,104.00 as filing fee
covering the damages alleged in the original complaint as it did not consider the damages to be merely an or incidental to the
action for recovery of ownership and possession of real property. An amended complaint was filed by plaintiff with leave of
8

court to include the government of the Republic as defendant and reducing the amount of damages, and attorney's fees prayed
for to P100,000.00. Said amended complaint was also admitted. 9

In the Magaspi case, the action was considered not only one for recovery of ownership but also for damages, so that the filing
fee for the damages should be the basis of assessment. Although the payment of the docketing fee of P60.00 was found to be
insufficient, nevertheless, it was held that since the payment was the result of an "honest difference of opinion as to the correct
amount to be paid as docket fee" the court "had acquired jurisdiction over the case and the proceedings thereafter had were
proper and regular." Hence, as the amended complaint superseded the original complaint, the allegations of damages in the
amended complaint should be the basis of the computation of the filing fee.

In the present case no such honest difference of opinion was possible as the allegations of the complaint, the designation and
the prayer show clearly that it is an action for damages and specific performance. The docketing fee should be assessed by
considering the amount of damages as alleged in the original complaint.

As reiterated in the Magaspi case the rule is well-settled "that a case is deemed filed only upon payment of the docket fee
regardless of the actual date of filing in court. Thus, in the present case the trial court did not acquire jurisdiction over the case
by the payment of only P410.00 as docket fee. Neither can the amendment of the complaint thereby vest jurisdiction upon the
Court. For an legal purposes there is no such original complaint that was duly filed which could be amended. Consequently,
the order admitting the amended complaint and all subsequent proceedings and actions taken by the trial court are null and
void.

ISSUE: WON CA correctly ruled in the present case that the basis of assessment of the docket fee should be the amount of
damages sought in the original complaint and not in the amended complaint.

HELD: YES. The Court cannot close this case without making the observation that it frowns at the practice of counsel who filed
the original complaint in this case of omitting any specification of the amount of damages in the prayer although the amount of
over P78 million is alleged in the body of the complaint. This is clearly intended for no other purpose than to evade the
payment of the correct filing fees if not to mislead the docket clerk in the assessment of the filing fee. This fraudulent practice
was compounded when, even as this Court had taken cognizance of the anomaly and ordered an investigation, petitioner
through another counsel filed an amended complaint, deleting all mention of the amount of damages being asked for in the
body of the complaint. It was only when in obedience to the order of this Court of October 18, 1985, the trial court directed that
the amount of damages be specified in the amended complaint, that petitioners' counsel wrote the damages sought in the
much reduced amount of P10,000,000.00 in the body of the complaint but not in the prayer thereof. The design to avoid
payment of the required docket fee is obvious.

The Court serves warning that it will take drastic action upon a repetition of this unethical practice.

To put a stop to this irregularity, henceforth all complaints, petitions, answers and other similar pleadings should specify the
amount of damages being prayed for not only in the body of the pleading but also in the prayer, and said damages shall be
considered in the assessment of the filing fees in any case. Any pleading that fails to comply with this requirement shall not be
accepted nor admitted, or shall otherwise be expunged from the record.

The Court acquires jurisdiction over any case only upon the payment of the prescribed docket fee. An amendment of the
complaint or similar pleading will not thereby vest jurisdiction in the Court, much less the payment of the docket fee based on
the amounts sought in the amended pleading. The ruling in the Magaspi case in so far as it is inconsistent with this
pronouncement is overturned and reversed.

18. Sun It is not simply the filing of the initiatory Facts:


Insurance Ltd. pleading, but the payment of prescribed Petitioner Sun Insurance Office, Ltd. (SIOL for brevity) filed a complaint with the Regional Trial Court of Makati, Metro Manila
vs. Asuncion docket fee, that vests a TC with jurisdiction for the consignation of a premium refund on a fire insurance policy with a prayer for the judicial declaration of its nullity against
79937-38 13 over the SM or nature of the action. Where private respondent Manuel Uy Po Tiong. Private respondent as declared in default for failure to file the required answer within
the filing of the initiatory pleading is not the reglementary period.
February 1989-
accompanied by payment of the docket fee,
NATO the court may allow payment of the fee On the other hand, private respondent filed a complaint in the Regional Trial Court of Quezon City for the refund of premiums
within a reasonable time but in no case and the issuance of a writ of preliminary attachment, initially against petitioner SIOL, and thereafter including E.B. Philipps and
beyond the applicable prescriptive or D.J. Warby as additional defendants. The complaint sought, among others, the payment of actual, compensatory, moral,
reglementary period. The same rule applies exemplary and liquidated damages, attorney's fees, expenses of litigation and costs of the suit. Although the prayer in the
to permissive counterclaims, third party complaint did not quantify the amount of damages sought said amount may be inferred from the body of the complaint to be
claims and similar pleadings. about Fifty Million Pesos (P50,000,000.00). Only the amount of P210.00 was paid by private respondent as docket fee which
prompted petitioners' counsel to raise his objection. Said objection was disregarded by respondent Judge Jose P. Castro who
was then presiding over said case.

Upon the order of this Court, the records of said case together with twenty-two other cases assigned to different branches of
the Regional Trial Court of Quezon City which were under investigation for underassessment of docket fees were transmitted
to this Court.

The Court en banc issued a Resolution in Administrative Case No. 85-10-8752-RTC directing the judges in said cases to
reassess the docket fees and that in case of deficiency, to order its payment. The Resolution also requires all clerks of court to
issue certificates of re-assessment of docket fees. All litigants were likewise required to specify in their pleadings the amount
sought to be recovered in their complaints.

Reassessment value: amounted to P39,786.00 as docket fee. This was subsequently paid by private respondent. Petitioners
then filed a petition for certiorari with the Court of Appeals questioning the said order of Judge Asuncion. During the pendency
of this petition and in conformity with the said judgment of respondent court, private respondent paid the additional docket fee
of P62,432.90.

The main thrust of the petition is that the Court of Appeals erred in not finding that the lower court did not acquire jurisdiction
over Civil Case No. Q-41177 on the ground of nonpayment of the correct and proper docket fee. Petitioners allege that while it
may be true that private respondent had paid the amount of P182,824.90 as docket fee as herein-above related, and
considering that the total amount sought to be recovered in the amended and supplemental complaint is P64,601,623.70 the
docket fee that should be paid by private respondent is P257,810.49, more or less. Not having paid the same, petitioners
contend that the complaint should be dismissed and all incidents arising therefrom should be annulled.

Issue:

WON the court acquired jurisdiction

Held:

It is not simply the filing of the complaint or appropriate initiatory pleading, but the payment of the prescribed docket fee, that
vests a trial court with jurisdiction over the subject matter or nature of the action. Where the filing of the initiatory pleading is not
accompanied by payment of the docket fee, the court may allow payment of the fee within a reasonable time but in no case
beyond the applicable prescriptive or reglementary period.

In the present case, a more liberal interpretation of the rules is called for considering that, unlike

Manchester, private respondent demonstrated his willingness to abide by the rules by paying the

additional docket fees as required. The promulgation of the decision in Manchester must have had that sobering influence on
private respondent who thus paid the additional docket fee as ordered by the respondent court. It triggered his change of
stance by manifesting his willingness to pay such additional docket fee as may be ordered.

Nevertheless, petitioners contend that the docket fee that was paid is still insufficient considering the total amount of the claim.
This is a matter which the clerk of court of the lower court and/or his duly authorized docket clerk or clerk in-charge should
determine and, thereafter, if any amount is found due, he must require the private respondent to pay the same.

"Awards of claims not specified in the Facts:


19. Ayala Corp. pleading" refers only to "damages arising Private respondents filed against petitioners an action for specific performance with damages in the Regional Trial Court of
vs. Madayag after the filing of the complaint or similar Makati. Petitioners filed a motion to dismiss on the ground that the lower court has not acquired jurisdiction over the case as
88421 30 pleading . . . as to which the additional filing private respondents failed to pay the prescribed docket fee and to specify the amount of exemplary damages both in the body
January 1990- fee therefor shall constitute a lien on the and prayer of the amended and supplemental complaint. The trial court denied the motion in an order dated April 5, 1989. A
judgment." The amount of any claim for motion for reconsideration filed by petitioners was likewise denied in an order dated May 18, 1989. Hence this petition.
PERES damages, therefore, arising on or before The main thrust of the petition is that private respondent paid only the total amount of P l,616.00 as docket fees instead of the
the filing of the complaint or any pleading, amount of P13,061.35 based on the assessed value of the real properties involved as evidenced by its tax declaration. Further,
should be specified. While it is true that the petitioners contend that private respondents failed to specify the amount of exemplary damages sought both in the body and
determination of certain damages as the prayer of the amended and supplemental complaint.
exemplary or corrective damages is left to In Manchester Development Corporation vs. Court of Appeals a similar case involving an action for specific performance with
the sound discretion of the court, it is the damages, this Court held that the docket fee should be assessed by considering the amount of damages as alleged in the
duty of the parties claiming such damages original complaint.
to specify the amount sought on the basis However, the contention of petitioners is that since the action concerns real estate, the assessed value thereof should be
of which the court may make a proper considered in computing the fees pursuant to Section 5, Rule 141 of the Rules of Court. Such rule cannot apply to this case
determination, and for the proper which is an action for specific performance with damages although it is in relation to a transaction involving real estate.
assessment of the appropriate docket fees. Pursuant to Manchester, the amount of the docket fees to be paid should be computed on the basis of the amount of damages
The exception contemplated as to claims stated in the complaint.
not specified or to claims although specified Petitioners also allege that because of the failure of the private respondents to state the amount of exemplary damages being
are left for determination of the court is sought, the complaint must nevertheless be dismissed in accordance to Manchester. The trial court denied the motion stating
limited only to any damages that may arise that the determination of the exemplary damages is within the sound discretion of the court and that it would be unwarrantedly
after the filing of the complaint or similar presumptuous on the part of the private respondents to fix the amount of exemplary damages being prayed for. The trial court
pleading for then it will not be possible for cited the subsequent case of Sun Insurance vs. Judge Asuncion 2 in support of its ruling.
the claimant to specify nor speculate as to The clarificatory and additional rules laid down in Sun Insurance are as follows:
the amount thereof. 1. It is not simply the filing of the complaint or appropriate initiatory pleading, but (also) the payment of the prescribed docket
fee that vests a trial court with jurisdiction over the subject-matter or nature of the action. Where the filing of the initiatory
The amended and supplemental complaint pleading is not accompanied by payment of the docket fee, the court may allow payment of the fee within a reasonable tune
in the present case, therefore, suffers from but in no case beyond the applicable prescriptive or reglementary period.
the material defect in failing to state the 2. The same rule applies to permissive counterclaims, third party claims and similar pleadings, which shall not be considered
amount of exemplary damages prayed for. filed until and unless the filing fee prescribed therefor is paid. The court may also allow payment of said fee within a reasonable
As ruled in Tacay the trial court may either time but also in no case beyond its applicable prescriptive or reglementary period.
order said claim to be expunged from the 3. Where the trial court acquires jurisdiction over a claim by the filing of the appropriate pleading and payment of the prescribed
record as it did not acquire jurisdiction over filing fee but, subsequently, the judgment awards a claim not specified in the pleading, or if specified, the same has been left
the same or on motion, it may allow, within for determination by the court, the additional filing fee therefor shall constitute a lien on the judgment. It shall be the
a reasonable time, the amendment of the responsibility of the Clerk of Court or his duly authorized deputy to enforce said lien and assess and collect the additional fee.
amended and supplemental complaint so Issue/s:
as to state the precise amount of the Whether or not the Trial Court is correct in denying the petition.
exemplary damages sought and require the
payment of the requisite fees therefor Held:
within the relevant prescriptive period. No. Apparently, the trial court misinterpreted paragraph 3 of the above ruling of this Court wherein it is stated that "where the
judgment awards a claim not specified in the pleading, or if specified, the same has been left for the determination of the court,
the additional filing fee therefor shall constitute a lien on the judgment" by considering it to mean that where in the body and
prayer of the complaint there is a prayer, say for exemplary or corrective damages, the amount of which is left to the discretion
of the Court, there is no need to specify the amount being sought, and that any award thereafter shall constitute a lien on the
judgment.

In the latest case Tacay vs. Regional Trial Court of Tagum, this Court had occasion to make the clarification that the phrase
"awards of claims not specified in the pleading" refers only to "damages arising after the filing of the complaint or similar
pleading . . . as to which the additional filing fee therefor shall constitute a lien on the judgment." The amount of any claim for
damages, therefore, arising on or before the filing of the complaint or any pleading, should be specified. While it is true that the
determination of certain damages as exemplary or corrective damages is left to the sound discretion of the court, it is the duty
of the parties claiming such damages to specify the amount sought on the basis of which the court may make a proper
determination, and for the proper assessment of the appropriate docket fees. The exception contemplated as to claims not
specified or to claims although specified are left for determination of the court is limited only to any damages that may arise
after the filing of the complaint or similar pleading for then it will not be possible for the claimant to specify nor speculate as to
the amount thereof.

The amended and supplemental complaint in the present case, therefore, suffers from the material defect in failing to state the
amount of exemplary damages prayed for.

As ruled in Tacay the trial court may either order said claim to be expunged from the record as it did not acquire jurisdiction
over the same or on motion, it may allow, within a reasonable time, the amendment of the amended and supplemental
complaint so as to state the precise amount of the exemplary damages sought and require the payment of the requisite fees
therefor within the relevant prescriptive period.
In case where the party does not Facts:
20. Negros deliberately intend to defraud the court in
Oriental payment of docket fees, and manifests its Campos filed a Complaint for Breach of Contract with Damages against NOPA before the Regional Trial Court (RTC) of
Planters willingness to abide by the rules by paying Negros Occidental, Bacolod City. Campos and NOPA entered into two separate contracts denominated as Molasses Sales
Association, Inc. additional docket fees when required by the Agreement. Campos allegedly paid the consideration of the Molasses Sales Agreement in full, but was only able to receive a
court, the liberal doctrine enunciated in Sun partial delivery of the molasses because of a disagreement as to the quality of the products being delivered.
vs. Hon.
Insurance and not the strict regulations set
Presiding Judge in Manchester will apply.
of Negros More than six years after NOPA filed its Answer, NOPA filed a Motion to Dismiss on the ground of an alleged failure of Campos
to file the correct filing fee. According to NOPA, Campos deliberately concealed in his Complaint the exact amount of actual
24 December damages by opting to estimate the value of the unwithdrawn molasses in order to escape the payment of the proper docket
2008- fees.
PURIFICACION
On 30 June 2006, the RTC issued an Order denying the Motion to Dismiss. NOPA received this Order on 17 July 2006. NOPA
filed a Motion for Reconsideration to which the RTC denied. NOPA then filed a Petition for Certiorari before the Court of
Appeals assailing the Orders of the RTC dated 30 June 2006 and 5 January 2007. On 23 May 2007, the Court of Appeals
dismissed the Petition for Certiorari

Issue/s: Whether or not the CA erred in denying the Petition for Certiorari.

Held:
NOPA seeks in its Petition for Certiorari for the application of this Court's ruling in Manchester Development
Corporation v. Court of Appeals, wherein we ruled that the court acquires jurisdiction over any case only upon payment of
the prescribed docket fee. An amendment of the complaint or similar pleading will not thereby vest jurisdiction in the court,
much less the payment of the docket fee based on the amount sought in the amended pleading.
In denying NOPA's Motion to Dismiss, the RTC cited Sun Insurance Office, Ltd. (SIOL) v. Asuncion, wherein we
modified our ruling in Manchester and decreed that where the initiatory pleading is not accompanied by the payment of the
docket fee, the court may allow payment of the fee within a reasonable period of time, but in no case beyond the applicable
prescriptive or reglementary period. The aforesaid ruling was made on the justification that, unlike in Manchester, the
private respondent in Sun Insurance Office, Ltd. (SIOL) demonstrated his willingness to abide by the rules by paying the
additional docket fees required. NOPA claims that Sun is not applicable to the case at bar, since Campos deliberately
concealed his claim for damages in the prayer.
In United Overseas Bank (formerly Westmont Bank) v. Ros, we discussed how Manchester was not applicable to
said case in view of the lack of deliberate intent to defraud manifested in the latter:
This Court wonders how the petitioner could possibly arrive at the conclusion that the private
respondent was moved by fraudulent intent in omitting the amount of damages claimed in its Second
Amended Complaint, thus placing itself on the same footing as the complainant in Manchester, when it is
clear that the factual milieu of the instant case is far from that of Manchester.
First, the complainant in Manchester paid the docket fee only in the amount of P410.00,
notwithstanding its claim for damages in the amount of P78,750,000.00, while in the present case,
the private respondent paid P42,000.00 as docket fees upon filing of the original complaint.
Second, complainant's counsel in Manchester claimed, in the body of the complaint, damages in
the amount of P78,750.00 but omitted the same in its prayer in order to evade the payment of docket fees.
Such fraud-defining circumstance is absent in the instant petition.
Finally, when the court took cognizance of the issue of non-payment of docket fees in
Manchester, the complainant therein filed an amended complaint, this time omitting all mention of
the amount of damages being claimed in the body of the complaint; and when directed by the court
to specify the amount of damages in such amended complaint, it reduced the same from
P78,750,000.00 to P10,000,000.00, obviously to avoid payment of the required docket fee. Again, this
patent fraudulent scheme is wanting in the case at bar. aAcDSC
This Court is not inclined to adopt the petitioner's piecemeal construction of our rulings in
Manchester and Sun Insurance. Its attempt to strip the said landmark cases of one or two lines and use
them to bolster its arguments and clothe its position with jurisprudential blessing must be struck down by
this Court.
All told, the rule is clear and simple. In case where the party does not deliberately intend to
defraud the court in payment of docket fees, and manifests its willingness to abide by the rules by
paying additional docket fees when required by the court, the liberal doctrine enunciated in Sun
Insurance and not the strict regulations set in Manchester will apply.
In the case at bar, Campos filed an amount of P54,898.50 as docket fee, based on the amounts of P10,000,000.00
representing the value of unwithdrawn molasses, P100,00.00 as storage fee, P200,00.00 as moral damages, P100,000.00
as exemplary damages and P500,000.00 as attorney's fees. The total amount considered in computing the docket fee was
P10,900,000.00. NOPA alleges that Campos deliberately omitted a claim for unrealized profit of P100,000.00 and an
excess amount of storage fee in the amount of P502,875.98 in its prayer and, hence, the amount that should have been
considered in the payment of docket fees is P11,502,875.98. The amount allegedly deliberately omitted was therefore only
P602,875.98 out of P11,502,875.98, or merely 5.2% of said alleged total. Campos's pleadings furthermore evince his
willingness to abide by the rules by paying the additional docket fees when required by the Court. ETaHCD
Since the circumstances of this case clearly show that there was no deliberate intent to defraud the Court in the
payment of docket fees, the case of Sun should be applied, and the Motion to Dismiss by NOPA should be denied.

While the payment of the prescribed docket Facts:


21. Spouses Go fee is a jurisdictional requirement, even its Petitioner Go issued a cashier’s check bearing the words “Final Payment/Quitclaim,” payable to respondent Tong. Tong
vs. Tong 151942 nonpayment at the time of filing does not deposited the check but the same was dishonored because the worse “Final Payment/Quitclaim” had been erased. His request
27 November automatically cause the dismissal of the for the replacement check was not granted, so he filed a complaint for sum of money, damages, and attorney’s fees. During
2003- SABIO case, as long as the fee is paid within the the pendency of the case, the petitioners’ son filed a criminal complaint against private respondent for falsification of the check,
applicable prescriptive or reglementary but the same was dismissed. Subsequently, a supplemental complaint was filed by private respondent praying for an increased
period. amount of damages sought to be recovered due to the damages caused by the filing of a criminal complaint for falsification
against him by the petitioners’ son.

The petitioners deposited to the court the money representing the amount of the check which was later on released to private
respondent Tong by an order issued by public respondent Judge Nabong. Considering the huge amount involved, public
respondent Judge Nabong issued another order allowing private respondent Tong to pay the docket fees on staggered basis.
Petitioners filed for a motion for reconsideration but was denied. Thereafter, petitioners filed a petition for certiorari before the
CA alleging that the respondent judge committed grave abuse of discretion in issuing the orders, but the CA ruled against the
petitioners.

Issue/s:
Whether or not public respondent Judge Nabong committed grave abuse of discretion by allowing private respondent Tong to
pay the docket fee on a staggered basis.

Held:
NO. While the payment of the prescribed docket fee is a jurisdictional requirement, even its nonpayment at the time of filing
does not automatically cause the dismissal of the case, as long as the fee is paid within the applicable prescriptive or
reglementary period; more so when the party involved demonstrates a willingness to abide by the rules prescribing such
payment.

While the cause of action of private respondent Tong was supposed to prescribe in four years, he was allowed to pay; in fact
he paid the docket fee in a year’s time. The SC did not see that period as unreasonable.

For certiorari to lie against public respondent Judge Nabong, the abuse of discretion committed must be grave, as when power
is exercised arbitrarily by reason of passion or personal hostility; and such exercise must be so patent and gross as to amount
to an evasion of positive duty, or to a virtual refusal to perform it or to act in contemplation of law.

Under The Judiciary Reorganization Act of FACTS:


22. Tamano vs. 1980, RTCs have jurisdiction over all On 31 May 1958 Senator Mamintal Abdul Jabar Tamano (Tamano) married private respondent Haja Putri Zorayda A. Tamano
Ortiz 126603 29 actions involving the contract of (PR Zorayda) in civil rites. Their marriage supposedly remained valid and subsisting until his death on 18 May 1994.
June 1998- marriage and marital relations. Personal Prior to his death, Tamano also married petitionerEstrellita J. Tamano (PET. Estrellita) in civil rites in Malabang, Lanao
actions, such as the instant complaint for del Sur.
SOMEROS
declaration of nullity of marriage, may be On 23 November 1994 PR Zorayda joined by her son Adib A. Tamano (Adib) filed a Complaint for Declaration of Nullity
commenced and tried where the plaintiff or of Marriage of Tamano and Estrellita on the ground that it was bigamous. They contended that Tamano and Estrellita
any of the principal plaintiffs resides, or misrepresented themselves as divorced and single, thus making the entries in the marriage contract false and fraudulent.
where the defendant or any of the principal PR Zorayda allegation:
defendants resides, at the election of the “Tamano never divorced Zorayda and that Estrellita was not single when she married Tamano as the decision annulling
plaintiff. her previous marriage with Romeo C. Llave never became final and executory for non-compliance with publication
PD 1083 does not provide for a situation requirements.”
where the parties were married both in civil Estrellita filed a motion to dismiss alleging that the RTC QC was without jurisdiction over the subject and nature of the
and Muslim rites. Consequently, sharia action.
courts are not vested with original and PET. Estellita allegation:
exclusive jurisdiction when it comes to “"only a party to the marriage" could file an action for annulment of marriage against the other spouse, hence, it was only
marriages celebrated under both civil and Tamano who could file an action for annulment of their marriage. since Tamano and Zorayda were both Muslims and married
Muslim laws. Therefore, the RTC is not in Muslim rites the jurisdiction to hear and try the instant case was vested in the shari'acourts pursuant to Art. 155 of the Code
divested of their GENERAL ORIGINAL of Muslim Personal Laws.”
JURISDICTION under BP 129, as The lower court denied the motion and ruled that the instant case was properly cognizable by the RTC QC since Estrellita
amended and Tamano were married in accordance with the Civil Code, and not exclusively in accordance with PD No. 1083 or the
Code of Muslim Personal laws. The motion for reconsideration was likewise denied.
Jurisdiction over the subject matter of a Petitioner filed the instant petition with this Court seeking to set aside the order of the RTC-Br. 89, QC, denying petitioner's
case is determined from the allegations motion to dismiss and reconsideration.
of the complaint as the latter comprises In a Resolution dated 13 December 1995 we referred the case to the CA for consolidation with G.R. No. 118371. Zorayda
a concise statement of the ultimate facts and Adib A. Tamano however filed a motion, which the CA granted, to resolve the Complaint for Declaration of Nullity of
constituting the plaintiff's causes of Marriage ahead of the other consolidated cases.
action. CA ruled that the instant case would fall under the exclusive jurisdiction of shari'a courts only when filed in places where
there are shari'a courts. But in places where there are no shari'a courts, like Quezon City, the instant case could properly
be filed before the RTC.
ISSUE: WON shari'a court and not the Regional Trial Court which has jurisdiction over the subject and nature of the action.
HELD: NO, Under The Judiciary Reorganization Act of 1980, RTCs have jurisdiction over all actions involving the contract of
marriage and marital relations. Personal actions, such as the instant complaint for declaration of nullity of marriage, may be
commenced and tried where the plaintiff or any of the principal plaintiffs resides, or where the defendant or any of the principal
defendants resides, at the election of the plaintiff. There should be no question by now that what determines the nature of an
action and correspondingly the court which has jurisdiction over it are the allegations made by the plaintiff in this case.
In the complaint for declaration of nullity of marriage filed by private respondents herein, it was alleged that Estrellita and
Tamano were married in accordance with the provisions of the Civil Code. Never was it mentioned that Estrellita and Tamano
were married under Muslim laws or PD No. 1083. Interestingly, Estrellita never stated in her Motion to Dismiss that she and
Tamano were married under Muslim laws. That she was in fact married to Tamano under Muslim laws was first mentioned only
in her Motion for Reconsideration.
Nevertheless, the RTC was not divested of jurisdiction to hear and try the instant case despite the allegation in the
Motion for Reconsideration that Estrellita and Tamano were likewise married in Muslim rites. This is because a court's
jurisdiction cannot be made to depend upon defenses set up in the answer, in a motion to dismiss, or in a motion for
reconsideration, but only upon the allegations of the complaint. Jurisdiction over the subject matter of a case is
determined from the allegations of the complaint as the latter comprises a concise statement of the ultimate facts constituting
the plaintiff's causes of action.
Petitioner argues that the shari'a courts have jurisdiction over the instant suit pursuant to Art. 13, Title, II, PD No. 1083, which
provides —
Art. 13. Application. — (1) The provisions of this Title shall apply to marriage and divorce wherein both
parties are Muslims, or wherein only the male party is a Muslim and the marriage is solemnized in
accordance with Muslim law or this Code in any part of the Philippines.
(2) In case of a marriage between a Muslim and a non-Muslim, solemnized not in accordance with Muslim
law or this Code, the Civil Code of the Philippines shall apply.
(3) Subject to the provisions of the preceding paragraphs, the essential requisites and legal impediments to
marriage, divorce, paternity and filiation, guardianship and custody of minors, support and maintenance,
claims for customary dower (mahr), betrothal, breach of contract to marry, solemnization and registration of
marriage and divorce, rights and obligations between husband and wife, parental authority, and the property
relations between husband and wife shall be governed by this Code and other applicable Muslim laws.
As alleged in the complaint, petitioner and Tamano were married in accordance with the Civil Code. Hence, contrary to
the position of petitioner, the Civil Code is applicable in the instant case. Assuming that indeed petitioner and Tamano
were likewise married under Muslim laws, the same would still fall under the general original jurisdiction of the
Regional Trial Courts.
Article 13 of PD No. 1083 does not provide for a situation where the parties were married both in civil and Muslim rites.
Consequently, the shari'a courts are not vested with original and exclusive jurisdiction when it comes to marriages celebrated
under both civil and Muslim laws. Consequently, the RTC are not divested of their general original jurisdiction under Sec. 19,
par. (6) of BP Blg. 129 which provides —
Sec. 19. Jurisdiction in Civil Cases. — Regional Trial Courts shall exercise exclusive original jurisdiction: . . . (6) In all cases not
within the exclusive jurisdiction of any court, tribunal, person or body exercising judicial or quasi-judicial functions . .

Shari'a District Courts have no jurisdiction Facts:


23. Villagracia over real actions where one of the parties is February 15, 1996, Roldan E. Mala purchased a 300-square-meter parcel of land located in Poblacion, Parang, Maguindanao,
vs. Fifth Shari’a not a Muslim. now Shariff Kabunsuan, from one Ceres Cañete. At the time of the purchase, Vivencio B. Villagracia occupied the parcel of
District Court land. When Roldan had the land surveyed, he found out that Villagracia was occupying the property. Roldan failed to settle
188832 23 April with Vivencio at the barangay level, he then filed an action to recover the possession of the parcel of land with respondent Fifth
Shari'a District Court.
2014- TAN
However, despite service of summons, Vivencio failed to file his answer. Thus, Roldan moved that he be allowed to present
evidence ex parte, which motion respondent Fifth Shari'a District Court granted in its order.

On January 13, 2009, Vivencio filed a petition for relief from judgment with prayer for issuance of writ of preliminary injunction.
Vivencio cited Article 155, paragraph (2) of the Code of Muslim Personal Laws of the Philippines and argued that Shari'a
District Courts may only hear civil actions and proceedings if both parties are Muslims. Considering that he is a Christian,
Vivencio argued that respondent Fifth Shari'a District Court had no jurisdiction to take cognizance of Roldan's action for
recovery of possession of a parcel of land.

Issue/s: W/N a Shari'a District Court has jurisdiction over a real action where one of the parties is not a Muslim.

Held:

When it became apparent that Vivencio is not a Muslim, respondent Fifth Shari'a District Court should have motu proprio
dismissed the case. Under Rule 9, Section 1 of the Rules of Court, if it appears that the court has no jurisdiction over the
subject matter of the action based on the pleadings or the evidence on record, the court shall dismiss the claim.

As a matter of law, Shari'a District Courts may only take cognizance of a real action "wherein the parties involved are Muslims."
Considering that one of the parties involved in this case is not a Muslim, respondent Fifth Shari'a District Court had no
jurisdiction to hear, try, and decide the action for recovery of possession of real property.

The judgment against Vivencio is void for respondent Fifth Shari'a District Court's lack of jurisdiction over the subject matter of
the action. That Vivencio raised the issue of lack of jurisdiction over the subject matter only after respondent Fifth Shari'a
District Court had rendered judgment is immaterial. A party may assail the jurisdiction of a court or tribunal over a subject
matter at any stage of the proceedings, even on appeal. The reason is that "jurisdiction is conferred by law, and lack of it
affects the very authority of the court to take cognizance of and to render judgment on the action."
VENUE OF ACTION
CASE TITLE DOCTRINE FACTS-ISSUE/s-HELD

1. Sps. Ang vs. Sps. It is a legal truism that the rules on Facts:
Ang 186993 22 August the venue of personal actions are On September 2, 1992, spouses Alan and Em Ang (respondents) obtained a loan in the amount of Three Hundred
2012- Ubay fixed for the convenience of the Thousand U.S. Dollars (US$300,000.00) from Theodore and Nancy Ang (petitioners). On even date, the respondents
plaintiffs and their witnesses. executed a promissory note in favor of the petitioners wherein they promised to pay the latter the said amount, with interest
Equally settled, however, is the at the rate of ten percent (10%) per annum, upon demand. However, despite repeated demands, the respondents failed to
principle that choosing the venue of pay the petitioners.
an action is not left to a plaintiff s Thus, on August 28, 2006, the petitioners sent the respondents a demand letter asking them to pay their outstanding debt
caprice; the matter is regulated by which, at that time, already amounted to Seven Hundred Nineteen Thousand, Six Hundred Seventy-One U.S. Dollars and
the Rules of Court. Twenty-Three Cents (US$719,671.23), inclusive of the ten percent (10%) annual interest that had accumulated over the
years. Notwithstanding the receipt of the said demand letter, the respondents still failed to settle their loan obligation.
On August 6, 2006, the petitioners, who were then residing in Los Angeles, California, United States of America (USA),
executed their respective Special Powers of Attorney in favor of Attorney Eldrige Marvin B. Aceron (Atty. Aceron) for the
purpose of filing an action in court against the respondents. On September 15, 2006, Atty. Aceron, in behalf of the
petitioners, filed a Complaint for collection of sum of money with the RTC of Quezon City against the respondents.
On November 21, 2006, the respondents moved for the dismissal of the complaint filed by the petitioners on the grounds of
improper venue and prescription. Insisting that the venue of the petitioners action was improperly laid, the respondents
asserted that the complaint against them may only be filed in the court of the place where either they or the petitioners
reside. They averred that they reside in Bacolod City while the petitioners reside in Los Angeles, California, USA. Thus, the
respondents maintain, the filing of the complaint against them in the RTC of Quezon City was improper.
The RTC denied the respondents motion to dismiss. The respondents then filed with the CA a Petition for Certiorari alleging
in the main that, pursuant to Section 2, Rule 4 of the Rules of Court, the petitioners complaint may only be filed in the court
of the place where they or the petitioners reside.
The CA rendered the herein Decision, which annulled and set aside the Orders of the RTC of Quezon City and,
accordingly, directed the dismissal of the complaint filed by the petitioners.
Hence, the instant petition.

Issue: Whether or not the court of appeals committed reversible error of law when it ruled that the complaint must be
dismissed on the ground that venue was not properly laid.

Held:
The petitioners complaint should have been filed in the RTC of Bacolod City, the court of the place where the respondents
reside, and not in RTC of Quezon City.
It is a legal truism that the rules on the venue of personal actions are fixed for the convenience of the plaintiffs and their
witnesses. Equally settled, however, is the principle that choosing the venue of an action is not left to a plaintiff s caprice;
the matter is regulated by the Rules of Court.
The petitioners complaint for collection of sum of money against the respondents is a personal action as it primarily seeks
the enforcement of a contract. The Rules give the plaintiff the option of choosing where to file his complaint. He can file it in
the place (1) where he himself or any of them resides, or (2) where the defendant or any of the defendants resides or may
be found. The plaintiff or the defendant must be residents of the place where the action has been instituted at the time the
action is commenced.
However, if the plaintiff does not reside in the Philippines, the complaint in such case may only be filed in the court of the
place where the defendant resides. In Cohen and Cohen v. Benguet Commercial Co., Ltd., this Court held that there can be
no election as to the venue of the filing of a complaint when the plaintiff has no residence in the Philippines. In such case,
the complaint may only be filed in the court of the place where the defendant resides. Thus:
Section 377 provides that actions of this character "may be brought in any province where the defendant or any necessary
party defendant may reside or be found, or in any province where the plaintiff or one of the plaintiffs resides, at the election
of the plaintiff." The plaintiff in this action has no residence in the Philippine Islands. Only one of the parties to the action
resides here. There can be, therefore, no election by plaintiff as to the place of trial. It must be in the province where the
defendant resides.

Here, the petitioners are residents of Los Angeles, California, USA while the respondents reside in Bacolod City. Applying
the foregoing principles, the petitioners complaint against the respondents may only be filed in the RTC of Bacolod City the
court of the place where the respondents reside. The petitioners, being residents of Los Angeles, California, USA, are not
given the choice as to the venue of the filing of their complaint.
Thus, the CA did not commit any reversible error when it annulled and set aside the orders of the RTC of Quezon City and
consequently dismissed the petitioners complaint against the respondents on the ground of improper venue.
In this regard, it bears stressing that the situs for bringing real and personal civil actions is fixed by the Rules of Court to
attain the greatest convenience possible to the litigants and their witnesses by affording them maximum accessibility to the
courts. And even as the regulation of venue is primarily for the convenience of the plaintiff, as attested by the fact that the
choice of venue is given to him, it should not be construed to unduly deprive a resident defendant of the rights conferred
upon him by the Rules of Court.

2.Marcos-Araneta vs. When there is more than one Petitioner, Irene Marcos-Araneta alleged that Far East Managers and Investors, Inc. (FEMII) and Universal Equity
Court of plaintiff in a personal action case, Corporation (UEC) was organized by Ambassador Benedicto and his business associates pursuant to a contract whereby
Appeals 154096 22 the residences of the principal Benedicto, as trustor, placed in his name and in the name of his associates, as trustees, the shares of stocks of both
August 2008- Yumul parties should be the basis for corporations with the obligation to hold such shares for the benefit of Irene to the extent of 65%. Irene, through her trustee-
determining proper venue. husband, Gregorio Ma. Araneta III, demanded the reconveyance of said 65% stockholdings, but the Benedicto Group
refused to oblige.

Irene then instituted before the RTC two similar complaints for conveyance of shares of stock with prayer for the issuance of
a temporary restraining order. Respondent Francisca Benedicto-Paulino, moved to dismiss the case on the ground that
venue was improperly laid.

Benedicto and Francisca, by way of bolstering their contentions on improper venue, presented the Joint Affidavit that Irene
did not maintain residence in Batac, Ilocos Norte and that she stayed in Makati City with her husband. RTC dismissed her
complaints, stating that these partly constituted “real action”, and that Irene did not actually reside in Ilocos Norte, therefore,
venue was improperly laid.

Irene then interposed a motion for reconsideration and pending its resolution, she filed a Motion to Admit Amended
Complaint, attaching therein a copy of the Amended Complaint which contained additional plaintiffs. All added plaintiffs
were from Ilocos Norte and were Irene’s new trustees. The amended complaint stated the same cause of actions but only
sought after FEMII shares only.
Respondent moved to dismiss the amended complaint, but was denied by the RTC. Respondent submitted an answer to
the amended complaint to evade default but went to CA to question the acceptance of the Amended Complaint.

Issue/s:
WON the CA erred in ruling that petitioner Irene was not a resident of Batac, Ilocos Norte and that none of the principal
parties are residents of Ilocos Norte.

Ruling:

There can be no serious dispute that the real party-in-interest plaintiff is Irene. As self-styled beneficiary of the disputed
trust, she stands to be benefited or entitled to the avails of the present suit. It is undisputed too that the added plaintiffs in
the amended complaint were all from Ilocos Norte. Sec. 2 of Rule 4 indicates quite clearly that when there is more than one
plaintiff in a personal action case, the residences of the principal parties should be the basis for determining proper venue.

Before the RTC in Batac, in Civil Case Nos. 3341-17 and 3342-17, Irene stands undisputedly as the principal plaintiff, the
real party-in-interest. Following Sec. 2 of Rule 4, the subject civil cases ought to be commenced and prosecuted at the
place where Irene resides. Withal, that court was an improper venue for her conveyance action.

3. Universal Robina Trial court may not motu proprio Facts:


Corporation vs. dismiss a complaint on the ground
Lim 154338 5 October of improper venue, thus: The present case arose from a contract of sale between Universal Robina Corporation, petitioner, and Albert Lim,
2007- Abad respondent. Pursuant to the contract, petitioner sold to respondent grocery products in the total amount of P808,059.88.
Rule 9, Section 1 of the 1997 Rules After tendering partial payments, respondent refused to settle his obligation despite petitioner's repeated demands.
of Civil Procedure states that
defenses and objections not Thus, petitioner filed with the RTC, Branch 227, Quezon City, a complaint against respondent for a sum of money.
pleaded either in a motion to
dismiss or in the answer are The trial court issued an Order dismissing the complaint motu proprio on grounds of lack of jurisdiction and improper venue,
deemed waived. The court may thus:
only dismiss an action motu proprio
in case of lack of jurisdiction over
the subject matter, litis pendentia, “There is not even a remote connection by the parties to Quezon City, where this RTC sits, the plaintiff corporation has
res judicata and prescription. T principal office at Pasig City and the defendant is, as provided in the complaint, from Laoag City.”

Petitioner filed a motion for reconsideration together with an amended complaint alleging that the parties agreed that the
proper venue for any dispute relative to the transaction is Quezon City.

The trial court granted the motion and admitted petitioner's amended complaint.

Summons was served upon respondent. For his failure to file an answer seasonably and upon motion of petitioner, the trial
court issued an Order dated declaring him in default and allowing petitioner to present its evidence ex parte.

However, the trial court, still unsure whether venue was properly laid, issued an Order directing petitioner to file a
memorandum of authorities on whether it can file a complaint in Quezon City. Subsequently, the trial court again issued an
Order dismissing the complaint on the ground of improper venue, thus:
“Plaintiff's official place of business is in Pasig whereas the defendant's residence is stated to be in Laoag City — both
stipulated in the Complaint. The filing is based on the stipulation at the back of the delivery receipt that venue shall be in
Quezon City — which is not even stated in the Complaint nor admitted to have been signed by the defendant.”

Petitioner filed a motion for reconsideration but it was denied by the trial court.

Petitioner then filed with the CA a petition for review. But it was dismissed. Petitioner filed a motion for reconsideration but it
was likewise denied by the appellate court.
Hence, this petition.

Issue:
Whether the trial court may dismiss motu proprio petitioner's complaint on the ground of improper venue.

Held: NO.

Sec. 2. Venue of personal actions. — All other actions may be commenced and tried where the plaintiff or any of the
principal plaintiffs resides, or where the defendant or any of the principal defendants resides, or in the case of a non-
resident defendant where he may be found, at the election of the plaintiff.

Sec. 4. When Rule not applicable. — This Rule shall not apply —

(a) In those cases where a specific rule or law provides otherwise; or

(b) Where the parties have validly agreed in writing before the filing of the action on the exclusive venue thereof.

Clearly, in personal actions, the plaintiff may commence an action either in the place of his or her residence or the place
where the defendant resides. However, the parties may agree to a specific venue which could be in a place where neither
of them resides.

Corollarily, Section 1, Rule 9 of the same Rules provides for the instances when the trial court may motu proprio dismiss a
claim.

Implicit from the above provision is that improper venue not impleaded in the motion to dismiss or in the answer is deemed
waived. Thus, a court may not dismiss an action motu proprio on the ground of improper venue as it is not one of the
grounds wherein the court may dismiss an action motu proprio on the basis of the pleadings.

Rudolf Lietz Holdings Inc. v. Registry of Deeds of Parañaque, the Court likewise held that a trial court may not motu proprio
dismiss a complaint on the ground of improper venue, thus:

Rule 9, Section 1 of the 1997 Rules of Civil Procedure states that defenses and objections not pleaded either in a motion to
dismiss or in the answer are deemed waived. The court may only dismiss an action motu proprio in case of lack of
jurisdiction over the subject matter, litis pendentia, res judicata and prescription. Therefore, the trial court in this case erred
when it dismissed the petition motu proprio. It should have waited for a motion to dismiss or a responsive pleading from
respondent, raising the objection or affirmative defense of improper venue, before dismissing the petition.

In the instant case, respondent, despite proper service of summons, failed to file an answer and was thus declared in
default by the trial court. Verily, having been declared in default, he lost his standing in court and his right to adduce
evidence and present his defense, including his right to question the propriety of the venue of the action.

4. United Overseas Bank FACTS: Respondent Rosemoor Mining and Development Corporation (Rosemoor), a Philippine mining corporation with
vs. Rosemoore Mining Venue of real actions affecting offices at Quezon City, applied for and was granted by petitioner Westmont Bank (Bank) a credit facility in the total amount
and properties found in different of ₱80 million consisting of ₱50,000,000.00 as long term loan and ₱30,000,000.00 as revolving credit line.
Development 159669 12 provinces is determined by the To secure the credit facility, a lone real estate mortgage agreement was executed by Rosemoor and Dr. Lourdes Pascual
singularity or plurality of the (Dr. Pascual), Rosemoor’s president, as mortgagors in favor of the Bank as mortgagee in the City of Manila. The
March 2007- De Gala
transactions involving said parcels agreement, however, covered six (6) parcels of land located in San Miguel, Bulacan (Bulacan properties), all
of land. Where said parcels are the registered under the name of Rosemoor, and two (2) parcels of land situated in Gapan, Nueva Ecija (Nueva Ecija
object of one and the same properties), owned and registered under the name of Dr. Pascual.
transaction, the venue is in the Rosemoor subsequently opened with the Bank four (4) irrevocable Letters of Credit (LCs) totaling US$1,943,508.11. To
court of any of the provinces cover payments by the Bank under the LCs, Rosemoor proceeded to draw against its credit facility and thereafter executed
wherein a parcel of land is situated. promissory notes amounting collectively to ₱49,862,682.50. Two (2) other promissory notes were also executed by
In the present case, there is only Rosemoor in the amounts of ₱10,000,000.00 and ₱3,500,000.00, respectively, to be drawn from its revolving credit line.
one proceeding sought to be Rosemoor defaulted in the payment of its various drawings under the LCs and promissory notes. In view of the
nullified and that is the extra-judicial default, the Bank caused the extra-judicial foreclosure of the Nueva Ecija properties on 22 May 1998 and the
mortgage foreclosure sale. And Bulacan properties on 10 August 1998. The Bank was the highest bidder on both occasions.
there is only one initial transaction On 8 October 1999, the Bank caused the annotation of the Notarial Certificate of Sale covering the Nueva Ecija properties
which served as the basis of the on the certificates of title concerned. Later, on 16 March 2001, the Notarial Certificate of Sale covering the Bulacan
foreclosure sale and that is the properties was annotated on the certificates of title of said properties.
mortgage contract. Indeed, The foregoing facts led to Rosemoor’s filing of separate complaints against the Bank, one before the Regional Trial
Rosemoor, through Dr. Pascual, Court of Manila (Manila RTC) and the other before the Regional Trial Court of Malolos, Bulacan (Malolos RTC).
executed a lone mortgage contract The Manila Case (G.R. No. 163521)
where it undertook to "mortgage the On 5 August 1998, Rosemoor and Dr. Pascual filed a Complaint, originally captioned as one for "Damages, Accounting and
land/real property situated in Release of Balance of Loan and Machinery and for Injunction" before the Manila RTC. Impleaded as defendants were the
Bulacan and Nueva Ecija," with the Bank and Notary Public Jose Sineneng, whose office was used to foreclose the mortgage. The complaint was twice
list of mortgaged properties amended, the caption eventually reflecting an action for "Accounting, Specific Performance and Damages." Through the
annexed thereto revealing six (6) amendments, Pascual was dropped as a plaintiff while several officers of the Bank were included as defendants.
properties in Bulacan and two (2) The Bank moved for the dismissal of the original and amended complaints on the ground that the venue had been
properties in Nueva Ecija subject of improperly laid.The motion was denied by the trial court.
the mortgage. The Bank challenged the Manila RTC’S denial of the Bank’s second motion to dismiss before the CA through a petition for
certiorari.
The Court of Appeals dismissed the petition.
The Bank filed Petition for Review on Certiorari in GR NO. 163521 before the Supreme Court.
The Bank argues that the CA erred in holding that no forum shopping attended the actions brought by Rosemoor.
MALOLOS RTC
After the complaint with the Manila RTC had been lodged, on 11 March 2002, Rosemoor and Dr. Pascual filed another
action against the Bank, this time before the Malolos RTC. Impleaded together with the Bank as respondent was the
Register of Deeds for the Province of Bulacan in the Petition for Injunction with Damages, with Urgent Prayer for Temporary
Restraining Order and/or Preliminary Injunction.
In the Malolos case, Rosemoor and Dr. Pascual alleged that the redemption period for the Bulacan properties would expire
on 16 March 2002. They claimed that the threatened consolidation of titles by the Bank is illegal, stressing that the
foreclosure of the real estate mortgage by the Bank was fraudulent and without basis, as the Bank had made them sign
two blank forms of Real Estate Mortgage and several promissory notes also in blank forms. It appeared later, according
to Rosemoor and Dr. Pascual, that the two Real Estate Mortgage blank forms were made as security for two loans, one for
₱80 million and the other for ₱48 million, when the total approved loan was only for ₱80 million. The Bank later released
only the amount of ₱10 million out of the ₱30 million revolving credit line, to the prejudice of Rosemoor, they added.
As it did before the Manila RTC, the Bank filed a motion to dismiss on 26 March 2002 on the ground that Rosemoor had
engaged in forum-shopping, adverting to the pending Manila case.The Bank further alleged that Dr. Pascual has no
cause of action since the properties registered in her name are located in Nueva Ecija.
The Malolos RTC denied the motion to dismiss in an Order dated 13 May 2002
COURT OF APPEALS: DENIED both motions.
ISSUE: Whether or not the action to invalidate the foreclosure sale was properly laid with the Malolos RTC even as regard
to the Nueva Ecija properties?
HELD: YES.
The rule on venue of real actions is provided in Section 1, Rule 4 of the 1997 Rules of Civil Procedure, which reads in part:
Section 1. Venue of Real Actions. Actions affecting title to or possession of real property, or interest therein, shall be
commenced and tried in the proper court which has jurisdiction over the area wherein the real property involved, or a
portion thereof, is situated.
The venue of the action for the nullification of the foreclosure sale is properly laid with the Malolos RTC although two of the
properties together with the Bulacan properties are situated in Nueva Ecija. Following the above-quoted provision of the
Rules of Court, the venue of real actions affecting properties found in different provinces is determined by the
singularity or plurality of the transactions involving said parcels of land. Where said parcels are the object of one
and the same transaction, the venue is in the court of any of the provinces wherein a parcel of land is situated.
Ironically, the Bank itself correctly summarized the applicable jurisprudential rule in one of the pleadings before the Court.
Yet the Bank itself has provided the noose on which it would be hung. Resorting to deliberate misrepresentation, the Bank
stated in the same pleading that "the Bulacan and Nueva Ecija [p]roperties were not the subject of one single real estate
mortgage contract."
In the present case, there is only one proceeding sought to be nullified and that is the extra-judicial mortgage
foreclosure sale. And there is only one initial transaction which served as the basis of the foreclosure sale and that
is the mortgage contract. Indeed, Rosemoor, through Dr. Pascual, executed a lone mortgage contract where it
undertook to "mortgage the land/real property situated in Bulacan and Nueva Ecija," with the list of mortgaged
properties annexed thereto revealing six (6) properties in Bulacan and two (2) properties in Nueva Ecija subject of
the mortgage.
This apparent deliberate misrepresentation cannot simply pass without action. The real estate mortgage form supplied to
Rosemoor is the Bank’s standard pre-printed form. Yet the Bank perpetrated the misrepresentation. Blame must be placed
on its doorstep. But as the Bank’s pleading was obviously prepared by its counsel, the latter should also share the blame. A
lawyer shall not do any falsehood, nor consent to the doing of any in court; nor shall he mislead, or allow the Court to be
misled by any artifice.Both the Bank’s president and counsel should be made to explain why they should not be sanctioned
for contempt of court.
WHEREFORE, considering the foregoing, the Decision of the Court of Appeals in G.R. 163521 dated 26 February 2004
and in G.R No. 159669 dated 20 June 2003 are AFFIRMED. Costs against petitioner. Petitioner, United Overseas Bank,
Phils. and its counsel, Siguion Reyna Montecillo & Ongsiako Law Offices, are given ten (10) days from notice to EXPLAIN
why they should not be held in contempt of court for making a misrepresentation before the Court as adverted to in this
Decision.

5. San Luis vs. San In the application of venue statutes Facts: The instant case involves the settlement of the estate of Felicisimo San Luis (Felicisimo), who was the former
Luis 133743 6 February and rules – Section 1, Rule 73 of governor of the Province of Laguna. During his lifetime, Felicisimo contracted three marriages. First with Virginia Sulit in
2007-Espiritu the Revised Rules of Court is of 1942 out of which were born six children but in 1963 Virginia predeceased Felicisimo. In 1968 or 5 years later, Felicisimo
such nature – residence rather married Merry Lee Corwin, with whom he had a son, Tobias. However, in 1971, Merry Lee, an American citizen, filed a
than domicile is the significant Complaint for Divorce which was granted by the Family Court of the Hawaii USA in 1973.
factor. In 1974, Felicisimo married respondent Felicidad San Luis (Felicidad) in California, U.S.A. He had no children with
respondent but lived with her for 18 years from the time of their marriage up to his death in 1992. Thereafter, respondent
The subject petition for letters of Felicidad sought the dissolution of their conjugal partnership assets and the settlement of Felicisimo’s estate. She filed a
administration was validly filed in petition for letters of administration before the RTC of Makati City.
the RTC which has territorial On February 4, 1994, petitioner Rodolfo San Luis, one of the children of Felicisimo by his first marriage, filed a motion to
jurisdiction over Alabang, dismiss on the grounds of improper venue and failure to state a cause of action. Rodolfo claimed that the petition for letters
Muntinlupa. During the filing of of administration should have been filed in the Province of Laguna because this was Felicisimo’s place of residence prior to
petition, Muntinlupa was still a his death.
municipality and the branches of On February 15, 1994, Linda invoked the same grounds and joined her brother Rodolfo in seeking the dismissal of the
the RTC of the National Capital petition. On February 28, 1994, the trial court issued an Order denying the two motions to dismiss. On September 12, 1995,
Judicial Region which had territorial the trial court dismissed the petition for letters of administration. Respondent appealed to the Court of Appeals which
jurisdiction over Muntinlupa were reversed and set aside the orders of the trial court in its assailed Decision dated February 4, 1998.The petition was
then seated in Makati City as per DENIED. The Decision of the Court of Appeals reinstating and affirming the February 28, 1994 Order of the Regional Trial
Supreme Court Administrative Court is AFFIRMED. The records of the case was REMANDED to the trial court for further proceedings.
Order No. 3. Thus, the subject Issue: Whether or not the letter of administration was filed in the proper venue
petition was validly filed before the Ruling: YES. Under Section 1, Rule 73 of the Rules of Court, the petition for letters of administration of the estate of
RTC of Makati City. Felicisimo should be filed in the Regional Trial Court of the province "in which he resides at the time of his death." In this
popular sense, the term means merely residence, that is, personal residence, not legal residence or domicile. No particular
length of time of residence is required though; however, the residence must be more than temporary.
Needless to say, there is a distinction between "residence" for purposes of election laws and "residence" for purposes of
fixing the venue of actions. In election cases, "residence" and "domicile" are treated as synonymous terms, that is, the fixed
permanent residence to which when absent, one has the intention of returning. However, for purposes of fixing venue under
the Rules of Court, the "residence" of a person is his personal, actual or physical habitation, or actual residence or place of
abode, which may not necessarily be his legal residence or domicile provided he resides therein with continuity and
consistency.
In the instant case, while petitioners established that Felicisimo was domiciled in Sta. Cruz, Laguna, respondent proved that
he also maintained a residence in Alabang, Muntinlupa from 1982 up to the time of his death. Respondent submitted in
evidence the Deed of Absolute Sale in 1983 showing that the deceased purchased the aforesaid property and other
documents such as billing statements, membership with Ayala Country club, letters from his children, etc.
From the foregoing, we find that Felicisimo was a resident of Alabang, Muntinlupa for purposes of fixing the venue
of the settlement of his estate. Consequently, the subject petition for letters of administration was validly filed in the RTC
which has territorial jurisdiction over Alabang, Muntinlupa. The subject petition was filed on December 17, 1993. At
that time, Muntinlupa was still a municipality and the branches of the Regional Trial Court of the National Capital Judicial
Region which had territorial jurisdiction over Muntinlupa were then seated in Makati City as per Supreme Court
Administrative Order No. 3. Thus, the subject petition was validly filed before the RTC of Makati City.
6. Spouses Lantin vs. The mere stipulation on the venue FACTS: Petitioners Renato and Angelina Lantin took several peso and dollar loans from respondent Planters Development
Lantion 160053 28 of an action, however, is not Bank and executed several real estate mortgages and promissory notes to cover the loans. They defaulted on the
August 2006- Nakagawa enough to preclude parties from payments so respondent bank foreclosed the mortgaged lots. The foreclosed properties, in partial satisfaction of petitioners’
bringing a case in other venues. debt, were sold at a public auction where the respondent bank was the winning bidder. On November 8, 2003, petitioners
The parties must be able to show filed against Planters Development Bank and its officers Elizabeth Umali, Alice Perce and Jelen Mosca (private
that such stipulation is exclusive. In respondents), a Complaint for Declaration of Nullity and/or Annulment of Sale and/or Mortgage, Reconveyance, Discharge
the absence of qualifying or of Mortgage, Accounting, Permanent Injunction, and Damages with the RTC of Lipa City, Batangas. Petitioners alleged that
restrictive words, the stipulation only their peso loans were covered by the mortgages and that these had already been fully paid, hence, the mortgages
should be deemed as merely an should have been discharged. They challenged the validity of the foreclosure on the alleged non-payment of their dollar
agreement on an additional forum, loans as the mortgages did not cover those loans.
not as limiting venue to the
specified place. Private respondents moved to dismiss the complaint on the ground of improper venue since the loan agreements restricted
the venue of any suit in Metro Manila.

On May 15, 2003, the respondent judge dismissed the case for improper venue.

Petitioners sought reconsideration. They argued that the trial court in effect prejudged the validity of the loan documents
because the trial court based its dismissal on a venue stipulation provided in the agreement. The motion for reconsideration
was denied and the lower court held that the previous order did not touch upon the validity of the loan documents but
merely ruled on the procedural issue of venue.

ISSUE: WON respondent judge committed grave abuse of discretion when she dismissed the case for improper venue.

HELD: NO. Petitioners contend that, since the validity of the loan documents were squarely put in issue, necessarily this
meant also that the validity of the venue stipulation also was at issue. Moreover, according to the petitioners, the venue
stipulation in the loan documents is not an exclusive venue stipulation under Section 4(b) of Rule 4 of the 1997 Rules of
Civil Procedure. The venue in the loan agreement was not specified with particularity. Besides, petitioners posit, the rule on
4

venue of action was established for the convenience of the plaintiff, herein petitioners. Further, petitioners also contend that
since the complaint involves several causes of action which did not arise solely from or connected with the loan documents,
the cited venue stipulation should not be made to apply.

Private respondents counter that, in their complaint, petitioners did not assail the loan documents, and the issue of validity
was merely petitioners’ afterthought to avoid being bound by the venue stipulation. They also aver that the venue stipulation
was not contrary to the doctrine in Unimasters, which requires that a venue stipulation employ categorical and suitably
5

limiting language to the effect that the parties agree that the venue of actions between them should be laid only and
exclusively at a definite place. According to private respondents, the language of the stipulation is clearly exclusive.

At the outset, we must make clear that under Section 4 (b) of Rule 4 of the 1997 Rules of Civil Procedure, the general rules
on venue of actions shall not apply where the parties, before the filing of the action, have validly agreed in writing on an
exclusive venue. The mere stipulation on the venue of an action, however, is not enough to preclude parties from bringing a
case in other venues. The parties must be able to show that such stipulation is exclusive. In the absence of qualifying or
6

restrictive words, the stipulation should be deemed as merely an agreement on an additional forum, not as limiting venue to
the specified place.The pertinent provisions of the several real estate mortgages and promissory notes executed by the
petitioner respectively read as follows:

18. In the event of suit arising out of or in connection with this mortgage and/or the promissory note/s secured by this
mortgage, the parties hereto agree to bring their causes of auction (sic) exclusively in the proper court of Makati, Metro
Manila or at such other venue chosen by the Mortgagee, the Mortgagor waiving for this purpose any other venue.

I/We further submit that the venue of any legal action arising out of this note shall exclusively be at the proper court of
Metropolitan Manila, Philippines or any other venue chosen by the BANK, waiving for this purpose any other venue
provided by the Rules of Court.

Clearly, the words "exclusively" and "waiving for this purpose any other venue" are restrictive and used advisedly to meet
the requirements.

Petitioners claim that effecting the exclusive venue stipulation would be tantamount to a prejudgment on the validity of the
loan documents. We note however that in their complaint, petitioners never assailed the validity of the mortgage contracts
securing their peso loans. They only assailed the terms and coverage of the mortgage contracts. What petitioners claimed
is that their peso loans had already been paid thus the mortgages should be discharged, and that the mortgage contracts
did not include their dollar loans. In our view, since the issues of whether the mortgages should be properly discharged and
whether these also cover the dollar loans, arose out of the said loan documents, the stipulation on venue is also applicable
thereto.

Considering all the circumstances in this controversy, we find that the respondent judge did not commit grave abuse of
discretion, as the questioned orders were evidently in accord with law and jurisprudence.

7. Saludo vs. American x x x [T]he term domicile is not


Express International, exactly synonymous in legal FACTS: Aniceto G. Saludo, Jr. filed a complaint for damages against the American Express International, Inc. (AMEX)
Inc. 159507 19 April contemplation with the and/or its officers Ian T. Fish, Vice-President and Country Manager, and Dominic Mascrinas, Head of Operations, with the
2006- Nato term residence, for it is [an] RTC of Maasin City, Southern Leyte. The case was raffled to Branch 25 of the said court.
established principle in Conflict of The complaint alleged, inter alia, that plaintiff (herein petitioner Saludo) "is a Filipino citizen, of legal age, and a member of
Laws that domicile refers to the the House of Representatives and a resident of Ichon, Macrohon, Southern Leyte, Philippines." On the other hand,
relatively more permanent abode of defendant (herein respondent AMEX, Inc.) "is a corporation doing business in the Philippines and engaged in providing
a person while residence applies to credit and other credit facilities and allied services with office address at 4th floor, ACE Building, Rada Street, Legaspi
a temporary stay of a person in a Village, Makati City." The other defendants (herein respondents Fish and Mascrinas) are officers of respondent AMEX, and
given place. In fact, this distinction may be served with summons and other court processes at their office address.
is very well emphasized in those The complaint's cause of action stemmed from the alleged wrongful dishonor of petitioner Saludo's AMEX credit card and
cases where the Domiciliary Theory the supplementary card issued to his daughter. The first dishonor happened when petitioner Saludo's daughter used her
must necessarily supplant the supplementary credit card to pay her purchases in the United States some time in April 2000. The second dishonor
Nationality Theory in cases occurred when petitioner Saludo used his principal credit card to pay his account at the Hotel Okawa in Tokyo, Japan while
involving stateless persons. he was there with other delegates from the Philippines to attend the Congressional Recognition in honor of Mr. Hiroshi
Tanaka.
xxx The dishonor of these AMEX credit cards were allegedly unjustified as they resulted from respondents' unilateral act of
suspending petitioner Saludo's account for his failure to pay its balance covering the period of March 2000. Petitioner
“There is a difference between Saludo denied having received the corresponding statement of account. Further, he was allegedly wrongfully charged for
domicile and residence. Residence late payment in June 2000. Subsequently, his credit card and its supplementary cards were canceled by respondents on
is used to indicate a place of July 20, 2000.
abode, whether permanent or Petitioner Saludo claimed that he suffered great inconvenience, wounded feelings, mental anguish, embarrassment,
temporary; domicile denotes a fixed humiliation and besmirched political and professional standing as a result of respondents' acts which were committed in
permanent residence to which gross and evident bad faith, and in wanton, reckless and oppressive manner. He thus prayed that respondents be adjudged
when absent, one has the intention to pay him, jointly and severally, actual, moral and exemplary damages, and attorney's fees.
of returning. A man may have a PROCEDURAL HISTORY: Before the Court is the Petition for Review on Certiorari filed by Aniceto G. Saludo, Jr. seeking
residence in one place and a to reverse and set aside the Decision dated May 22, 2003 of the Court of Appeals in CA-G.R. SP No. 69553. The assailed
1

domicile in another. Residence is decision directed the Regional Trial Court (RTC) of Maasin City, Southern Leyte, Branch 25 thereof, to vacate and set aside
not domicile, but domicile is its Orders dated September 10, 2001 and January 2, 2002 in Civil Case No. R-3172, and enjoined the presiding
residence coupled with intention to judge thereof from conducting further proceedings in said case, except to dismiss the complaint filed therewith on ground of
2

remain for an unlimited time. A man improper venue. The petition also seeks to reverse and set aside the appellate court's Resolution dated August 14, 2003
can have but one domicile for one denying the motion for reconsideration of the assailed decision.
and the same purpose at any time, ISSUES:
but he may have numerous places 1. Whether the term resides' means the actual residence or domicile of the decedent at the time of his death?
of residence. His place of residence 2. whether the appellate court committed reversible error in holding that venue was improperly laid in the court a quo in Civil
generally is his place of domicile, Case No. R-3172 because not one of the parties, including petitioner Saludo, as plaintiff therein, was a resident of Southern
but is not by any means, Leyte at the time of filing of the complaint
necessarily so since no length of 3. Whether Saludo's motive in filing his complaint with the court a quo was only to vex and unduly inconvenience
residence without intention of respondents or even to wield influence in the outcome of the case, petitioner Saludo being a powerful and influential figure
remaining will constitute in the said province could be regarded as a "specie of forum shopping"
domicile.”[6] HOLDING: The term "resides" as employed in the rule on venue on personal actions filed with the courts of first instance
means the place of abode, whether permanent or temporary, of the plaintiff or the defendant, as distinguished from
"domicile" which denotes a fixed permanent residence to which, when absent, one has the intention of returning.
Residence in civil law is a material fact, referring to the physical presence of a person in a place. A person can have two or
more residences, such as a country residence and a city residence. (Quetulio v. Ruiz, S.C. Off. Gaz. 156, Commentaries
and Jurisprudence in Civil Law, Vol. 1, page 211, Tolentino). Residence is acquired by living in a place; on the other hand,
domicile can exist without actually living in the place. The important thing for domicile is that, once residence has been
established in one place, there be an intention to stay there permanently, even if residence is also established in some
other place.
Thus, if a person lives with his family habitually in Quezon City, he would have his domicile in Quezon City. If he also has a
house for vacation purposes in the City of Baguio, and another house in connection with his business in the City of Manila,
he would have residence in all three places (Tolentino, Commentaries and Jurisprudence on Civil Law, Vol. 1, Page 212,
1990 Edition) so that one[']s legal residence or domicile can also be his actual, personal or physical residence or habitation
or place of abode if he stays there with intention to stay there permanently.
'resides' should be viewed or understood in its popular sense, meaning, the personal, actual or physical habitation of a
person, actual residence or place of abode. It signifies physical presence in a place and actual stay thereat. In this popular
sense, the term means merely residence, that is, personal residence, not legal residence or domicile. Residence simply
requires bodily presence as an inhabitant in a given place, while domicile requires bodily presence in that place and also an
intention to make it one's domicile. No particular length of time of residence is required though; however, the residence
must be more than temporary
In the instant case, since plaintiff has a house in Makati City for the purpose of exercising his profession or doing business
and also a house in Ichon, Macrohon, Southern Leyte, for doing business and/or for election or political purposes where he
also lives or stays physically, personally and actually then he can have residences in these two places. Because it would
then be preposterous to acknowledge and recognize plaintiff Aniceto G. Saludo, Jr. as congressman of Southern Leyte
without also recognizing him as actually, personally and physically residing thereat, when such residence is required by
law.28

DISPOSITION: WHEREFORE, premises considered, the petition is GRANTED. The Decision dated May 22, 2003 and
Resolution dated August 14, 2003 of the Court of Appeals in CA-G.R. SP No. 69553 are REVERSED and SET ASIDE. The
Orders dated September 10, 2001 and January 2, 2002 of the Regional Trial Court of Maasin City, Southern Leyte, Branch
25 thereof, in Civil Case No. R-3172 are REINSTATED.

8. Hyatt Elevators vs. Residence is the permanent home - FACTS:


Goldstar - the place to which, whenever Petitioner [herein Respondent] Goldstar Elevator Philippines, Inc. (GOLDSTAR for brevity) is a domestic corporation
Elevators 161026 24 absent for business or pleasure, primarily engaged in the business of marketing, distributing, selling, importing, installing, and maintaining elevators and
October 2005- Perez one intends to return. escalators, with address at 6th Floor, Jacinta II Building, 64 EDSA, Guadalupe, Makati City.

Residence is vital when dealing Private respondent [herein petitioner] Hyatt Elevators and Escalators Company (HYATT for brevity) is a domestic
with venue. A corporation, however, corporation similarly engaged in the business of selling, installing and maintaining/servicing elevators, escalators and
has no residence in the same parking equipment, with address at the 6th Floor, Dao I Condominium, Salcedo St., Legaspi Village, Makati, as stated in its
sense in which this term is applied Articles of Incorporation.
to a natural person.
A corporation is in a metaphysical HYATT filed a Complaint for unfair trade practices and damages against LG Industrial Systems Co. Ltd. (LGISC) and LG
sense a resident of the place where International Corporation (LGIC), alleging among others, that:
its principal office is located as in 1988, it was appointed by LGIC and LGISC as the exclusive distributor of LG elevators and escalators in the Philippines
stated in the articles of under a Distributorship Agreement; LGISC, in the latter part of 1996, made a proposal to change the exclusive
incorporation. Even before this distributorship agency to that of a joint venture partnership;
ruling, it has already been while it looked forward to a healthy and fruitful negotiation for a joint venture, however, the various meetings it had with
established that the residence of a LGISC and LGIC, through the latter's representatives, were conducted in utmost bad faith and with malevolent intentions;
corporation is the place where its in the middle of the negotiations, in order to put pressures upon it, LGISC and LGIC terminated the Exclusive
principal office is established. Distributorship Agreement; as a consequence, [HYATT] suffered actual damages, representing loss of earnings and
For purposes of venue, the term business opportunities, P20,000,000.00 as damages for its reputation and goodwill, P1,000,000.00 as and by way of
'residence is synonymous with exemplary damages, and P500,000.00 as and by way of attorney's fees.
'domicile.
LGISC and LGIC filed a Motion to Dismiss which was denied by the [trial] court .LGISC and LGIC filed an Answer with
Compulsory Counterclaim ex abundante cautela. Thereafter, they filed a 'Motion for Reconsideration and to Expunge
Complaint which was denied.
HYATT filed a motion for leave of court to amend the complaint, alleging that subsequent to the filing of the complaint, it
learned that LGISC transferred all its organization, assets and goodwill, as a consequence of a joint venture agreement with
Otis Elevator Company of the USA, to LG Otis Elevator Company (LG OTIS, for brevity). Thus, LGISC was to be
substituted or changed to LG OTIS, its successor-in-interest. Likewise, the motion averred that GOLDSTAR was being
utilized by LG OTIS and LGIC in perpetrating their unlawful and unjustified acts against HYATT. Consequently, in order to
afford complete relief, GOLDSTAR was to be additionally impleaded as a party-defendant. Hence, in the Amended
Complaint, HYATT impleaded GOLDSTAR as a party-defendant, and all references to LGISC were correspondingly
replaced with LG OTIS.

On January 8, 2001, the [trial] court admitted the Amended Complaint. LG OTIS (LGISC) and LGIC filed a motion for
reconsideration thereto but was similarly rebuffed.

GOLDSTAR filed a Motion to Dismiss the amended complaint, raising the following grounds: (1) the venue was improperly
laid, as neither HYATT nor defendants reside in Mandaluyong City, where the original case was filed; and (2) failure to state
a cause of action against [respondent], since the amended complaint fails to allege with certainty what specific ultimate acts
x x x Goldstar performed in violation of Hyatt's rights. In the Order dated May 27, 2002, which is the main subject of the
present petition, the [trial] court denied the motion to dismiss, ratiocinating that the factual and legal arguments raised by
the movants-defendants, hare substantially the same issues posed by the then defendant LG Industrial System Co.
particularly the matter dealing [with] the issues of improper venue, failure to state cause of action as well as this court's lack
of jurisdiction. Under the circumstances obtaining, the court resolves to rule that the complaint sufficiently states a cause of
action and that the venue is properly laid. It is significant to note that in the amended complaint, the same allegations are
adopted as in the original complaint with respect to the Goldstar Philippines to enable this court to adjudicate a complete
determination or settlement of the claim subject of the action it appearing preliminarily as sufficiently alleged in the plaintiff's
pleading that said Goldstar Elevator Philippines Inc., is being managed and operated by the same Korean officers of
defendants LG-OTIS Elevator Company and LG International Corporation.
GOLDSTAR filed a motion for reconsideration thereto. On June 18, 2002, without waiving the grounds it raised in its motion
to dismiss, [it] also filed an 'Answer Ad Cautelam. On October 1, 2002, [its] motion for reconsideration was denied.
From the aforesaid Order denying Goldstar's motion for reconsideration, it filed the petition for certiorari [before the CA]
alleging grave abuse of discretion amounting to lack or excess of jurisdiction on the part of the [trial] court in issuing the
assailed Orders dated May 27, 2002 and October 1, 2002.
The CA ruled that the trial court had committed palpable error amounting to grave abuse of discretion when the latter
denied respondent's Motion to Dismiss. The appellate court held that the venue was clearly improper, because none of the
litigants 'resided in Mandaluyong City, where the case was filed.

According to the appellate court, since Makati was the principal place of business of both respondent and petitioner, as
stated in the latter's Articles of Incorporation, that place was controlling for purposes of determining the proper venue. The
fact that petitioner had abandoned its principal office in Makati years prior to the filing of the original case did not affect the
venue where personal actions could be commenced and tried.

Issue:
Whether or not the Court of Appeals, in reversing the ruling of the Regional Trial Court, erred as a matter of law and
jurisprudence, as well as committed grave abuse of discretion, in holding that in the light of the peculiar facts of this case,
venue was improper

Held:
No.
Sec. 2. Venue of personal actions. ' All other actions may be commenced and tried where the plaintiff or any of the principal
plaintiff resides, or where the defendant or any of the principal defendant resides, or in the case of a non-resident defendant
where he may be found, at the election of the plaintiff.
Since both parties to this case are corporations, there is a need to clarify the meaning of 'residence. The law recognizes two
types of persons: (1) natural and (2) juridical. Corporations come under the latter in accordance with Article 44(3) of the Civil
Code.
Residence is the permanent home -- the place to which, whenever absent for business or pleasure, one intends to return.
Residence is vital when dealing with venue. A corporation, however, has no residence in the same sense in which this term
is applied to a natural person. This is precisely the reason why the Court in Young Auto Supply Company v. Court of
Appeals ruled that 'for practical purposes, a corporation is in a metaphysical sense a resident of the place where its
principal office is located as stated in the articles of incorporation. Even before this ruling, it has already been established
that the residence of a corporation is the place where its principal office is established.
This Court has also definitively ruled that for purposes of venue, the term 'residence is synonymous with 'domicile.
Correspondingly, the Civil Code provides:
Art. 51. When the law creating or recognizing them, or any other provision does not fix the domicile of juridical persons, the
same shall be understood to be the place where their legal representation is established or where they exercise their
principal functions.

It now becomes apparent that the residence or domicile of a juridical person is fixed by 'the law creating or recognizing it.
Under Section 14(3) of the Corporation Code, the place where the principal office of the corporation is to be located is one
of the required contents of the articles of incorporation, which shall be filed with the Securities and Exchange Commission
(SEC).
In the present case, there is no question as to the residence of respondent. What needs to be examined is that of petitioner.
Admittedly, the latter's principal place of business is Makati, as indicated in its Articles of Incorporation. Since the principal
place of business of a corporation determines its residence or domicile, then the place indicated in petitioner's articles of
incorporation becomes controlling in determining the venue for this case.

urisprudence has, however, settled that the place where the principal office of a corporation is located, as stated in the
articles, indeed establishes its residence. This ruling is important in determining the venue of an action by or against a
corporation, as in the present case.

Without merit is the argument of petitioner that the locality stated in its Articles of Incorporation does not conclusively
indicate that its principal office is still in the same place. We agree with the appellate court in its observation that the
requirement to state in the articles the place where the principal office of the corporation is to be located 'is not a
meaningless requirement. That proviso would be rendered nugatory if corporations were to be allowed to simply disregard
what is expressly stated in their Articles of Incorporation
.
Inconclusive are the bare allegations of petitioner that it had closed its Makati office and relocated to Mandaluyong City, and
that respondent was well aware of those circumstances. Assuming arguendo that they transacted business with each other
in the Mandaluyong office of petitioner, the fact remains that, in law, the latter's residence was still the place indicated in its
Articles of Incorporation. Further unacceptable is its faulty reasoning that the ground for the CA's dismissal of its Complaint
was its failure to amend its Articles of Incorporation so as to reflect its actual and present principal office. The appellate
court was clear enough in its ruling that the Complaint was dismissed because the venue had been improperly laid, not
because of the failure of petitioner to amend the latter's Articles of Incorporation.
Indeed, it is a legal truism that the rules on the venue of personal actions are fixed for the convenience of the plaintiffs and
their witnesses. Equally settled, however, is the principle that choosing the venue of an action is not left to a plaintiff's
caprice; the matter is regulated by the Rules of Court. Allowing petitioner's arguments may lead precisely to what this Court
was trying to avoid in Young Auto Supply Company v. CA: the creation of confusion and untold inconveniences to party
litigants.
The rules on venue, like the other procedural rules, are designed to insure a just and orderly administration of justice or the
impartial and evenhanded determination of every action and proceeding. Obviously, this objective will not be attained if the
plaintiff is given unrestricted freedom to choose the court where he may file his complaint or petition.

The choice of venue should not be left to the plaintiff's whim or caprice. He may be impelled by some ulterior motivation in
choosing to file a case in a particular court even if not allowed by the rules on venue.

9. Diaz vs. VENUE OF LIBEL CASE WHERE Facts:


Adiong 106847 5 March OFFENDED PARTY IS AN PUBLIC The Mindanao Kris, a newspaper of general circulation in Cotabato City, published in its front page the news article
OFFICIAL. — From the provision of captioned "6-Point Complaint Filed vs. Macumbal," and in its Publisher's Notes the editorial, "Toll of Corruption," which
1993- Purificacion Article 360, third paragraph of the exposed alleged anomalies by key officials in the Regional Office of the Department of Environment and Natural
Revised Penal Code as amended Resources.
by R.A. 4363, it is clear that an The public officers alluded to were the private respondents Sultan Macorro L. Macumbal, Sultan Linog M. Indol, Atty.
offended party who is at the same Macabangkit M. Lanto and Atty. Mohamadali Abedin, who then instituted separate criminal and civil complaints arising from
time a public official can only the libel.
institute an action arising from libel
in two (2) venues: the place where In the interim, the civil complaint for damages was raffled to Branch 10 of the Regional Trial Court in Marawi City.
he holds office, and the place
where the alleged libelous articles The City Prosecutor’s Office dismissed the criminal case.
were printed and first published.
Petitioner Diaz moved for the dismissal of the action for damages on the ground that the trial court did not have jurisdiction
IMPROPER VENUE; MUST BE over the subject matter. He vehemently argued that the complaint should have been filed in Cotabato City and not in
RAISED IN A NOTION TO Marawi City.
DISMISS PRIOR TO A
RESPONSIVE PLEADING. —
Unless and until the defendant Petitioner Diaz contends that the civil action for damages could not be rightfully filed in Marawi City as none of the private
objects to the venue in a motion to respondents, who are all public officers, held office in Marawi City; neither were the alleged libelous news items published
dismiss prior to a responsive in that city. Consequently, it is petitioner's view that the Regional Trial Court in Marawi City has no jurisdiction to
pleading, the venue cannot truly be entertain the civil action for damages.
said to have been improperly laid
since, for all practical intents and Issues:
purposes, the venue though 1. Whether or not the RTC of Marawi has jurisdiction over the case - NO
technically wrong may yet be 2. Whether or not the venue was improperly laid. - YES
considered acceptable to the
parties for whose convenience Held:
the rules on venue had been 1. No. The RTC of Marawi has no jurisdiction over the case. Not one of the respondents then held office in Marawi
devised. City. Moreover, it is admitted that the libelous articles were published and printed in Cotabato City. Thus,
respondents were limited in their choice of venue for their action for damages only to Cotabato City where
WAIVED IN CASE AT BAR BY Macumbal, Lanto and Abedin had their office and Lanao del Norte where Indol worked. Marawi City is not among
FILING ANSWER. — Well-settled is those where venue can be laid.
the rule that improper venue may
be waived and such waiver may According to the third paragraph of Art. 360 of the Revised Penal Code, as amended by R.A. No. 4363, it is clear
occur by laches. Sec. 1 of Rule 16 that an offended party who is at the same time a public official can only institute an action arising from libel in two
provides that objections to improper (2) venues: the place where he holds office, and the place where the alleged libelous articles were printed and first
venue must be made in a motion to published.
dismiss before any responsive
pleading is filed. Responsive
pleadings are those which seek
affirmative relief and set up 2. Venue was improperly laid. However, unless and until the defendant objects to the venue in a motion to dismiss
defenses. Consequently, having prior to a responsive pleading, the venue cannot truly be said to have been improperly laid since, for all practical intents
already submitted his person to the and purposes, the venue though technically wrong may yet be considered acceptable to the parties for whose
jurisdiction of the trial court, convenience the rules on venue had been devised.
petitioner may no longer object to Petitioner Diaz should have timely challenged the venue laid in Marawi City in a motion to dismiss, pursuant to
the venue which, although Sec. 4, Rule 4, of the Rules of Court. Unfortunately, petitioner had already submitted himself to the
mandatory in the instant case, is
nevertheless waivable. As such, jurisdiction of the trial court when he filed his Answer to the Complaint with Counterclaim.
improper venue must be
His motion to dismiss was therefore belatedly filed and could no longer deprive the trial court of jurisdiction to
seasonably raised, otherwise, it hear and decide the instant civil action for damages. Well-settled is the rule that improper venue may be waived
may be deemed waived.
and such waiver may occur by laches.

Indeed, the laying of venue is Withal, objections to venue in civil actions arising from libel may be waived; it does not, after all, involve a
procedural rather than substantive, question of jurisdiction. Indeed, the laying of venue is procedural rather than substantive, relating as it does
relating as it does to jurisdiction of to jurisdiction of the court over the person rather than the subject matter. Venue relates to trial and not to
the court over the person rather jurisdiction.
than the subject matter. Venue Finally, Sec. 1 of Rule 16 provides that objections to improper venue must be made in a motion to dismiss before
relates to trial and not to any responsive pleading is filed. Responsive pleadings are those which seek affirmative relief and set up
jurisdiction. defenses. Consequently, having already submitted his person to the jurisdiction of the trial court,
petitioner may no longer object to the venue which, although mandatory in the instant case, is nevertheless
waivable. As such, improper venue must be seasonably raised, otherwise, it may be deemed waived.

10. Nocum vs. Jurisdiction is not the same as FACTS: Respondent Lucio Tan filed a complaint against petitioner reporter Nocum, Capt. Umali, ALPAP, and Inquirer with
Tan 145022 23 venue. the RTC of Makati, seeking moral and exemplary damages for the alleged malicious and defamatory imputations contained
September 2005- Sabio 1. Jurisdiction is the authority in a news article.
to hear and determine a
case; venue is the place Petitioner filed their answers and alleged that, among others, venue was improperly laid. It appeared that the complaint
where the case is to be failed to state the residence of the complainant at the time of the alleged commission of the offense and the place where
heard or tried. the libelous article was printed and first published.
2. Jurisdiction is a matter of
substantive law; venue is of The RTC of Makati dismissed Tan’s complaint without prejudice on the ground of improper venue.
procedural law.
3. Jurisdiction establishes a
relation between the court Nocum, et al. argued that since the original complaint only contained the office address of Tan and not his actual residence
and the subject matter; or the place where the allegedly offending news reports were printed and first published, the original complaint, by reason
venue establishes a of the deficiencies in its allegations, failed to confer jurisdiction on the lower court.
relation between plaintiff
and defendant, or petitioner ISSUE: Whether or not petitioner’s contentions are correct.
and respondent.
4. Jurisdiction is fixed by law HELD: NO. Nocum, et al. are confusing jurisdiction with venue.
and cannot be conferred by 5. Jurisdiction is the authority to hear and determine a case; venue is the place where the case is to be heard or tried.
parties; venue may be 6. Jurisdiction is a matter of substantive law; venue is of procedural law.
conferred by the act or 7. Jurisdiction establishes a relation between the court and the subject matter; venue establishes a relation between
agreement of the parties. plaintiff and defendant, or petitioner and respondent.
8. Jurisdiction is fixed by law and cannot be conferred by parties; venue may be conferred by the act or agreement of
the parties.

In the case at bar, the allegations in the amended complaint that the article and caricature were printed and published first
in Makati referred only to the question of venue and not jurisdiction. These allegations would neither confer jurisdiction on
the RTC nor would Tan’s failure to include the same in the original complaint divest the lower court of its jurisdiction over
the case.
Tan’s failure to allege these allegations gave the lower court the power, upon motion by a party, to dismiss the complaint on
the ground that venue was improperly laid.

11. Phil. Banking Corp. It is settled in this jurisdiction that FACTS: Petitioner Philippine Banking Corporation is a commercial banking corporation with principal office at Makati,
vs. Tensuan 106920 10 the parties, by written agreement, Manila. Petitioner Bank instituted a complaint for collection of a sum of money, with a prayer for preliminary attachment, at
December 1993- may change or transfer the venue RTC Makati. It appears from the allegations of the Bank's complaint that respondent Circle Financial Co., through its
Someros of an action from one province to representatives, obtained several loans aggregating P1million from petitioner. Respondent Circle, for value received,
another. (Section 3 Rule 4 of the delivered to petitioner Bank four (4) promissory notes, each of which contained the stipulation that jurisdiction of the courts
Rules of Court) We have many of Valenzuela will take cognizance if any legal action arise.
times sustained the validity and
enforceability of contractual As security for the re-payment by respondent Circle of the sums loaned by petitioner Bank, eight (8) individuals, were
stipulations relating to venue. In the impleaded as defendants in the complaint and executed a Continuing Surety Agreement and undertook to pay jointly and
enforcement of the parties' severally respondent Circle's obligations. Only five (5) out of the eight (8) individual obligors are respondents in present
stipulations concerning venue, it is, case, namely: Domingo Santiago, Hilario Lopez, Avelino Deato, Benjamin P. Santiago and Socorro Gomez.
of course, the tenor of their Respondent Circle and the 8 other individual failed to pay its obligations under the promissory notes.
agreement which is of critical
relevance. The relevant task, in Petitioner moved for issuance of a writ of preliminary attachment, alleging that respondent Circle had become insolvent
other words, is determining the and had been placed under receivership by the Central Bank. The trial judge granted the motion and issued a writ of
intent of the parties as manifested preliminary attachment.
in the words employed by them A motion to dismiss was filed by the respondents and averred that the venue of the action was improperly laid since an
and, where such words are less agreement had fixed the venue of actions arising from the promissory notes in Valenzuela, Metro Manila, only.
than clear, in other recognized Respondents called the trial court's attention to the stipulation contained in the promissory note.
indicators of the will of the RTC: dismissed the petition and granted respondent’s motion. It is with merit consistent with Sec. 13, Rule 14 of the Rules
contracting parties. of Court as well as in line with the doctrinal rule in Bautista vs. Hon. Juan de Borja, et al. (18 SCRA 474) that the proper
The Court noted, that no one of the venue for an action is that stipulated in a document 'in case of any litigation herefrom or in connection herewith' upon a
private respondents has claimed to rationale that had the parties intended to reserve the right to choose venue under Section 2 (b), Rule 4 of the Rules of
have been put to undue hardship or Court, such reservation should have been reflected in the document as against the rationale in Polytrade Corporation vs.
inconvenience as a result of the Blanco (30 SCRA 187) which should allow choice of venue where an actionable document does not set forth qualifying or
institution of the action in Makati. restrictive words in point,
Venue relates to the trial and Petitioner moved for reconsideration, but denied.
touches more upon the Hence, this Petition.
convenience of the parties rather ISSUE: WON there was an error in the judgement of TENSUAN
than upon the substance or merits
of the case. HELD: YES, We consider that the Petition is meritorious.
It is settled in this jurisdiction that the parties, by written agreement, may change or transfer the venue of an action from one
province to another. We have many times sustained the validity and enforceability of contractual stipulations relating to
venue. In the enforcement of the parties' stipulations concerning venue, it is, of course, the tenor of their agreement which
is of critical relevance. The relevant task, in other words, is determining the intent of the parties as manifested in the words
employed by them and, where such words are less than clear, in other recognized indicators of the will of the contracting
parties.
A careful reading of the terms of the stipulation — "I/We hereby expressly submit to the jurisdiction of the courts of
Valenzuela any legal action which may arise out of this promissory note" — shows that the stipulation does not require the
laying of venue in Valenzuela exclusively or mandatorily. The plain or ordinary import of the stipulation is the authorizing of,
or permission to bring, suit in Valenzuela; there is not the slightest indication of an intent to bar suit in other competent
courts.
Permissive stipulations like the one here considered have invariably received judicial approval and we have declared that
either of the parties is authorized to lay venue of an action in the court named in the stipulation. The stipulation here does
not purport to deprive either party of its right to elect, or option to have resort to, another competent court as expressly
permitted by Section 2(b) of Rule 4 of the Rules of Court, should such party choose to initiate a suit. The stipulation here
merely operated to confer or confirm a right upon a party to elect recourse to the courts of Valenzuela or, alternatively, to go
before any of the tribunals envisaged by the rules on venue, i.e., the courts of Makati, Quezon City and Bulacan.
In Polytrade Corporation v. Blanco,
". . . the venue in personal actions is fixed for the convenience of the plaintiff and his witnesses and to
promote the ends of justice.
While the parties have agreed to submit their dispute to the jurisdiction of the Manila courts, there is
nothing in the language used . . . which clearly shows that the intention of the parties was to limit the
venue of the action to the City of Manila only. Such agreements should be construed reasonably and
should not be applied in such a manner that it would work more to the inconvenience of the parties
without promoting the ends of justice."
We note, finally, that no one of the private respondents has claimed to have been put to undue hardship or inconvenience
as a result of the institution of the action in Makati. Venue relates to the trial and touches more upon the convenience of the
parties rather than upon the substance or merits of the case.

12. Anita Mangila vs. AGREED VENUE THAT IS NOT FACTS:


Court of MADE EXCLUSIVE DOES NOT
Appeals 125027 12 PRECLUDE OTHER VENUES. — Respondent Loreta Guina, president and gen. manager of Airswift International filed a case for collection of sum of money
August 2002- Tan The Rules of Court provide that amounting to P109, 376.95 against petitioner Anita Mangila exporter of seafoods under the name Seafoods Products with
parties to an action may agree in Pasay RTC. Summons, however, was not personally served on Mangila (but rather to a household help in San Fernando,
writing on the venue on which an Pampanga) on the allegations that she had transferred residence and later found by the sheriff, that she had already left the
action should be brought. However, country (believed was done with the intention to defraud). Without recourse to service by publication, Guina filed a motion
a mere stipulation on the venue of for a writ of preliminary attachment and the trial court granted the same.
an action is not enough to preclude
parties from bringing a case in Petitioner filed a Motion to Dismiss the Complaint on the ground of improper venue. Private respondent's invoice for the
other venues. The parties must be freight forwarding service stipulates that "if court litigation becomes necessary to enforce collection . . . the agreed venue for
able to show that such such action is Makati, Metro Manila."

stipulation is exclusive. Thus, Pasay RTC: Ordered the petitioner to pay respondent P109,376.95 plus 18 percent interest per annum, 25 percent
absent words that show the parties' attorney's fees and costs of suit.CA: Affirmed RTC decision of RTC.
intention to restrict the filing of a
suit in a particular place, courts will ISSUE: W/N there was improper venue
allow the �ling of a case in any
venue, as long as jurisdictional HELD: On the issue of venue, wherein the case was filed in a place other than that stipulated by the parties, the Court ruled
requirements are followed. Venue the same is proper where the agreement does not preclude other venues. However, bringing the case to the venue where
stipulations in a contract, while the sole proprietorship business is found, there is improper venue. Sole proprietorship business has no separate juridical
considered valid and enforceable, personality that could file a suit in court.
do not as a rule supersede the
general rule set forth in Rule 4 of We resolve to dismiss the case on the ground of improper venue but not for the reason stated by petitioner. The Rules of
the Revised Rules of Court. In the Court provide that parties to an action may agree in writing on the venue on which an action should be brought.
absence of qualifying or restrictive However, a mere stipulation on the venue of an action is not enough to preclude parties from bringing a case in
words, they should be considered other venues. The parties must be able to show that such stipulation is exclusive. Thus, absent words that show the
merely as an agreement on parties' intention to restrict the filing of a suit in a particular place, courts will allow the filing of a case in any venue, as long
additional forum, not as limiting as jurisdictional requirements are followed. Venue stipulations in a contract, while considered valid and enforceable,
venue to the specified place. do not as a rule supersede the general rule set forth in Rule 4 of the Revised Rules of Court. In the absence of
qualifying or restrictive words, they should be considered merely as an agreement on additional forum, not as limiting venue
to the specified place.

WHEREFORE, the petition is GRANTED on the grounds of improper venue and invalidity of the service of the writ of
attachment. The decision of the Court of Appeals and the order of respondent judge denying the motion to dismiss are
REVERSED and SET ASIDE. Civil Case No. 5875 is hereby dismissed without prejudice to re�ling it in the proper venue.
The attached properties of petitioner are ordered returned to her immediately.

13. Unimasters the invariable construction placed


Conglomeration Inc. vs. on venue stipulations is that they do Facts:
Court of not negate but merely complement
Appeals 119657 7 or add to the standards of Rule 4. Private respondent Kubota Agri-Machinery Philippines, Inc. (KUBOTA) and petitioner Unimasters Conglomeration, Inc.
February 1997- Tañada In other words, unless the parties (UNIMASTERS) entered into a Dealership Agreement for Sales and Services of the former’s products in Samar and Leyte
make very clear, by employing Provinces. The contract contains, among others, that “All suits arising out of this Agreement shall be filed with/in the proper
categorical and suitably limiting Courts of Quezon City”. Some five years after the agreement was made, UNIMASTERS filed an action in the RTC of
language, that they wish the venue Tacloban against KUBOTA, a certain Reynaldo Go, and METROBANK for damages from breach of contract and injunction
of actions between them to be laid with prayer for TRO. KUBOTA filed two motions: a motion to dismiss on the ground of improper venue which was set at a
only and exclusively at a definite later date, and a motion to transfer the injunction hearing which was also set at a later date. Notwithstanding that its motion
place, and to disregard the to transfer the injunction hearing had been granted, the RTC went with the hearing prior to the reset date. Thereafter, during
prescriptions of Rule 4, agreements the injunction hearing, the RTC authorized the issuance of a preliminary injunction and denied KUBOTA’s motion to
on venue are not to be regarded as dismiss. Both orders were challenged by KUBOTA in a special civil action of certiorari and prohibition with the CA alleging,
mandatory or restrictive, but merely among others, that RTC had no jurisdiction over the matter as venue was improperly laid. The CA ruled in favor of
permissive, or complementary of KUBOTA, holding that the stipulation on venue limited the venue of all suits arising thereunder only and exclusively to the
said rule. The fact that in their proper courts of Quezon City, thus, the venue was improperly laid. It also held that the participation of KUBOTA’s counsel at
agreement the parties specify only the hearing on the injunction incident did not operate as a waiver to object on the venue. Hence, UNIMASTER filed the
one of the venues mentioned in instant petition before the SC.
Rule 4, or fix a place for their
actions different from those Issue:
specified by said rule, does not
suffice to characterize the Whether or not venue was properly laid.
agreement as a restrictive one.
There must be accompanying Held: Yes.
language clearly and categorically
expressing their purpose and Rule 4 of the Rules of Court expressly allows parties, by stipulation, to waive the legal venue, and such waiver is valid and
design that actions between them effective being merely a personal privilege, which is not contrary to public policy or prejudicial to third persons. Written
be litigated only at the place named stipulations as to venue may be restrictive in the sense that the suit may be filed only in the place agreed upon, or merely
by them, regardless of the general permissive in that the parties may file their suit not only in the place agreed upon but also in the places fixed by law (Rule 4,
precepts of Rule 4; and any doubt specifically). What is essential is the ascertainment of the intention of the parties respecting the matter. Since convenience
or uncertainty as to the parties' is the rationale for the rules on venue, venue stipulations should be deemed permissive merely, and that interpretation
intentions must be resolved against should be adopted which most serves the parties' convenience. In other words, stipulations designating venues other than
giving their agreement a restrictive those assigned by Rule 4 should be interpreted as designed to make it more convenient for the parties to institute actions
or mandatory aspect. arising from or in relation to their agreements; that is to say, as simply adding to or expanding the venues indicated in said
Rule 4. Because restrictive stipulations are in derogation of this policy, the language of the parties must be so clear and
categorical as to leave no doubt of their intention to limit the place or places, or to fix places other than those indicated in
Rule 4, for their actions. Pursuant to an analysis of previous jurisprudence on venue, the invariable construction placed on
venue stipulations is that they do not negate but merely complement or add to the standards of Rule 4. In other words,
unless the parties make very clear, by employing categorical and suitably limiting language, that they wish the venue of
actions between them to be laid only and exclusively at a definite place, and to disregard the prescriptions of Rule 4,
agreements on venue are not to be regarded as mandatory or restrictive, but merely permissive, or complementary of said
rule. The fact that in their agreement the parties specify only one of the venues mentioned in Rule 4, or fix a place for their
actions different from those specified by said rule, does not suffice to characterize the agreement as a restrictive one. There
must be accompanying language clearly and categorically expressing their purpose and design that actions between them
be litigated only at the place named by them, regardless of the general precepts of Rule 4; and any doubt or uncertainty as
to the parties' intentions must be resolved against giving their agreement a restrictive or mandatory aspect. In this case, the
question is whether the stipulation by the parties on venue of their actions had the effect of effectively eliminating Tacloban
(UNIMASTERS’ residence) as an optional venue and limiting litigation between the parties only and exclusively to Quezon
City. The SC answered this in the negative. Absent additional words and expressions definitely and unmistakably denoting
the parties' desire and intention that actions between them should be ventilated only at the place selected by them, Quezon
City — or other contractual provisions clearly evincing the same desire and intention — the stipulation should be construed,
not as confining suits between the parties only to that one place, Quezon City, but as allowing suits either in Quezon City or
Tacloban City, at the option of the UNIMASTERS.
CASE TITLE DOCTRINE FACTS-ISSUE/s-HELD

1. Remelita The statutory requirements of Miralles, respondent, filed with the RTC a complaint for a sum of money against Robinson. Respondent alleged
ROBINSON v. substituted service must be followed that the petitioner borrowed from here $20,054.00. Service of summons could not be served due to the refusal of
Celita strictly, faithfully, and fully and any the guard of Alabang under instruction to let anyone enter the defendant’s house if she was not present. A copy
Miralles G.R. No. substituted service other than that of the summons was then left with the guard who refused to affix his signature therein. Eventually, Robinson was
163584, Dec 12, authorized by the Rules is declared in default and judgment was rendered against her ordering her to pay the sum of money. A copy of the
2006- YUMUL considered ineffective. However, we decision was then sent to her registered mail at her new address.
frown upon an overly strict
application of the Rules. It is the Petitioner filed with the trial court a petition for relief from the judgment by default. She claimed that summons was
spirit, rather than the letter of the improperly served upon her, thus, the trial court never acquired jurisdiction over her and that all its proceedings
procedural rules, that governs. are void. Stating Section 7, Rule 14; that the guard was “not related to her or staying at her residence” nor was he
authorized to receive summons for the residents in the village. Hence, invalidating the summons.

Issue/s:
WON a substituted service of summons upon petitioner was valid.

Held:
YES. We have ruled that the statutory requirements of substituted service must be followed strictly, faithfully, and
fully and any substituted service other than that authorized by the Rules is considered ineffective. However, we
frown upon an overly strict application of the Rules. It is the spirit, rather than the letter of the procedural rules, that
governs.

Robinson’s instruction to the guard to not let anyone enter when she is absent made it impossible for the sheriff to
properly serve her the summons. Considering her strict instruction to the guard, she now bears the consequences
of her actions.

GENERAL PROVISION

2.Spouses Yu vs. It has been held in a long line of Facts:


Pacleb 172172 24 cases that an action for specific Respondent Baltazar Pacleb and his late first wife (Angelita Chan) are the owners of a parcel of land located in
February 2009- performance is an action in Barrio Langcaan, Dasmarinas Cavite (Langcaan Property). The Langcaan Property was subject to three
TAÑADA personam. Hence, as an action in documents purporting to transfer its ownership: 1) a Deed of Absolute Sale between Sps. Pacleb and Angelita
personam, Civil Case No. 741-93 is Chan, and Rebecca Del Rosario; 2) a Deed of Absolute Sale between Rebecca del Rosario and Ruperto Javier;
binding only upon the parties and 3) a Contract to Sell between Javier and spouses Yu (petitioners). All these sales were not registered.
properly impleaded therein and duly Petitioners filed with the RTC a complaint for specific performance (Civil Case No. 741-93) against Javier, to
heard or given an opportunity to be compel him to deliver to them the ownership, possession, title to the Langcaan Property, alleging that Javier
heard. represented to them that the property was not tenanted, but later found out that it was occupied by Ramon Pacleb
(Ramon). They demanded for the cancellation of their agreement and for the return of their initial payment.
Thereafter, the RTC rendered a decision in favor of petitioners. Respondent then filed a complaint for annulment
of deed of sale and other documents arising from it, alleging that his and his first wife’s signatures appearing on
the first deed of sale were forged, and hence, the deed was spurious. The trial court dismissed the case.
Petitioners filed an action for forcible entry against respondent with the MTC, which the MTC ruled in favor of the
petitioners, affirmed by the RTC, and reversed by the CA. Respondent then filed an action for removal of cloud
from title, alleging that the sale made between him and his first wife and Rebecca del Rosario was not known to
them and was invalid. The trial court ruled against the respondent; the CA reversed the decision. In the instant
case filed with the SC, petitioners argued, among other things, that the RTC decision in Civil Case No. 741-93 is
conclusive and binding upon the respondent even if the latter was not a party since it involved the question of
possession and ownership of real property, and is thus not merely an action in personam but an action quasi in
rem.

Issue:
Whether or not Civil Case No. 741-93 is an action quasi in rem.

Held: No.
Domangas vs. Jensen provided that: “A proceeding in personam is a proceeding to enforce personal rights and
obligations brought against the person and is based on the jurisdiction of the person, although it may involve his
right to, or the exercise of ownership of, specific property, or seek to compel him to control or dispose of it in
accordance with the mandate of the court. Here, it is said to be one which has for its object a judgment against the
person. It has been held that an action in personam is a proceeding to enforce personal rights or obligations; such
action is brought against the person…. In an action quasi in rem, an individual is named as defendant and the
purpose of the proceeding is to subject his interests therein to the obligation or loan burdening the property. These
actions deal with the status, ownership, or liability of a particular property but which are intended to operate on
these questions only as between the particular parties to the proceedings and not to ascertain or cut off the rights
or interests of all possible claimants. The judgments herein are binding only upon the parties who joined the
action.”
Civil Case No. 741-93 is an action for specific performance. Its object is to compel Javier to accept full payment of
the purchase price and to execute a deed of absolute sale over the subject property in their favor. The obligations
of Javier under the contract to sell attach to him only, and do not burden the subject property. It has been held in a
long line of cases that an action for specific performance is an action in personam. Hence, as an action in
personam, Civil Case No. 741-93 is binding only upon the parties properly impleaded therein and duly heard or
given an opportunity to be heard.

3. Domagas vs. An action in personam is said to be one On February 19, 1999, Domagas filed a complaint for forcible entry against Jensen before the MTC of Calasiao,
Jensen 158407 17 which has for its object a judgment Pangasinan. Domagas alleged that she was the owner of a parcel of land in Pangasinan with an area of 827 square meters. She
January 2005- against the person, as distinguished further alleged that on January 9, 1999, Jensen, by means of force, strategy and stealth, gained entry into her property by
Zapanta from a judgment against the propriety to excavating a portion thereof and constructing a fence. Domagas was deprived of 60 square meters of her property along the
determine its state. It has been held that boundary line.
an action in personam is a proceeding to
enforce personal rights or obligations; The summons and the complaint were not served on Jensen because she was apparently out of the country. This was relayed
such action is brought against the to the Sheriff by her brother, Oscar Layno, who was then in Jensen’s house. The Sheriff left the summons and complaint with
person. An action for unlawful detainer Oscar Layno, who received the same.
or forcible entry is a real action and in
personam because the plaintiff seeks to On May 17, 1999, the court rendered judgment ordering Jensen and all persons occupying the property to vacate the disputed
enforce a personal obligation or liability area and to pay monthly rentals. Jensen failed to appeal the decision. A writ of execution was issued on September 27, 1999.
on the defendant under Art. 539 of the
New Civil Code. On August 16, 2000, Jensen filed a complaint against Domagas before the RTC of Dagupan City for the annulment of the
decision of the MTC on the ground that due to the Sheriff’s failure to serve the complaint and summons, the MTC
never acquired jurisdiction over her person. Apparently Jensen was in Oslo, Norway when summons was sent to her
house. Jensen alleged that: (a) when the complaint was filed, she was no longer a resident of Calasiao, Pangasinan but Oslo,
Norway, and she had leased her house to Eduardo Gonzales; (b) she was at Oslo at the time the complaint was served; (c) her
brother was not a resident of the house and had no authority to receive the summons and complaint for and in her behalf.

Jensen filed a Manifestation and appended thereto: (a) copy of her passport showing she left the country; (b) affidavit that she
was married to Jarl Jensen, and had resided in Norway with her husband since 1993; (c) contract of lease between Jensen and
Gonzales; (d) affidavit of Oscar stating he was there to collect the rental payment from Gonzales.

RTC rendered judgment in favor of Jensen. The trial court declared that there was no valid service of the complaint and
summons on Jensen, considering that she left the Philippines on February 17, 1999, for Oslo, Norway and her brother was
never authorized to receive the said complaint and summons for and in her behalf.

Upon appeal, the appellate court ruled that the complaint was one for ejectment, which is an action quasi in rem. CA ruled
that since Jensen was temporarily out of the country, the summons and complaint should have been served via extraterritorial
service under Sec. 15 in relation to Sec. 16, Rule 14 of the Rules of Court, which requires leave of court. Considering that
there was no prior leave of court and Domagas followed none of the modes of service prescribed by the Rules of Court, the
CA concluded that there was really no valid service of summons and complaint upon Jensen.

ISSUE(S):

w/n the action of the petitioner in the MTC against respondent is an action in personam or quasi in rem.

HELD:

An action in personam is said to be one which has for its object a judgment against the person, as distinguished from a
judgment against the propriety to determine its state. It has been held that an action in personam is a proceeding to enforce
personal rights or obligations; such action is brought against the person. An action for unlawful detainer or forcible entry is a
real action and in personam because the plaintiff seeks to enforce a personal obligation or liability on the defendant under Art.
539 of the New Civil Code.

Citing Asiavest Limited vs. CA, In an action in personam, jurisdiction over the person of the defendant is necessary for the
court to validly try and decide the case. Jurisdiction over the person of a resident defendant who does not voluntarily appear
in court can be acquired by personal service of summons as provided under Section 7, Rule 14 of the Rules of Court. If he
cannot be personally served with summons within a reasonable time, substituted service may be made in accordance with
Section 8 of said Rule. If he is temporarily out of the country, any of the following modes of service may be resorted to: (a)
substituted service set forth in Section 8; (2) personal service outside the country, with leave of court; (3) service by
publication, also with leave of court; or (4) any other manner the court may deem sufficient.
Thus, any judgment of the court which has no jurisdiction over the person of the defendant is null and void.
In the present case, the records show that the respondent, before and after his marriage to Jarl Jensen on August 23, 1987,
remained a resident of Barangay Buenlag, Calasiao, Pangasinan. This can be gleaned from the Deed of Absolute Sale dated
August 26, 1992 in which she declared that she was a resident of said barangay. Moreover, in the Real Estate Mortgage
Contract dated February 9, 1999, ten days before the complaint in Civil Case No. 879 was filed, the petitioner categorically
stated that she was a Filipino and a resident of Barangay Buenlag, Calasiao, Pangasinan. Considering that the respondent was
in Oslo, Norway, having left the Philippines on February 17, 1999, the summons and complaint in Civil Case No. 879 may
only be validly served on her through substituted service under Section 7, Rule 14 of the Rules of Court, which reads:

SEC. 7. Substituted service. If, for justifiable causes, the defendant cannot be served within a reasonable time as provided in
the preceding section, service may be effected (a) by leaving copies of the summons at the defendants residence with some
person of suitable age and discretion then residing therein, or (b) by leaving the copies at defendants office or regular place of
business with some competent person in charge thereof.

Strict compliance with the mode of service is required in order that the court may acquire jurisdiction over the person of the
defendant. The statutory requirement of substituted service must be followed faithfully and strictly and any substituted service
other than that authorized by the statute is rendered ineffective.
As gleaned from the said return, there is no showing that as of April 5, 1999, the house where the Sheriff found Oscar Layno
was the latters residence or that of the respondent herein. Neither is there any showing that the Sheriff tried to ascertain where
the residence of the respondent was on the said date. It turned out that the occupant of the house was a lessor, Eduardo
Gonzales, and that Oscar Layno was in the premises only to collect the rentals from him. The service of the summons on a
person at a place where he was a visitor is not considered to have been left at the residence or place or abode, where he has
another place at which he ordinarily stays and to which he intends to return

4. Go vs. UCPB The cancellation of the real estate Petitioner Go and Looyuko are co-owners of Noah’s Ark International and other Noah’s Ark Sugar companies.
156187 11 mortgage is a real action, They applied for an Omnibus Line accommodation with respondent bank for Php900,000.00 which was approved.
November 2004- considering that a real estate They secured it by way of real estate mortgage over two (2) parcels of land located in Mandaluyong City belonging
Yumul mortgage is a real right and a real to Looyuko and Noah’s Ark Sugar Refinery. Subsequently, UCPB cancelled the Omnibus Line accommodation
property by itself; and an action for prompting petitioner Go to demand the return of the TCT of the two(2) parcels of land covering the real estate
cancellation of real estate mortgage mortgage. UCPB refused and continued to have the same notarized and cause its registration before the registry
is necessarily an action affecting the of deeds of Mandaluyong City. UCPB then filed for the extrajudicial foreclosure of the real estate mortgage for
title to the property. failure to pay off the obligation. Therefore, the public auction sale of the mortgaged property was set.

To protect his interest, Go filed a complaint for the cancellation of Real Estate Mortgage plus damages, against
respondent bank and its officers with the RTC of Pasig City. Instead of filing an answer, UCPB filed a motion to
dismiss on the ground that the court has no jurisdiction due to the failure of payment of filing and docket fees, that
it was filed in the wrong venue.

Petitioner contends that a case for cancellation of mortgage is a personal action and since he resides in Pasig
City, venue was properly laid therein.

Issue/s:
WON petitioner’s complaint for cancellation of the real estate mortgage is a personal or real action for the purpose
of determining the venue.

Held:
It is a REAL ACTION. Therefore, the venue for real action is the same for RTCs and MTCs, the court which has
territorial jurisdiction over the area where the real property or any part thereof lies. It was succinctly stated by
jurisprudence that the prayer for the nullification of the mortgage is a prayer affecting real property, hence, is a real
action.

In contrast, the venue for personal actions likewise the same for the RTC and MTC, the court of the place where
the plaintiff or any of the principal plaintiffs reside, or where the defendant or any of the principal defendants
reside, at the election of the plaintiff.

Wherefore, the instant petition is DENIED.

5.Cabutihan vs. Sections 1 and 2, Rule 4 of the Rules of Facts:


Landcenter 146594 Court provide an answer to the issue of
10 June 2002- venue. Actions affecting title to or On December 3, 1996, herein respondent — Landcenter Construction & Development Corporation, represented by Wilfredo
Abad possession of real property or an B. Maghuyop — entered into an Agreement with Petitioner Rebecca Cabutihan.
interest therein (real actions), shall be Armed with Board Resolution No. 01, Series of 1997, which had authorized her to represent the corporation, Luz Baylon
commenced and tried in the proper Ponce entered into a February 11, 1997 Deed of Undertaking with a group composed of petitioner, Wenifredo P. Forro,
court that has territorial jurisdiction over Nicanor Radan Sr. and Atty. Prospero A. Anave.
the area where the real property is
situated. On the other hand, all other An action for specific performance with damages was filed by petitioner on October 14, 1999 before the RTC of Pasig City,
actions, (personal actions) shall be Branch 263. She alleged:
commenced and tried in the proper
courts where the plaintiff or any of the
principal plaintiffs resides or where the
defendant or any of the principal a. Petitioner accomplished her undertakings under the subject Agreement and the Undertaking. So in a
defendants resides. In the present case, letter [respondent corporation] was informed accordingly thereof. Simultaneously, [petitioner] demanded upon [respondent
petitioner seeks payment of her services corporation] to execute the corresponding Deed of Assignment of the lots in the subject property, as compensation for the
in accordance with the undertaking the services rendered in favor of the [respondent corporation]. The subject letter was duly received and acknowledged receipt, by
parties signed. Breach of contract gives then Acting Corporate Secretary of the [respondent corporation].
rise to a cause of action for specific b. Respondent corporation failed and refused to act on said demand of petitioner.
performance or for rescission. If the c. The subject property was already transferred to and registered in the name of respondent corporation
petitioner had filed an action in rem for d. Said title of the property now in the possession of the respondent corporation, petitioner is apprehensive that the
the conveyance of real property, the more that she will not be able to obtain from respondent corporation, compliance with the aforestated Agreement and
dismissal of the case would have been Undertaking ;
proper on the ground of lack of cause of e. Petitioner through counsel sent a Formal Demand, to comply with its obligation but respondent corporation did not
action. heed the demand.

Petitioner prayed, inter alia, that respondent corporation be ordered to execute the appropriate document assigning,
conveying, transferring and delivering the particular lots in her favor. The lots represented compensation for the undertakings
she performed and accomplished, as embodied in the Agreement.

Respondent sought the dismissal of the Complaint on the grounds of (1) improper venue, (2) lack of jurisdiction over the
subject matter, and (3) nonpayment of the proper docket fees.

RTC- the allegations in the Complaint show that its primary objective was to recover real property. The prayer was to compel
respondent to execute the necessary deeds of transfer and conveyance of a portion of the property corresponding to 36.5
percent of its total area or, in the alternative, to hold respondent liable for the value of the said portion, based on the prevailing
market price. The RTC further ruled that, since the suit would affect the title to the property, it should have been instituted in
the trial court where the property was situated.

The action was filed only by petitioner. There was no allegation that she had been authorized by Forro, Radan and Anave to
represent their respective shares in the compensation. Finally, since this case was an action in rem, it was imperative for
petitioner to pay the appropriate docket or filing fees equivalent to the pecuniary value of her claim, a duty she failed to
discharge

Issue/s:
Whether or not the dismissal of the complaint was in accordance with the pertinent law and jurisprudence on the matter.

Held:
First Issue:

Proper Venue

We agree with petitioner. Sections 1 and 2, Rule 4 of the Rules of Court provide an answer to the issue of venue. 17 Actions
affecting title to or possession of real property or an interest therein (real actions), shall be commenced and tried in the proper
court that has territorial jurisdiction over the area where the real property is situated. On the other hand, all other actions,
(personal actions) shall be commenced and tried in the proper courts where the plaintiff or any of the principal plaintiffs
resides or where the defendant or any of the principal defendants resides.
A close scrutiny of National Steel and Ruiz reveals that the prayers for the execution of a Deed of Sale were not in any way
connected to a contract, like the Undertaking in this case. Hence, even if there were prayers for the execution of a deed of
sale, the actions filed in the said cases were not for specific performance. In the present case, petitioner seeks payment of
her services in accordance with the undertaking the parties signed. Breach of contract gives rise to a cause of action for
specific performance or for rescission. 23 If petitioner had filed an action in rem for the conveyance of real property, the
dismissal of the case would have been proper on the ground of lack of cause of action

Second Issue:

Non-Joinder of Proper Parties

Petitioner claims that she was duly authorized and empowered to represent the members of her group and to prosecute their
claims on their behalf via a Special Power of Attorney executed by Forro, Radan and Anave. Besides, she argues that the
omission of her companions as plaintiffs did not prevent the RTC from proceeding with the action, because whatever
judgment would be rendered would be without prejudice to their rights. In the alternative, she avers that the trial court may
add or drop a party or parties at any stage of the action and on such terms as are just.

Again, we side with petitioner. Neither a misjoinder nor a non-joinder of parties is a ground for the dismissal of an action.
Parties may be dropped or added by order of the court, on motion of any party or on the court's own initiative at any stage of
the action. The RTC should have ordered the joinder of such party, and noncompliance with the said order would have been
ground for dismissal of the action.

Although the Complaint prayed for the conveyance of the whole 36.5 percent claim without impleading the companions of
petitioner as party-litigants, the RTC could have separately proceeded with the case as far as her 20 percent share in the
claim was concerned, independent of the other 16.5 percent. This fact means that her companions are not indispensable
parties without whom no final determination can be had. At best, they are mere necessary parties who ought to be impleaded
for a complete determination or settlement of the claim subject of the action. The non-inclusion of a necessary party does not
prevent the court from proceeding with the action, and the judgment rendered therein shall be without prejudice to the rights
of such party.

Third Issue:

Correct Docket Fees

Petitioner insists that the value of the real property, which was the subject of the contract, has nothing to do with the
determination of the correct docket or filing fees. The RTC ruled that although the amount of damages sought had not been
specified in the body of the Complaint, one can infer from the assessed value of the disputed land that it would amount to P50
million. Hence, when compared to this figure, the P210 paid as docket fees would appear paltry. We hold that the trial court
and respondent used technicalities to avoid the resolution of the case and to trifle with the law. True, Section 5, Rule 141 of
the Rules of Court requires that the assessed value of the real estate, subject of an action, should be considered in computing
the filing fees. But the Court has already clarified that the Rule does not apply to an action for specific performance, which is
classified as an action not capable of pecuniary estimation.
Besides, if during the course of the trial, petitioner's 20 percent claim on the Fourth Estate Subdivision can no longer be
satisfied and the payment of its monetary equivalent is the only solution left, Sunlife Insurance Office, Ltd. v. Asuncion holds
as follows: "Where the filing of the initiatory pleading is not accompanied by payment of the docket fee, the court may allow
payment of the fee within a reasonable time but in no case beyond the applicable prescriptive or reglementary period."

6. Gochan vs. The court acquire jurisdiction over FACTS:


Gochan 146089 13 any case only upon the payment of Respondent wrre stockholders of the Felix Gochan and Sons Realty Corporation and the Mactan Realty
December 2001- the prescribed docket fees. Development Corporation. Respondents offered to sell their shares in the two corporations to the petitioner for and
De Gala in consideration of the sum of 200k. It was accepted by the petitioner and paid the amount to respondents.

Respondent executed their respective “Release, Waiver, and Quitclaim” wherein they undertook that they would
initiate any suit, action, or complaint against petitioner for whatever reason or purpose. Respondents, through
Crispo Gochan Jr. required individual petitioners to execute a promissory note, undertaking not to divulge the
actual consideration they paid for the shares of stock. Unknown to the petitioner, Gochan Jr. inserted in the
promissory note a phrase that says, “Said amount is in partial consideration of the sale.”

ISSUE:
Whether or not CA acted with grave abuse of discretion amount to lack or excess of jurisdiction in ruling that the
correct docket fee have been paid

HELD:
Yes. The rule is well-settled that the court acquire jurisdiction over any case only upon the payment of the
prescribed docket fees.

It is necessary to determine the true nature of the complaint in order to resolve the issue of whether or not the
respondents paid the correct amount of docket fees therefor. In this jurisdiction, the dictum adhered to is that the
nature of an action is determined by the allegations in the body of the pleading or complaint itself, rather than by
its title or heading. The caption of the complaint below was dominated as one for specific performance and
damages. The relief sought, however, is the conveyance or transfer of real property, or ultimately, the execution of
deeds of conveyance in favor of the real properties enumerated in the provisional memorandum of agreement.
Under these circumstances, the case below was actually a real action, affecting as it is the trial court was in the
nature of real action, although ostensibly denominated as one for specific performance.

Consequently, the basis for determining the correct docket fees shall be the assessed value of the property, or the
estimated value thereof as alleged by the claimant.

7. Tacay vs. RTC of


Tagum 88075-77 BP 129 provides that RTCs shall Facts: Three (3) actions for recovery of possession (acciones publicianas) were separately instituted by Godofredo Pineda
exercise exclusive original jurisdiction against three (3) defendants. The complaints all alleged the same essential facts (1) Pineda was the owner of a parcel of land
20 December 1989- inter alia over "all civil actions which measuring 790 square meters, his ownership being evidenced by a TCT; (2) the previous owner had allowed the defendants
Espitiru involve the title to, or possession of, real to occupy portions of the land by mere tolerance; (3) having himself need to use the property, Pineda had made demands on
property, or any interest therein, except the defendants to vacate the property and pay reasonable rentals therefor, but these demands had been refused; and (4) the
actions for forcible entry into and last demand had been made more than a year prior to the commencement of suit. The prayer of each complaint contained a
unlawful detainer of lands or buildings. handwritten notation (evidently made by plaintiff's counsel) reading, "P5,000.00 as and for," immediately above the
typewritten words, "Actual damages, as proven," the intention apparently being to make the entire phrase read, " P5,000.00
The rule applies regardless of the value as and for actual damages as proven.
of the real property involved, whether it
be worth more than P20,000.00 or not. Motions to dismiss were filed in behalf of each of the defendants by common counsel. Every motion alleged that the Trial
The rule also applies even where the Court had not acquired jurisdiction of the case for the reason that the complaint violates the mandatory and clear provision of
complaint involving realty also prays for Circular No. 7 of the Supreme Court dated March 24,1988, by failing to specify all the amounts of damages which plaintiff is
an award of damages; the amount of claiming from defendant. Judge Matas denied the motion to dismiss. The defendants in all three (3) actions have filed with
those damages would be immaterial to this Court a "Joint Petition" for certiorari, prohibition and mandamus, with prayer for temporary restraining order and/or writ of
the question of the Court's jurisdiction. preliminary prohibitory injunction," praying essentially that said orders be annulled and respondent judges directed to dismiss
all the complaints. The joint petition re-asserted the proposition that because the complaints had failed to state the amounts
being claimed as actual, moral and nominal damages, the Trial Courts a quo had not acquired jurisdiction over the three (3)
actions in question-indeed.

Issue: Whether or not the court acquired jurisdiction

Held: YES. While it is true that the complaints do not state the amounts being claimed as actual, moral and nominal damages,
it is also true, however, that the actions are not basically for the recovery of sums of money. They are principally for recovery
of possession of real property, in the nature of an accion publiciana. Determinative of the court's jurisdiction in this type of
actions is the nature thereof, not the amount of the damages allegedly arising from or connected with the issue of title or
possession, and regardless of the value of the property. Quite obviously, an action for recovery of possession of real property
(such as an accion plenaria de possesion) or the title thereof, or for partition or condemnation of, or the foreclosure of a
mortgage on, said real property - in other words, a real action-may be commenced and prosecuted without an accompanying
claim for actual, moral, nominal or exemplary damages; and such an action would fall within the exclusive, original jurisdiction
of the Regional Trial Court.

8. Redeña vs.
Court of Appeals
146611 06
February 2007-
Nakagawa

Additional Cases

9. Felicisimo
Fernandez, Sps. Courts, however, are not shorn of This is a Petition for Review on Certiorari[1] under Rule 45 seeking to annul and set aside the Decision[2] and the
Danilo & Generosa the discretion to suspend the rules Resolution[3] of the Court of Appeals (CA) dated 22 December 2008 and 17 April 2009, respectively, in CA-
or except a particular case from their
Vitug-Ligon vs Sps. operation when their rigid G.R.SP No. 85011.
Isaac & application would frustrate rather
Concepcion Ronulo than promote justice. The policy is to FACTS
(G.R.No. 187400, maintain a healthy balance between
July 13, 2016)- the strict enforcement of procedural Sometime in 1970, Tomas Fernandez filed a Free Patent Application over a parcel of land with an area of 9,478
NATO laws and the guarantee that litigants square meters located in Sitio Kuala, Barangay Wawa in Nasugbu, Batangas. When he died, his son Felicisimo
are given the full opportunity for a (herein petitioner) pursued the application. On 24 April 1984, the Bureau of Lands (BoL) approved Survey Plan
just and proper disposition of their Psu No. 04-008565 covering the entire property.
cause. In some cases, it is a far
better and more prudent cause of In 1985, respondents asked the OP to investigate their claim that the approved Survey Plan in the name of Tomas
action for the court to excuse a Fernandez included the 1,000 square meters of land they had been occupying since the 1950s. The OP referred
technical lapse and afford the the matter to the BoL, which then referred it to the DENR Region IV Office for appropriate action.
parties a review of the case to attain
the ends of justice. In those cases, Acting on that same request of respondents, Presidential Executive Assistant Juan C. Tuvera also issued a
in which technicalities are dispensed Memorandum dated 12 April 1985 regarding the matter. The request became the subject of a Memorandum Order
with, the courts do not mean to of Investigation dated 25 April 1985 sent by Assistant Regional Director Claudio C. Batilles, Regional Lands Office
undermine the force and effectivity No. IV, Quezon City, to Atty. Raymundo L. Apuhin of the same office.
of the periods set by law. When the
courts do so, it is because of the Findings of the DENR Region IV Office
existence of a clear need to prevent
the commission of a grave injustice. On 20 March 1985, Land Inspector Julian B. De Roxas of the Sub Office of the BoL in Balayan, Batangas,
conducted an investigation and ocular inspection to determine the veracity of respondents' claim. Roxas submitted
his Report of Investigation on 21 May 1985 recommending the dismissal of the claim. Concurring with the Report,
the officer-in-charge of the sub office indorsed it to the Regional Land Director, Regional Office No. IV, Quezon
City, on the same date.

Findings of Regional Lands Office No. IV

Atty. Apuhin likewise conducted his own investigation and ocular inspection covering the subject land on 20 May
1985. In his initial report dated 21 May 1985 submitted to Assistant Regional Director Batilles, Atty. Apuhin verified
and ascertained that (1) the land was situated at Sitio Kuala, Barangay Wawa, Nasugbu, Batangas; (2) there were
improvements on the property allegedly introduced by respondents; and (3) respondents had previously stayed
outside the land and only transferred their house within in 1984. The report also mentioned that Fernandez could
not pinpoint the improvements that he and his predecessors-in-interest might have introduced on the land.

On 26 November 1987, Atty. Apuhin wrote a letter to the Regional Technical Director (RTD) of the Land
Management Sector in Region IV. The former requested that the continuation of the investigation be referred to
the District Land Officer of Balayan, Batangas, up to its termination. RTD Pedro Calimlim acted on the request in a
1st Indorsement dated 04 December 1987.
On 18 April 1991, Atty. Apuhin submitted his Final Report of Investigation to the Regional Executive Director of
DENR Region IV in Ermita, Manila. The former recommended that the survey plan in the name of Tomas
Fernandez be cancelled.

Regional Executive Director (Provision Region IV-A) Antonio G. Principe subsequently issued an Order dated 09
October 1995 in DENR Case No. IV-5516, Isaac and Conception Ronulo v. Felicisimo Fernandez, adopting in toto
the report and recommendation of Atty. Apuhin. The Order stated that petitioner Fernandez failed to establish his
claim of ownership over the land in question and was found to have never occupied or possessed even a portion
thereof. It was ruled that respondents had a better preferential right to the land in question for being its actual
occupants and possessors for quite a number of years already.

On 20 November 1995, petitioner Fernandez moved for reconsideration of the Order dated 9 October 1995.
Director Principe denied the motion on 8 January 1996. The Order became final and executory, as no appeal
thereon was filed within the allowed period. The DENR Region IV Office issued a Certificate of Finality dated 5
March 1996. A day before the issuance of the certification or on 4 March 1996, however, petitioner Fernandez
filed a notice of appeal on the Order of Director Principe at the Office of the DENR Secretary. The appeal was
docketed as DENR Case No. 5102.

THE ISSUES

Whether or not the respondents' second Motion for Reconsideration of the Decision of the DENR Secretary tolled
the period of appeal to the OP; and

THE RULING OF THE COURU

The Petition has no merit

Petitioners argue that the basis of the MTC Decision - which was subsequently affirmed by the RTC, CA, and this
Court - was erroneous. They contend that Director Principe's Order, being the subject of the case at bar, has yet
to become final. Hence, they say that the second Motion for Reconsideration was not based on indubitable
grounds and should not have tolled the appeal.

Petitioners further contend that the second Motion for Reconsideration was not filed under any extraordinary
circumstance to warrant a liberal interpretation of the rules and a waiver of the procedural proscription against the
filing thereof. Citing various cases, they stress that (1) procedural rules are not to be disdained as mere
technicalities that may be ignored at will to suit the convenience of a party; (2) justice is to be administered
according to the rules in order to obviate arbitrariness, caprice, or whimsicality; (3) rules of procedure are intended
to ensure the orderly administration of justice and the protection of substantive rights in judicial and extrajudicial
proceedings; (4) procedural rules are not to be belittled or dismissed simply because their nonobservance may
have resulted in prejudice to a party's substantive rights, they are required to be followed except only when, for the
most persuasive of reasons, they may be relaxed to relieve litigants of an injustice not commensurate with the
degree of their thoughtlessness in not complying with the procedure prescribed; and (5) liberality in the
interpretation and application of the rules applies only in proper cases and under justifiable causes and
circumstances.

In light of these arguments, petitioners conclude that the CA should have ruled that the appeal of respondents to
the OP was not interposed within the reglementary period, resulting in the finality of the DENR Secretary's
Decision.

On their end, respondents aver that filing a second Motion for Reconsideration is not absolutely prohibited and is
allowed in exceptionally meritorious circumstances, as in the instant case. They claim that this case is imbued with
utmost public interest, since it involves the integrity and validity of a public land grant and, as such, warrants a
liberal interpretation of the rules. They cite Allied Banking Corporation and Pacita Uy v. Spouses David and
Zenaida Eserjose,[99] in which the Court held that "[t]he period for appeal set by law must be deemed mandatory
save for the most extraordinary of circumstances."

Respondents assert that petitioners' Motion for Reconsideration before DENR Region IV and appeal filed with the
DENR Secretary were the ones actually time-barred. They said petitioners' counsel received the Order on 20
October 1995, but filed the Motion for Reconsideration only on 20 November 1995. They also claim that
petitioners' counsel received the notice of the denial of the Motion for Reconsideration on 3 February 1996, but
filed an appeal only on 4 March 1996. They present the Certificate of Finality dated 5 March 1995 on file with the
DENR Region IV Office to prove that the Order of Director Principe had long become final and executory.

Both parties presented allegations that the other committed technical procedural lapses in the course of this case.
Clearly they are aware that observance of the rules of procedure should not be lightly estimated, as the Court
considers it a matter of public policy. Indeed, the rules were conceived and promulgated not only to effectively
dispense justice, but also to fully protect the rights of the parties.

Courts, however, are not shorn of the discretion to suspend the rules or except a particular case from their
operation when their rigid application would frustrate rather than promote justice. The policy is to maintain a
healthy balance between the strict enforcement of procedural laws and the guarantee that litigants are given the
full opportunity for a just and proper disposition of their cause. In some cases, it is a far better and more prudent
cause of action for the court to excuse a technical lapse and afford the parties a review of the case to attain the
ends of justice. In those cases, in which technicalities are dispensed with, the courts do not mean to undermine
the force and effectivity of the periods set by law. When the courts do so, it is because of the existence of a clear
need to prevent the commission of a grave injustice.

10. Lamberto
Casalla vs People
of the Philippines
and Milagros
Estevanes; G.R.
No. 138855,
October 29, 2002-
Perez
CIVIL PROCEDURE: MARCH 20, 2021 CASES
CASE DOCTRINE FACTS / ISSUE / HELD

UNIFORM PROCEDURE IN TRIAL COURTS


1. DE LIMA V. In many instances, the Court adopted Facts:
GATDULA a policy of liberally construing its rules Respondent Gatdula filed a Petition for the Issuance of a Writ of Amparo in the RTC- Manila. The Amparo was directed
[GR 204528] in order to promote a just, speedy and against the petitioner De Lima, et al, Gatdula wanted De Lima et, et al. “to cease and desist from framing up petitioner
19 FEB 2013 inexpensive disposition of every (Gatdula) for the fake ambush incident by filing bogus charges of Frustrated Murder against petitioner in relation to the
CASTRO action and proceeding. The rules can alleged ambush incident.” Thereafter, the judge issued summons and order De Lima et al. to file an Answer. He also set the
be suspended on the following case for hearing. The hearing was held allegedly for determining whether a temporary protection order may be issued.
grounds: (1) matters of life, liberty, During that hearing, counsel for De Lima et al. manifested that a Return, not an Answer, is appropriate in Amparo cases. In
honor or property, (2) the existence of an order, the judge insisted that since no writ has been issued, return is not the required pleading but answer. The judge
special or compelling circumstances, noted that the ROC applies suppletory in Amparo cases. He stated that the Revised Rules of Summary Procedure applied
and thus required an Answer. The judge the proceeded to conduct a hearing on the main case. Even without a Return nor
(3) the merits of the case, (4) a cause
an Answer, he ordered the parties to file their respective memoranda. The court further decided that the memorandum of de
not entirely attributable to the fault or
Lima et at would be filed in lieu of their answer.
negligence of the party favored by the
suspension of the rules, (5) a lack of
Issue:
any showing that the review sought is
Whether or not there are procedural irregularities in the issuing of Writ of Amparo in RTC.
merely frivolous and dilatory, and (6)
the other party will not be unjustly
Ruling:
prejudiced thereby.
Yes.
A) The insistence on filing of an Answer
It is the return that serves as the responsive pleading for petitions for the issuance of writs ofamparo. The requirement to file
an answer is contrary to the intention of the court to provide a speedy remedy to those whose right to life, liberty and security
are violated or are threatened to be violated. Judge Pampilo’s basis for requiring an Answer was section 5 of Revised Rules
of Summary Procedure since the Rules of Court shall apply suppletorily insofar as it is not inconsistent with the Rule on the
Writ of Amparo. But the Judge’s basis is misplaced since this type of summary procedure only applies to lower courts. Aside
from that, this Court limited the application of summary procedure to certain civil and criminal cases. A writ of Amparo is a
special proceeding. It is a remedy by which a party seeks to establish a status. A right or particular fact. It is not a civil nor
criminal action, hence the application of the Revised Rules on Summary Procedure is seriously misplaced.
B) Holding of a hearing on the main case prior to the issuance of the writ and filing of a Return
C) RTC requiring a memorandum in lieu of responsive pleading (Answer) of De Lima, et al
The Return in amparo cases allows the respondents to frame the issues subject to a hearing. Hence, it should be done prior
to the hearing, not after. A memorandum, on the other hand is a synthesis of the claims of the party litigants and is a final
pleading usually required before the case is submitted for decision. One cannot substitute for the other since theses
submissions have different functions in facilitating the suit. More importantly, a memorandum is prohibited pleading under the
rules on the writ of Amparo.
D) The decision dated 20 march 2012
In the body of its decision, The RTC stated. “Accordingly this court Grants the privilege of the writ and the interim reliefs
prayed for the by petitioner”
This gives the impression that the decision was the judgement since the phraseology is similar to Section 19 of the Rule on
the Writ of Amparo
The privilege of the writ of Amparo should be distinguished from the actual order called the writ of Amparo. The privilege
includes availment of the entire procedure outlined in A.M No. 07-9-12- SC, the Rule on Writ of Amparo. After examining the
petition and its attached affidavits, the Return and the evidence presented in the summary hearing, the judgement should
detail the required
acts from the respondents what will mitigate, if not totally eradicate the violation of or the threat to a petitioner’s life.

A judgement which simply grants “the privilege of the writ” cannot be executed. It is tantamount to a failure of the judge to
intervene and grand judicial succor to the petitioner. Petitions filed to avail of the privilege of the Writ of Amparo arise out of
very real and concrete circumstances. Judicial responses cannot be as tragically symbolic or ritualistic “granting the privilege
of writ of amparo”

2. PASCUAL V. All cases of forcible entry and unlawful Facts: Complainant filed a complaint for forcible entry against a certain Manaois. The complaint was dismissed without
JOVELLANOS [AM detainer are governed by the Rules on prejudice for being insufficient in some material allegations. Thus, she filed a corrected complaint. Instead of filing an
MTJ-02-1429] 04 OCT Summary Procedure. [Section 1, Rule 5 answer, defendant filed a Motion to Strike Out arguing that the new allegations in the complaint are false. After the period to
2002 of the RoC provides that the procedure answer lapsed and no answer was submitted, complainant filed a Motion for Summary Judgment. However, defendant's
LIMIN in the Municipal Trial Courts shall be the motion to strike out was granted by respondent Judge.
same as in the Regional Trial Courts,
except ... (b) in civil cases governed by Meanwhile, defendant, taking advantage of the lull in the proceedings, started the construction of a one-storey building on
the Rule on Summary Procedure.] the subject land. To protect her interest, complainant filed an Application for Preliminary Injunction. However, 2 years later or
Municipal trial court judges ought to be until the present, respondent Judge has not ruled on her application on preliminary injunction.
familiar with the Rules on Summary
Procedure governing ejectment cases. Instead of obeying the TRO, defendant continued with the construction of the building and even started with a new one.
Failure to observe them constitutes Hence, a contempt charge was filed by herein complainant. Defendant moved to dismiss the contempt charge. However,
gross ignorance of the law. respondent Judge still has not resolved the aforesaid motion to the prejudice of herein complainant.

Thus, an administrative complaint was filed by complainant charging Judge Jovellanos of the Municipal Circuit Trial Court of
Alcala, Pangasinan with gross ignorance of the law, bias and partiality, abuse of discretion and neglect of duty. The OCA
found that respondent failed to apply the Rule on Summary Procedure, which he ought to have been very conversant with,
because it was a common procedure in municipal courts.

Issue: W/N Respondent Judge failed to apply the Rules of Summary Procedure, holding him administratively liable

Held: Yes. In this case, it is clear that respondent lacks awareness of the relevant provisions on ejectment. He has evidently
been remiss in resolving the forcible entry case, pursuant to the Revised Rules on Summary Procedure. Verily, judgment
should have been rendered based on the allegations of the Complaint and the evidence presented therein, inasmuch as the
defendant failed to file his answer after the lapse of 10 days from the service of the summons. Section 6 of the Rule allows
the trial court to render judgment, even motu proprio, upon failure of the defendant to file an answer within the reglementary
period. Moreover, under Section 10 of the Rule, respondent was duty-bound to render his decision within 30 days from
receipt of the last affidavits and position papers, or the expiration of the period for filing them. This notwithstanding, he has
not yet ruled on the Motion for Summary Judgment dated December 15, 1999, filed in accordance with Section 6 of the Rule
on Summary Procedure.
Furthermore, respondent failed to apply these very basic rules when he granted the defendant's Motion to Strike Out which
was in reality a motion to dismiss, a prohibited pleading. In his Order, he ruled that the Complaint in 2nd civil case filed by
complainant was a mere rehash of the dismissed Complaint in the 1st civil case. He cited Section 12 of Rule 8 of the 1997
Rules on Civil Procedure as basis for this ruling. In doing so, he committed an obvious mistake showing gross ignorance of
the law. All cases of forcible entry and unlawful detainer are governed by the Rules on Summary Procedure. [Section 1, Rule
5 of the RoC provides that the procedure in the Municipal Trial Courts shall be the same as in the Regional Trial Courts,
except ... (b) in civil cases governed by the Rule on Summary Procedure.]

3. LUCAS V. The motion for reconsideration Facts: Complainant Lucas was the defendant in an ejectment case pending before respondent judge. She alleges that
FABROS [AM MTJ- prohibited by Section 19 (c) of the Rules Judge Fabros granted the plaintiff’s motion for reconsideration after the case had been dismissed the case for failure of
99-1266] 31 JAN 2000 of Summary Procedure is that which plaintiff and her counsel to appear at the Preliminary Conference. She averred that it is elementary, under Section 19(c) of
UBAY seeks reconsideration of the judgment the Rules of Summary Procedure, that a motion for reconsideration is prohibited, but respondent judge, in violation of the
rendered by the court after trial on the rule, granted the motion for reconsideration. She added that, notwithstanding the fact that the respondent herself had
merits of the case. The order of pointed out in open court that the case is governed by the Rules on Summary Procedure, the judge ordered the revival of the
dismissal issued by respondent judge case out of malice, partiality and with intent to cause an injury to complainant. Thus, the instant complaint, charging
due to failure of a party to appear during respondent judge with Gross Ignorance of the Law and Grave Abuse of Discretion
the preliminary conference is obviously
not a judgment on the merits after trial Issue: Whether or not Judge Fabros erred in granting the motion for reconsideration
of the case.
Held: Yes. The Court find that respondent Judge Fabros abused her discretion in granting the Motion for Reconsideration.
Respondent Judge Fabros maintained that she could not be guilty of gross ignorance of the law as she knows that a motion
for reconsideration of judgment is a prohibited motion in an ejectment case. She explained that although there is already a
judgment dismissing the case, she granted the plaintiffs motion for reconsideration in the interest of justice since the reasons
stated in the motion for reconsideration are meritorious.
Respondent failed to realize that the first duty of the court is to apply the law and that when the law is clear and
unambiguous, there is no room for interpretation. Although her intention was good, this could not free her from liability.
Respondent should have denied the motion since the plaintiff had other judicial remedies like appeal.
The Office of the Court Administrator recommended that respondent judge be fined in the amount of P2,000.00 for grave
abuse of discretion. The Court, however, finds this recommendation without factual and legal basis.
As a rule, a motion for reconsideration is a prohibited pleading under Section 19 of the Revised Rule on Summary
Procedure. Thus,
"SEC. 19. Prohibited pleadings and motions. The following pleadings, motions, or petitions shall not be allowed in the cases
covered by this Rule.
(c) Motion for new trial, or for reconsideration of a judgment, or for reopening of trial;
This rule, however, applies only where the judgment sought to be reconsidered is one rendered on the merits. As held by the
Court in an earlier case involving Sec. 15 (c) of the Rules on Summary Procedure, later Sec. 19 (c) of the Revised Rules on
Summary Procedure effective November 15, 1991: "The motion prohibited by this Section is that which seeks
reconsideration of the judgment rendered by the court after trial on the merits of the case." Here, the order of dismissal
issued by respondent judge due to failure of a party to appear during the preliminary conference is obviously not a judgment
on the merits after trial of the case. Hence, a motion for the reconsideration of such order is not the prohibited pleading
contemplated under Section 19 (c) of the present Rule on Summary Procedure. Thus, respondent judge committed no grave
abuse of discretion, nor is she guilty of ignorance of the law, in giving due course to the motion for reconsideration subject of
the present complaint.
KINDS OF PLEADINGS (RULE 6, SECS. 1 TO 13)
4. MONGAO V. There is joinder of issues when the A complaint for rescission and damages was filed by petitioner Mongao against respondent Pryce Properties Corp. before
PRYCE PROPERTIES answer makes a specific denial of the the RTC. The complaint alleged that petitioner Mongao and respondent corporation executed a Memorandum of Agreement,
[GR 156474] 16 AUG material allegations in the complaint wherein the former agreed to sell to the latter for the total price of ₱5,028,800.00 a parcel of land in South Cotabato
2005 or asserts affirmative defenses which registered in the name of petitioner Mongao only. According to the Memorandum, respondent corporation allegedly paid
YUMUL would bar recovery by the plaintiff. petitioners the sum of ₱550,000.00 as earnest money considered as part of the purchase price. The complaint further
Where there is proper joinder of alleged that after considerable delay, respondent corporation offered to pay the balance of the purchase price by issuing a
issues, the trial court is barred from check payable to petitioner Mongao and her mother, Nellie Animas, which was rejected. Respondent corporation
rendering judgment based only on the continuously refused to heed petitioners’ written and oral demands to pay the balance.
pleadings filed by the parties and
must conduct proceedings for the Respondent corporation averred that the true agreement between respondent corporation and the Animas family was for the
reception of evidence. former to purchase the two parcels of land belonging to the late Pedro Animas, father of petitioner Mongao. It admitted the
execution of the Memorandum of Agreement but qualified that respondent corporation did not pay the earnest money directly
On the other hand, an answer fails to and solely to petitioner Mongao.
tender an issue where the allegations
admit the allegations in support of the Respondent corporation averred that petitioner Mongao and Pedro Animas, Jr., the registered owners of the subject
plaintiff’s cause of action or fail to properties, executed simultaneously the corresponding Deed of Sale and Memorandum of Agreement after respondent
address them at all. In either case, corporation’s representative delivered the checks to the bank as payment for redemption of the properties. Controversy
there is no genuine issue and arose after respondent corporation had allegedly manifested its intent to complete payments but petitioner Mongao
judgment on the pleadings is proper. demanded that payment be made to her alone to the exclusion of the rest of the Animas family.

Petitioners moved for judgment on the pleadings on the ground that the answer admitted the material allegations of the
complaint and, therefore, failed to tender an issue.In particular, the answer allegedly admitted the existence of the contract of
sale and respondent corporation’s refusal to satisfy the unpaid balance of the purchase price despite demand.

Respondent corporation opposed petitioners’ motion for judgment on the pleadings, arguing that two material allegations in
the complaint, namely: that petitioner Mongao did not execute the Deed of Sale and that petitioner Mongao was the owner of
the subject property, were disputed in the answer.

The trial court granted petitioners’ motion for judgment on the pleadings. With the adverse decision, respondent corporation
elevated the case to the Court of Appeals, which reversed the trial court’s Decision and remanded the case for trial on the
merits

ISSUE/S:

WON judgment on the pleadings on the ground that respondent corporation’s allegation did not tender an issue is proper.

HELD:

NO. Judgment on the pleadings is governed by Section 1, Rule 34 of the 1997 Rules of Civil Procedure, essentially a
restatement of Section 1, Rule 19 of the 1964 Rules of Court then applicable to the proceedings before the trial court.
Section 1, Rule 19 of the Rules of Court provides that where an answer "fails to tender an issue, or otherwise admits the
material allegations of the adverse party's pleading, the court may, on motion of that party, direct judgment on such
pleading." The answer would fail to tender an issue, of course, if it does not comply with the requirements for a specific
denial set out in Section 10 (or Section 8) of Rule 8; and it would admit the material allegations of the adverse party's
pleadings not only where it expressly confesses the truthfulness thereof but also if it omits to deal with them at all.

WHEREFORE, the instant petition for review is GRANTED.

5. DIO V. SBMA If the dismissal of the complaint FACTS:


[GR 189532] 11 JUN somehow eliminates the cause of the Petitioner H.S. Equities, Ltd., (HSE) is a foreign corporation while Respondent Subic Bay Marine Exploratorium, Inc. (SBME)
2014 counterclaim, then the counterclaim is a domestic corporation. SBME decided to expand its business and needs investors for the construction and operation of
ZAPANTA cannot survive. Conversely, if the the beach resort project.
counterclaim itself states sufficient HSE thru its authorized director, Dio, agreed to invest the amount of US$2.5M. The agreement was reduced into writing
cause of action then it should stand wherein HSE, was afforded minority protection rights such as the right to appoint a member of the board of directors and the
independently of and survive the right to veto certain board resolutions. After HSE initially paid US$200,000.00 for its subscription, it refused to further lay out
dismissal of the complaint. Now, money for the expansion project of the SBME due to the alleged mismanagement in the handling of corporate funds.
having been directly confronted with SBME initiated an intra-corporate dispute before the RTC of Balanga City, Bataan against petitioners HSE and Dio for
the problem of whether the refusing to pay the its unpaid subscription jeopardizing the expansion. Petitioners maintained in their Answer with
compulsory counterclaim by reason of Compulsory Counterclaim that it would be highly preposterous for them to dissuade investors and banks from putting in
the unfounded suit may prosper even money to SBME considering that HSE and Dio are stakeholders of the company with substantial investments therein. In turn,
if the main complaint had been petitioners countered that their reputation and good name in the business community were tarnished as a result of the filing
dismissed, we rule in the affirmative. of the instant complaint, and thus prayed that they be indemnified
RTC dismissed the case motu proprio based on the defective certificate of non-forum shopping which was signed by
Desmond without specific authority from the Board of Directors of SBME. Respondents moves for the reinstatement of the
case after securing a board res but RTC still denied the motion
CA dismissed the case for the failure of the respondents to file their appellants’ brief. Petitioners went back to the RTC to file
a motion to set their counterclaims for hearing but RTC denied for it has already been affirmed with finality by the appellate
court, it has already lost its jurisdiction to act on petitioners’ counterclaim, the compulsory counterclaim being merely
ancillary to the principal controversy.
Petitioners filed this instant Petition for Review on Certiorari

ISSUE:
WON the dismissal of the complaint carries with it the dismissal of the counterclaim

HELD:
YES. The dismissal of the complaint resulted from respondents’ failure to append to the complaint a copy of the board
resolution authorizing Desmond to sign the certificate of non-forum shopping on behalf of SBME. The subsequent dismissal
of the counterclaim, in turn, erroneously proceeded from the ratio that since the main action has already been dismissed with
finality by the appellate court, the lower court has lost its jurisdiction to grant any relief under the counterclaim.
In the significant case of Pinga v. Heirs of German Santiago, this Court speaking through Justice Dante Tinga, resolved the
nagging question as to whether or not the dismissal of the complaint carries with it the dismissal of the counterclaim. Putting
to rest the remaining confusion occasioned by Metals Engineering Resources Corp. v. Court of Appeals and BA Finance
Corporation v. Co, the Court articulated that, in light of the effectivity of the 1997 Rules of Civil Procedure, the correct and
prevailing doctrine is as follows:
A counterclaim may be necessarily dismissed along with the complaint, CLEARLY CONFLICTS with the 1997 Rules of Civil
Procedure.
Thus, the present rule embodied in Sections 2 and 3 of Rule 17 ordains a more equitable disposition of the counterclaims by
ensuring that any judgment thereon is based on the merit of the counterclaim itself and not on the survival of the main
complaint.

As the rule now stands, the nature of the counterclaim notwithstanding, the dismissal of the complaint does not ipso jure
result in the dismissal of the counterclaim, and the latter may remain for independent adjudication of the court, provided that
such counterclaim, states a sufficient cause of action and does not labor under any infirmity that may warrant its outright
dismissal. Stated differently, the jurisdiction of the court over the counterclaim that appears to be valid on its face, including
the grant of any relief thereunder, is not abated by the dismissal of the main action. The court’s authority to proceed with the
disposition of the counterclaim independent of the main action is premised on the fact that the counterclaim, on its own,
raises a novel question which may be aptly adjudicated by the court based on its own merits and evidentiary support.
In Perkin Elmer Singapore Pte Ltd. v. Dakila Trading Corporartion, a case on all fours with the present one, we expounded
our ruling in Pinga and pointed out that the dismissal of the counterclaim due to the fault of the plaintiff is without prejudice to
the right of the defendant to prosecute any pending counterclaims of whatever nature in the same or separate action, thus:
Based on the aforequoted ruling of the Court, if the dismissal of the complaint somehow eliminates the cause of the
counterclaim, then the counterclaim cannot survive. Conversely, if the counterclaim itself states sufficient cause of action
then it should stand independently of and survive the dismissal of the complaint. Now, having been directly confronted with
the problem of whether the compulsory counterclaim by reason of the unfounded suit may prosper even if the main
complaint had been dismissed, we rule in the affirmative.

It bears to emphasize that petitioner's counterclaim against respondent is for damages and attorney's fees arising from the
unfounded suit. While respondent's Complaint against petitioner is already dismissed, petitioner may have very well already
incurred damages and litigation expenses such as attorney's fees since it was forced to engage legal representation in the
Philippines to protect its rights and to assert lack of jurisdiction of the courts over its person by virtue of the improper service
of summons upon it.

WHEREFORE The case is REMANDED to the RTC for further proceedings, on the matter of petitioners Virginia S. Dio and
H.S. Equities, Ltd. 's counterclaims.

6. PADILLA V. With respect to the fact that when an Facts:


GLOBE ASIATIQUE appeal raises only pure questions of Pursuant to and as a condition for the CTS Facility availments, respondents executed in favor of PNB several Deeds of
[GR 207376] 06 AUG law, this Court has jurisdiction to Assignment covering accounts receivables in the aggregate amount of One Billion One Hundred Ninety-Five Million Nine
2014 entertain the same. Hundred Twenty-Six Thousand Three Hundred Ninety Pesos and Seventy-two centavos (₱1,195,926,390.72). In the said
ABAD instruments, respondents acknowledged the total amount of One Billion Three Hundred Ninety FiveMillion Six Hundred
A counterclaim is any claim which a Sixty-Five Thousand Five Hundred Sixty-FourPesos and Sixty-nine centavos (₱1,395,665,564.69) released to them by PNB
defending party may have against an in consideration of the aforesaid accounts receivables.
opposing party. Sometime in the first quarter of 2010, respondents defaulted in the payment of their outstanding balance and delivery to PNB
Under the 1997 Rules of Civil of transfer certificates of title corresponding to the assigned accounts receivables, for which PNB declared them in default
Procedure, it is now explicitly provided under the CTS Facility Agreements. Subsequently, respondents made partial payments and made proposals for paying in
that the dismissal of the complaint full its obligation to PNB as shown in the exchange of correspondence between respondents and PNB. PNB made a formal
due to failure of the plaintiff to and final demand upon respondents to pay/settle the total amount of ₱974,377,159.10 representing their outstanding
prosecute his case is "without obligation. In their complaint, PNB alleged in detail the fraudulent acts and misrepresentations committed by respondents in
prejudice to the right of the defendant obtaining PNB’s conformity to the CTS Facility Agreements and the release of various sums to respondents in the total
amount of ₱974,377,159.10. PNB accused respondents of falsely representing that they have valid and subsisting contracts
to prosecute his counterclaim in the to sell, which evidently showed they had no intention to pay their loan obligations.
same or in a separate action." Respondents filed a suit against petitioner for filing a malicious and devastating unfounded civil action. Petitioner filed her
The doctrine that a counterclaim may Answer With Compulsory Counterclaims,17 praying for the dismissal of respondents’ complaint on the following grounds: (1)
be necessarily dismissed along with submission of a false certification of non-forum shopping by respondents and their blatant commission of willful, deliberate
the complaint, clearly conflicts with and contumacious forum shopping (respondents failed to disclose a criminal complaint entitled "Tbram Cuyugan v. Aida
the 1997 Rules of Civil Procedure. Padilla and Members of the Board of Directors of PNB", docketed as I.S. No. XV-13-INV-11-H-01208 pending before the
office of the CityProsecutor of Pasay City); (2) litis pendentia; (3) respondents’ failure to attach the alleged actionable
The only apparent exception to this document, i.e.the supposed "new term loan", inviolation of Section 7, Rule 8 of the Rules of Court; (4) failure to state a cause
circumstance is if it is alleged in the of action against petitioner; and (5) petitioner cannot be held personally liable for her official acts done for and in behalf of
counterclaim that the very act of the PNB.
plaintiff in filing the complaint Petitioner filed a motion for preliminary hearing on affirmative defenses, contending that respondents are parroting the very
precisely causes the violation of the same arguments raised and relying on the same evidence they presented before the Pasay City RTC to establish the
defendant's rights. Yet even in such alleged novation and purported insufficiency of the attachment bond, which issues are still pending in the said court. It was
an instance, it remains debatable thus stressed that respondents are evidently guilty of forum shopping
whether the dismissal or withdrawal of Defendant Padilla argues that this Court has jurisdictional competence and authority to resolve her counterclaims
the complaint is sufficient to obviate notwithstanding the dismissal of the Complaint dated August 10, 2011 for violation of the principle of judicial stability.
the pending cause of action
maintained by the defendant against Issue:
the plaintiff.
Whether or not a court can take cognizance of a compulsory counterclaim despite the fact that the corresponding complaint
was dismissed for lack of jurisdiction.

Held:
Respondents are incorrect in arguing that petitioner adopted the wrong mode of appeal stating that the remedy from the
dismissal of her counterclaims without prejudice is a petition for certiorari under Rule 65 and not an appeal under Rule 45,
petitioner raises the lone issue of whether the Pasig City RTC was correct in refusing to hear her counterclaims after the
dismissal of respondents' complaint for lack of jurisdiction.
petitioner's counterclaim for damages raised in her answer before the Pasig City RTC is compulsory, alleging suffering and
injury caused to her as a consequence of the unwarranted filing of the baseless complaint the RTC of Pasig City should have
allowed petitioner's counterclaim to proceed notwithstanding the dismissal of respondents' complaint, the same being
compulsory in nature and with its cause not eliminated by such dismissal.
Petitioner was hailed to a separate court (Pasig City RTC) even while the dispute between PNB and respondents was still
being litigated, and she already incurred expenses defending herself.

7. BUNGCAYAO To determine whether a FACTS: Manuel C. Bungcayao, Sr. (petitioner) claimed to be one of the two entrepreneurs who introduced
V. FORT ILOCANDIA counterclaim is compulsory or not, improvements on the foreshore area of Calayab Beach in 1978 when Fort Ilocandia Hotel started its construction in the
PROPERTIES [GR the Court has devised the following area. Thereafter, other entrepreneurs began setting up their own stalls in the foreshore area. They later formed
170483] 19 APR 2010 tests: themselves into the D’Sierto Beach Resort Owner’s Association, Inc.
BRIONES (a) Are the issues of fact and law
raised by the claim and by the In July 1980, six parcels of land in Barrio Balacad (now Calayad) were transferred, ceded, and conveyed to the
counterclaim largely the same? Philippine Tourism Authority (PTA). Fort Ilocandia Resort Hotel was erected in the area. Fort Ilocandia Property
(b) Would res judicata bar a Holdings and Development Corporation (respondent) filed a foreshore application over a 14-hectare area abutting the
subsequent suit on defendant’s Fort Ilocandia Property, including the 5-hectare portion applied for by D’Sierto members. The foreshore applications
claims, absent the compulsory became the subject matter of a conflict case between respondent and D’Sierto members.
counterclaim rule? DENR Regional Executive Director denied the foreshore lease applications of the D’Sierto members, including
(c) Will substantially the same petitioner, on the ground that the subject area applied for fell either within the titled property or within the foreshore
evidence support or refute plaintiff’s areas applied for by respondent. The D’Sierto members appealed the denial of their applications.
claim as well as the defendant’s
counterclaim? and In a letter dated 18 September 2003, respondent, through its Public Relations Manager Arlene de Guzman, invited the
(d) Is there any logical relation D’Sierto members to a luncheon meeting to discuss common details beneficial to all parties concerned. Atty. Liza
between the claim and the Marcos (Atty. Marcos), wife of Governor Bongbong Marcos, was present as she was asked by Fort Ilocandia hotel
counterclaim? A positive answer to officials to mediate over the conflict among the parties.
all four questions would indicate that
the counterclaim is compulsory. Petitioner then filed an action for declaration of nullity of contract before the Regional Trial Court of Laoag City, against
the respondent. Respondent countered that the area upon which petitioner and the other D’Sierto members
constructed their improvements was part of its titled property under Transfer Certificate of Title No. T-31182.
Respondent alleged that petitioner’s sons, Manuel, Jr. and Romel, attended the luncheon meeting on their own volition
and they were able to talk to their parents through a cellular phone before they accepted respondent’s offer.

ISSUE: W/N respondent’s counterclaim is compulsory?

HELD: No. The rule in permissive counterclaim is that for the trial court to acquire jurisdiction, the counterclaimant is
bound to pay the prescribed docket fees. Any decision rendered without jurisdiction is a total nullity and may be struck
down at any time, even on appeal before this Court. In this case, respondent did not dispute the non-payment of
docket fees and only insisted that its claims were all compulsory counterclaims. As such, the judgment by the trial
court in relation to the second counterclaim is considered null and void without prejudice to a separate action which
respondent may file against petitioner.

8. FERNANDO In the case of a written instrument or Facts: Petitioner Fernando Medical Enterprices, Inc., a domestic corporation dealing with medical equipment and supplies
MEDICAL document upon which an action or delivered to and installed medical equipment and supplies at the respondent's hospital with the contracts Memorandum of
ENTERPRISES, INC. V. defense is based, which is also known Agreement and several Deeds of Undertaking. According to the petitioner, the respondent only paid a portion of its total
WESLEYAN as the actionable document, the obligation. However, on February 2009, the petitioner and the respondent, respectively represented by counsels, entered
UNIVERSITY pleader of such document is required into an agreement, where the former agreed to reduce its claim, and allowed the latter to pay the adjusted obligation on
PHILIPPINES, INC. [GR either to set forth the substance of installment basis within 36 months. In the letter dated May 27, 2009, the respondent notified the petitioner that its new
207970] 20 JAN 2016 such instrument or document in the administration had reviewed their contracts and had found the contracts defective and
CASTRO pleading, and to attach the original or rescissible due to economic prejudice or lesion; and that it was declining to recognize the February 2009 agreement because
a copy thereof to the pleading as an of the lack of approval by its Board of Trustees and for having been signed by Maglaya whose term of office had expired. On
exhibit, which shall then be deemed to June 2009, the petitioner sent a demand letter to the respondent. Due to the failure of the respondent to pay upon demand,
be a part of the pleading, or to set the petitioner filed its complaint for sum of money in the RTC. Respondent moved to dismiss the complaint. The RTC denied
forth a copy in the pleading. The the motion to dismiss. Respondent then filed its answer which then the petitioner filed a reply to the answer. The petitioner
adverse party is deemed to admit the filed a Motion for Judgment Based on Pleadings which was opposed by the respondent. The RTC in its Order denied the
genuineness and due execution of the Motion for Judgment Based on Pleadings. The petitioner moved for reconsideration but was denied by the RTC.
actionable document unless he
specifically denies them under oath, Issue: Whether or not the Court of Appeals erred in affirming the RTC’s denial of the petitioner’s motion for judgment on the
and sets forth what he claims to be pleadings.
the facts, but the requirement of an
oath does not apply when the adverse Ruling: Yes. The Supreme Court ruled that the essential query in resolving a motion for judgment on the pleadings is
party does not appear to be a party to whether or not there are issues of fact generated by the pleadings. Whether issues of fact exist in a case or not depends on
the instrument or when compliance how the defending party's answer has dealt with the ultimate facts alleged in the complaint. The defending party's answer
with an order for an inspection of the either admits or denies the allegations of ultimate facts in the complaint or other initiatory pleading. The allegations of
original instrument is refused. ultimate facts the answer admit, being undisputed, will not require evidence to establish the truth of such facts, but the
allegations of ultimate facts the answer properly denies, being disputed, will require evidence. The answer admits the
material allegations of ultimate facts of the adverse party's pleadings not only when it expressly confesses the truth of such
Trial court may render a judgment on allegations but also when it omits to deal with them at all. The controversion of the ultimate facts must only be by specific
the pleadings upon motion of the denial. Section 10, Rule 8 of the Rules of Court recognizes only three modes by which the denial in the answer raises an
claiming party when the defending issue of fact. The first is by the defending party specifying each material allegation of fact the truth of which he does not
party's answer fails to tender an issue, admit and, whenever practicable, setting forth the substance of the matters upon which he relies to support his denial. The
or otherwise admits the material second applies to the defending party who desires to deny only a part of an averment, and the denial is done by the
allegations of the adverse party's defending party specifying so much of the material allegation of ultimate facts as is true and material and denying only the
pleading. For that purpose, only the remainder. The third is done by the defending party who is without knowledge or information sufficient to form a belief as to
pleadings of the parties in the action the truth of a material averment made in the complaint by stating so in the answer. Any material averment in the complaint
are considered. It is error for the trial not so specifically denied are deemed admitted except an
court to deny the motion for judgment averment of the amount of unliquidated damages.
on the pleadings because the
defending party's pleading in another In the case of a written instrument or document upon which an action or defense is based, which is also known as the
case supposedly tendered an issue of actionable document, the pleader of such document is required either to set forth the substance of such instrument or
fact. document in the pleading, and to attach the original or a copy thereof to the pleading as an exhibit, which shall then be
deemed to be a part of the pleading, or to set forth a copy in the pleading. The adverse party is deemed to admit the
genuineness and due execution of the actionable document unless he specifically denies them under oath, and sets forth
what he claims to be the facts, but the requirement of an oath does not apply when the adverse party does not appear to be
a party to the instrument or when compliance with an order for an inspection of the original instrument is refused.

PARTS AND CONTENTS OF PLEADINGS (RULE 7, SECS. 1 TO 6)


9. VALLACAR FACTS: Petitioner is engaged in the business of transportation and the franchise owner of a Ceres Bulilit bus with Plate
TRANSIT INC. V. In contrast, all complaints, No. T-0604-1348. Quirino C. Cabanilla (Cabanilla) is employed as a regular bus driver of petitioner.
JOCELYN CATUBIG petitions, applications, and other On January 27, 1994, respondent’s husband, Quintin Catubig, Jr. (Catubig), was on his way home from Dumaguete
[GR 175512] 30 MAY initiatory pleadings must be City riding in tandem on a motorcycle with his employee, Teddy Emperado (Emperado). Catubig was the one driving
2011 accompanied by a certificate the motorcycle. While approaching a curve at kilometers 59 and 60, Catubig tried to overtake a slow moving ten-
DE GALA against forum shopping, first wheeler cargo truck by crossing-over to the opposite lane, which was then being traversed by the Ceres Bulilit bus
prescribed by Administrative driven by Cabanilla, headed for the opposite direction. When the two vehicles collided, Catubig and Emperado were
Circular No. 04-94, which took thrown from the motorcycle. Catubig died on the spot where he was thrown, while Emperado died while being rushed to
effect on April 1, 1994, then later the hospital.
on by Rule 7, Section 5 of the 1997 On February 1, 1994, Cabanilla was charged with reckless imprudence resulting in double homicide in Criminal Case
Rules of Court. It is not disputed No. M-15-94 before the Municipal Circuit Trial Court (MCTC) of Manjuyod-Bindoy-Ayungon of the Province of Negros
herein that respondent’s complaint Oriental. After preliminary investigation, the MCTC issued a Resolution on December 22, 1994, dismissing the criminal
for damages was accompanied by charge against Cabanilla. It found that Cabanilla was not criminally liable for the deaths of Catubig and Emperado,
such a certificate. because there was no negligence, not even contributory, on Cabanilla’s part.
Thereafter, respondent filed before the RTC on July 19, 1995 a Complaint for Damages against petitioner, seeking
actual, moral, and exemplary damages, in the total amount of ₱484,000.00, for the death of her husband, Catubig,
based on Article 2180, in relation to Article 2176, of the Civil Code. Respondent alleged that petitioner is civilly liable
because the latter’s employee driver, Cabanilla, was reckless and negligent in driving the bus which collided with
Catubig’s motorcycle.
Petitioner, in its Answer with Counterclaim, contended that the proximate cause of the vehicular collision, which resulted
in the deaths of Catubig and Emperado, was the sole negligence of Catubig when he imprudently overtook another
vehicle at a curve and traversed the opposite lane of the road. As a special and affirmative defense, petitioner asked for
the dismissal of respondent’s complaint for not being verified and/or for failure to state a cause of action, as there was
no allegation that petitioner was negligent in the selection or supervision of its employee driver.
On January 26, 2000, the RTC promulgated its Decision favoring petitioner. Based on the sketch prepared by PO2
Elnas, which showed that "the point of impact x x x occurred beyond the center lane near a curve within the lane of the
Ceres bus[;]"17 plus, the testimonies of PO2 Elnas and Cadimas that the motorcycle recklessly tried to overtake a truck
near a curve and encroached the opposite lane of the road, the RTC ruled that the proximate cause of the collision of
the bus and motorcycle was the negligence of the driver of the motorcycle, Catubig. The RTC, moreover, was
convinced through the testimony of Maypa, the Administrative and Personnel Manager of the Dumaguete branch of
petitioner, that petitioner had exercised due diligence in the selection and supervision of its employee drivers, including
Cabanilla.

Respondent appealed to the Court of Appeals. In its Decision dated November 17, 2005, the appellate court held that
both Catubig and Cabanilla were negligent in driving their respective vehicles. Catubig, on one hand, failed to use
reasonable care for his own safety and ignored the hazard when he tried to overtake a truck at a curve. Cabanilla, on
the other hand, was running his vehicle at a high speed of 100 kilometers per hour. The Court of Appeals also brushed
aside the defense of petitioner that it exercised the degree of diligence exacted by law in the conduct of its business.
Maypa was not in a position to testify on the procedures followed by petitioner in hiring Cabanilla as an employee driver
considering that Cabanilla was hired a year before Maypa assumed his post at the Dumaguete branch of petitioner.

ISSUE: Whether or not the complaint should be dismissed to verify the same

HELD:
General rule, a pleading need not be verified, unless there is a law or rule specifically requiring the same. Examples of
pleadings that require verification are: (1) all pleadings filed in civil cases under the 1991 Revised Rules on Summary
Procedure; (2) petition for review from the Regional Trial Court to the Supreme Court raising only questions of law
under Rule 41, Section 2; (3) petition for review of the decision of the Regional Trial Court to the Court of Appeals under
Rule 42, Section 1; (4) petition for review from quasi-judicial bodies to the Court of Appeals under Rule 43, Section 5;
(5) petition for review before the Supreme Court under Rule 45, Section 1; (6) petition for annulment of judgments or
final orders and resolutions under Rule 47, Section 4; (7) complaint for injunction under Rule 58, Section 4; (8)
application for preliminary injunction or temporary restraining order under Rule 58, Section 4; (9) application for
appointment of a receiver under Rule 59, Section 1; (10) application for support pendente lite under Rule 61, Section 1;
(11) petition for certiorari against the judgments, final orders or resolutions of constitutional commissions under Rule 64,
Section 2; (12) petition for certiorari, prohibition, and mandamus under Rule 65, Sections 1 to 3; (13) petition for quo
warranto under Rule 66, Section 1; (14) complaint for expropriation under Rule 67, Section 1; (15) petition for indirect
contempt under Rule 71, Section 4, all from the 1997 Rules of Court; (16) all complaints or petitions involving intra-
corporate controversies under the Interim Rules of Procedure on Intra-Corporate Controversies; (17) complaint or
petition for rehabilitation and suspension of payment under the Interim Rules on Corporate Rehabilitation; and (18)
petition for declaration of absolute nullity of void marriages and annulment of voidable marriages as well as petition for
summary proceedings under the Family Code.
In contrast, all complaints, petitions, applications, and other initiatory pleadings must be accompanied by a certificate
against forum shopping, first prescribed by Administrative Circular No. 04-94, which took effect on April 1, 1994, then
later on by Rule 7, Section 5 of the 1997 Rules of Court. It is not disputed herein that respondent’s complaint for
damages was accompanied by such a certificate.
In addition, verification, like in most cases required by the rules of procedure, is a formal, not jurisdictional, requirement,
and mainly intended to secure an assurance that matters which are alleged are done in good faith or are true and
correct and not of mere speculation. When circumstances warrant, the court may simply order the correction of
unverified pleadings or act on it and waive strict compliance with the rules in order that the ends of justice may thereby
be served.

10. CHUA V.
METROPOLITAN The rule against splitting a cause of Facts: In 1988, petitioner Fidel Chua president of co-petitioner Filiden obtained from respondent Metrobank a loan of
BANK AND TRUST action is intended to prevent ₱4,000,000.00 secured by a real estate mortgage (REM) on parcels of land registered in Chua’s name. Thereafter, petitioners
COMPANY [GR repeated litigation between the obtained other loans from Metrobank. Failing to pay, Metrobank sought to extra-judicially foreclose the REM constituted on
182311] 19 AUG 2009 same parties in regard to the same the subject properties. Upon a verified Petition for Foreclosure filed by Metrobank, respondent Atty. Romualdo Celestra (Atty.
ESPIRITU subject of controversy, to protect Celestra) issued a Notice of Sale. Chua filed before Branch 257 of RTC of Parañaque, a Complaint for Injunction with Prayer
the defendant from unnecessary for Issuance of TRO against respondents Atty. Celestra. After the expiration of the TRO, and no injunction having been
vexation; and to avoid the costs issued, the auction sale proceeded, and a Certificate of Sale was accordingly issued to respondent Metrobank as the highest
and expenses incident to bidder of the foreclosed properties.
numerous suits.
In 2002, petitioners filed a Motion to Admit Amended Complaint which impleaded as additional defendant the incumbent
Register of Deeds of Parañaque. Petitioners alleged that the Certificate of Sale was a falsified document since there was no
It comes from the old maxim nemo
actual sale that took place and even if an auction sale was conducted, the Certificate of Sale would still be void because the
debet bis vexari, pro una et eadem
auction sale was done in disobedience to a lawful order of the RTC. Petitioners also sought, in their Amended Complaint, the
causa (no man shall be twice issuance of a TRO or a writ of preliminary injunction to enjoin respondent Atty. Celestra and all other persons from proceeding
vexed for one and the same with the foreclosure sale, on the premise that no auction sale existed. The RTC denied petitioners’ application for injunction
cause). on the ground that the sale of the foreclosed properties rendered the same moot and academic. The auction sale, which was
conducted by respondents Metrobank and Atty. Celestra, after the expiration of the TRO was considered as proper and valid.
Petitioners filed a Motion for Reconsideration then with the CA a Petition for Certiorari. The SC dismissed the appeal of
respondents with finality. Thereafter, petitioners sought the inhibition of Acting Executive Judge of RTC-Branch 257 wherein
the case was re-raffled to RTC-Branch 258.
In 2005, petitioners filed with Branch 195 of RTC Paranaque a Verified Complaint for Damages against respondents
Metrobank, Atty. Celestra, and other three Metrobank lawyers. Petitioners sought in their Complaint the award of damages
against the respondents for making it appear that an auction sale of the subject properties took place, as a result of which, the
prospective buyers of the said properties lost their interest and petitioner Chua was prevented from realizing a profit of
₱70,000,000.00 from the intended sale.
Petitioners filed with RTC-Branch 195 a Motion to Consolidate the second case re: the action for damages pending before
said court, and the first case re: the injunction case that was being heard before RTC-Branch 258.The respondents filed with
RTC-Branch 195 an Opposition to Motion to Consolidate with Prayer for Sanctions, praying for the dismissal of the Complaint
for Damages of the second case on the ground of forum shopping.

Issue: Whether or not there is forum shopping

Ruling: YES. There is no dispute that petitioners failed to state in the Certificate of Non-Forum Shopping, attached to their
Verified Complaint in their case before RTC-Branch 195, the existence of case pending before RTC-Branch 258. Petitioners
committed forum shopping by filing multiple cases based on the same cause of action, although with different prayers. Forum
shopping occurs although the actions seem to be different when there is a splitting of a cause of action. Petitioners would like
to make it appear that first case was solely concerned with the nullification of the auction sale and certification of sale, while
the second case was a totally separate claim for damages. Yet, a review of the records reveals that petitioners also included
an explicit claim for damages in their Amended Complaint of the first case. There is no question that the claims of petitioners
for damages in both cases are premised on the same cause of action, i.e., the purportedly wrongful conduct of respondents in
connection with the foreclosure sale of the subject properties.

Petitioners’ contention that the outcome of the first case will not determine that of the second does not justify the filing of
separate cases. Even if it were assumed that the two cases contain two separate remedies that are both available to
petitioners, these two remedies that arose from one wrongful act cannot be pursued in two different cases.

11. ZARSONA Verification of a pleading is a formal, Facts: A complaint was filed against petitioner Zarsona Medical Clinic (ZMC) for violation of Section 149 of the Revised
MEDICAL V. PHIC [GR not jurisdictional, requirement Implementing Rules and Regulations of Republic Act No. 7875 or the National Health Insurance Act of 1995. Section 149
191225] 13 OCT 2014 intended to secure the assurance penalizes any health care provider that increases the period of actual confinement of any patient with revocation of
LEE that the matters alleged in a pleading accreditation. ZMC filed a claim with the Philippine Health Insurance Corporation (PhilHealth) on the confinement of National
are true and correct. Thus, the court Health Insurance Program (NHIP) member Lorna M. Alestre (Alestre). Said claim was denied on the ground of "extended
may simply order the correction of confinement." It was stated on the claim form that Alestre was admitted to ZMC on 6 August 2003 and was discharged on 12
unverified pleadings or act on them August 2003. It was also revealed in her Salaysay that Alestre's actual confinement at ZMC was on 10-11 August 2003.
and waive strict compliance with the Alestre, who is a teacher at Rizal Elementary School, was found to have reported for work on 12 August 2003. In defense of
rules. It is deemed substantially ZMC, Dr. Sylvia Bragat (Dr. Bragat), its Medical Director, stated that ZMC's Midwife/Clerk Jennifer R. Acuram (Acuram)
complied with when one who has committed an honest mistake when she wrote 6-12 August 2003 as the confinement period in the claim form. Dr. Bragat
ample knowledge to swear to the asserted that the hospital had in fact claimed only for two (2) days. So, ZMC was found liable for the charge of "Extending
truth of the allegations in the Period of Confinement" in violation of Section 149 of the Revised Rules and Regulations of Republic Act No. 7875 and was
complaint or petition signs the meted the penalty of suspension from participating in the NHIP for a period of three (3) months and a fine of P10,000.00.
verification, and when matters
alleged in the petition have been Health Insurance Arbiter: The Health Insurance Arbiter Michael Troy Polintan considered the admission date of 6 August
made in good faith or are true and 2003 reflected in Alestre's clinical record as a mere clerical error, he refused to believe Alestre's claim that she was
correct. As to certification against discharged only on 12 August 2003 but on that day, she was travelling back and forth from hospital to the school where she
forum shopping, non-compliance teaches. The PhilHealth Arbiter gave more evidentiary weight to the signature of Alestre in the school's attendance logbook
therewith or a defect therein, unlike which established the fact that she reported for work on 12 August 2003. ZMC appealed but the PhilHealth Board of Directors
in verification, is generally not (the Board) issued the PhilHealth Board Resolution dismissing such appeal and affirming the decision of the Arbiter.
curable by its subsequent
submission or correction thereof, Court of Appeals: ZMC filed a petition for review with the Court of Appeals putting in the forefront of its arguments Alestre's
unless there is a need to relax the Affidavit of Explanation. ZMC admitted to Alestre's recantation but in its defense, ZMC emphasized that the Affidavit, being
Rule on the ground of "substantial notarized and executed under oath, should weigh more than the Salaysay, which was not so. ZMC added that Alestre's
compliance" of presence of "special retraction rang true because she was willing to incriminate herself in exchange for telling the truth. So the CA had to rectify
circumstances or compelling the deficiencies in its petition and ZMC complied. However it was dismissed for failure on the part of ZMC to attach a valid
reasons." Rule 7, Section 5 of the SPA. The appellate court found the SPA defective on the ground that it does not explicitly authorize Dr. Bragat to sign and
Rules of the Court, requires that the execute the required verification and certification of non-forum shopping in this case. The appellate court noted that the
certification should be signed by the powers granted to Dr. Bragat pertain only to her administrative functions as Medical Director of ZMC. ZMC moved for
"petitioner principal party" himself. reconsideration but was denied.
The rationale behind this is "because
only the petitioner himself has actual Issue: Whether the CA erred in ruling that the Special power of Attorney (SPA) executed in favor of Dr. Sylvia P. Bragat was
knowledge of whether or not he has insufficient to cover the authority granted upon her to sign the verification and certification of non-forum shopping
initiated similar actions or
proceedings in different courts or Held: Yes. ZMC insists that the SPA, provided that the Attorney-in-fact can make, execute and sign any contract, documents
agencies." or all other writing of whatever kind and nature which are necessary to the power granted to it which is to represent, process,
follow-up, transact and facilitate claims in PhilHealth. This also covers the execution of verification and certification of non-
forum shopping. ZMC then asserts that it will not gain anything in extending the period of confinement and reiterates that its
clerk committed a mistake in entering the exact period of confinement.
Verification of a pleading is a formal, not jurisdictional, requirement intended to secure the assurance that the matters alleged
in a pleading are true and correct. Thus, the court may simply order the correction of unverified pleadings or act on them and
waive strict compliance with the rules. It is deemed substantially complied with when one who has ample knowledge to swear
to the truth of the allegations in the complaint or petition signs the verification, and when matters alleged in the petition have
been made in good faith or are true and correct. As to certification against forum shopping, non-compliance therewith or a
defect therein, unlike in verification, is generally not curable by its subsequent submission or correction thereof, unless there
is a need to relax the Rule on the ground of "substantial compliance" of presence of "special circumstances or compelling
reasons." Rule 7, Section 5 of the Rules of the Court, requires that the certification should be signed by the "petitioner
principal party" himself. The rationale behind this is "because only the petitioner himself has actual knowledge of whether or
not he has initiated similar actions or proceedings in different courts or agencies."
It was ruled in Lim v CA that the submission of an SPA authorizing an attorney-in-fact to sign the verification and certification
against forum-shopping in behalf of the principal party is considered as substantial compliance with the Rules. However, the
SPA granted by Dr. Zarsona to his attorneys-in-fact, Dr. Bragat and William Bragat was defective since there is no explicit
authorization for Dr. Bragat to sign and execute the requirement verification and certification in this case. At the very least, the
SPA should have granted the attorneys-in-fact the power and authority to institute civil and criminal actions which would
necessarily include the signing of the verification and certification against forum-shopping. However, despite the defects of the
SPA, we rule in favor of ZMC. ZMC should be given the opportunity to rectify the defects in the petition since it had in good
faith complied by submitting an SPA which it thought was sufficient and encompasses the filing of the instant suit. the rule
requiring a certification of forum shopping to accompany every initiatory pleading, or the verification for that matter "should not
be interpreted with such absolute literalness as to subvert its own ultimate and legitimate objective or the goal of all rules of
procedure — which is to achieve substantial justice as expeditiously as possible."

12. ANDERSON V. Non-compliance with the rules on the Facts: Anderson filed a Complaint for Ejectment against Ho before MeTC. She alleged that through her mere tolerance, Ho is
HO [GR 172590] 07 certification against forum shopping in possession of the Roosevelt property. As she was already in need of the said property, Anderson served upon Ho a
JAN 2013 or a defect therein is generally not Demand Letter to Vacate but despite receipt thereof, Ho refused. Because of this, Anderson prayed that the MeTC order Ho
LIMIN curable by its subsequent to vacate the Roosevelt property and pay her damages and attorney's fees.
submission or correction thereof,
unless there is a need to relax the The MeTC rendered a Decision dismissing the case for lack of cause of action. This is affirmed by the RTC. Intending to file
Rule on the ground of 'substantial with the CA a Petition for Review, Anderson's counsel, Atty. Oliva filed a Motion for Extension of Time of 15 days to file a
compliance' or presence of 'special petition. The motion was granted by the CA. Subsequently, said counsel sought another extension of 15 days. However, as of
circumstances or compelling the last day of the extended period, the petition has not yet been sent back, hence, the additional extension being sought. In
reasons'. Moreover, the certificate the interest of justice, the CA once again granted the said motion for extension. Atty. Oliva was finally able to file the Petition
must be executed by the party- for Review, but the certification against forum shopping attached thereto was signed by him on Anderson's behalf without any
pleader, not by his counsel. If, accompanying authority to do so. Hence, the CA dismissed the Petition. Anderson then filed a Motion for Reconsideration.
however, for reasonable or justifiable During its pendency, she also filed a Manifestation to which was attached an Affidavit and an SPA authorizing her counsel to
reasons, the party-pleader is unable cause the preparation and filing of the Petition for Review and to sign and execute the verification and certification against
to sign, he must execute a Special forum shopping on her behalf. She explained in the Affidavit that at the time the petition was filed, her health condition
Power of Attorney designating his hindered her from going to the proper authority to execute the necessary SPA so she just verbally instructed her lawyer to
counsel of record to sign on his draft the petition and cause the filing of the same. Nevertheless, upon learning of the dismissal of her case, she returned to
behalf. the Philippines from. the US., even against her doctor's advice and executed an SPA in favor of her counsel. The CA,
however, remained unswayed and denied the Motion for Reconsideration.

Issue: W/N the rules on certification against forum shopping should be relaxed in this case.

Held: No. The certificate of non-forum shopping has, time and again, been declared as basic, necessary and mandatory for
procedural orderliness. The requirement that it is the petitioner, not her counsel, who should sign the certificate of non-forum
shopping is due to the fact that “Obviously, it is the petitioner, and not always the counsel, whose professional services have
been retained for a particular case, who is in the best position to know whether [she] actually filed or caused the filing of a
petition in that case." However, if a petitioner is unable to sign a certification for reasonable or justifiable reasons, she must
execute an SPA designating her counsel of record to sign on her behalf.

While the Court notes that Anderson tried to correct this error by later submitting an SPA and by explaining her failure to
execute one prior to the filing of the petition, this does not automatically denote substantial compliance. It is true that in some
cases the Court considered such a belated submission as substantial compliance, but it "did so only on sufficient and
justifiable grounds that compelled a liberal approach while avoiding the effective negation of the intent of the rule on non-
forum shopping."

In the case of Donato v. CA, when the CA dismissed the Petition for Review on the ground, among others, that the
certification against forum shopping was signed by his counsel, petitioner submitted a certification duly signed by himself in
filing a motion for reconsideration. When this reached the SC, petitioner has drawn the Court's attention to the physical
impossibility of filing the Petition within the 15-day reglementary period to appeal considering that he is a resident of 1125
South Jefferson Street, Roanoke, Virginia, U.S.A. where he [needs] to personally accomplish and sign the verification.
Meanwhile, in this case, Anderson had a total of 45 days to comply with the requirements of a Petition for Review as against
the 15 days afforded to the petitioner in Donato. The 45 days is more than enough time for Anderson to execute an SPA
before the nearest Philippine Consulate, which again unlike in Donato, was located in the same state where Anderson was
(Hawaii), and thereafter to send it to the Philippines. Moreover, anent her allegation that her health condition at that time
hindered her from going to the proper authorities to execute an SPA, the same deserves scant consideration as no medical
certificate was submitted to support this. Lastly, simultaneous with the filing of a Motion for Reconsideration, the proper
certificate of non-forum shopping was submitted by the petitioner in Donato. Notably in this case, the SPA was submitted two
months after the filing of Anderson's Motion for Reconsideration.
13. VDA. DE FACTS: Records show that on October 14, 1989, Nellie Panelo Vda. De Formoso (Nellie) and her children namely: Ma.
FORMOSO V. PNB Verification is deemed substantially Theresa Formoso-Pescador, Roger Formoso, Mary Jane Formoso, Bernard Formoso, and Benjamin Formoso,
[GR154704] 01 JUN complied with when one who has executed a special power of attorney in favor of Primitivo Malcaba (Malcaba) authorizing him, among others, to secure
2011 ample knowledge to swear to the all papers and documents including the owner’s copies of the titles of real properties pertaining to the loan with real
NAKAGAWA truth of the allegations in the estate mortgage originally secured by Nellie and her late husband, Benjamin S. Formoso, from Philippine National
complaint or petition signs the Bank, Vigan Branch (PNB) on September 4, 1980.
verification, and when matters
alleged in the petition have been On April 20, 1990, the Formosos sold the subject mortgaged real properties to Malcaba through a Deed of Absolute
made in good faith or are true and Sale. Subsequently, on March 22, 1994, Malcaba and his lawyer went to PNB to fully pay the loan obligation including
correct. interests in the amount of ₱2,461,024.74.

As to certification against forum PNB, however, allegedly refused to accept Malcaba’s tender of payment and to release the mortgage or surrender the
shopping, non-compliance titles of the subject mortgaged real properties.
therewith or a defect therein, unlike
in verification, is generally not On March 24, 1994, the petitioners filed a Complaint for Specific Performance against PNB before the Regional Trial
curable by its subsequent Court of Vigan, Ilocos Sur (RTC) praying, among others, that PNB be ordered to accept the amount of ₱2,461,024.74
submission or correction thereof, as full settlement of the loan obligation of the Formosos.
unless there is a need to relax the
Rule on the ground of "substantial After an exchange of several pleadings, the RTC finally rendered its decision on October 27, 1999 favoring the
compliance" or presence of petitioners. The petitioners’ prayer for exemplary or corrective damages, attorney’s fees, and annual interest and daily
"special circumstances or interest, however, were denied for lack of evidence.
compelling reasons.
PNB filed a motion for reconsideration but it was denied for failure to comply with Rule 15, Section 5 of the 1997 Rules
of Civil Procedure. PNB then filed a Notice of Appeal but it was dismissed for being filed out of time.

The petitioners received their copy of the decision on November 26, 1999, and on January 25, 2001, they filed their
Petition for Relief from Judgment questioning the RTC decision that there was no testimonial evidence presented to
warrant the award for moral and exemplary damages. They reasoned out that they could not then file a motion for
reconsideration because they could not get hold of a copy of the transcripts of stenographic notes. In its August 6, 2001
Order, the RTC denied the petition for lack of merit.

On September 7, 2001, the petitioners moved for reconsideration but it was denied by the RTC in its Omnibus Order of
September 26, 2001.

ISSUE: WON THE COURT OF APPEALS PATENTLY ERRED IN RULING THAT ALL THE PETITIONERS MUST
SIGN THE VERIFICATION AND CERTIFICATION OF NON-FORUM SHOPPING IN A PETITION FOR CERTIORARI
WHEREIN ONLY QUESTIONS OF LAW ARE INVOLVED

HELD: NO. The Court disagrees.

Sections 4 and 5 of Rule 7 of the 1997 Rules of Civil Procedure provide:

SEC. 4. Verification. – Except when otherwise specifically required by law or rule, pleadings need not be under oath,
verified or accompanied by affidavit.

A pleading is verified by an affidavit that the affiant has read the pleadings and that the allegations therein are true and
correct of his personal knowledge or based on authentic records.

A pleading required to be verified which contains a verification based on "information and belief" or upon "knowledge,
information and belief" or lacks a proper verification, shall be treated as an unsigned pleading.

SEC. 5. Certification against forum shopping. – The plaintiff or principal party shall certify under oath in the complaint or
other initiatory pleading asserting a claim for relief, or in a sworn certification annexed thereto and simultaneously filed
therewith: (a) that he has not theretofore commenced any action or filed any claim involving the same issues in any
court, tribunal or quasi-judicial agency and, to the best of his knowledge, no such other action or claim is pending
therein; (b) if there is such other pending action or claim, a complete statement of the present status thereof; and (c) if
he should thereafter learn that the same or similar action or claim has been filed or is pending, he shall report that fact
within five (5) days therefrom to the court wherein his aforesaid complaint or initiatory pleading has been filed.

Failure to comply with the foregoing requirements shall not be curable by mere amendment of the complaint or other
initiatory pleading but shall be cause for the dismissal of the case without prejudice, unless otherwise provided, upon
motion and after hearing. The submission of a false certification or non-compliance with any of the undertakings therein
shall constitute indirect contempt of court, without prejudice to the corresponding administrative and criminal actions. If
the acts of the party or his counsel clearly constitute willful and deliberate forum shopping, the same shall be ground for
summary dismissal with prejudice and shall constitute direct contempt, as well as a cause for administrative sanctions.
x x x.

In this regard, the case of Oldarico S. Traveno v. Bobongon Banana Growers Multi-Purpose Cooperative, is
enlightening:

Respecting the appellate court’s dismissal of petitioners’ appeal due to the failure of some of them to sign the therein
accompanying verification and certification against forum-shopping, the Court’s guidelines for the bench and bar in
Altres v. Empleo, which were culled "from jurisprudential pronouncements," are instructive:

For the guidance of the bench and bar, the Court restates in capsule form the jurisprudential pronouncements already
reflected above respecting non-compliance with the requirements on, or submission of defective, verification and
certification against forum shopping:

1) A distinction must be made between non-compliance with the requirement on or submission of defective verification,
and non-compliance with the requirement on or submission of defective certification against forum shopping.

2) As to verification, non-compliance therewith or a defect therein does not necessarily render the pleading fatally
defective. The Court may order its submission or correction or act on the pleading if the attending circumstances are
such that strict compliance with the Rule may be dispensed with in order that the ends of justice may be served thereby.

3) Verification is deemed substantially complied with when one who has ample knowledge to swear to the truth of the
allegations in the complaint or petition signs the verification, and when matters alleged in the petition have been made
in good faith or are true and correct.

4) As to certification against forum shopping, non-compliance therewith or a defect therein, unlike in verification, is
generally not curable by its subsequent submission or correction thereof, unless there is a need to relax the Rule on the
ground of "substantial compliance" or presence of "special circumstances or compelling reasons."

5) The certification against forum shopping must be signed by all the plaintiffs or petitioners in a case; otherwise, those
who did not sign will be dropped as parties to the case. Under reasonable or justifiable circumstances, however, as
when all the plaintiffs or petitioners share a common interest and invoke a common cause of action or defense, the
signature of only one of them in the certification against forum shopping substantially complies with the Rule.

6) Finally, the certification against forum shopping must be executed by the party-pleader, not by his counsel. If,
however, for reasonable or justifiable reasons, the party-pleader is unable to sign, he must execute a Special Power of
Attorney designating his counsel of record to sign on his behalf.

In Docena v. Lapesura, we ruled that the certificate of non-forum shopping should be signed by all the petitioners or
plaintiffs in a case, and that the signing by only one of them is insufficient. The attestation on non-forum shopping
requires personal knowledge by the party executing the same, and the lone signing petitioner cannot be presumed to
have personal knowledge of the filing or non-filing by his co-petitioners of any action or claim the same as similar to the
current petition.

The certification against forum shopping in CA-G.R. SP No. 72284 is fatally defective, not having been duly signed by
both petitioners and thus warrants the dismissal of the petition for certiorari. We have consistently held that the
certification against forum shopping must be signed by the principal parties. With respect to a corporation, the
certification against forum shopping may be signed for and on its behalf, by a specifically authorized lawyer who has
personal knowledge of the facts required to be disclosed in such document.

14. METROBANK
V. SANTOS [GR A petition for a writ of possession is Facts:
157867] 15 DEC 2009 neither a complaint nor an initiatory
NATO pleading, a certificate against non- Respondent Manfred Jacob De Koning (De Koning) obtained a loan from Metrobank to secure the payment of this loan, De
forum shopping is not required. The Koning executed a real estate mortgage (REM) in favor of Metrobank over a condominium unit and all its improvements.
certificate that Metrobank attached to
its petition is thus a superfluity that When De Koning failed to pay his loan despite demand, Metrobank instituted extrajudicial foreclosure proceedings against the
the lower court should have REM. Metrobank was the highest bidder at the public auction of the condominium unit and a Certificate of Sale was issued in
disregarded. the bank's favor.

Metrobank duly registered this Certificate of Sale with the Registry of Deeds. The redemption period lapsed without De
Koning redeeming the property. Thus, Metrobank demanded that he turn over possession of the condominium unit.

WRIT OF POSSESSION

When De Koning refused, Metrobank filed with the RTC Makati, Branch 65, an ex parte petition for a writ of possession over
the foreclosed property the lower court issued an order setting the ex parte hearing of Metrobank's petition

MTD

During the scheduled ex parte hearing De Koning's counsel appeared and manifested that he filed a motion to dismiss on the
ground that Metrobank's petition violated Section 5, Rule 7 of the Rules of Court (Rules) which requires the attachment of a
certification against forum shopping to a complaint or other initiatory pleading.

Metrobank's petition for the issuance of a writ of possession involved the same parties, the same issues and the same subject
matter as the case he had filed with the RTC of Makati, to question Metrobank's right to foreclose the mortgage.

De Koning also had a pending petition for certiorari with the CA, which arose from the RTC case he filed.

RTC: The RTC agreed with De Koning and dismissed Metrobank's petition.

CA: The CA affirmed the dismissal of Metrobank's petition.

Issues:Whether or not, an ex parte petition for the issuance of a writ of possession is not an initiatory pleading asserting a
claim.

Ruling:

YES, an ex parte petition for the issuance of a writ of possession is not an initiatory pleading asserting a claim.

A writ of possession is defined as "a writ of execution employed to enforce a judgment to recover the possession of land. It
commands the sheriff to enter the land and give its possession to the person entitled under the judgment."

There are three instances when a writ of possession may be issued:

(a) in land registration proceedings

(b) in judicial foreclosure, provided the debtor is in possession of the mortgaged realty and no third person, not a party to the
foreclosure suit and

(c) in extrajudicial foreclosure of a real estate mortgage

In order to obtain a writ of possession, the purchaser in a foreclosure sale must file a petition, in the form of an ex parte
motion, in the registration or cadastral proceedings of the registered property. The certification against forum shopping is
required only in a complaint or other initiatory pleading. The ex parte petition for the issuance of a writ of possession filed by
the respondent is not an initiatory pleading.

Although the private respondent denominated its pleading as a petition, it is, nonetheless, a motion. The office of a motion is
not to initiate new litigation, but to bring a material but incidental matter arising in the progress of the case in which the motion
is filed. An application for a writ of possession is a mere incident in the registration proceeding. Hence, although it was
denominated as a "petition," it was in substance merely a motion.

The right to possess a property merely follows the right of ownership. Thus, after the consolidation of title in the buyer's name
for failure of the mortgagor to redeem, the writ of possession becomes a matter of right and its issuance to a purchaser in an
extrajudicial foreclosure is merely a ministerial function.

Since a petition for a writ of possession is neither a complaint nor an initiatory pleading, a certificate against non-forum
shopping is not required. The certificate that Metrobank attached to its petition is thus a superfluity that the lower court should
have disregarded.

15. ARGALLON- The lack of a certification against FACTS


JOCSON AND TUSING forum shopping or a defective
V. CA [GR 149660] 20 certification is generally not curable Petitioner Ceferina Argallon-Jocson (Jocson) filed a complaint for Reconveyance and Damages against Marcelo Steel
JAN 2009 by its subsequent submission or Corporation and Maria Cristina Fertilizer Corporation (MCFC), which were represented by Jose Marcelo as president of both
PEREZ correction, unless there is a need to companies.
relax the rule under special On 24 February 1999, the trial court rendered a decision in favor of the plaintiff [Jocson] and against the defendants [Marcelo
circumstances or for compelling Steel Corporation and MCFC].
reasons. We find no compelling
reason for a liberal application of the Marcelo Steel Corporation and MCFC (private respondents) appealed to the Court of Appeals, which affirmed the trial court’s
rules especially in this case where decision. Private respondents did not appeal the Court of Appeals’ decision, which became final and executory. Jocson then
the petitioner who did not sign the filed a Motion for Issuance of a Writ of Execution. On 9 December 2002, the trial court issued an order for the issuance of a
verification and certification for non- writ of execution in accordance with the tenor of the decision.
forum shopping already filed with the
trial court a Motion for Issuance of
Alias Writ of Execution. On 20 December 2002, a Writ of Execution was issued to the Sheriff of the Office of the Clerk of Court of Manila,
The certification of non-forum commanding the Sheriff to implement the writ upon private respondents in accordance with the tenor of the decision. The
shopping is rooted in the principle Sheriffs levied upon the properties of Marcelo Steel Corporation in full satisfaction of the judgment debt. The execution sale
that a party-litigant should not be was then scheduled on 17 February 2003. On 14 February 2003, Midas International Development Corporation (Midas Corp.)
allowed to pursue simultaneous filed a third-party claim, alleging that some of the levied properties were previously mortgaged to Midas Corp. The execution
remedies in different fora, such act sale was postponed to 21 February 2003. On 20 February 2003, Jocson posted an indemnity bond so that the levied
being detrimental to an orderly properties would not be released to claimant Midas Corp.
judicial procedure. The Sheriffs then proceeded with the execution sale on 21 February 2003 and sold the properties of Marcelo Steel
Corporation for the full satisfaction of the judgment against private respondents. A certificate of sale was issued to petitioner
Rodolfo Tuising (Tuising), who was the highest bidder at the auction sale.

On 28 February 2003, Jocson filed with the trial court a Very Urgent Ex-Parte Motion for Issuance of a Break-Open Order and
Petition for Contempt of Court. On 3 March 2003, Marcelo Steel Corporation filed an Extremely Urgent Omnibus Motion,
praying for the annulment of the execution sale and for the issuance of an order directing the Sheriffs not to deliver the
properties sold to Tuising pending resolution of Marcelo Steel Corporation’s motion. Marcelo Steel Corporation alleged that its
obligation was merely joint with MCFC and that the total price of the properties sold on execution was unconscionably
inadequate.

On 14 April 2003, the trial court issued an order declaring null and void and the Certificate of Sale dated February 21, 2003
issued pursuant thereto is hereby set aside and cancelled.
The motion for the issuance of a break-open order is hereby denied for lack of merit and basis.

Jocson moved for reconsideration of the trial court’s order, claiming that the nature of the obligation to pay the balance of the
purchase price was solidary. Tuising filed a Motion for Intervention with Leave of Court with Motion for Reconsideration and
Entry of Appearance. On the other hand, Marcelo Steel Corporation filed, on 7 May 2003, a Manifestation and Motion on
Satisfaction of Judgment, depositing with the trial court a Manager’s Check in the amount of ₱4,260,198.11 representing full
satisfaction of Marcelo Steel Corporation’s obligation to Jocson. On 14 July 2003, the trial court denied Jocson’s motion for
reconsideration and Tuising’s motion for intervention and reconsideration, and granted Marcelo Steel Corporation’s prayer for
entry of satisfaction of judgment on its behalf.
On 18 August 2003, Jocson filed with the trial court a Notice of Appeal, which she later withdrew on 4 September 2003, and in
lieu thereof, petitioners Jocson and Tuising filed a Petition for Certiorari with the Court of Appeals. The Court of Appeals
dismissed the petition for lack of merit.

Jocson and Tuising filed a motion for reconsideration, which the Court of Appeals denied on 25 March 2004. Hence, this
petition.
Meanwhile, on 23 February 2004, Jocson filed with the trial court a Motion for Issuance of Alias Writ of Execution to
implement the decision as against MCFC, stating that in view of the Court of Appeals’ decision, there is a need to execute the
decision as against the other defendant MCFC.
The Trial Court’s Ruling
The trial court ruled that the liability of Marcelo Steel Corporation was limited to its proportional share in the entire money
judgment. Considering that the dispositive portion of the Decision dated 24 February 1999 in this case did not state that the
obligation of private respondents was solidary, then their obligation was merely joint.
The Court of Appeals’ Ruling
The Court of Appeals held that in consonance with Section 1, Rule 65 of the Rules of Civil Procedure, certiorari is not a
substitute for lost appeal. Moreover, the Court of Appeals found that the assigned issues were factual issues not proper in a
petition for certiorari, which is limited to the issues of jurisdiction and grave abuse of discretion.
The Court of Appeals found no grave abuse of discretion on the part of the respondent judge. On the merits of the case, the
Court of Appeals held that the obligation of private respondents to Jocson was merely joint. The Court of Appeals noted that
the trial court’s Decision dated 24 February 1999 was silent as to the nature of the liability. Solidary obligations are not
presumed in the absence of an express determination thereof in the judgment. When the judgment does not provide that the
defendants are liable to pay jointly and severally a certain amount of money, none of them may be compelled to satisfy in full
said judgment.
The Court of Appeals found that the Sheriffs disregarded the trial court’s 24 February 1999 Decision, and deviated from the
trial court’s Order dated 9 December 2002 and the writ of execution dated 20 December 2002, which directed them to
execute the writ in accordance with the tenor of the decision.

ISSUE:
Whether or not the Court of Appeals erred in dismissing the case.

HELD:
NO. We find the petition without merit.
At the outset, the Court notes that the petition supposedly filed by petitioners Jocson and Tuising was not signed by Jocson’s
counsel. It was Tuising’s counsel who signed in behalf of Jocson’s counsel. Tuising’s counsel had no authority to sign the
petition in behalf of Jocson. The records are bereft of any proof that Jocson ever authorized Tuising’s counsel to be her
counsel or to act in her behalf. Under Section 3, Rule 7 of the Rules of Civil Procedure, every pleading must be signed by the
party or counsel representing him, otherwise the pleading produces no legal effect.
Furthermore, only Tuising signed the Verification and Certification for Non-Forum Shopping. Jocson did not sign the
Verification and Certification. Section 1, Rule 45 of the Rules of Civil Procedure requires the petition for review on certiorari to
be verified. A pleading required to be verified which lacks proper verification shall be treated as an unsigned pleading.
Although Tuising belatedly filed on 24 September 2004 a "Special Power of Attorney" allegedly signed by Jocson and
authorizing Tuising to file the petition for review and to verify and to certify the petition, no explanation was given by Tuising
why the Special Power of Attorney was belatedly filed four months after the petition for review was filed on 12 May 2004. The
lack of a certification against forum shopping or a defective certification is generally not curable by its subsequent submission
or correction, unless there is a need to relax the rule under special circumstances or for compelling reasons. We find no
compelling reason for a liberal application of the rules especially in this case where the petitioner who did not sign the
verification and certification for non-forum shopping already filed with the trial court a Motion for Issuance of Alias Writ of
Execution. By filing the Motion for Issuance of Alias Writ of Execution, Jocson was in effect abiding by the Court of Appeals’
Decision dated 16 January 2004.
In Athena Computers, Inc. v. Reyes, the Court held that the appellate court was correct in dismissing the petition where the
verification and certification for non-forum shopping were signed by only one of the two petitioners. The Court held:
The verification of the petition and certification on non-forum shopping before the Court of Appeals were signed only by
Jimenez. There is no showing that he was authorized to sign the same by Athena, his co-petitioner.

Section 4, Rule 7 of the Rules states that a pleading is verified by an affidavit that the affiant has read the pleading and that
the allegations therein are true and correct to his knowledge and belief. Consequently, the verification should have been
signed not only by Jimenez but also by Athena’s duly authorized representative.

In Docena v. Lapesura, we ruled that the certificate of non-forum shopping should be signed by all the petitioners or plaintiffs
in a case, and that the signing by only one of them is insufficient. The attestation on non-forum shopping requires personal
knowledge by the party executing the same, and the lone signing petitioner cannot be presumed to have personal knowledge
of the filing or non-filing by his co-petitioners of any action or claim the same as similar to the current petition.

In this case, the flaw is fatal considering that Jocson, the co-petitioner who did not sign the verification and certification of non-
forum shopping and whose counsel did not sign the petition, was the principal party in the original case. Jocson was the
plaintiff in the trial court who sought reconveyance of her properties while her co-petitioner Tuising was not a party in the
original case but was merely the highest bidder in the execution sale which was declared void by the trial court.
The certification of non-forum shopping is rooted in the principle that a party-litigant should not be allowed to pursue
simultaneous remedies in different fora, such act being detrimental to an orderly judicial procedure. The petition, signed only
by Tuising’s counsel, conveniently failed to mention the fact that on 23 February 2004, prior to the filing of the petition, Jocson
already filed with the trial court a Motion for Issuance of Alias Writ of Execution.
Clearly, such an action is incompatible with this petition for review. Even at the appellate court’s level, the Motion for
Reconsideration supposedly filed by petitioners Jocson and Tuising on 3 February 2004 was also signed by Tuising’s counsel
only. Jocson’s filing of a Motion for Issuance of Alias Writ of Execution to implement the decision as against MCFC clearly
indicates that she already acceded to the Court of Appeals’ Decision dated 16 January 2004 and no longer intended to move
for its reconsideration, much less appeal to this Court. Besides, a party should not be allowed to abuse and make a mockery
of the judicial process by pursuing simultaneous and incompatible remedies in different courts.
WHEREFORE, we DENY the petition. We AFFIRM the Decision dated 16 January 2004 and the Resolution dated 25 March
2004 of the Court of Appeals in CA-G.R. SP No. 79179.
SO ORDERED.
16. MARANAW The certificate of non-forum Facts:
HOTEL AND RESORT shopping is a mandatory Private respondent Sheryl Oabel was hired as an extra beverage attendant from April 24, 1995 to February 7,1997 in Century
CORP. V. CA [GR requirement. Substantial compliance Park Hotel which was owned by the petitioner (Maranaw Hotels and Resort Corp). On September 16, 1996, petitioner
149660] 20 JAN 2009 applies only with respect to the contracted with Manila Resource Development Corporation. Subsequently, private respondent Oabel was transferred to
PURIFICACION contents of the certificate but not as MANRED. On July 20, 1998, Oabel filed before the Labor Arbiter a petition for regularization. However, on August 1, 1998,
to its presence in the pleading Oabel was dismissed from employment. Respondent converted her petition for regularization into a complaint for illegal
wherein it is required. dismissal.

Labor Arbiter Madjayran H. Ajan - dismissed the complaint against the petitioner.

Oabel - appealed before the National Labor Relations Commission (NLRC)

NLRC - reversed the ruling of the Labor Arbiter and held that: (1) MANRED is a labor-only contractor, and (2) private
respondent was illegally dismissed.

CA - dismissed the petition on account of the failure of the petitioner to append the board resolution authorizing the
counsel for petitioner to file the petition before the Court of Appeals.

Petitioner - motion for reconsideration was denied.

Petitioner - invokes substantial justice as justification for a reversal of the resolution of the Court of Appeals.
 contends that the filing of a motion for reconsideration with the certificate of non-forum shopping attached
constitutes substantial compliance with the requirement.

Issue: Whether or not the petitioner is correct in the claim that the filing of a motion for reconsideration with an appended
certificate of non forum-shopping suffices to cure the defect in the pleading. - NO.

Held:
Well-settled is the rule that the certificate of non-forum shopping is a mandatory requirement. Substantial compliance
applies only with respect to the contents of the certificate but not as to its presence in the pleading wherein it is
required.

Petitioner's contention that the filing of a motion for reconsideration with an appended certificate of non forum-shopping
suffices to cure the defect in the pleading is absolutely specious. It negates the very purpose for which the certification against
forum shopping is required: to inform the Court of the pendency of any other case which may present similar issues and
involve similar parties as the one before it. The requirement applies to both natural and juridical persons.

This Court has not wavered in stressing the need for strict adherence to procedural requirements. The rules of procedure
exist to ensure the orderly administration of justice. They are not to be trifled with lightly.

For this reason alone, the petition must already be dismissed. However, even if this grave procedural infirmity is set aside, the
petition must still fail. In the interest of averting further litigation arising from the present controversy, and in light of the
respective positions asserted by the parties in the pleadings and other memoranda filed before this Court, the Court now
proceeds to resolve the case on the merits.

17. CAGAYAN A pleading is verified by an affidavit FACTS: Petitioner Cagayan Valley is a corporation duly organized retailer of medicine and other pharmaceutical products. In
VALLEY V. that the affiant has read the pleading 1995 Cagayan Valley alleged it granted 20% sales discounts to qualified senior citizens on purchases of medicine.
COMMISSIONER OF and that the allegations therein are
INTERNAL REVENUE true and correct of his knowledge Cagayan Valley treated the 20% sales discounts granted to qualified senior citizens in 1995 as deductions from the gross
[GR 151413] 13 FEB and belief. sales in order to arrive at the net sales, instead of treating them as tax credit. In 1996, Cagayan Valley filed with the BIR a
2008 claim for tax refund/tax credit of the full amount of the 20% sales discount it granted to senior citizens in 1995.
SABIO
BIR’s inaction prompted Cagayan Valley to file a petition before the CTA. The CTA dismissed Cagayan’s petition for lack of
merit. Cagayan Valley appealed to CA but the petition was dismissed on procedural grounds. The CA held that the person
who signed the verification and certification of absence of forum shopping, a certain Concepcion, President of Cagayan
Valley, failed to adduce proof that he was duly authorized by the board of directors to do so. The CA found no sufficient proof
to show that Concepcion was duly authorized by the Board of Directors of Cagayan Valley.

ISSUE: Whether or not the verification and certification of non-forum shopping signed by the President of Cagayan Valley is
sufficient compliance with Secs. 4 and 5, Rule 7 of the Rules of Civil Procedure.

HELD: YES. There is substantial compliance with Secs. 4 and 5, Rule 7 of the Rules of Civil Procedure. A pleading is verified
by an affidavit that the affiant has read the pleading and that the allegations therein are true and correct of his knowledge and
belief.

The SC found that Cagayan Valley’s President has substantially complied with such. First, the requisite board resolution has
been submitted albeit belatedly by Cagayan Valley. Second, in the ruling of Lepanto with the rationale that the President of
Cagayan Valley is in a position to verify the truthfulness and correctness of the allegations in the petition. Third, the President
of Cagayan Valley has signed the complaint. before the CTA at the inception of this judicial claim for refund or tax credit.

18. FUENTEBELLA the petitioner or the principal party FACTS: Respondent Darlica Castro (widow) of the late Freddie Castro who died on September 18, 1997 in Bacolod City.
V. CASTRO [GR must execute the certification against Respondent engaged the funeral services of petitioner Rolling Hills Memorial Park, Inc. in Bacolod City for the interment of
150865] 30 JUN 2006 forum shopping. The reason for this the remains of her husband on September 27, 1997 at 3PM.
SOMEROS is that the principal party has actual During the burial, when the casket were about to be lowered into the vault, it was discovered that the casket does not fit on it.
knowledge whether a petition has while they were preparing for the casket it was measured by using a spade and it was placed under the heat of the sun for an
previously been filed involving the hour in front of all the mourners.. The Insulted respondent, demanding an explanation for its negligence but was no avail.
same case or substantially the same Consequently, respondent filed a COMPLAINT FOR DAMAGES against the corporation and its Park-in-Charge Art
issues. If, for any reason, the Fuentebella, jointly and solidarily, BEFORE THE MTCC of Bacolod City.
principal party cannot sign the Petitioners filed a motion to dismiss on the ground that the MTCC has no jurisdiction because the damages claimed is more
petition, the one signing on his behalf than P200,000. Respondent subsequently filed a motion to withdraw the complaint, which was granted by the MTCC.
must have been duly authorized. On April 15, 1999, respondent filed a SIMILAR COMPLAINT WITH THE RTC of Negros Occidental. Attached in the
complaint was the Verification and Certification against Forum Shopping required under Section 5, Rule 7 of ROC
Petitioners filed a motion to dismiss on the ground that the certification is false because the respondent had previously filed
an identical complaint with the MTCC but was denied.
Petitioners filed a motion for reconsideration and argued that the submission by the latter is a false certification but it was
denied again.
Petitioners filed with CA a petition for certiorari with preliminary injunction and/or restraining order but was dismissed because
the certification was signed by a certain Lourdes Pomperada without any showing or indication that she is duly authorized by
the petitioners to sign for and in their behalf.
A motion for reconsideration of the above resolution was filed by petitioner Rolling Hills Memorial Park, Inc. attaching thereto
a Secretary's Certificate signed by Monico A. Puentevella, Jr., Corporate Secretary of petitioner corporation, affirming therein
the authority of Lourdes A. Pomperada to file the aforementioned petition but it was denied on the ground that there is still no
showing that the said Mrs. Lourdes Pomperada is duly authorized to act for and in behalf of the other petitioner.
ISSUES:
(1) WON CA acted erroneously in dismissing outright the petition for certiorari on the basis of a non-existent rule;
and
(2) WON trial court committed grave abuse of discretion when it did not dismiss the petition due to FALSE
CERTIFICATION.
HELD:
(1), NO, under Rule 7, Section 5 of the 1997 Revised Rules on Civil Procedure it is obligatory that the one signing the
verification and certification against forum shopping on behalf of the principal party or the other petitioners has the authority to
do the same.
The provision mandates that the petitioner or the principal party must execute the certification against forum shopping. The
reason for this is that the principal party has actual knowledge whether a petition has previously been filed involving the same
case or substantially the same issues. If, for any reason, the principal party cannot sign the petition, the one signing on his
behalf must have been duly authorized.
This requirement is intended to apply to both natural and juridical persons. Where the petitioner is a corporation, the
certification against forum shopping should be signed by its duly authorized director or representative.
Likewise, where there are several petitioners, it is insufficient that only one of them executes the certification, absent a
showing that he was so authorized by the others. That certification requires personal knowledge and it cannot be presumed
that the signatory knew that his co-petitioners had the same or similar actions filed or pending.
Hence, a certification which had been signed without the proper authorization is defective and constitutes a valid cause for the
dismissal of the petition.
In the case above, Lourdes Pomperada, the Administrative Manager of petitioner corporation, who signed the verification and
certificate on non-forum shopping, initially failed to submit a secretary's certificate or a board resolution confirming her
authority to sign for the corporation, and a special power of attorney to sign on behalf of co-petitioner Art Fuentebella, who
was sued jointly and solidarily with the corporation in his capacity as officer of the latter.
(2) NO. An omission in the certificate of non-forum shopping about any event that would not constitute res judicata and litis
pendentia, as in the present case, is not fatal as to merit the dismissal and nullification of the entire proceedings considering
that the evils sought to be prevented by the said certificate are not present.
Hence, in any event, the trial court correctly held that the submission of a false certification shall constitute indirect contempt
of court, without prejudice to the corresponding administrative and criminal sanctions. This is in accordance with Section 5,
Rule 7 of ROC

19. SAMEER FACTS: December 5, 1995 : private respondents Lord Nelson Santos, Danilo Balcita,Nicson Cruz, Pepito Manglicmot, and
OVERSEAS An unsigned pleading produces Allan Aranes (Santos, et al.) were recruited by petitioner Sameer Overseas Placement Agency, Inc. (Sameer) as aluminum
PLACEMENT AGENCY, products manufacturer operators for Ensure Company Ltd. of Taiwan (Ensure), under a one-year employment contract with a
INC. V. SANTOS [GR no legal effect. basic monthly salary of NT$14,800.00.
152579]
TAN The rule allows the pleadings to Santos, et al. were deployed and were able to work for Ensure. However, they were repatriated even prior to the expiration of
be signed by either the party to their contracts. Consequently, in July and August 1996, Santos, et al. �led complaints against Sameer before the National
the case or the counsel Labor Relations Commission (NLRC) for illegal dismissal, underpayment of salaries, and unauthorized salary deductions.
representing that party
On November 3, 1997, Sameer filed a third party complaint against private respondent ASBT International Management
Section 3, Rule 7 of the Rules of Civil Service, Inc. (ASBT). It claimed that the latter should be liable for all the contractual obligations of Ensure since Sameer's
Procedure provides — accreditation was transferred to ASBT on June 9, 1997.

SEC. 3. Signature and address. — Labor Arbiter: ordered Sameer to pay each respondent their due wages.
Every pleading must be signed by
the party or counsel representing Sameer appealed to NLRC: the appealed decision is hereby SET ASIDE and a new one entered absolving SAMEER
him, stating in either case his Overseas Placement Agency,Inc. from its liabilities in view of the transfer of accreditation to ASBT Management
address which should not be a Services, Inc. and ordering the latter to pay the respondents.
post office box. The signature of
counsel constitutes a certificate by Court of Appeals: ASBT filed motion for reconsideration but was denied by NLRC they then filed a petition for certiorari with
him that he has read the pleading; the CA, the Court of Appeals denied due course and dismissed ASBT's petition on the ground that the attached
that to the best of his knowledge, Verification and Certification of Non-Forum Shopping was signed by Mildred R. Santos as President of ASBT without
information, and belief there is good any proof of authority to sign for and bind ASBT in the proceedings.
ground to support it; and that it is not
interposed for delay. ASBT filed a motion for reconsideration of the June 19, 2001 Resolution, submitting therewith the necessary board resolution
authorizing corporate president Mildred R. Santos to represent ASBT before the Court of Appeals. The appellate court
granted the motion and reinstated the petition. (CA ruled in favor of ASBT)

ISSUE:

W/N both the Petition and the Motion for Reconsideration filed by ASBT and signed by Mildred Santos, as corporate
president, should be considered unsigned pleadings.

HELD: NO.

Section 3, Rule 7 of the Rules of Civil Procedure provides —

SEC. 3. Signature and address. — Every pleading must be signed by the party or counsel representing him, stating in
either case his address which should not be a post office box. The signature of counsel constitutes a certificate by him
that he has read the pleading; that to the best of his knowledge, information, and belief there is good ground to support it; and
that it is not interposed for delay.

An unsigned pleading produces no legal effect. However, the court may, in its discretion, allow such deficiency to be
remedied if it shall appear that the same was due to mere inadvertence and not intended for delay.

Obviously, the rule allows the pleadings to be signed by either the party to the case or the counsel representing that
party. In this case, ASBT, as petitioner, opted to sign its petition and its motion for reconsideration in its own behalf,
through its corporate president, Mildred R. Santos, who was duly authorized by ASBT's Board of Directors to
represent the company in prosecuting this case. Therefore, the said pleadings cannot be considered unsigned and
without any legal effect.

MANNER OF MAKING ALLEGATIONS IN PLEADINGS (RULE 8, SECS. 1 TO 13)


20. ELIZA ZUNIGA- The petitioner’s cause of action
SANTOS V. SANTOS- should not merely be "stated" but, Facts:
GRAN [GR 197380] 08 importantly, the statement thereof
OCT 2014 should be "sufficient." This is why Petitioner, through her authorized representative, filed a complaint for annulment of sale and revocation of title against
TANADA the elementary test in a motion to respondents and the Register of Deeds, Marikina City. In her amended complaint, petitioner alleged, among others, that the
dismiss on such ground is whether or sale was made pursuant to void and voidable documents – a void deed of sale which petitioner cannot locate despite diligent
not the complaint alleges facts which efforts. Respondent filed a motion to dismiss contending that the actions has prescribed and that the amended complaint
if true would justify the relief failed to state a cause of action as the void and voidable documents sought to be nullified were not properly identified nor the
demanded. It has been held that only substance thereof set forth. The RTC dismissed the amended complaint for its failure to state a cause of action, considering
ultimate facts and not legal that the deed of sale sought to be nullified was not attached. It likewise held that the certificates of title covering the subject
conclusions or evidentiary facts are properties cannot be collaterally attacked and that since the action was based on a written contract, the same had already
considered for purposes of applying prescribed under Art. 1144 of the Civil Code. On appeal, the CA dismissed petitioner’s amended complaint but on the ground
the test. This is consistent with of insufficiency of factual basis. It held, among others, that since the Deed of Sale sought to be annulled was not attached to
Section 1, Rule 8 of the Rules of the Amended Complaint, it was impossible for the court to determine whether petitioner's signature therein was a forgery
Court which states that the and thus, would have no basis to order the surrender or reconveyance of the subject properties. Petitioner moved for
complaint need only allege the reconsideration and attached for the first time, a copy of the questioned deed of sale which she claimed to have recently been
ultimate facts or the essential facts recovered. The CA denied the MR.
constituting the plaintiff's cause of
action. A fact is essential if they Issue:
cannot be stricken out without
leaving the statement of the cause of Whether or not the dismissal of petitioner’s amended complaint should be sustained.
action inadequate. Since the inquiry
is into the sufficiency, not the Held: Yes.
veracity, of the material allegations,
it follows that the analysis should be The SC held that failure to state a cause of action is different from lack of cause of action. The former refers to the
confined to the four corners of the insufficiency of the allegations in the pleading, while the latter to the insufficiency of the factual basis for the action. The CA
complaint, and no other. dismissed the complaint on an incorrect ground (insufficiency of factual basis). Insufficiency of factual basis is not a ground
for a motion to dismiss; but rather, it becomes available only after the questions of fact have been resolved on the basis of
stipulations, admissions, or evidence presented by the plaintiff. The remedy is to raise such ground in a demurrer to evidence
after the plaintiff presented his/her evidence. Further, the complaint is still dismissible but on the ground of failure to state a
cause of action, as correctly held by the RTC, which was properly raised by respondent Gran in a motion to dismiss. The
petitioner’s cause of action should not merely be "stated" but, importantly, the statement thereof should be "sufficient." This
is why the elementary test in a motion to dismiss on such ground is whether or not the complaint alleges facts which if true
would justify the relief demanded. It has been held that only ultimate facts and not legal conclusions or evidentiary facts are
considered for purposes of applying the test. This is consistent with Section 1, Rule 8 of the Rules of Court which states that
the complaint need only allege the ultimate facts or the essential facts constituting the plaintiff's cause of action. A fact is
essential if they cannot be stricken out without leaving the statement of the cause of action inadequate. Since the inquiry is
into the sufficiency, not the veracity, of the material allegations, it follows that the analysis should be confined to the four
corners of the complaint, and no other. A judicious examination of petitioner's Amended Complaint readily shows its failure
to sufficiently state a cause of action. The allegations therein do not proffer ultimate facts which would warrant an action for
nullification of the sale and recovery of the properties in controversy; hence, rendering the same dismissible. While the
Amended Complaint does allege that petitioner was the registered owner of the subject properties in dispute, nothing in the
said pleading or its annexes would show the basis of that assertion, either through statements/documents tracing the root of
petitioner's title or copies of previous certificates of title registered in her name. Instead, the certificates of title covering the
said properties that were attached to the Amended Complaint are in the name of Gran. At best, the attached copies of the
certificate of title only mention petitioner as the representative of Gran at the time of the covered property's registration
when she was a minor. Nothing in the pleading, however, indicates that the former had become any of the properties' owner.
This leads to the logical conclusion that her right to the properties in question — at least through the manner in which it was
alleged in the Amended Complaint — remains ostensibly unfounded. Indeed, while the facts alleged in the complaint are
hypothetically admitted for purposes of the motion, it must, nevertheless, be remembered that the hypothetical admission
extends only to the relevant and material facts well pleaded in the complaint as well as to inferences fairly deductible
therefrom. Aside from the insufficiency of petitioner's allegations with respect to her right to the subject properties sought to
be recovered, the ultimate facts supposedly justifying the "annulment of sale," by which the reconveyance of the subject
properties is sought, were also insufficiently pleaded. Clearly, the claim that the sale was effected through "voidable and void
documents" partakes merely of a conclusion of law that is not supported by any averment of circumstances that will show
why or how such conclusion was arrived at. In fact, what these "voidable and void documents" are were not properly stated
and/or identified. Hence, by merely stating a legal conclusion, the Amended Complaint presented no sufficient allegation
upon which the Court could grant the relief petitioner prayed for. Thus, said pleading should be dismissed on the ground of
failure to state cause of action, as correctly held by the RTC. That a copy of the Deed of Sale adverted to in the Amended
Complaint was subsequently submitted by petitioner does not warrant a different course of action. An examination of the
present petition, however, reveals no counter-argument against the foregoing actions; hence, the SC considered any
objection thereto as waived.

21. LAZARO V. In a complaint, the cause of action Facts: The complaint alleges that petitioners obtained on credit from respondent Brewmaster beer and other products in the
BREWMASTER may be determined on its face. he total amount of 138,502.92 evidenced by invoices presented before the Court.
INTERNATIONAL [GR basic requirement under the rules of Photocopies of sales invoices showed that they were sold to TOTAL. And that despite repeated demands, petitioners refuse
182779] 23 AUG 2010 procedure is that a complaint must to pay.Petitioner denied having transacted with respondent and averred that the documents attached to the complaint
CASTRO make a plain, concise, and direct showed that it was Total which purchased goods from respondent. Prescillo filed an answer with counterclaim in which he
statement of the ultimate facts on denies the obligation. He contested that he himself never authorized Victorina to purchase anything from Brewmaster. He
which the plaintiff relies for his claim. pointed out that the purchaser of the items, as borne out by the sales invoices attached to the complaint, was Total.Likewise,
Victorina, in her own answer with counterclaim denied the transaction with the beer company, and averred that the
documents attached to the complaint showed that it was Total which purchased goods from Brewmaster.During the
preliminary conference, both Victorina and Prescillo did not appear. Hence, a default.

Issue: Whether or not in a complaint, the cause of action may be determined on its face.

Ruling: Yes. The basic requirement under the rules of procedure is that a complaint must make a plain, concise, and direct
statement of the ultimate facts on which the plaintiff relies for his claim. Ultimate facts mean the important and substantial
facts which either directly form the basis of the plaintiff's primary right and duty or directly make up the wrongful acts or
omissions of the defendant. Contrary to what petitioner thinks, the Court ruled that the Complaint sufficiently states a cause
of action. The following allegations in the complaint were found to be sufficiently make up a cause of action for collection of
sum of money:
(1) that petitioner and her husband obtained beer and other products worth a total ofP138,502.92 on
credit from respondent; and
(2) that they refused to pay the said amount despite demand.

22. VDA. DE It should be stressed that in the Facts: Petitioner Concepcion Villamor Vda. de Daffon (Concepcion) was married to the late Amado Daffon, with whom she
DAFFON V. CA [GR determination of whether a complaint begot one son, Joselito Daffon (Joselito). Joselito married Lourdes Osmeña (Lourdes), and they bore six children, namely,
129017] 20 AUG 2002 fails to state a cause of action, only Aileen, Joselito Jr., Ana Vanesa, Leila, Julius and Suzette. Amado passed away on January 21, 1982. His son, Joselito, died
LIMIN the statements in the complaint may on October 25, 1990. Lourdes Osmeña Vda. De Daffon, together with her six minor children (Respondents), instituted an
be properly considered. Moreover, a action for partition against Petitioner. Respondents alleged that Amado left several real and personal properties which
defendant who moves to dismiss the formed part of his conjugal partnership with Petitioner. Joselito being a forced heir of Amado was entitled to at least 1/2 of
complaint on the ground of lack of Amado's estate, consisting of his share in the said conjugal properties. However, the said properties were never partitioned
cause of action hypothetically admits between petitioner and Joselito. Petitioner filed a Motion to Dismiss on the grounds of (1) lack of jurisdiction over the subject
all the averments thereof. In the matter of the case; (2) failure of the complaint to state a cause of action; and (3) waiver, abandonment and extinguishment of
same vein, the complaint needs only the obligation.
to allege the ultimate facts on which
the plaintiffs rely for their claim. Issue: W/N the complaint fails to state a cause of action

Held: No. In the determination of whether a complaint fails to state a cause of action, only the statements in the complaint
may be properly considered. Moreover, a defendant who moves to dismiss the complaint on the ground of lack of cause of
action hypothetically admits all the averments thereof. The test of sufficiency of the facts found in a complaint as constituting
a cause of action is whether or not admitting the facts alleged the court can render a valid judgment upon the same in
accordance with the prayer thereof. The hypothetical admission extends to the relevant and material facts well pleaded in the
complaint and inferences fairly deducible therefrom. Hence, if the allegations in the complaint furnish sufficient basis by
which the complaint can be maintained, the same should not be dismissed regardless of the defense that may be assessed
by the defendants.

In the case at bar, the complaint sufficiently alleged that "defendant (i.e., petitioner herein) was married to Amado Quiros
Daffon" and that "they begot an only son in Joselito Daffon." The complaint further alleged that "Joselito Daffon later got
married to herein plaintiff Lourdes Osmeña and before the former died on October 25, 1990 he sired the six (6) children who
are now plaintiffs with their mother." This, to our mind, was sufficient allegation that Joselito Daffon was a legitimate son of
the spouses Amado and Concepcion Daffon; and that plaintiffs (i.e., respondents herein) were likewise legitimate heirs of
Joselito Daffon. Admitting the truth of these averments, there was, therefore, no need to inquire whether respondent minor
children were duly acknowledged by the deceased Amado Daffon. To be sure, the illegitimacy of the said children and the
lack of acknowledgment are matters which petitioner may raise as a defense in her answer and threshed out by the court
during a full-blown trial.

In the same vein, there is no need for the complaint to specifically allege respondents' claim of co-ownership of the
properties. The complaint needs only to allege the ultimate facts on which the plaintiffs rely for their claim. The allegations
contained therein are sufficient to establish respondents' right to the estate of Amado Daffon. By stating their relationship to
the deceased, they established their line of succession as the basis for their claim. Their rights to succeed as heirs were
transmitted from the moment of death of the decedent.

23. CANETE V. A pleading should state the ultimate Facts: On January 11, 1999, petitioners filed a complaint for cancellation of title to property covered by Transfer Certificate
GENUINO ICE CO. [GR facts essential to the rights of action of Title (TCT) Nos. N-140441; 14399; RT-94384 (292245); RT-94794 (292246); and 292247. Petitioners alleged that said
154080] 22 JAN 2008 or defense asserted, as titles are spurious, fictitious and were issued "under mysterious circumstances," considering that the holders thereof –
UBAY distinguished from mere conclusions including their predecessors-in-interest – were never in actual, adverse and physical possession of the property, rendering
of fact, or conclusions of law. them ineligible to acquire title to the said property under the Friar Lands Act. Petitioners also sought to nullify Original
General allegations that a contract is Certificate of Title (OCT) No. 614 from which the foregoing titles sought to be cancelled originated or were derived.
valid or legal, or is just, fair and
reasonable, are mere conclusions of Petitioners allege the following:
law. Likewise, allegations that a That the OCT 614 and all transfer certificate of title derived therefrom should be declared as null and void ab initio;
contract is void, voidable, invalid, That the defendants’ transfer certificates of title over the property in litigation should be declared as null and void;
illegal, ultra vires, or against public Ordering defendant Register of Deeds of Quezon City to cancel defendants’ transfer certificates of title and all transfer
policy, without stating facts showing certificates of title dereived therefrom;
its invalidity, are mere conclusions of To declare the plaintiffs as bona fide occupants of the property in litigation pursuant to the provisions of the Friar Lands Act
law. and other existing laws.
Respondent Genuino Ice Co., Inc, filed a motion to dismiss on the ground that the complaint states no cause of action
because petitioners are not real-parties-in-interest; that no relief may be granted as a matter of law; and that petitioners
failed to exhaust administrative remedies, but it was denied by the trial court. Respondent moved for reconsideration but the
same was denied.
On January 2001, the trial court denied respondent’s motion to dismiss the Second Amended Complaint. Its motion for
reconsideration was likewise denied respondent filed a petition for certiorari with the Court of Appeals.
The appellate court granted respondent’s petition for certiorari and dismissed petitioner’s Second Amended Complaint for
failure to state a cause of action. Hence, the instant petition.

Issue: Whether or not the court of appeals erred in declaring that the complaint filed by the petitioners with the RTC does not
state a valid of cause of action

Held: The court denied the petition.

The basic rules of proper pleading and procedure require that every pleading shall contain in a methodical and logical form,
a plain, concise and direct statement of the ultimate facts on which the party pleading relies for his claim or defense, as the
case may be, omitting the statement of mere evidentiary facts. And in all averments of fraud or mistake, the circumstances
constituting fraud or mistake must be stated with particularity.
It is axiomatic that the averments of the complaint determine the nature of the action, and consequently, the
jurisdiction of the courts. This is because the complaint must contain a concise statement of the ultimate facts constituting
the plaintiff's cause of action and must specify the relief sought. No rule is better established than that which requires the
complaint to contain a statement of all the facts constituting the plaintiff's cause of action. Additionally, Section 5, Rule 8 of
the Rules of Court provides that in all averments of fraud or mistake, the circumstances constituting fraud or mistake must be
stated with particularity. In the case at bar, while there are allegations of fraud in the above quoted complaints, the same are
not particular enough to bring the controversy within the SEC's jurisdiction. The said allegations are not statements of
ultimate facts but are mere conclusions of law.
A pleading should state the ultimate facts essential to the rights of action or defense asserted, as distinguished from
mere conclusions of fact, or conclusions of law. General allegations that a contract is valid or legal, or is just, fair and
reasonable, are mere conclusions of law. Likewise, allegations that a contract is void, voidable, invalid, illegal, ultra vires, or
against public policy, without stating facts showing its invalidity, are mere conclusions of law.
Corollarily, the question of whether or not a complaint states a cause of action against a defendant or the action is premature
is one of law. The trial court can consider all the pleadings filed, including annexes, motions and the evidence on record.
However in so doing, the trial court does not rule on the truth or falsity of such documents. It merely includes such
documents in the hypothetical admission. Any review of a finding of lack of cause of action based on these documents would
not involve a calibration of the probative value of such pieces of evidence but would only limit itself to the inquiry of whether
the law was properly applied given the facts and these supporting documents. Therefore, what would inevitably arise from
such a review are pure questions of law, and not questions of fact.
Petitioners’ Second Amended Complaint betrays no more than an incomplete narration of facts unsupported by documentary
or other exhibits; the allegations therein partake of conclusions of law unsupported by a particular averment of circumstances
that will show why or how such inferences or conclusions were arrived at. It is replete with sweeping generalizations and
inferences derived from facts that are not found therein. While there are allegations of fraud upon the claim that the subject
titles were fictitious, spurious and obtained under "mysterious circumstances," the same are not specific to bring the
controversy within the trial court’s jurisdiction. There is no explanation or narration of facts as would show why said titles are
claimed to be fictitious or spurious, contrary to the requirement of the Rules that the circumstances constituting fraud must
be stated with particularity; otherwise, the allegation of fraud would simply be an unfounded conclusion of law. In the
absence of specific averments, the complaint is defective, for it presents no basis upon which the court should act, or for the
defendant to meet it with an intelligent answer.

24. STEELCASE V. No person ought to derive any Steelcase is a foreign corporation existing under the laws of Michigan, United States of America. Respondent Design
DESIGN advantage of his own wrong. This is International Selections, Inc. (DISI) is a corporation existing under Philippine Laws. Steelcase and DISI orally entered into a
INTERNATIONAL [GR as it should be for as mandated by dealership agreement whereby Steelcase granted DISI the right to market, sell, distribute, install, and service its products to
171995] 18 APR 2012 law, “every person must in the end-user customers within the Philippines. The business relationship continued smoothly until it was terminated sometime in
YUMUL exercise of his rights and in the January 1999 after the agreement was breached with neither party admitting any fault.
performance of his duties, act with
justice, give everyone his due, and Steelcase filed a complaint for sum of money against DISI alleging an unpaid account. In his Order dated November 15,
observe honesty and good faith. 1999, Acting Presiding Judge Bonifacio Sanz Maceda dismissed the complaint, granted the TRO prayed for by DISI, and
denied Steelcase’s Motion to Admit Second Amended Complaint. The RTC stated that in requiring DISI to meet the Dealer
Performance Expectation and in terminating the dealership agreement with DISI based on its failure to improve its
performance in the areas of business planning, organizational structure, operational effectiveness, and efficiency, Steelcase
unwittingly revealed that it participated in the operations of DISI.

It then concluded that Steelcase was “doing business” in the Philippines and since it did not have the license to do business
in the country, it was barred from seeking redress from our courts until it obtained the requisite license to do so.

ISSUE/S:

WON a foreign corporation may sue in Philippines courts although not authorized to do business within.

HELD:

YES. A foreign corporation doing business in the Philippines may sue in Philippine Courts although not authorized to do
business here against a Philippine citizen or entity who had contracted with and benefited by said corporation. To put it in
another way, a party is estopped to challenge the personality of a corporation after having acknowledged the same by
entering into a contract with it. And the doctrine of estoppel to deny corporate existence applies to a foreign as well as to
domestic corporations. One who has dealt with a corporation of foreign origin as a corporate entity is estopped to deny its
corporate existence and capacity: The principle will be applied to prevent a person contracting with a foreign corporation
from later taking advantage of its noncompliance with the statutes chiefly in cases where such person has received the
benefits of the contract.
25. LUISTRO V. CA In all averments of fraud or mistake, FACTS
[GR 158819] 16 APR the circumstances constituting fraud First Gas Power Corporation operates gas fired power generating facility by virtue of a Power Purchase Agreement (PPA)
2009 or mistake must be stated with and sells the electric power generated by its facility to Meralco. It entered into a Substation Interconnection Agreement with
ZAPANTA particularity. Malice, intent, Meralco and NPC which required First Gas to finance, construct, commission, and energize a 230-kilovolt electric power
knowledge or other condition of the transmission line, approximately 25 km in Batangas. Under the SIA, First gas had the to acquire easements of right-of-way
mind of a person may be averred over affected lands located along the designated route of the transmission line.
generally. First Gas entered into a Contract of Easement of Right-of-Way (Contract) with Luistro, an owner of a parcel of land in
Batangas. Under the contract, First Gas was granted perpetual easement over a 100-sq. m. portion of his property for the
erection of the transmission line tower and a 25-year easement over 2,453.60 sq. m. portion of the property for the right to
pass overhead line cables in consideration of P 88,608.
Luistro’s counsel wrote a letter to First Gas asking for a temporary stoppage of all kinds of work within the vicinity of
petitioner’s residential house pending settlement of petitioner’s grievance that the house and other improvements lay
underneath the transmission wire/line being constructed and would endanger the life and health of the persons in the vicinity.

Petitioner filed a complaint for “Rescission/Amendment And Or Modification of Contract Of Easement With Damages",
docketed as Civil Case No. 142-2000, against respondent and First Balfour Beatty Realty, Inc. (defendants). Petitioner
alleged that
respondent, by means of fraud and machinations of words, was able to convince him to enter into the Contract. Petitioner
alleged that he entered into the Contract under misrepresentation, promises, false and fraudulent assurances, and tricks of
respondent. Petitioner alleged that while his house was supposed to be 20 to 25 meters away from the transmission
wire/line, it turned out after the installation of Posts 97 and 98 that his house was only 7.23 meters directly underneath the
transmission wire/line. Petitioner
alleged that the powerful 230 kilovolts passing the transmission wire/line continuously endanger the lives, limbs, and
properties of petitioner and his family.

RTC denied MTC. RTC also denied MR.


CA: First Gas filed petition for ceriorari before the CA. CA ordered dismissal for failure to state cause of action. CA denied
the MR filed by Luistro.

ISSUE:
W/N complaint alleged fraud with particularity as required under Section 5, Rule 8 of the 1997 Rules of Civil Procedure.

HELD: No.
Section 5, Rule 8 of the 1997 Rules of Civil Procedure states: Fraud, mistake, condition of the mind. — In all averments of
fraud or mistake, the circumstances constituting fraud or mistake must be stated with particularity. Malice, intent, knowledge
or other condition of the mind of a person may be averred generally.

Complaint fell short of the requirement that fraud must be stated with sufficient particularity. Complaint merely states: “That
sometime in the year of 1997, the consolidator-facilitator of the Defendants FGPC and Balfour by means of fraud and
machinations of words were able to convince[] the plaintiff to enter into 'CONTRACT OF EASEMENT OF RIGHT OF WAY'
wherein the latter granted in favor of the defendant FGPC the right to erect [its] Tower No. 98 on the land of the plaintiff
situated at Barangay Maigsing Dahilig, Lemery 4209 Batangas including the right to Install Transmission Lines over a portion
of the same property for a consideration therein stated, a xerox copy of said contract is hereto attached as [] ANNEXES "A"
up to "A-4" of the complaint;
That the said contract, was entered into by the plaintiff under the "MISREPRESENTATION, PROMISES, FALSE AND
FRAUDULENT ASSURANCES AND TRICKS" of the defendants.

Not only did petitioner fail to allege with particularity the fraud allegedly committed by respondent. A review of the Contract
shows that its contents were explained to petitioner. The Contract states:
Bago ko/namin nilagdaan ang kasulatang ito ay ipinaliwanag muna sa akin/amin sa wikang Tagalog/ o sa wikang
aking/aming naiintindihan. Ang nilalaman nito'y lubusan ko/naming nauunawaan kaya't lumagda kami rito ng kusang loob,
walang sinumang pumilit o tumakot sa akin/amin.
There is clearly no basis for the allegation that petitioner only signed the Contract because of fraud perpetrated by
respondent.

26. TITAN Where a party acted in complete FACTS:


CONSTSRUCTION V. disregard of or wholly overlooked Manuel A. David, Sr. (Manuel) and Martha S. David (Martha) were married on March 25, 1957. In 1970, the spouses
DAVID [GR 169548] 15 Section 8, Rule 8 and did not object acquired a 602 square meter lot located at White Plains, Quezon City, which was registered in the name of "MARTHA S.
MAR 2010 to the introduction and admission of DAVID, of legal age, Filipino, married to Manuel A. David" and covered by Transfer Certificate of Title (TCT) No. 156043
ABAD evidence questioning the issued by the Register of Deeds of Quezon City. In 1976, the spouses separated de facto, and no longer communicated with
genuineness and due execution of a each other.
document, he must be deemed to Sometime in March 1995, Manuel discovered that Martha had previously sold the property to Titan Construction Corporation
have waived the benefits of said (Titan) for ₱1,500,000.00 through a Deed of Sale dated April 24, 1995, and that TCT No. 156043 had been cancelled and
Rule. replaced by TCT No. 130129 in the name of Titan.
Thus, on March 13, 1996, Manuel filed a Complaint for Annulment of Contract and Recovenyance against Titan before the
RTC of Quezon City. Manuel alleged that the sale executed by Martha in favor of Titan was without his knowledge and
consent, and therefore void. He prayed that the Deed of Sale and TCT No. 130129 be invalidated, that the property be
reconveyed to the spouses, and that a new title be issued in their names.
In its Answer with Counterclaim, Titan claimed that it was a buyer in good faith and for value because it relied on a Special
Power of Attorney (SPA) dated January 4, 1995 signed by Manuel which authorized Martha to dispose of the property on
behalf of the spouses. Titan thus prayed for the dismissal of the complaint.
In his unverified Reply, Manuel claimed that the SPA was spurious, and that the signature purporting to be his was a forgery;
hence, Martha was wholly without authority to sell the property.
Subsequently, Manuel filed a Motion for Leave to File Amended Complaint which was granted by the trial court. Thus, on
October 15, 1996, Manuel filed an Amended Complaint impleading Martha as a co-defendant in the proceedings. However,
despite personal service of summons upon Martha, she failed to file an Answer. Thus, she was declared in default. Trial then
ensued.
RTC issued a Decision which (i) invalidated both the Deed of Sale and TCT No. 130129; (ii) ordered Titan to reconvey the
property to Martha and Manuel; (iii) directed the Register of Deeds of Quezon City to issue a new title in the names of
Manuel and Martha; and (iv) ordered Titan to pay ₱200,000.00 plus ₱1,000.00 per appearance as attorney’s fees, and
₱50,000.00 as costs of suit.
The CA affirmed the Decision of the trial court but deleted the award of attorney’s fees and the amount of ₱50,000.00 as
costs.
Titan moved for reconsideration but the motion was denied on August 31, 2005.
Hence, this petition.

Issue:
Whether or not there was failure to deny the genuineness and due execution of the SPA.

Held:

The Supreme Court fully concurred with the findings of the Court of Appeals that it is true that the reply filed by Manuel
alleging that the special power of attorney is a forgery was not made under oath. However, the complaint, which was verified
by Manuel under oath, alleged that the sale of the subject property executed by his wife, Martha, in favor of Titan was
without his knowledge, consent, and approval, express or implied; and that there is nothing on the face of the deed of sale
that would show that he gave his consent thereto. While Section 8, Rule 8 is mandatory, it is a discovery procedure and must
be reasonably construed to attain its purpose, and in a way as not to effect a denial of substantial justice. The interpretation
should be one which assists the parties in obtaining a speedy, inexpensive, and most important, a just determination of the
disputed issues.1

Moreover, during the pre-trial, Titan requested for stipulation that the special power of attorney was signed by Manuel
authorizing his wife to sell the subject property, but Manuel refused to admit the genuineness of said special power of
attorney and stated that he is presenting an expert witness to prove that his signature in the special power of attorney is a
forgery. However, Titan did not register any objection x x x. Furthermore, Titan did not object to the presentation of Atty.
Desiderio Pagui, who testified as an expert witness, on his Report finding that the signature on the special power of attorney
was not affixed by Manuel based on his analysis of the questioned and standard signatures of the latter, and even cross-
examined said witness. Neither did Titan object to the admission of said Report when it was offered in evidence by Manuel
on the ground that he is barred from denying his signature on the special power of attorney. In fact, Titan admitted the
existence of said Report and objected only to the purpose for which it was offered. In Central Surety & Insurance Company
v. C.N. Hodges, it was held that where a party acted in complete disregard of or wholly overlooked Section 8, Rule 8 and did
not object to the introduction and admission of evidence questioning the genuineness and due execution of a document, he
must be deemed to have waived the benefits of said Rule. Consequently, Titan is deemed to have waived the mantle of
protection given [it] by Section 8, Rule 8.

It is true that a notarial document is considered evidence of the facts expressed therein. A notarized document enjoys a
prima facie presumption of authenticity and due execution and only clear and convincing evidence will overcome such legal
presumption. However, such clear and convincing evidence is present here.1avvph!1 While it is true that the SPA was
notarized, it is no less true that there were defects in the notarization which mitigate against a finding that the SPA was either
genuine or duly executed. Curiously, the details of Manuel’s Community Tax Certificate are conspicuously absent, yet
Martha’s are complete. The absence of Manuel’s data supports his claim that he did not execute the same and that his
signature thereon is a forgery. Moreover, we have Manuel’s positive testimony that he never signed the SPA, in addition to
the expert testimony that the signature appearing on the SPA was not Manuel’s true signature.

27. CONSOLIDATED When the defendant fails to deny FACTS: Consolidated Bank, a domestic banking and trust corporation, alleged that it extended a loan in favor of
BANK V. DEL MONTE specifically and under oath the due respondents Del Monte Motor Works, Inc. and Narciso Morales as evidenced by a promissory note executed by
MOTOR WORKS [GR execution and genuineness of a respondents, but that the respondents defaulted on their monthly installments despite oral and written demands.
143338] 29 JUL 2005 document copied in a complaint,
BRIONES the plaintiff need not prove that fact Petitioner filed before the RTC of Manila a complaint for recovery of sum of money against respondents. When petitioner
as it is considered admitted by the made its formal offer of evidence, they had lost the original copy of Exhibit A (promissory note) and instead sought the
defendant. admission of the duplicate original of the promissory note which was identified and marked as Exhibit E. RTC initially
admitted Exhibit E into evidence. Del Monte filed a manifestation and MR claiming that Exhibit E should not have been
admitted as it was immaterial, irrelevant, was not properly identified, and was hearsay evidence. Morales likewise filed
manifestation and MR raising the same arguments. RTC granted respondent’s Motion for Reconsideration and denied
petitioner’s MR. Respondents filed their Motions to Dismiss on the similar ground that with the exclusion of Exhibits A and E,
petitioner no longer possessed any proof of respondent’s alleged indebtedness.

ISSUE: W/N respondents were able to generally and specifically deny under oath the genuineness and due execution of the
promissory notes.

HELD: Both the court a quo and the CA erred in ruling that the respondents were able to specifically deny the allegations in
the petitioner's complaint in the manner specifically required by the rules. In effect, respondents had, to all intent and
purposes, admitted the genuineness and due execution of the subject promissory note and recognized their obligation to
petitioner. Defendant must declare under oath that he didn’t sign the document or that it is otherwise false or fabricated. It
should be noted that respondents never disputed the terms and conditions of the promissory note, leaving the Court to
conclude that as far as the parties are concerned, the wording or content of said note is clear enough and leaves no room for
disagreement. When a defendant fails to deny specifically and under oath the due execution and genuineness of a
document copied in a complaint, the plaintiff no longer needs to prove that fact as it is considered admitted by the defendant.

28. GUEVARRA V. Where a fact is alleged with Facts: Joselano Guevarra and Irene Moje got married. Irene introduced Atty. Jose Emmanuel Eala to him. Atty Eala has a
EALA [GR 7136] 01 AUG qualifying or modifying language and wife and children. After Guevarra and Irene Moje got married, the former noticed how his wife was
2007 the words of the allegation asso continuously receiving phone calls and text messages from Atty. Eala. The messages had “I love you”. Irene started going
CASTRO qualified or modified are literally home very late at night. There was confrontation and Irene started not going home. Guevarra saw a love letter from Atty.
denied, it has been held that the Eala to Irene. Such love letter contained a manifestation of Atty. Eala’s love for Irene. Eventually, Irene got pregnant of
qualifying circumstance alone are Eala’s child.
denied while the fact itself is Hence, Guevarra filed a complaint for disbarment before the Integrated Bar of the Philippines (IBP) against Atty. Eala. From
admitted. respondent's answer, he does not deny carrying on an adulterous relationship with Irene.

To respondent's answer, complainant filed a reply, alleging that Irene gave birth to a girl and Irene named respondent in the
Certificate of Live Birth as the girl's father. Complainant attached to the reply, a copy of a Certificate of Live Birth bearing
Irene's signature and naming respondent as the father of her daughter The Commissioner thus recommended that
respondent
be disbarred for violating Rule 1.01 of Canon 1 of the Code of Professional Responsibility reading: Rule 1.01:A lawyer shall
not engage in unlawful, dishonest, immoral or deceitful conduct. The IBP Board of Governors, however, annulled and set
aside the Recommendation of the Investigating
Commissioner and accordingly dismissed the case for lack of merit.
ISSUE:
Whether or not there is proper and sufficient denial by respondent in his answer with regard to the material allegations in the
complaint.
RULING:
No. It should be noted that in his Answer, respondent
through counsel made the following statements to wit: "Respondent specifically denies having [ever] flaunted an adulterous
relationship with Irene as alleged in paragraph [14] of the Complaint, the truth of the matter being [that] their relationship was
low profile and known only to immediate members of their respective families . . . , and Respondent specifically denies the
allegations in paragraph 19 of the complaint, the reason being that under the circumstances the acts of the respondents with
respect to his purely personal and low profile relationship with Irene is neither under scandalous circumstances nor
tantamount to grossly immoral conduct . . ." According to the Supreme Court, these statements of respondent in his Answer
are an admission that there is indeed a "special" relationship between him and
complainant's wife, Irene, [which] taken together with the Certificate of
Live Birth of Samantha Louise Irene Moje (daughter) sufficiently prove that there was indeed an illicit relationship between
respondent and Irene which
resulted in the birth of the child "Samantha".

29. HALIMAO V. FACTS:


VILLANUEVA [AC 3825] The rule does not unqualifiedlyThis is a complaint for disbarment against Attorneys Daniel Villanueva and Inocencio Ferrer, Jr., for serious
01 FEB 1996 apply to a case where the misconduct.
DE GALA defendant files a motion to dismiss
The complaint originated from a letter dated April 14, 1992 which complainant Reynaldo Halimao wrote to the Chief
based on lack of jurisdiction of the
Justice, alleging that respondents, without lawful authority and armed with armalites and handguns, forcibly entered
court or tribunal over the person of
the Oo Kian Tiok Compound in Cainta, Rizal, of which complainant was caretaker, on April 4, 1992 at 11:00 A.M.
the defendant or over the subject
Complainant prayed that an investigation be conducted and respondents disbarred. To the complaint were attached
matter or over the nature of the
the affidavits of alleged witnesses, including that of Danilo Hernandez, a security guard at the compound, who had
action; or on improper venue; or on
also filed a similar complaint against herein respondents.
lack of capacity to sue of the In its resolution dated July 1, 1992, the Court required respondents to comment.
plaintiff or on litis pendentia, res
On August 14, 1992, respondents filed a comment in which they claimed that the complaint is a mere duplication of the
judicata, prescription,
complaint filed by Danilo Hernandez in Administrative Case No. 3835, which this Court had already dismissed on
unenforceability, or on the
August 5, 1992 for lack of merit. They pointed out that both complaints arose from the same incident and the same
allegation that the suit is between
acts complained of and that Danilo Hernandez, who filed the prior case, is the same person whose affidavit is attached
members of the same family and to the complaint in this case.
no earnest efforts towards a Respondent Ferrer claimed that he was nowhere near the compound when the incident took place. He submitted
compromise have been made. In affidavits attesting to the fact that he had spent the whole day of April 4, 1992 in Makati with his family.
such cases, the hypothetical Additionally, Ferrer claimed that the two complaints were filed for the purpose of harassing him because he was the
admission is limited to the facts
principal lawyer of Atty. Daniel Villanueva in two cases before the Securities and Exchange Commission. The cases
alleged in the complaint which involved the ownership and control of Filipinas Textile Mills (Filter), which is owned by Villanueva's family and whose
relate to and are necessary for the
premises are the Oo Kian Tiok compound.
resolution of these grounds as This case was thereafter referred to the Integrated Bar of the Philippines for investigation, report and recommendation.
preliminary matters involving
In its Resolution No. XI-94-017 dated January 22, 1994, the Board of Governors of the IBP dismissed the case against
substantive or procedural laws, but
respondents. It acted on the basis of the report and recommendation of Atty. Victor C. Fernandez, Investigating
not to the other facts of the case.
Commissioner, who found that the complaint is barred by the decision in Administrative Case No. 3835 which involved
This case was dismissed on the the same incident. Atty. Fernandez noted that in fact the complaints in the two cases were similarly worded.
ground of res judicata. On March 28, 1994, complainant filed a motion for reconsideration of the resolution of the IBP Board of Governors. His
motion was referred to the Court in view of the fact that the records of the case had earlier been forwarded to the Court
on March 11, 1994.
In his aforesaid motion, complainant contends that by filing a motion to dismiss the complaint in this case, private
respondents must be deemed to have hypothetically admitted the material allegations in the complaint and, therefore,
private respondents must be deemed to have confessed to the charge of serious misconduct. Hence, it was error for
the IBP to dismiss his complaint.
ISSUE:
Whether or not by filing a motion to dismiss, respondents are deemed to have admitted the allegations of the complaint
against them
HELD:
Suffice it to say that the rule that a motion to dismiss is to be considered as a hypothetical admission of the facts
alleged in the complaint applies more particularly to cases in which the ground for dismissal is the failure of the
complaint to state a cause of action. When it appears on the face of the complaint that the plaintiff is not entitled to any
relief under the facts alleged, the defendant may file a motion to dismiss hypothetically admitting the facts alleged in
the complaint.By filing such a motion, the defendant in effect says that even assuming the facts to be as alleged by the
plaintiff, the latter has failed to prove that he has a right which the former has violated.
The rule does not unqualifiedly apply to a case where the defendant files a motion to dismiss based on lack of
jurisdiction of the court or tribunal over the person of the defendant or over the subject matter or over the nature of the
action; or on improper venue; or on lack of capacity to sue of the plaintiff or on litis pendentia, res judicata, prescription,
unenforceability, or on the allegation that the suit is between members of the same family and no earnest efforts
towards a compromise have been made. In such cases, the hypothetical admission is limited to the facts alleged in the
complaint which relate to and are necessary for the resolution of these grounds as preliminary matters involving
substantive or procedural laws, but not to the other facts of the case.
On the other hand, when a motion to dismiss is based on payment, waiver, abandonment, release, compromise, or
other form of extinguishment, the motion to dismiss does not hypothetically, but actually, admits the facts alleged in the
complaint, i.e., the existence of the obligation or debt, only that the plaintiff claims that the obligation has been
satisfied. So that when a motion to dismiss on these grounds is denied, what is left to be proven in the trial is no longer
the existence of the debt but the fact vel non of payment by the defendant.
The Investigating Commissioner properly dismissed the complaint in this case on the ground of res judicata, it
appearing that it involves the same incident and the same cause of action as Administrative Case No. 3825. Indeed, it
appears that on August 5, 1995, the First Division of the Court dismissed a similar complaint filed in Administrative
Case No. 3835.
Two motions for reconsideration of this resolution were filed by the complainant therein, both of which were denied, the
first one on September 23, 1992 and the second one on November 9, 1992.
While the complainant (Danilo Hernandez) in Administrative Case No. 3835 is different from the complainant in the
present case, the fact is that they have an identity of interest, as the Investigating Commissioner ruled. Both
complainants were employed at the Oo Kian Tiok Compound at the time of the alleged incident. Both complain of the
same act allegedly committed by respondents. The resolution of this Court in Administrative Case No. 3835 is thus
conclusive in this case, it appearing that the complaint in this case is nothing but a duplication of the complaint of
Danilo Hernandez in the prior case. In dismissing the complaint brought by Danilo Hernandez in the prior case, this
Court categorically found "want of a prima facie showing of professional misconduct on the part of the respondents
[Attorneys Daniel Villanueva and Inocencio Ferrer, Jr.]".

30. PACSPORT INC.


PHILS. V. NICCOLO One of two actions will be dismissed Facts: Petitioner Pacsports Phils. Inc., (PPI) and respondent Niccolo Sports, Inc. (NSI) entered into two (2) separate
SPORTS [GR 141601] 22 on ground of litis pendentia if the Exclusive Retail Agreements by virtue of which petitioner supplied respondent, on consignment basis, assorted Bridgestone
NOV 2001 following requisites concur: and Cross Creek golf products to be sold by the latter in its store outlet. Petitioner PPI claimed that after months of operation,
ESPIRITU respondent's obligations to it amounted to about P1.5 Million. Despite demand, respondent failed to pay, and eventually it
(a) identity of parties, or at least such pre-terminated the contracts. Petitioner was then prompted to file with the RTC -Branch 141, Makati City, Civil Case No. 99-
as representing the same interest in 221 for damages with application for a writ of replevin against respondent. For its part, respondent NSI filed with the RTC-
both actions; Branch 91, Quezon City, Civil Case No. Q-99-36797 for "Breach and Confirmation of Termination of Contracts and
Damages" against petitioner. Respondent NSI filed with the Makati RTC a motion to dismiss or suspend the proceedings in
(b) identity of rights asserted and Civil Case No. 99-221 citing the pendency of the Quezon City case. Thereupon, petitioner PPI also filed with the Quezon
relief prayed for, the relief being City RTC a motion to dismiss Civil Case No. Q-99-36797 on the ground of pendency of the Makati City case. The Makati
founded on the same facts; and RTC issued an order denying respondent's motion to dismiss. In the same order, the Makati Court granted petitioner's
application for a writ of preliminary mandatory injunction. Respondent filed a motion for reconsideration,but was denied.
(c) the identity in the two (2) cases Respondent NSI questioned the orders of the Makati RTC in a petition for certiorari and prohibition before the Court of
should be such that judgment in one Appeals. The appellate court granted the petition and ordered the dismissal of Civil Case No. 99-221 pending before the
would amount to res judicata in the Makati RTC on the ground that the Quezon City case involves a broader scope of inquiry as it goes to the pith of the
other.||| controversy, which is the pre-termination of the agreement. Hence, the present petition.

In several cases that when the The SC reversed the decision of the CA. The Court ruled that in ordering the dismissal of the Makati City case filed earlier
elements of litis pendentia exist, the than the Quezon City case, the CA deviated from the general rule that when the elements of litis pendentia exist, the later
action filed later should be abated to action should be abated to avoid multiplicity of suits. The Court also rejected the appellate court's rationale that the Quezon
avoid multiplicity of suits. City case involves a broader scope of inquiry than the Makati case. The Court pointed out that the only basic issue between
the parties in both cases is whether the pre-termination of the agreements is valid as claimed by respondent or invalid as
This is based on the maxim Qui prior claimed by petitioner. The complaints may differ from each other in some details, but such details are mere incidents to the
est tempore, potior est jure (He who basic issue of the validity of the pre-termination of the exclusive retail agreements.
is before in time is the better in right).
This is the general rule. Issue: Which of the two cases should be dismissed by reason of litis pendentia — the Makati City case which was filed
earlier or the Quezon City case which was filed later|||
Ruling: The Quezon City case. In this case, the parties concede that the Makati City case and the Quezon City case involve
the same parties, rights asserted and reliefs prayed for, being founded on the same facts; and that judgment in one would
constitute res judicata on the other. Because of the concurrence of these similarities, petitioner and respondent sought the
abatement of each other's suit on the ground of litis pendentia. Undisputably, the parties in the Makati case and the Quezon
City case are the same. Petitioner is the plaintiff in the Makati case and the defendant in the Quezon City case; and
respondent is the defendant in the Makati case and the plaintiff in the Quezon City case.
The rights asserted and the reliefs prayed for by petitioner in the Makati City case and the rights asserted and the reliefs
prayed for by respondent in the Quezon City case are all based on the validity of the pre-termination of the Exclusive Retail
Agreements. In ordering the dismissal of the Makati City case filed earlier than the Quezon City case, the CA deviated from
the said general rule. It correctly observed that: (1) both actions arose from the two (2) Exclusive Retail Agreements entered
into by the parties, and the asserted rights are founded on an identical set of facts; and (2) there is a claim of breach of the
said Agreements by one of the parties against the other. However, we cannot go along with CA in concluding that the
Quezon City case "involves a broader scope of inquiry" than the Makati case. The Appellate Court did not explain why the
Quezon City case is broader in scope than the Makati case. Clearly, the Quezon City RTC's deviation from the general rule
cannot be sustained on the ground that the case before it involves a broader scope of inquiry.

31. PANGANIBAN The following requisites of litis Facts: Petitioner, Carmelita Panganiban entered into a Sublease and Dealer Agreement (SLDA) with respondent, Shell. In a
V. PILIPINAS SHELL pendentia are present in this case: letter, Shell notified Panganiban that the SLDA was expiring on July 31, 1995. Shell then advised Panganiban to wind up her
CORP. [GR 131471] 22 (a) the identity of parties, or at least business on or before July 31, 1995. Believing that the SLDA had not yet expired and was still effective until December 31,
JAN 2003 such as representing the same 2002, Panganiban continued to pay rentals for the gasoline station. Shell refused to accept the payments. Thereafter,
LEE interests in both actions; (b) the Panganiban filed a petition for declaratory relief with the Regional Trial Court of Makati City. On the other hand, Shell filed
identity of rights asserted and relief an unlawful detainer case against petitioner with the Metropolitan Trial Court of Caloocan City. During the preliminary
prayed for, the relief being founded conference of the unlawful detainer case, Panganiban moved for the suspension of the proceedings since the declaratory
on the same facts; and © the identity relief case filed with the Regional Trial Court involved the same parties and issues. The Metropolitan Trial Court denied
of the two cases such that judgment Panganiban's motion and the court ordered the parties to submit their position papers. The Metropolitan Trial Court issued its
in one, regardless of which party Decision in the unlawful detainer case in favor of Shell. Panganiban appealed the decision of the Metropolitan Trial Court,
issuccessful, would amount to res and the said appeal is still pending with the Regional Trial Court of Caloocan City. Subsequently, the Regional Trial Court
judicata in the other. ordered the dismissal of the petition for declaratory relief. Panganiban filed a motion for reconsideration of the Order, but was
denied. Panganiban filed a petition for review with the Supreme Court. The Court, however, referred the petition to the Court
Litis pendentia as a ground for the of Appeals, which upheld the order of the trial court dismissing the petition for declaratory relief on the ground of litis
dismissal of a civil action refers to pendentia. Hence, the present petition.
that situation wherein another action
is pending between the same parties Issue: Whether the CA erred in affirming RTC’s Dismissal of the Civil Case on motion of shell on the ground of litis
for the same cause of actions and pendentia which was filed long after Shell had filed its answer.
that the second action becomes
unnecessary and vexatious. We Held: No. The CA is correct in sustaining the dismissal of the action for declaratory relief to give way to the ejectment suit.
have set the relevant factors that a The requirement that a motion to dismiss should be filed within the time for filing the answer is not absolute. Even after an
court must consider when it has to answer has been filed, a defendant can still file a motion to dismiss on the following grounds: (1) lack of jurisdiction, (2) litis
determine which case should be pendentia (3) lack of cause of action, and (4) discovery during trial of evidence that would constitute a ground for dismissal.
dismissed given the pendency of two Litis pendentia is also one of the grounds that authorize a court to dismiss a case motu proprio. In this case, the bona fide
actions. These are: existence of litis pendentia is beyond dispute.

"(1) the date of filing, with preference The following requisites of litis pendentia are present in this case: (a) the identity of parties, or at least such as representing
generally given to the first action filed the same interests in both actions; (b) the identity of rights asserted and relief prayed for, the relief being founded on the
to be retained; same facts; and © the identity of the two cases such that judgment in one, regardless of which party issuccessful, would
amount to res judicata in the other.
(2) whether the action sought to be
dismissed was filed merely to Petitioner questions the preference given by the Regional Trial Court and the Court of Appeals to the unlawful detainer case
preempt the latter action or to filed by private respondent. Petitioner maintains that based on priority in time, the action for declaratory relief, the case filed
anticipate its filing and lay the basis earlier, should not have been abated in favor of the ejectment suit, a case filed much later.
for its dismissal; and
Litis pendentia as a ground for the dismissal of a civil action refers to that situation wherein another action is pending
(3) whether the action is the between the same parties for the same cause of actions and that the second action becomes unnecessary and vexatious.
appropriate vehicle for litigating the We have set the relevant factors that a court must consider when it has to determine which case should be dismissed given
issues between the parties." the pendency of two actions. These are: "(1) the date of filing, with preference generally given to the first action filed to be
retained; (2) whether the action sought to be dismissed was filed merely to preempt the latter action or to anticipate its filing
and lay the basis for its dismissal; and (3) whether the action is the appropriate vehicle for litigating the issues between the
parties."

The mere fact that the action for declaratory relief was filed earlier than the case for unlawful detainer does not necessarily
mean that the first case will be given preference. In Cruz v. Court of Appeals, we have ruled that the earlier case can be
dismissed in favor of the later case if the later case is the more appropriate forum for the ventilation of the issues between
the parties. Clearly, the interpretation of a provision in the SLDA as to when the SLDA would expire is the key issue that
would determine petitioner's right to possess the gasoline service station. When the primary issue to be resolved is physical
possession, the issue should be threshed out in the ejectment suit, and not in any other case such as an action for
declaratory relief to avoid multiplicity of suits. Indeed, the action for declaratory relief had become vexatious. It would have
been an exercise in futility for the Regional Trial Court to continue the proceedings in the action for declaratory relief when
the Metropolitan Trial Court had already ruled that the term of the SLDA was for only five years or until July 31, 1995.
Moreover, the decision of the Metropolitan Trial Court once it attains finality would amount to res judicata. The proper forum
for petitioner to clarify the provision of the SLDA on the expiration of the term of the contract is in her appeal of the decision
of the Metropolitan Trial Court in the unlawful detainer case.

32. SUNVILLE The doctrine of exhaustion of Facts: The petitioner was granted a Timber License Agreement (TLA), authorizing it to cut, remove and utilize timber within
TIMBER PRODUCTS administrative remedies calls for the concession area covering 29,500 hectares of forest land in Zamboanga del Sur, for a period of ten years expiring on
INC. V. ABAD [GR resort first to the appropriate September 31, 1992. On July 31, 1987, the herein private respondents filed a petition with the DENR for the cancellation of
85502] 24 FEB 1992 administrative authorities in the the TLA on the ground of serious violations of its conditions and the provisions of forestry laws and regulations. The same
LIMIN resolution of a controversy falling charges were subsequently made, also by the herein private respondents, in a complaint for injunction with damages against
under their jurisdiction before the the petitioner. Among others, the petitioner moved to dismiss this case on the grounds that the plaintiffs had not yet
same may be elevated to the courts exhausted administrative remedies. RTC Judge Respondent denied the motion to dismiss. The petitioner then elevated the
of justice for review. Non-observance matter to the Respondent CA, which sustained the decision of the trial court. The Respondent court cited, in support of their
of the doctrine results in lack of a conclusion that this case falls under the exceptions of the doctrine, the case of De Lara v. Cloribel, where "irreparable
cause of action, which is one of the damage and injury" was allowed as an exceptional ground, and Arrow Transportation Corporation v. Board of
grounds allowed in the Rules of Transportation, where the doctrine was waived because of "the strong public interest in having the matter settled" as soon as
Court for the dismissal of the possible.
complaint. However, the deficiency is
not jurisdictional. Issue: W/N the exceptions to the doctrine of exhaustion of administrative remedies applies in the case at bar as held by the
lower courts.

Held: No. The doctrine of exhaustion of administrative remedies calls for resort first to the appropriate administrative
authorities in the resolution of a controversy falling under their jurisdiction before the same may be elevated to the courts of
justice for review. Non-observance of the doctrine results in lack of a cause of action, which is one of the grounds allowed in
the Rules of Court for the dismissal of the complaint. The deficiency is not jurisdictional. Failure to invoke it operates as a
waiver of the objection as a ground for a motion to dismiss and the court may then proceed with the case as if the doctrine
had been observed. The private respondents now submit that their complaint comes under the exceptions of the doctrine
because forestry laws do not require observance of the doctrine as a condition precedent to judicial action; the question they
are raising is purely legal; application of the doctrine will cause great and irreparable damage; and public interest is
involved.

The case at bar does not fall under the exceptions for the following reasons: 1.) There would still be the explicit language of
pertinent laws vesting in the DENR the power and function"to regulate the development, disposition, extraction, exploration
and use of the country's forests" and "to exercise exclusive jurisdiction" in the "management and disposition of all lands of
the public domain," and in the Forest Management Bureau the responsibility for the enforcement of the forestry laws and
regulations here claimed to have been violated. This comprehensive conferment clearly implies at the very least that the
DENR should be allowed to rule in the first instance on any controversy coming under its express powers before the courts
of justice may intervene.

2.) The argument that the questions raised in the petition are purely legal is also not acceptable. The private respondents
have charged that the petitioner has violated the terms and conditions of the TLA and the provisions of forestry laws and
regulations. The charge involves factual issues calling for the presentation of supporting evidence. Such evidence is best
evaluated first by the administrative authorities, employing their specialized knowledge of the agreement and the rules
allegedly violated.

3.) As for the alleged urgent necessity for judicial action and the claimed adverse impact of the case on the national interest,
the record does not show that the petitioners have satisfactorily established these extraordinary circumstances to justify
deviation from the doctrine of exhaustion of administrative remedies and immediate resort to the courts of justice. In fact, this
particular submission must fall flat against the petitioner's uncontested contention that it has since 1988 stopped its
operations under the TLA in compliance with the order of the DENR. In the memorandum filed by the petitioner with this
Court, it is informed that "the Secretary of the DENR suspended petitioner's logging operations until further investigation. The
suspension is still in force up to this date after the lapse of almost 3 years.

There is no question that the case at bar comes within the jurisdiction of the respondent court. Nevertheless, as the wrong
alleged in the complaint was supposedly committed as a result of the unlawful logging activities of the petitioner, it will be
necessary first to determine whether or not the TLA and the forestry laws and regulations had indeed been violated. To
repeat for emphasis, determination of this question is the primary responsibility of the Forest Management Bureau of the
DENR.

33. HACIENDA
BIGAA INC. V. CHAVEZ Where no identity of causes of FACTS: On June 5, 1996, petitioner Hacienda Bigaa, Inc. filed with the Municipal Trial Court of Calatagan,
[GR 174160] 20 APR action but only identity of issues Batangas a complaint for ejectment (forcible entry) and damages with application for writ of preliminary injunction
2010 exists, res judicata comes under against respondent Epifanio V. Chavez (Chavez), docketed as Civil Case No. 129. The complaint alleged that
NAKAGAWA
the second concept – i.e., under Chavez, by force, strategy and/or stealth, entered on April 29, 1996 the premises of Hacienda Bigaa's properties
conclusiveness of judgment. covered by Transfer Certificate of Title (TCT) Nos. 44695 and 56120 by cutting through a section of the barbed
Under this concept, the rule bars wire fence surrounding the properties and destroying the lock of one of its gates, subsequently building a house
the re-litigation of particular facts on the property, and occupying the lots without the prior consent and against the will of Hacienda Bigaa.
or issues involving the same
parties even if raised under The records show that the lots were originally covered by TCT No. 722 owned by Ayala y Cia and/or Alfonso,
different claims or causes of Jacobo and Enrique Zobel, with an area of 9,652.583 hectares, known as Hacienda Calatagan. Ayala and/or the
action. Conclusiveness of Zobels expanded TCT No. 722 to cover an additional 2,000 hectares of land consisting, among others, of beach,
judgment finds application when foreshore and bay areas, and navigable waters, making it appear that these excess areas are part of Hacienda
a fact or question has been Calatagan's TCT No. 722. The Ayalas and/or the Zobels later ordered the subdivision of the hacienda, including
squarely put in issue, judicially these excess areas, and sold the subdivided lots to third parties.
passed upon, and adjudged in a
former suit by a court of Among the buyers or transferees of the expanded and subdivided areas was Hacienda Bigaa which caused the
competent jurisdiction. issuance of titles – TCT Nos. 44695 and 56120 – under its name covering the purchased subdivided areas. Thus,
in his answer before the MTC of Calatagan, then defendant (now respondent) Epifanio V. Chavez alleged that
then plaintiff (now petitioner) Hacienda Bigaa is the successor-in-interest of Ayala y Cia, Hacienda Calatagan,
Alfonso Zobel, Jacobo Zobel and Enrique Zobel – the original titular owners of TCT No. 722.

Portions of the same lands – foreshore lands – were leased out by the Republic, through the Bureau of Fisheries,
to qualified applicants in whose favor fishpond permits were issued. The government-issued fishpond permits
pertaining to lands covered by titles derived from TCT No. 722 of Ayala y Cia and/or the Zobels, gave rise to
ownership and/or possessory disputes between the owners of Hacienda Calatagan and their privies and/or
successors-in-interest, on the one hand, and the Republic or its lessees or fishpond permittees, on the other.

Suits were filed in various courts in Batangas for the recovery of the areas in excess of the area originally
covered by TCT No. 722, which suits ultimately reached the Supreme Court. In the Court's 1965 decisions in
Dizon v. Rodriguez (for quieting of title) and Republic v. Ayala y Cia and/or Hacienda Calatagan, et al. (for
annulment of titles), the excess areas of TCT No. 722 were categorically declared as unregisterable lands of the
public domain such that any title covering these excess areas are necessarily null and void. In these cases, the
Ayalas and the Zobels were found to be mere usurpers of public domain areas, and all subdivision titles issued to
them or their privies and covering these areas were invalidated; the wrongfully registered public domain areas
reverted to the Republic. In Dizon, the Court declared as void the Zobels' TCT No. 2739 and its derivative titles
covering subdivision Lots 1 and 49 – areas sold to the Dizons – as areas in excess of TCT No. 722 and are
properly part of the public domain. In Ayala y Cia, the Court invalidated TCT No. 9550 and "all other subdivision
titles" issued in favor of Ayala y Cia and/or the Zobels of Hacienda Calatagan over the areas outside its private
land covered by TCT No. 722. These areas, including the lots covered by TCT No. 9550, reverted to public
dominion.

The pronouncement in the above cases led to the Court's 1988 decision in Republic v. De los Angeles, a case
covering the same excess areas under a reinvindicatory claim of the Republic aimed at recovering lands usurped
by the Ayalas and the Zobels and at placing the Republic’s lessees and fishpond permittees in possession. The
Court effectively held that as owner of the excess lands, the Republic has the right to place its lessees and
fishpond permittees – among them Zoila de Chavez, predecessor-in-interest of Chavez – in possession. The
Court invalidated TCT Nos. 3699 and 9262 for being among the "other subdivision titles" declared void and
ordered reverted to public dominion.

To return to the forcible entry case, then defendant (now respondent) Chavez alleged in his answer before the
MTC of Calatagan that his mother, Zoila de Chavez (who died intestate on September 14, 1979) was a fishpond
permittee/lessee under Fishpond Permit Nos. F-4572-0 and F-24735 issued by the Bureau of Fisheries on April
21, 1959 and June 3, 1966, respectively; that the areas covered by the permits are the same parcels of land
which he presently occupies as Zoila's successor-in-interest and which Hacienda Bigaa also claims.

Chavez likewise asserted that Hacienda Bigaa is the successor-in-interest of Ayala y Cia, Hacienda Calatagan,
Alfonso Zobel, Jacobo Zobel and Enrique Zobel who owned land with an area of 9,652.583 hectares, covered by
TCT No. 722 in the Registry of Deeds of Batangas; that Ayala y Cia, the Zobels, or Hacienda Calatagan, illegally
expanded the original area of TCT No. 722 by 2,000 hectares; that suits were filed to recover the expanded area;
that these suits reached the Supreme Court which declared that these excess areas are part of the public domain
and ordered their reversion to the Republic; that the Supreme Court likewise declared certain TCTs covering the
subdivision lots outside the area of TCT No. 722 and issued to transferees as null and void; therefore, Hacienda
Bigaa's titles – TCT Nos. 44695 and 56120 – carry no probative value as they are of dubious origins and have
been nullified by the Supreme Court. 11

Chavez further argued that the suit is barred by prior judgment in two prior cases – (1) Civil Case No. 78, a suit
for unlawful detainer filed by the Zobels against Chavez’s predecessor-in-interest, Zoila de Chavez, before the
then Justice of the Peace Court of Calatagan, Batangas; and (2) Civil Case No. 653, a case of accion
reinvindicatoria with prayer for preliminary mandatory injunction filed by the Republic, Zoila de Chavez, and other
lessees or fishpond permittees of the Republic, against Enrique Zobel (Hacienda Bigaa's predecessor-in-interest)
before the then Court of First Instance of Batangas. This case reached this Court as G.R. No. L-30240 entitled
"Republic of the Philippines v. De los Angeles, Enrique Zobel, et al." and was decided in 1988. Chavez asserts
that the subject matter and the issues involved in these cases are squarely similar and/or identical to the subject
matter and issues involved in the present forcible entry suit; the rulings in these two cases, therefore constitute
res judicata with respect to the present case.

ISSUE: WON THERE IS RES JUDICATA

HELD: YES. The doctrine of res judicata is set forth in Section 47 of Rule 39 of the Rules of Court, which in its
relevant part reads:

Sec. 47. Effect of judgments or final orders. — The effect of a judgment or final order rendered by a court of the
Philippines, having jurisdiction to pronounce the judgment or final order, may be as follows:

(b) In other cases, the judgment or final order is, with respect to the matter directly adjudged or as to any other
matter that could have been raised in relation thereto, conclusive between the parties and their successors in
interest by title subsequent to the commencement of the action or special proceeding, litigating for the same thing
and under the same title and in the same capacity; and

(c) In any other litigation between the same parties or their successors in interest, that only is deemed to have
been adjudged in a former judgment or final order which appears upon its face to have been so adjudged, or
which was actually and necessarily included therein or necessary thereto.

This provision comprehends two distinct concepts of res judicata: (1) bar by former judgment and (2)
conclusiveness of judgment. Under the first concept, res judicata absolutely bars any subsequent action when
the following requisites concur: (a) the former judgment or order was final; (b) it adjudged the pertinent issue or
issues on their merits; (c) it was rendered by a court that had jurisdiction over the subject matter and the parties;
and (d) between the first and the second actions, there was identity of parties, of subject matter, and of causes of
action.

Where no identity of causes of action but only identity of issues exists, res judicata comes under the second
concept – i.e., under conclusiveness of judgment. Under this concept, the rule bars the re-litigation of particular
facts or issues involving the same parties even if raised under different claims or causes of action.
Conclusiveness of judgment finds application when a fact or question has been squarely put in issue, judicially
passed upon, and adjudged in a former suit by a court of competent jurisdiction. The fact or question settled by
final judgment or order binds the parties to that action (and persons in privity with them or their successors-in-
interest), and continues to bind them while the judgment or order remains standing and unreversed by proper
authority on a timely motion or petition; the conclusively settled fact or question furthermore cannot again be
litigated in any future or other action between the same parties or their privies and successors-in-interest, in the
same or in any other court of concurrent jurisdiction, either for the same or for a different cause of action. Thus,
only the identities of parties and issues are required for the operation of the principle of conclusiveness of
judgment.

While conclusiveness of judgment does not have the same barring effect as that of a bar by former judgment that
proscribes subsequent actions, the former nonetheless estops the parties from raising in a later case the issues
or points that were raised and controverted, and were determinative of the ruling in the earlier case. In other
words, the dictum laid down in the earlier final judgment or order becomes conclusive and continues to be
binding between the same parties, their privies and successors-in-interest, as long as the facts on which that
judgment was predicated continue to be the facts of the case or incident before the court in a later case; the
binding effect and enforceability of that earlier dictum can no longer be re-litigated in a later case since the issue
has already been resolved and finally laid to rest in the earlier case.

a. Identity of Parties

As already stated above, the parties to the present case are virtually the same as those in the antecedent cases.
Specifically in De los Angeles, the parties were Enrique Zobel, the predecessor-in-interest of petitioner Hacienda
Bigaa, and Zoila de Chavez, the mother and predecessor-in-interest of Chavez.

b. Identity of Subject Matter

Hacienda Bigaa and Chavez are litigating the same properties subject of the antecedent cases inasmuch as they
claim better right of possession to parcels of land covered by subdivision titles derived from Hacienda
Calatagan's TCT No. 722 and by government-issued fishpond permits. Specifically in De los Angeles, the Zobels
and Zoila de Chavez litigated the disputed lots covered by subdivision titles in Zobel’s name and by fishpond
permits the Republic issued in favor of de Chavez.

In ruling that the subject lots are the same lots litigated in the previously decided cases, the courts below based
their findings on De los Angeles that in turn was guided by our rulings in Dizon and Ayala y Cia. For emphasis,
we reiterate our ruling in De los Angeles: all areas the Ayalas and/or the Zobels made to appear to be covered by
TCT No. 722 are owned by the Republic because they form part of the public domain; specifically, portions of the
navigable water or of the foreshores of the bay converted into fishponds are parts of the public domain that
cannot be sold by the Ayalas and/or the Zobels to third parties.

In his answer before the MTC, Chavez asserted that the areas covered by the fishpond permits of Zoila de
Chavez are the same parcels of land that he now occupies as Zoila's successor-in-interest. Given the rulings in
the antecedent cases that Chavez invoked, Hacienda Bigaa never bothered to object to or to rebut this allegation
to show that the presently disputed lots are not part of the expanded areas that, apart from the specifically
described titles, Ayala y Cia described as "other subdivision titles" covering unregisterable lands of the public
domain that must revert to the Republic. Hacienda Bigaa should have objected as we held in De los Angeles that
the onus is on Ayala and the Zobels – Hacienda Bigaa’s predecessors-in-interest – to show that their titles do not
cover the expanded areas whose titles were declared null and void. We find no cogent reason to depart from our
past rulings in the antecedent cases, and from the ruling of the courts below in this case that the lots claimed by
Hacienda Bigaa are the same lots covered by our rulings in the antecedent cases.

c. Identity of Issues

This case and the antecedent cases all involve the issue of ownership or better right of possession.

34. CHU V. SPS.


CUNANAN [GR 156185] Absolute identity of parties is not a Facts:
12 SEPT 2001 condition sine qua non for res
NATO judicata to apply because a shared The Chus executed a deed of sale with assumption of mortgage involving five (5) parcels of land in favor of the Cunanans
identity of interest sufficed. stipulating that ownership would remain with the Chus until complete payment and total compliance with the terms of the
deed of sale with mortgage. Thereafter, the Chus executed and SPA authorizing the Cunanans to borrow money from any
banking institution and to mortgage the five (5) lots as security, then to deliver the proceeds to the Chus net of the balance of
the mortgage and downpayment. The Cunanans were able to transfer the title of the five lots in her name without the
knowledge of the Chus and to borrow money with the lots as security without paying the balance of the price to the Chus.
She later transferred two of the lots to the Carloses and three to Cool Town Realty, despite the annotation of a vendor’s lien
on the last three lots. The Chus commenced a civil case in the RTC to recover the unpaid balance from the Cunanans. Five
(5) years later they amended the complaint seeking annulment of the deed of sale with assumption of mortgage and the
TCTs issued pursuant to the deed with damages. They impleaded Cool Town Realty and the RD of Pampanga. The
Carloses had meanwhile sold the two lots to Benelda Estate Development, so the Chus amended the complaint impleading
Benelda. Benelda filed its answer with a motion to dismiss, claiming that the amended complaint stated no cause of action
because they had acted in good faith in buying the lots. Their motion was denied by the RTC, so they assailed the denial on
certiorari in the CA, which annulled the RTC’s decision for being tainted with grave abuse and dismissing the civil case
against Benelda. The Chus, Cunanans and Cool Town Realty entered into a compromise agreement. The RTC approved it.
After, the Chus brought another suit against the Carloses and benelda, seeking cancellation of the TCTs plus damages.
They amended this complaint to included the Cunanans. The Cunananas and Benelda moved to dismiss mainly on the
reason that the action was barred by res judicata. The RTC denied the motions, holding that the amended complaint stated a
cause of action against all defendants; that the action was not barred by res judicata because there was no identity of parties
and subject matter with the prior case. On appeal the CA granted the petition for certiorari nullifying the RTC’s orders, ruling
that the compromise agreement had ended the controversy and that the filing of the case violated the rule against splitting of
a cause of action, rendering the case subject to a motion to dismiss on bar by res judicata.

Issue: Was the case barred by res judicata, although Benelda was not a party to the compromise agreement?

Ruling:

Petition denied. The compromise agreement was not limited merely to the three lots sold to Cool Town, it included the two
sold to Benelda as it would contravene the object of the civil case to enforce or rescind the deed of sale. It was apparent that
petitioners were guilty of splitting their single cause of action to enforce or rescind the deed of sale with assumption of
mortgage. Their splitting violated the policy against multiplicity of suit, the purpose of which is to avoid unduly burdening the
courts. Their contravention of the policy merited the dismissal of the case on the ground of bar by res judicata. In order that
res judicata may be invoked, the following requisites must concur:

a. The former judgment must be final;

b. It must have been rendered by a court having jurisdiction over the subject matter or the parties

c. It must be a judgment on the merits;

d. There must be between the first and second actions:

a. Identity of parties

b. Identity of subject matter

c. Identity of cause of action

Although Benelda was not made a party to the compromise agreement is inconsequential as they were deemed successors
in interest of the Cunanans. It has been held that absolute identity of parties is not a condition sine qua non for res judicata
to apply because a shared identity of interest sufficed. As both actions involved the same properties the requisite of same
subject matter was met, and it is obvious that there has been a final judgment on the compromise agreement. Hence, the
Chus are guilty of splitting their cause of action. They should not be allowed to make a single cause of action the basis of
several suits.

35. GALINDO V. The general rule is that only those FACTS:


HEIRS OF ROXAS [GR parties in a case and their privies When Marciano A. Roxas died intestate, he was survived by his widow, Cirila Roxas and their nine children, namely,
147969] 17 JAN 2005 and successors-in-interest are Maximiano, Virginia, Benjamin, Eleazar, Elisa, Prescilla, Fortunato, Lydia and Uriel, all surnamed Roxas; and the
PEREZ bound by the order or decision of children of their son Vicente, who predeceased Marciano, namely, Rosalinda, Vicente, Jr. and Sergio, all surnamed
the trial court. Persons or entities Roxas.
who are not parties to the case are On April 14, 1955, the said heirs filed an action for specific performance against the heirs of Gregorio Galindo,
not and should not be bound or namely, Florencio, Felisa, Mercedes and Urbano, all surnamed Galindo, including his grandson Federico de
adversely affected by the said Guzman, with the then Court of First Instance (CFI) of Bulacan to compel the latter to execute a deed of absolute
order or decision; otherwise, they sale over Lot 1048 located in Sta. Maria, Bulacan. The said lot had an area of 48,089 square meters and was covered
will be deprived of their right to by Transfer Certificate of Title (TCT) No. T-2145. The case was docketed as Civil Case No. 1067. After the trial, the
due process. following facts emerged:
The absence of an indispensable Lot 1048 which formed part of the Sta. Maria de Pandi Estate was possessed by the late Gregorio Galindo who,
party renders all subsequent during his lifetime, had been paying rentals thereon to the government. On July 5, 1911, the Government of the
actions of the court null and void Philippines, through the then Acting Director of Lands, agreed to sell to Gregorio Galindo the said Lot 1048 payable
for want of authority to act, not in annual installments of P42.00 (Exhibit "F"). On July 4, 1916, Gregorio Galindo died intestate leaving as his only
only as to the absent parties but heirs his children Florencio, Felisa, Mercedes and Urbano, all surnamed Galindo, and a grandson Federico de
even as to those present. Guzman. Subsequently on December 1, 1916, Florencio, Felisa, Mercedes, all surnamed Galindo, and Federico de
Section 1, Rule 3 of the Rules of Guzman, represented by his father Luis de Guzman, sold whatever rights and interests Gregorio Galindo had in Lot
Court provides that only persons 1048 in favor of Marciano A. Roxas (Exhibit "B," deposition).
or juridical persons or entities Urbano Galindo, being then a minor, did not sign the document. In the said document, the signatories thereto
authorized by law may be parties obligated themselves to transfer the land to Marciano A. Roxas as soon as it would become feasible to do so, and
in a civil action. Section 4, Rule 8 as security that Urbano Galindo would ratify the same upon reaching the age of majority, Florencio Galindo ceded
of the said Rules further provides to Marciano A. Roxas title to Lot 833.
that facts showing the capacity of Since the execution of the document Exhibit "B"-deposition, possession and enjoyment of Lot 1048 were
a party to sue or be sued or the immediately transferred to Marciano A. Roxas but the possession and enjoyment of Lot 833, although given as
authority of a party to sue or be security by Florencio Galindo, remained in the latter. On May 23, 1931, Urbano Galindo, who was a minor at the time
sued in a representative capacity of the execution of the first document, executed an affidavit signifying his conformity to the sale executed by his
must be averred in the complaint. brother and sisters.
In order to maintain an action in a Since Marciano A. Roxas took possession of Lot 1048 he had been religiously paying the installments due to the
court of justice, the plaintiff must government. While the receipts for payment of the installments due to the government were issued in the name of
have an actual legal existence, Gregorio Galindo, yet the property had been declared for tax purposes in the name of Marciano A. Roxas who paid
that is, he or she or it must be a the real estate taxes thereon during his lifetime.
person in law and possessed of a On February 13, 1948, Transfer Certificate of Title No. T-2145 covering the property in question was issued by the
legal entity as either a natural or Register of Deeds of Bulacan in the name of the legal heirs of Gregorio Galindo. This, on account of the fact that the
an artificial person, and no suit sale of the right of Gregorio Galindo made by his heirs in favor of Marciano A. Roxas was not registered with the
can lawfully be prosecuted in the Bureau of Lands because of the minority of Urbano Galindo at the time.
name of that person.18 The party In the light of the facts and circumstances surrounding this case, the Court is convinced that plaintiffs are entitled
bringing suit has the burden of to the title over Lot 1048. Plaintiffs' deposition which is the "Documento de Compromiso" clearly states that the
proving the sufficiency of the signatories thereto sold whatever rights they have to the property in favor of plaintiffs' predecessor Marciano A.
representative character that he Roxas, obligating themselves to execute the final deed of sale must have been occasioned by the minority of
claims. If a complaint is filed by Urbano Galindo who was not made a signatory to the document. But on May 23, 1931, when Urbano Galindo was
one who claims to represent a already of age, he ratified the actuation of his brother and sisters by executing an affidavit whereby he signified that
party as plaintiff but who, in fact, he was agreeable thereto.
is not authorized to do so, such Since 1931 up to the date of the filing of the complaint, 24 years had elapsed without any of the defendants having
complaint is not deemed filed and taken any step to nullify the two documents aforementioned.
the court does not acquire Florencio Galindo's testimony contained in his deposition taken at the instance of plaintiffs is quite enlightening to
jurisdiction over the complaint. An the mind of the Court. He had acknowledged in all sincerity that he and his brother and sisters have, indeed, sold
unauthorized complaint does not their rights to Lot 1048 in favor of Marciano A. Roxas. Without prejudice, therefore, to defendants taking the
produce any legal effect.19 necessary legal steps to recover title to Lot 833 which admittedly was only placed has guarantee that defendant
Corollary, the defendants can Urbano Galindo would ratify the sale made by his brother and sisters.
assail the facts alleged in the On August 12, 1965, the trial court rendered judgment2 in favor of the plaintiffs.
complaint through a motion to The defendants appealed to the Court of Appeals (CA) which rendered judgment on February 5, 1973, affirming the
dismiss on the ground that the decision of the CFI. The decision became final and executory. However, the said decision was not enforced under
plaintiff has no capacity to sue Section 10, Rule 39 of the Rules of Court.
under Section 1(d) of Rule 16 of In the meantime, Urbano Galindo died intestate. On December 16, 1997, Felicissima, the widow of Urbano, and their
the Rules of Court, that is, that he children, Nestor, Juanita, Beatriz, Catalina, Danilo, Librada and Cesar, all surnamed Galindo, executed an
does not have the representative "Extrajudicial Settlement of the Estate of the Deceased Urbano Galindo and/or the Deceased Gregorio Galindo with
he claims.20 Waiver of Rights" in which they adjudicated unto themselves, as owners, the undivided rights and interests which
they claimed Urbano had over Lot 1048. In the said deed, they also waived their respective claims over the lot in
Section 1, Rule 7 of the Rules of favor of Juanita Galindo Rivera. The latter, as one of the legal heirs of Gregorio, undertook to preserve the property
Court, likewise, provides that the left by the said deceased and, thereafter, executed an affidavit of loss of the owner's copy of TCT No. T-2145 and,
names of the parties should be on the basis thereof, secured TCT No. 335593(M) over the property in her name on September 8, 1998.
included in the caption of the On April 7, 1999, a complaint was filed by the Heirs of Marciano Roxas, through Reginald S. Roxas, as plaintiffs,
original complaint. against Juanita Galindo Rivera in the Regional Trial Court (RTC) of Bulacan, for annulment of documents,
cancellation of title and damages with prayer for a writ of preliminary injunction.
A deceased person does not have The complaint alleged, inter alia, that: Reginald S. Roxas was the representative of the heirs of Marciano Roxas who
such legal entity as is necessary died on June 4, 1950, the extrajudicial settlement of the estates of the deceased Gregorio and Urbano Galindo
to bring action so much so that a executed in favor of the defendant, the affidavit of loss executed by her and TCT No. 335593(M) issued by the
motion to substitute cannot lie Register of Deeds are null and void for being contrary to the decision of the CFI in Civil Case No. 1067 declaring
and should be denied by the Marciano Roxas to be the lawful owner of the property, which was affirmed by the CA in CA-G.R. No. 37697-R and
court. An action begun by a had long become final and executory.
decedent's estate cannot be said The defendant filed a motion to dismiss the complaint on the following grounds:
to have been begun by a legal I. THAT THE PLAINTIFFS HAVE NO LEGAL CAPACITY TO SUE.
person, since an estate is not a II. THAT THE CAUSE OF ACTION IS BARRED BY STATUTE OF LIMITATIONS.
legal entity; such an action is a On the first ground, the defendant alleged that there was no allegation in the complaint that the heirs of Marciano
nullity and a motion to amend the Roxas had authorized Reginald S. Roxas to file the complaint against her in their behalf.
party plaintiff will not, likewise, lie, On the second ground, the defendant alleged that the plaintiffs' cause of action based on the decision of the CFI
there being nothing before the dated August 12, 1965 and affirmed by the CA, on February 5, 1973, not having been enforced was barred by the
court to amend. Considering that statute of limitations. Citing Article 1144 of the New Civil Code, the defendant argued that the plaintiffs should have
capacity to be sued is a filed their complaint within ten years from the finality of the said decision of the CFI. The defendant, likewise, cited
correlative of the capacity to sue, the decision of this Court in Heirs of Juan Sindiong v. Committee on Burnt Areas and Improvements of Cebu to
to the same extent, a decedent support her submission that a final and executory decision must be enforced within ten years from the finality
does not have the capacity to be thereof.
sued and may not be named a The trial court issued an Order denying the motion on September 7, 1999. It ruled that based on the plaintiffs'
party defendant in a court action. allegation in the complaint that they were the legitimate children of the deceased Marciano Roxas, such plaintiffs
are the real parties-in-interest. On the second ground, the trial court ruled that the complaint of the plaintiffs was an
The nature of an action and the action for quieting of title, the prescriptive period for which commenced only in 1998.
court which has jurisdiction over In her motion for the reconsideration of the order, the defendant averred, this time, that the action of the plaintiffs
the case are determined by the was one for the enforcement of a constructive trust which prescribed in ten years.
material allegations of the The defendant also reiterated her original submission that the complaint was one to enforce the decision of the CFI
complaint, the law existing at the which was affirmed by the CA; hence, it prescribes within ten years from the finality of the said decision. She also
time of the filing of the action and maintained that the action was barred by the decision of the RTC and the CA.
the character of the relief prayed The trial court denied the motion of the defendant in its Order dated November 26, 1999.
for by the plaintiff irrespective of The defendant, now the petitioner, joined by Felicissima, Nestor, Beatriz, Catalina, Danilo, Librada and Cesar, all
whether or not the plaintiff is surnamed Galindo, filed a petition for certiorari in the CA, assailing the September 7, 1999 and the November 26,
entitled to such relief prayed for. 1999 Orders of the trial court. However, the CA rendered judgment on April 4, 2001, denying due course and
The jurisdiction of the court dismissing the petition, on the ground that the petitioners failed to show that the RTC committed a grave abuse of
cannot be made to depend upon discretion in denying the motion to dismiss. The appellate court, likewise, ruled that even if the assailed orders of
the consent or waiver of the the RTC were erroneous, the same were merely errors of judgment, correctable through an appeal by writ of error.
parties. Neither is the caption of The petitioners now seek relief from the Court via a Petition for Review on Certiorari , contending that the CA erred
the complaint determinative of the in denying due course and dismissing their Petition for Certiorari. In support of their petition at bar, the petitioners
nature of an action. reiterated the same grounds and arguments which they raised in the CA.
In their comment on the petition, the respondents assert that the petitioners, except petitioner Juanita Galindo
Rivera, were not parties in the RTC and the CA; hence, are not proper parties as petitioners. They also aver that the
grounds raised by the petitioners in their petition are factual, besides being matters of defense which should be
threshed out and resolved only after trial.
ISSUES:
(1) whether Felicissima, Nestor, Beatriz, Catalina, Danilo, Librada and Cesar, all surnamed Galindo, are proper
parties as petitioners in this case; and
(2) whether the trial court committed a grave abuse of its discretion in denying petitioner Juanita Galindo Rivera's
motion to dismiss on the ground that Reginald S. Roxas had no legal capacity to sue for and in behalf of the heirs
of Marciano Roxas, and that the action of the respondents had already prescribed when they filed their complaint
on April 7, 1999.

HELD:
1. On the first issue, the respondents aver that of the eight petitioners in this case, only petitioner Juanita
Galindo Roxas was the petitioner in the CA and the sole defendant-movant in the RTC. The seven other petitioners
were not parties in the CA and in the RTC. Hence, the respondents aver, the said seven petitioners are not bound by
the orders of the trial court; as such, they are not the proper parties in this case as petitioners.

While we agree with the respondents' premise that the petitioners herein, except petitioner Juanita Galindo Rivera,
were not parties-defendants in the RTC and parties-petitioners in the CA, and, as a general rule, are not proper
parties as petitioners in this case, the Court finds and so holds that they are indispensable parties and should be
impleaded as parties-petitioners in this case.

The general rule is that only those parties in a case and their privies and successors-in-interest are bound by the
order or decision of the trial court. Persons or entities who are not parties to the case are not and should not be
bound or adversely affected by the said order or decision; otherwise, they will be deprived of their right to due
process. Since the petitioners, except petitioner Juanita Galindo Rivera, were not parties in the RTC and in the CA,
they are not bound by the assailed orders of the RTC and the decision of the CA against petitioner Juanita Galindo
Rivera; hence, they are not the proper parties to appeal from and assail the said orders of the RTC and the decision
of the CA.

It bears stressing, however, that the respondents, the plaintiffs in the RTC, sought the nullification of the
"Extrajudicial Settlement of the Estate of the Deceased Urbano Galindo and/or the Deceased Gregorio Galindo with
Waiver of Rights" executed by all the petitioners in which they adjudicated unto themselves as co-owners thereof
the rights and interests which they claimed the deceased Urbano Galindo had over Lot 1048 covered by TCT No.
335593(M) which was issued to and under the name of petitioner Juanita Galindo Rivera on the basis of the said
deed, and the retention of TCT No. T-2145 under the names of the heirs of Gregorio Galindo. Thus, all the
petitioners, who executed the said deed, are indispensable parties as parties-defendants in the RTC and as parties-
petitioners in the CA under Section 7, Rule 3 of the Rules of Court, and should have been impleaded by the
respondents in their complaint. Without the presence of the said petitioners as defendants, the trial court could not
validly render judgment and grant relief to the respondents. The failure of the respondents to implead all the
petitioners as parties-defendants constituted a legal obstacle to the trial court and the appellate court's exercise of
judicial power over the said cases and rendered any orders or judgments rendered therein a nullity. The absence of
an indispensable party renders all subsequent actions of the court null and void for want of authority to act, not
only as to the absent parties but even as to those present. The RTC should have ordered the dismissal of the
complaint.
The petitioners themselves must have realized the need to join petitioner Juanita Galindo Rivera as a party-
petitioner without waiting for the court to order the same because they eventually joined petitioner Juanita Galindo
Rivera as such party.

The respondents even failed to implead the seven brothers/sisters of respondent Reginald Roxas, all of whom
inherited the undivided share of their father Eleazar Roxas in the property subject of the deed of extrajudicial
settlement, as well as the children of their uncle Vicente Roxas, namely, Rosalinda, Vicente, Jr. and Sergio, all
surnamed Roxas. Respondent Reginald Roxas, his siblings and the children of Vicente Roxas were co-owners of
the property; hence, they are, likewise, indispensable parties as plaintiffs in the RTC.15 Their absence warranted
the dismissal of the complaint as well.

2. On the second issue, we agree with the petitioners that the RTC committed a grave abuse of its discretion
amounting to excess or lack of jurisdiction in denying their motion to dismiss the complaint on the ground of the
respondents' failure, as plaintiffs, to sufficiently allege in their complaint and prove that Reginald Roxas had the
representative capacity to sue as such representative of all the heirs of the deceased Marciano Roxas.

Section 1, Rule 3 of the Rules of Court provides that only persons or juridical persons or entities authorized by law
may be parties in a civil action. Section 4, Rule 8 of the said Rules further provides that facts showing the capacity
of a party to sue or be sued or the authority of a party to sue or be sued in a representative capacity must be
averred in the complaint. In order to maintain an action in a court of justice, the plaintiff must have an actual legal
existence, that is, he or she or it must be a person in law and possessed of a legal entity as either a natural or an
artificial person, and no suit can lawfully be prosecuted in the name of that person.18 The party bringing suit has
the burden of proving the sufficiency of the representative character that he claims. If a complaint is filed by one
who claims to represent a party as plaintiff but who, in fact, is not authorized to do so, such complaint is not
deemed filed and the court does not acquire jurisdiction over the complaint. An unauthorized complaint does not
produce any legal effect.19 Corollary, the defendants can assail the facts alleged in the complaint through a motion
to dismiss on the ground that the plaintiff has no capacity to sue under Section 1(d) of Rule 16 of the Rules of
Court, that is, that he does not have the representative he claims.20

Section 1, Rule 7 of the Rules of Court, likewise, provides that the names of the parties should be included in the
caption of the original complaint.

In this case, the caption of the complaint of the respondents states that the "Heirs of Marciano Roxas," represented
by Reginald S. Roxas, are the plaintiffs.

As gleaned from the averments of the complaint, of the nine plaintiffs, six are already deceased, namely, Maximiano
Roxas, Benjamin Roxas, Eleazar Roxas, Prescilla Roxas-de Perio, Virginia Roxas-Santos and Uriel Roxas. There is
no allegation in the complaint that a special proceeding to settle the estate of the said deceased had been filed and
was pending. Indeed, neither a dead person nor his estate may be a party-plaintiff in a court action. As explained by
this Court:

'A deceased person does not have such legal entity as is necessary to bring action so much so that a motion to
substitute cannot lie and should be denied by the court. An action begun by a decedent's estate cannot be said to
have been begun by a legal person, since an estate is not a legal entity; such an action is a nullity and a motion to
amend the party plaintiff will not, likewise, lie, there being nothing before the court to amend. Considering that
capacity to be sued is a correlative of the capacity to sue, to the same extent, a decedent does not have the
capacity to be sued and may not be named a party defendant in a court action.

In fine, the deceased Maximiano, Benjamin, Eleazar and Uriel, all surnamed Roxas, Prescilla Roxas-de Perio and
Virginia Roxas-Santos have no capacity to sue and may not be sued as parties-plaintiffs. Neither does respondent
Reginald Roxas have the capacity to represent the said deceased as party-plaintiff, nor is there any allegation in the
complaint that Lydia Roxas, Elisa Medina and Fortunato Roxas are of legal age and have the capacity to sue.

We agree with the ruling of the trial court that the action of the respondents against petitioner Juanita Galindo
Rivera was one for quieting of title under the second paragraph of Rule 63 of the Rules of Court and not one for the
enforcement of the decision of the CFI in Civil Case No. 1067. It bears stressing that the nature of an action and the
court which has jurisdiction over the case are determined by the material allegations of the complaint, the law
existing at the time of the filing of the action and the character of the relief prayed for by the plaintiff irrespective of
whether or not the plaintiff is entitled to such relief prayed for. The jurisdiction of the court cannot be made to
depend upon the consent or waiver of the parties. Neither is the caption of the complaint determinative of the
nature of an action.

As gleaned from the averments of the complaint, the respondents herein, who were the plaintiffs in the RTC, sought
to quiet their ownership over the property and remove any cloud thereon caused by (a) the Deed of Extrajudicial
Settlement of the Estate of Gregorio and Urbano Galindo executed by the petitioners; (b) the Affidavit of Loss
executed by petitioner Juanita Galindo Rivera; and (c) the issuance of TCT No. 335593(M) in favor of the latter.

While it may be true that the respondents prayed, in their complaint, that the said deed and TCT No. 335593(M) be
nullified by the court, it cannot thereby be concluded that the action of the respondents was for the enforcement of
the decision of the CFI in Civil Case No. 1067. The respondents, in fact, did not pray, in their complaint, that the
petitioners be ordered to execute a deed of absolute sale over the property as decreed in the decision of the CFI in
Civil Case No. 1067; that the property covered by the said title be reconveyed to them; or that a new title over the
property be issued to under their names as owners thereof. The plea of the respondents for the nullification of the
said deed and title is but the consequences of and incidental to their plea that their ownership over the lot be not
clouded by the contrary claim of the petitioners. It must be stressed that an action to quiet title is imprescriptible.

IN LIGHT OF ALL THE FOREGOING, the petition is GRANTED. The Decision of the Regional Trial Court and the
assailed Decision of the Court of Appeals are REVERSED AND SET ASIDE. The Regional Trial Court of Bulacan is
ordered to dismiss the complaint. No costs.

SO ORDERED.

36. SWEDISH For a note or memorandum to satisfy Facts:


MATCH V. CA [GR the Statute, it must be complete in Swedish Match AB (SMAB) had 3 subsidiary corporations in the Philippines, all organized under Philippine laws, to wit:
128120] 20 OCT 2004 itself and cannot rest partly in writing Phimco Industries (Phimco), Provident Tree Farms (PTF), and OTT/Louie. STORA, the parent company of SMAB, decided
PURIFICACION and partly in parol. The note or to sell SMAB of Sweden and its worldwide match, lighter, and shaving products operation to Swedish Match NV (SMNV).
memorandum must contain the
names of the parties, the terms and Ed Enriquez, VP of Swedish Match Sociedad Anonimas (SMSA) which is SMAB’s management company, was held under
conditions of the contract, and a strict instructions that the sale of Phimco shares should be executed on or before 30 June 1990 in view of the tight loan
description of the property sufficient covenants of SMNV. He came to the Philippines and informed the Philippine financial and business circles that the Phimco
to render it capable of identification. shares were for sale. Several interested parties tendered offers to acquire the Phimco shares one of which was private
Such note or memorandum must respondent, Antonio Litonjua, the president and general manager of ALS Management & Development Corporation.
contain the essential elements of the
contract expressed with certainty that On November 1989, Litonjua submitted to SMAB a firm offer to buy all of the latter’s shares in Phimco and all of Phimco’s
may be ascertained from the note or shares in PTF and OTT for P750,000,000.00. However, CEO Massimo Rossi informed respondents that their price offer was
memorandum itself, or some other below their expectations. Again, on May 1990, Litonjua offered to buy the disputed shares, excluding the lighter division for
writing to which it refers or within US$36M. Rossi wrote that ALS should undertake a due diligence process or pre-acquisition audit and review of the draft
which it is connected, without contract for the Match and Forestry activities of Phimco at ALS convenience. 2 days prior to the deadline for submission of
resorting to parol evidence. the final bid, Litonjua told Rossi that they would be unable to submit the final offer by 30 June 1990, considering that the
acquisition audit of Phimco and the review of the draft agreements had not yet been completed. Thus, Enriquez sent notice
to Litonjua that they would be constrained to entertain bids from other parties in view of Litonjua’s failure to make a firm
commitment for the shares of Swedish Match. In his letter, Litonjua asserted that they submitted the best bid and that they
were already finalizing the terms of the sale.

More than 2 months from receipt of Litonjua’s last letter, Enriquez advised the former that the proposed sale of SMAB’s
shares in Phimco with local buyers did not materialize. Enriquez then invited Litonjua to resume negotiations with SMAB for
the sale of Phimco shares. He indicated that SMAB would be prepared to negotiate with ALS on an exclusive basis for a
period of 15 days from 26 September 1990 subject to the terms contained in the letter. Additionally, Enriquez clarified that if
the sale would not be completed at the end of the 15-day period, SMAB would enter into negotiations with other buyers.
Litonjua emphasized that the new offer constituted an attempt to reopen the already perfected contract of sale of the shares
in his favor. CA ruled that the series of written communications between petitioners and respondents collectively constitute a
sufficient memorandum of their agreement under Article 1403 of the Civil Code. Thus, letters exchanged by and between the
parties, taken together, were sufficient to establish that an agreement to sell the disputed shares to respondents was
reached. On the other hand, petitioners stress that Litonjua made it clear in his letters that the quoted prices were merely
tentative and still subject to further negotiations between him and the seller. They point out that there was no meeting of the
minds on the essential terms and conditions of the sale because SMAB did not accept respondent’s offer that consideration
would be paid in Philippine pesos. They argued as well that the foregoing circumstances prove that they failed to reach an
agreement on the sale of the Phimco shares.

Issues:
Whether the appellate court erred in reversing the trial court's decision dismissing the complaint for being unenforceable
under the Statute of Frauds. - NO.
Whether the series of communication between petitioners and respondents collectively constitute a sufficient memorandum
of their agreement under Art. 1403 of the Civil Code. - NO.

Held:
The Statute of Frauds embodied in Article 1403, paragraph (2), of the Civil Code requires certain contracts
enumerated therein to be evidenced by some note or memorandum in order to be enforceable. The term "Statute of Frauds"
is descriptive of statutes which require certain classes of contracts to be in writing. The Statute does not deprive the parties
of the right to contract with respect to the matters therein involved, but merely regulates the formalities of the contract
necessary to render it enforceable. Evidence of the agreement cannot be received without the writing or a secondary
evidence of its contents.
The Statute, however, simply provides the method by which the contracts enumerated therein may be proved but
does not declare them invalid because they are not reduced to writing. By law, contracts are obligatory in whatever form they
may have been entered into, provided all the essential requisites for their validity are present. However, when the law
requires that a contract be in some form in order that it may be valid or enforceable, or that a contract be proved in a certain
way, that requirement is absolute and indispensable. Consequently, the effect of non-compliance with the requirement of the
Statute is simply that no action can be enforced unless the requirement is complied with. Clearly, the form required is for
evidentiary purposes only. Hence, if the parties permit a contract to be proved, without any objection, it is then just as binding
as if the Statute has been complied with.
The purpose of the Statute is to prevent fraud and perjury in the enforcement of obligations depending for their
evidence on the unassisted memory of witnesses, by requiring certain enumerated contracts and transactions to be
evidenced by a writing signed by the party to be charged.
However, for a note or memorandum to satisfy the Statute, it must be complete in itself and cannot rest partly in
writing and partly in parol. The note or memorandum must contain the names of the parties, the terms and conditions of the
contract, and a description of the property sufficient to render it capable of identification. Such note or memorandum must
contain the essential elements of the contract expressed with certainty that may be ascertained from the note or
memorandum itself, or some other writing to which it refers or within which it is connected, without resorting to parol
evidence.
Contrary to the Court of Appeals' conclusion, the exchange of correspondence between the parties hardly
constitutes the note or memorandum within the context of Article 1403 of the Civil Code. Rossi's letter dated 11 June 1990,
heavily relied upon by respondents, is not complete in itself. First, it does not indicate at what price the shares were being
sold. In paragraph (5) of the letter, respondents were supposed to submit their final offer in U.S. dollar terms, at that after the
completion of the due diligence process. The paragraph undoubtedly proves that there was as yet no definite agreement as
to the price. Second, the letter does not state the mode of payment of the price. In fact, Litonjua was supposed to indicate in
his final offer how and where payment for the shares was planned to be made.
Evidently, the trial court's dismissal of the complaint on the ground of unenforceability under the Statute of Frauds is
warranted.
Even if we were to consider the letters between the parties as a sufficient memorandum for purposes of taking the
case out of the operation of the Statute the action for specific performance would still fail.

EFFECT OF FAILURE TO PLEAD (RULE 9, SECS. 1 TO 3)


WHEN TO FILE RESPONSIVE PLEADINGS (RULE 11, SECS. 1 TO 11)
 SECS. 5, 6, 9 OF THE 1991 REVISED RULE ON SUMMARY PROCEDURE
 SECS. 9, 11, 12, 13, 14 OF THE REVISED RULES OF PROCEDURE ON SMALL CLAIMS CASES

37. HEIRS OF FAVIS The failure to allege earnest but FACTS: Dr. Favis was married to Capitolina with whom he had 7 children. When Capitolina died, Dr. Favis took Juana as his
SR. V. GONZALES [GR failed efforts at a compromise in a common-law wife with whom he had one child named Mariano. When Dr. Favis and Juana got married, he executed an
185922] 15 JAN 2014 complaint among members of the affidavit acknowledging Mariano as one of his legitimate children. Mariano married Larcelita with whom he has 4 children.
SABIO same family, is not a jurisdictional
defect but merely a defect in the Dr. Favis died intestate and before his death, he allegedly executed a Deed of Donation transferring and conveying properties
statement of a cause of action.
in favor of his grandchildren with Juana. Claiming that the donation prejudiced their legitime, Dr. Favis’ children with
Capitolina, petitioners herein, filed an action for annulment of the Deed of Donation before the RTC of Ilocos Sur.

In their Answer, respondents Juana, sps. Mariano and Larcelita and their grandchildren assert that the properties donated do
not form part of the estate of the late Dr. Favis because said donation was made inter vivos, hence the heirs of Favis
(petitioners) have no stake over said properties.

The RTC ruled in favor of petitioners and nullified the Deed of Donation stating that, at 92 and plagued with illnesses, Dr.
Favis could not have had full control of his mental capacities to execute such donation.

Respondent appealed to the CA who reversed the decision, the CA motu proprio ordered the dismissal for failure of
petitioners to make an averment that earnest efforts toward a compromise have been made in accordance with the Family
Code. Petitioners filed a MR but was rejected by the CA.

ISSUE: Whether or not the CA may dismiss the order of dismissal of the complaint for failure to allege therein that earnest
efforts towards a compromise have been made.

HELD: NO. Sec. 1, Rule 9 provides for only 4 instances when the court may motu proprio dismiss the claim, namely:
1. Lack of jursdiction over the subject matter,
2. Litis pendentia,
3. Res judicata, and
4. Prescription of action.

It is so inferable from the opening sentence of Sec. 1, Rule 9 stating that defense and objections not pleaded either in a
motion to dismiss or in the answer are deemed waived, except when one of the 4 exceptions stated above are present.
Failure to allege in the complaint that earnest efforts at a compromise has been made but had failed is not one of the
exceptions. Upon such failure, the defense is deemed waived.

The tenor of the second sentence of Sec. 1, Rule 9 is that the allowance of a motu proprio dismissal can proceed only from
the exemption from the rule on waiver; which is but logical because there can be no ruling on a waived ground.

The failure to allege earnest but failed efforts at a compromise in a complaint among members of the same family, is not a
jurisdictional defect but merely a defect in the statement of a cause of action.

In the case at bar, no motion to dismiss the complaint based on the failure to comply with a condition precedent was filed with
the trial court; neither was such failure assigned as error in the appeal that respondent brought before the CA. therefore, the
rule on deemed waiver of non-jurisdictional defense or objection is wholly applicable to respondent. If respondents did not,
after filing their answer to petitioner’s complaint, invoke the objection of absence of the required allegation on earnest efforts
at a compromise, the appellate court unquestionably did not have any authority or basis to motu proprio order the dismissal of
petitioner’s complaint.

38. CARNIYAN V.
Section 3 (b) of Rule 9 of ROC will Facts:
HOME GUARANTY
reveal that one of the defending Civil Case No. Q-09-64015-Judge Villordon of Branch 224.
CORPORATION [GR
228516] 14 AUG 2019 party's remedies against an order of On September 7, 2010, Home Guaranty Corporation (HGC) filed before the Quezon City RTC a complaint for recovery of
SOMEROS default is to file a motion under oath possession against the Carniyan (the petitioners), of the parcel of land situated in Constitution Hills, Quezon City, with TCT
to set it aside on the ground of No. 262715.
fraud, accident, mistake, or petitioners filed a Motion to dismiss and, subsequently, a Motion to Archive the Case as May Be Possible in Lieu of Dismissal.
excusable negligence. Additionally, In the former, the petitioners argued that the RTC had no jurisdiction to resolve the complaint (1) due to the fact that HGC has
the defending party must append to not yet acquired ownership over the contested property; and (2) because the assessed value thereof fell below P400,000.00,
the said motion an affidavit showing the alleged jurisdictional amount of civil actions filed in Metro Manila. On the other hand, in the latter motion, they essentially
that he or she has a meritorious sought to hold in abeyance the proceedings in Civil Case No. Q-09-64015 until HGC submitted a certified true copy of TCT
defense. No. 262715, among other things.
an order of default may be lifted, the The Challenged Trial Court Orders
following requisites must be met: (a) First challenged Order
that a motion be filed under oath by On March 18, 2011, Judge Villordon ruled that the trial court was possessed of the requisite jurisdiction to take cognizance of
one who has knowledge of the the complaint and denied the motion to archive the case on the ground that the said motion was merely dilatory. (fallo
facts; (b) that the defending party's decision)
failure to file answer was due to On June 29, 2011, the petitioners filed a Motion to Expunge/Rescind the Interlocutory Order Dated March 18, 2011 with
fraud, accident, mistake, or Motion for Inhibition. First, they contended that the trial court failed to pass upon their allegation on the non-existence of a
excusable negligence; and (c) that cause of action on the part of HGC. Second, they asserted that their previous motions were not intended to delay the
the defending party shows the resolution of the issues in the case. The petitioners therefore prayed that Judge Villordon inhibit herself from hearing the
existence of a meritorious defense motion to expunge and that the records of the case be returned to the Executive Judge of the Quezon City RTC for re-raffle to
through an affidavit of merit. another branch thereof.
It appears, however, that the petitioners had previously sought Judge Villordon's inhibition, which was denied in an earlier
order dated August 2, 2010.
second challenged order.
On February 8, 2012, Judge Villordon denied the petitioners' motion to expunge. She then disposed of the motions and
directed the petitioners to file their answer within a non-extendable period of 10 days.
petitioners failed to file an answer. HGC moved to declare the petitioners in default.
petitioners filed a Motion to Amend the February 8, 2012 Order to Resolve the Actual Controversy and to Judiciously Resolve
the Instant Motion for Inhibition Upon Receipt Hereof along with the motion to declare them in default.
third challenged order
Judge Villordon denied the petitioners' motion and declaring them in default. She ruled that the said motion partook of the
nature of a second motion for inhibition, which is proscribed under A.M. No. 11-6-10-SC. Hence, the same was held to be a
mere scrap of paper, and was stricken from the records. On the other hand, HGC's motion was held to be impressed with
merit. For this reason, HGC was allowed to present its evidence ex parte before the branch clerk of court on December 9,
2012. (fallo decision)
last challenged trial court orders
Finally, on November 21, 2012, Judge Villordon, rescheduled the ex parte presentation of HGC's evidence, viz.
Aggrieved, the petitioners challenged the four aforesaid trial court orders before the CA via a Petition for Certiorari,
Prohibition, and Mandamus, arguing that Judge Villordon had acted with grave abuse of discretion in issuing the same.
The CA's Ruling
CA denied the said petition on the ground that the same was an inappropriate remedy. The appellate court ruled that the
petitioners should have instead filed a motion under oath to set aside the order of default and shown that they had a
meritorious defense through an affidavit of merit. Moreover, the CA held that the petitioners' failure to file an answer was
attributable solely to their own negligence.
Hence this petition.
ISSUE: Whether or not the 2 and 3 challenged trial court were issued with grave abuse of discretion.
nd rd
HELD: No, The petition lacks merit.
second challenged trial court order:
At this juncture, it bears noting that the second challenged trial court order contained a directive to the petitioners to file an
answer to HGC's complaint within a non-extendible period of 10 days from notice. To which the petitioners never complied. As
a result, HGC filed a motion to declare them in default, which Judge Villordon granted through the third challenged trial
court order, dated October 31, 2012.
Section 3 (b) of Rule 9 of ROC will reveal that one of the defending party's remedies against an order of default is to file a
motion under oath to set it aside on the ground of fraud, accident, mistake, or excusable negligence. Additionally, the
defending party must append to the said motion an affidavit showing that he or she has a meritorious defense. Section 3 (b) of
Rule 9 relevantly provides:
(b) Relief from order of default. — A party declared in default may at any time after notice thereof and before judgment file a motion under oath to set aside
the order of default upon proper showing that his failure to answer was due to fraud, accident, mistake or excusable negligence and that he has a meritorious
defense. In such case, the order of default may be set aside on such terms and conditions as the judge may impose in the interest of justice.
an order of default may be lifted, the following requisites must be met: (a) that a motion be filed under oath by one who has
knowledge of the facts; (b) that the defending party's failure to file answer was due to fraud, accident, mistake, or excusable
negligence; and (c) that the defending party shows the existence of a meritorious defense through an affidavit of merit.
In addition to a motion to lift the order of default, jurisprudence provides several other remedies at the disposal of the
defendant who fails to file an answer. These were enumerated in Lina v. CA, et al. The availability of these alternative
remedies, however, depends on when the defending party discovers that he or she has been declared in default, or whether
the judgment in the suit is contrary to law, jurisprudence, or the evidence on record, thus:
b) If the judgment has already been rendered when the defendant discovered the default, but before the same has become
final and executory, he may file a motion for new trial under Section 1(a) of Rule 37;
c) If the defendant discovered the default after the judgment has become final and executory, he may file a petition for relief
under Section 2 of Rule 38; and
d) He may also appeal from the judgment rendered against him as contrary to the evidence or to the law, even if no petition to
set aside the order of default has been presented by him. (Sec. 2, Rule 41)
As discussed above, resort may be had to a petition for certiorari only in the absence of an appeal or any other plain, speedy,
and adequate remedy in the ordinary course of law. Considering that no judgment had yet been rendered a quo, the
petitioners, pursuant to Section 3 (b) of Rule 9 of the ROC, should have filed a motion to lift the order declaring them
in default. Failing to do so, their recourse to the CA via a petition for certiorari was improper.
As a consequence of declaring the petitioners in default, Judge Villordon allowed HGC to present its evidence ex parte before
the branch clerk of court.
It is well-settled that a petition for certiorari will prosper only if the act or omission constituting grave abuse of discretion is
alleged and proved. Without such a showing, the Court is left with no alternative other than to uphold the CA's denial of their
petition for certiorari.

39. OTERO V. TAN - A defendant who fails to file an Facts:


[GR 200134] 15 AUG answer may, upon motion, be A Complaint for collection of sum of money and damages was filed by Roger Tan (Tan) with the MTCC, Cagayan de Oro City
2012 declared by the court in default. on July 28, 2005 against Roberto Otero (Otero). Tan alleged that on several occasions from February 2000 to May 2001,
TAN Loss of standing in court, the Otero purchased on credit petroleum products from his Petron outlet in Valencia City, Bukidnon. That Otero failed to settle his
forfeiture of one's right as a party obligation despite demands.
litigant, contestant or legal
adversary, is the consequence of an Despite receipt of the summons served through his wife Grace R. Otero on August 31, 2005, Otero failed to file his answer
order of default. A party in default with the MTCC.
loses his right to present his
defense, control the proceedings,
and examine or cross-examine On November 18, 2005, Tan filed a motion with the MTCC to declare Otero in default for his failure to file his answer. Otero
witnesses. He has no right to opposed Tan's motion, claiming that he did not receive any of the summons.
expect that his pleadings would be
acted upon by the court nor may be The motion was reset for 3times and Otero failed to appear and the
object to or refute evidence or MTCC issued an order declaring him in default.
motions filed against him.
Tan was then allowed to present his evidence ex parte presenting the
- A defendant who has been testimonies of Rosemarie Doblado and Zita Sara, his employees in his Petron outlet who attended Otero when the latter
declared in default is precluded made purchases of petroleum products.As well the statements of account prepared by certain Lito Betache (Betache),
from raising any other ground in his apparently likewise an employee of Tan.
appeal from the judgment by default
since, otherwise, he would then be MTCC: MTCC ruled against Otero to pay Tan his outstanding obligation.
allowed to adduce evidence in his The MTCC opined that Otero's failure to file an answer despite notice is a tacit admission of Tan's claim.
defense, which right he had lost
after he was declared in default. Otero appealed to the RTC, asserting that the MTCC's disposition is factually baseless and that he was deprived of due
process.
RTC: RTC affirmed the MTCC Decision. The RTC held that the statements
of account that were presented by Tan were solid. Further, brushing aside Otero's claim of denial of due process.

Otero sought reconsideration but it was denied by the RT. Otero then filed a petition for review with the CA asserting that both
the RTC and the MTCC erred in giving credence to the pieces of evidence presented by Tan in support of his complaint
because a certain Betache was not presented as a
witness by Tan and that the said statements of account failed to establish the genuineness and due execution of the same.

CA: CA denied the petition for review filed by Otero. CA held that any defense which Otero may have against Tan's claim is
already deemed waived due to Otero's failure to file his answer. Otero's Motion for Reconsideration was denied by the CA

ISSUE: whether Otero, having been declared in default by the MTCC, may, in the appellate proceedings, still raise the failure
of Tan to authenticate the statements of account which he adduced in evidence

HELD: NO. The petition is denied.


Authentication of the Statements of Account - A defendant who fails to file an answer loses his standing in court.

The effect of a defendant's failure to file an answer within the time allowed therefor is primarily governed by Section 3, Rule 9
of the Rules of Court, viz.:
Sec. 3. Default; declaration of. — If the defending party fails to answer within the time allowed therefor, the court shall, upon
motion of the claiming party with notice to the defending party, and proof of such failure, declare the defending party in
default. Thereupon,the court shall proceed to render judgment granting the claimant such relief as his pleading may warrant,
unless the court in its discretion requires the claimant to submit evidence. Such reception of evidence may be delegated to
the clerk of court. . . .

A defendant who fails to file an answer may, upon motion, be declared by the court in default. Loss of standing in court, the
forfeiture of one's right as a party litigant, contestant or legal adversary, is the consequence of an order of default. A party in
default loses his right to present his defense, control the proceedings, and examine or cross-examine witnesses. He has no
right to expect that his pleadings would be acted upon by the court nor may be object to or refute evidence or motions filed
against him.

A defendant who was declared in default may nevertheless appeal from the judgment by default, albeit on limited grounds.

In Lina v. CA, et al., this Court enumerated the remedies available to party who has been declared in default, to wit:
a) The defendant in default may, at any time after discovery
thereof and before judgment, file a motion, under oath, to set aside the order of default on the ground that his failure to
answer was due to fraud, accident, mistake or excusable neglect, and that he has meritorious defenses; (Sec 3,Rule 18)
b) If the judgment has already been rendered when the defendant discovered the default, but before the same has become
final and executory, he may file a motion for new trial under Section 1(a) of Rule 37;
c) If the defendant discovered the default after the judgment has become final and executory, he may file a petition for relief
under Section 2 of Rule 38; and
d) He may also appeal from the judgment rendered against him as contrary to the evidence or to the law, even if no
petition to set aside the order of default has been presented by him. (Sec. 2, Rule 41) (Emphasis ours)

Indeed, a defending party declared in default retains the right to appeal from the judgment by default. However, the grounds
that may be raised in such an appeal are restricted to any of the following:

First, the failure of the plaintiff to prove the material allegations of the complaint;
Second, the decision is contrary to law; and
Third, the amount of judgment is excessive or different in kind from that prayed for.

In these cases, the appellate tribunal should only consider the pieces of evidence that were presented by the plaintiff during
the ex parte presentation of his evidence.

A defendant who has been declared in default is precluded from raising any other ground in his appeal from the judgment by
default since, otherwise, he would then be allowed to adduce evidence in his defense, which right he had lost after he was
declared in default. Indeed, he is proscribed in the appellate tribunal from adducing any evidence to bolster his defense
against the plaintiff's claim.

In the case at bar, the court stated that notwithstanding the inadmissibility of the said statements of account, this Court finds
that Tan was still able to prove by a preponderance of evidence the material allegations of his complaint against Otero.

Therefore,
having it proved by preponderance of evidence, Otereo is not entitled to avail
the right to appeal from the judgment by default.

40. DAVID V. The defendant in default may, at any


FACTS:
GUTIERREZ-FRUELDA time after discovery thereof and
[GR 170427] 30 JAN before judgment, file a motion under On Sept. 17, 2004, private respondents filed a complaint for accounting, reconveyance and damages with prayer for
2009 oath to set aside the order of default preliminary attachment against petitioner, his wife Marissa David, and the Register of Deeds of Pampanga. Private
TANADA on the ground that his failure to respondents alleged that petitioner fraudulently exceeded his special power of attorney to cause the conversion of their
answer was due to fraud, accident, agricultural lands to those for residential, commercial and industrial purposes by registering in his name some of the lands,
mistake or excusable negligence, and mortgaging others, failing to remit and account any money received from any transaction involving their lands, and
that he has a meritorious defense absconding.
(Sec. 3, Rule 18 [now Sec. 3(b), Rule Service of summons failed as petitioner was abroad. On January 24, 2005, the RTC ordered service by
9]); publication. Thereafter, private respondents moved that petitioner be declared in default since he failed to answer within 60
days from date of last publication on March 19, 2005.
On July 14, 2005, petitioner filed a motion for extension of 15 days with opposition to the motion to declare him in default.
RTC: In its Order dated July 15, 2005, the RTC declared petitioner in default. The RTC noted that petitioner failed to answer
despite the "many opportunities" given to him. The RTC also denied petitioner's motion for extension to file Answer.
Petitioner moved to lift the order of default and sought another extension of 15 days within which to file Answer.
The RTC denied the motion and ruled that while judgments by default are generally looked upon with disfavor, petitioner's
motion to lift the order of default was fatally flawed under Section 3 (b), Rule 9 of the Rules of Court. The RTC noted that
petitioner's motion was not under oath; unaccompanied by an affidavit of merit; and without any allegation that his failure to
file Answer was due to fraud, accident, mistake or excusable negligence. The RTC also ruled that it was not sufficient for
petitioner to merely allege that he has a meritorious defense.
Petitioner filed an appeal arguing that the respondent judge gravely abused her discretion amounting to lack of jurisdiction

Private respondents counter that the RTC did not commit grave abuse of discretion in denying petitioner's motion to lift
order of default because the motion was not under oath; did not contain an allegation that petitioner's failure to file Answer
was due to fraud, accident, mistake or excusable negligence; and was unaccompanied by an affidavit of merit showing that
petitioner has a meritorious defense.
ISSUE: WON the RTC commit grave abuse of discretion in denying petitioner's motion to lift order of default?
HELD: NO,
One declared in default has the following remedies:
a) The defendant in default may, at any time after discovery thereof and before judgment, file a motion under oath to set
aside the order of default on the ground that his failure to answer was due to fraud, accident, mistake or excusable
negligence, and that he has a meritorious defense (Sec. 3, Rule 18 [now Sec. 3(b), Rule 9]);
b) If the judgment has already been rendered when the defendant discovered the default, but before the same has become
final and executory, he may file a motion for new trial under Section 1(a) of Rule 37;
c) If the defendant discovered the default after the judgment has become final and executory, he may file a petition for relief
under Section 2 [now Section 1] of Rule 38; and
d) He may also appeal from the judgment rendered against him as contrary to the evidence or to the law, even if no petition
to set aside the order of default has been presented by him (Sec. 2, Rule 41).
Moreover, a petition for certiorari to declare the nullity of a judgment by default is also available if the trial court improperly
declared a party in default, or even if the trial court properly declared a party in default, if grave abuse of discretion attended
such declaration. (Emphasis added.)
Petitioner used the first remedy. But the RTC denied his motion to lift the order of default.
in this case, petitioner failed to comply with the basic requirements of Section 3 (b), Rule 9 of the Rules of Court. The motion
was not under oath. There was no allegation that petitioner's failure to file an Answer or any responsive pleading was due to
fraud, accident, mistake, or excusable negligence.
Petitioner merely stated that declarations of default are frowned upon, that he should be given the opportunity to present
evidence in the interest of substantial justice, and that he has meritorious defenses. Unfortunately, his claim that he has
meritorious defenses is unsubstantiated. He did not even state what evidence he intends to present if his motion is granted.

41. MONZON V.
SPS. RELOVA [GR Mere non-appearance of Facts:Respondents alleged that Monzon executed a promissory note in favor of the spouses Perez for
171827] 17 APR 2009 defendants at an ordinary the amount of P600,000.00, with interest of five percent per month. This was secured by a 300-square
CASTRO hearing and to adduce evidence meter lot in Barangay Kaybagal, Tagaytay City. Denominated as Lot No. 2A, this lot is a portion of
does not constitute default, when Psu232001. Thereafter, Monzon executed a Deed of Absolute Sale over the said parcel of land in favor of the spouses Perez.
they have already filed their Respondents also claim that, Monzon executed another promissory note, this time in favor of the spouses Relova for the
answer to the complaint within amount of P200,000.00 with interest of five percent per month. This loan was secured by a 200-square meter lot,
the reglementary period. denominated as Lot No. 2B, a portion of the aforementioned Psu232001. Thereafter, Monzon executed a Deed of Conditional
Sale over said parcel of land in favor of the spouses Relova.

Issue: Whether the RTC erred in applying the effects of default to petitioner

Ruling: Yes. It can be seen that despite the fact that Monzon was not declared in default by the RTC, the RTC
nevertheless applied the effects of a default order upon petitioner under Section 3, Rule 9 of the Rules of Court.
Former Justice Florenz D. Regalado writes that failure to appear in hearings is not a ground for the declaration of
a defendant in default:

“Failure to file a responsive pleading within the reglementary period, and not failure to appear at the hearing, is the
sole ground for an order of default (Rosario, et al. vs. Alonzo, et al., L-17320, June 29, 1963), except the failure to
appear at a pre-trial conference wherein the effects of a default on the part of the defendant are followed, that is,
the plaintiff shall be allowed to present evidence ex parte and a judgment based thereon may be rendered against
the defendant (Section 5, Rule 18). Also, a default judgment may be rendered, even if the defendant had filed his
answer, under the circumstance in Sec.3(c), Rule 29.”

In the case at bar, petitioner had not failed to file her answer. Neither was notice sent to petitioner that she would
be defaulted, or that the effects of default shall be imposed upon her. “Mere non-appearance of defendants at an
ordinary hearing and to adduce evidence does not constitute default, when they have already filed their answer to
the complaint within the reglementary period. It is error to default a defendant after the answer had already been
filed. It should be borne in mind that the policy of the law is to have every litigant’s case tried on the merits as
much as possible; it is for this reason that judgments by default are frowned upon.”failure to attend, when
committed during hearing dates for the presentation of the complainant’s evidence, would amount to the waiver of
such defendant’s right to object to the evidence presented during such hearing, and to cross-examine the
witnesses presented therein. However, it would not amount to a waiver of the defendant’s right to present
evidence during the trial dates scheduled for the reception of evidence for the defense. It would be an entirely
different issue if the failure to attend of the defendant was on a hearing date set for the presentation of the
evidence of the defense, but such did not occur in the case at bar.

42. GAJUDO V. The mere fact that a defendant is Facts: Petitioners filed a complaint against Respondents Traders Royal Bank, the City Sheriff of Quezon City and the
TRADERS ROYAL declared in default does not Register of Deeds of Quezon City. It sought the annulment of the extra-judicial foreclosure and auction sale, the conventional
BANK [GR 151098] 21 automatically result in the grant of redemption thereof, and prayed for damages and the issuance of a writ of preliminary injunction.
MAR 2006 the prayers of the plaintiff. To win,
LIMIN the latter must still present the same The complaint alleged that Petitioner Danilo Chua obtained a loan from Respondent bank in the amount of P75,000.00
quantum of evidence that would be secured by a real estate mortgage. When the loan was not paid, Respondent bank commenced extra-judicial foreclosure
required if the defendant were still proceedings on the property. Petitioners claim that when the Sheriff of Quezon City sold the property to the Respondent bank,
present. A party that defaults is not the highest bidder, the auction sale was tainted with irregularities. Traversing Petitioners' complaint, Respondent bank filed its
deprived of its rights, except the answer with counterclaim, specifically denying the allegations.
right to be heard and to present
evidence to the trial court. If the After numerous circumstances that occurred during the pendency of the case, the Trial Court eventually dismissed the case
evidence presented does not 'without prejudice'. So, Petitioners re-filed the complaint with the same Court. The amended complaint in the previous civil
support a judgment for the plaintiff, case substantially reproduced the allegations of the original complaint. Summons was then served to Respondent bank, but
the complaint should be dismissed, they were not able to file an answer. So, Petitioners filed a motion to declare Respondent bank in default.Upon proof that
even if the defendant may not have Petitioners had served Respondent bank with a copy of said motion, the Trial Court issued an Order of default against
been heard or allowed to present Respondent bank. They were by the Court allowed to present evidence ex parte insofar as Respondent bank was concerned.
any countervailing evidence. Then, the Trial Court rendered the questioned partial decision.

Respondent bank appealed the Partial Decision to the CA. The CA ruled in favor of respondent bank. On the issue of whether
petitioners had convincingly established their right to relief, the appellate court held that there was no ground to invalidate the
foreclosure sale of the mortgaged property. Moreover, petitioners failed to prove that the bank had agreed to sell the property
back to them.

Issue: W/N the CA erred in failing to apply the provisions of Section 3, Rule 9 of the 1997 Rules of Civil Procedure [and in
applying instead] the rule on preponderance of evidence under Section 1, Rule 133 of the Rules of Court

Held: No. In essence, petitioners argue that the quantum of evidence for judgments flowing from a default order under
Section 3 of Rule 9 is not the same as that provided for in Section 1 of Rule 133. Between the two rules, there is no
incompatibility that would preclude the application of either one of them. To begin with, Section 3 of Rule 9 governs the
procedure which the trial court is directed to take when a defendant fails to file an answer. As in other civil cases, basic is the
rule that the party making allegations has the burden of proving them by a preponderance of evidence. Needless to say, the
extent of the relief that may be granted can only be as much as has been alleged and proved with preponderant evidence
required under Section 1 of Rule 133.

Regarding judgments by default, it was explained in Pascua v. Florendo that complainants are not automatically entitled to the
relief prayed for, once the defendants are declared in default. Favorable relief can be granted only after the court has
ascertained that the relief is warranted by the evidence offered and the facts proven by the presenting party. In Pascua, this
Court ruled that "... it would be meaningless to require presentation of evidence if every time the other party is declared in
default, a decision would automatically be rendered in favor of the non-defaulting party and exactly according to the tenor of
his prayer. This is not contemplated by the Rules nor is it sanctioned by the due process clause."

In sum, while petitioners were allowed to present evidence ex parte under


Section 3 of Rule 9, they were not excused from establishing their claims for damages by the required quantum of proof under
Section 1 of Rule 133. Stated differently, any advantage they may have gained from the ex parte presentation of evidence
does not lower the degree of proof required. Clearly then, there is no incompatibility between the two rules.

43. INDIANA The remedies available to a Facts: Dr. Reynaldo B. Vera, Chairman, TPRAM of CHED, received a letter from Douglas R. Macias, Chairman, Board of
AEROSPACE V. defendant declared in default, as Aeronautical Engineering, Professional Regulatory Commission (PRC) and Chairman, Technical Committee for Aeronautical
COMMISSION ON follows: (1) a motion to set aside the Engineering (TPRAME) inquiring whether petitioner had already acquired university status in view of the latter's
HIGHER EDUCATION order of default under Section 3(b), advertisement in the Manila Bulletin.
[GR 139371] 04 APR Rule 9 of the Rules of Court, if the
2001 default was discovered before Dr. Vera formally referred the aforesaid letter to Chairman Alcala with a request that the concerned Regional Office of [CHED]
UBAY judgment could be rendered; (2) a be directed to conduct appropriate investigation on the alleged misrepresentation by petitioner. Thereafter, [CHED] referred
motion for new trial under Section the matter to its Regional Director in Cebu City, requesting said office to conduct an investigation and submit its report.
1(a) of Rule 37, if the default was
discovered after judgment but while The report stated that there was a violation committed by his institution when it used the term university unless the school
appeal is still available; (3) a petition ha[d] complied with the basic requirement of being a university as prescribed in CHED Memorandum Order No. 48, s. 1996.
for relief under Rule 38, if judgment As a consequence of said Report, respondent's Legal Affairs Service was requested to take legal action against petitioner.
has become final and executory; Subsequently, [respondent] directed petitioner to desist from using the term University, including the use of the same in any
and (4) an appeal from the of its alleged branches.
judgment under Section 1, Rule 41,
even if no petition to set aside the In the course of its investigation, was able to verify from the Securities and Exchange Commission (SEC) that petitioner had
order of default has been resorted filed a proposal to amend its corporate name from Indiana School of Aeronautics to Indiana Aerospace University, which was
to. supposedly favorably recommended by the Department of Education, Culture and Sports (DECS) and on that basis, SEC
issued Certificate of Registration. After due hearing, formally offered its evidence while respondent made a formal offer of
These remedies, however, are evidence to which filed its Comments/Objections and finally, respondent submitted its Memorandum relative thereto. The RTC
available only to a defendant who denied respondent's Motion to Dismiss and at the same time, issued a Writ of Preliminary Injunction in favor of petitioner.
has been validly declared in default. Respondent, in the same Order, was directed to file its Answer within fifteen (15) days from receipt of said Order.
Such defendant irreparably loses
the right to participate in the trial. Petitioner filed before the RTC a Motion To Declare Respondent in Default pursuant to Section 3, Rule 9 in relation to Section
4, Rule 16 of the Rules of Court, as amended, and at the same time praying for the Motion to Set for Hearing on October 30,
1998 at 8:30 a.m. On the same date, respondent filed a Motion For Extension of Time to File its Answer.

RTC then granted Petitioner's Motion to Declare Respondent in Default. Respondent filed with the CA a Petition for Certiorari,
arguing that the RTC had committed grave abuse of discretion (a) in denying the former's Motion to Dismiss, (b) in issuing a
Writ of Preliminary Injunction, and (c) in declaring respondent in default despite its filing an Answer.

The CA ruled that petitioner had no cause of action against respondent. Petitioner failed to show any evidence that it had
been granted university status by respondent as required under existing law and CHED rules and regulations.

CA also ruled that respondent should not have been declared in default, because its Answer had been filed long before the
RTC ruled upon petitioner's Motion to declare respondent in default. Thus, respondent had not obstinately refused to file an
Answer; on the contrary, its failure to do so on time was due to excusable negligence.

Thus this petition.

Issue: Whether or not CA erred in not requiring Respondent CHED to first file a Motion to Set Aside the Order
of Default

Held: Petitioner avers that the RTC was justified in declaring respondent in default, because the August 14, 1998 Order
directing the filing of an answer had been served on August 25, 1998. And as late as October 30, 1998, respondent could
only file a Motion for Extension of Time, which the trial court denied because of the expiry of the fifteen-day period. Petitioner
adds that respondent’s proper remedy would have been a Motion to Set Aside the Order of Default, pursuant to Section 3(b),
Rule 9 of the Rules of Court.

Respondent, in turn, avers that certiorari was the only plain, speedy and adequate remedy in the ordinary course of law,
because the default Order had improvidently been issued.

We agree with Respondent. Lina v. Court of Appeals 7 discussed the remedies available to a defendant declared in default,
as follows: (1) a motion to set aside the order of default under Section 3(b), Rule 9 of the Rules of Court, if the default was
discovered before judgment could be rendered; (2) a motion for new trial under Section 1(a) of Rule 37, if the default was
discovered after judgment but while appeal is still available; (3) a petition for relief under Rule 38, if judgment has become
final and executory; and (4) an appeal from the judgment under Section 1, Rule 41, even if no petition to set aside the order of
default has been resorted to.

These remedies, however, are available only to a defendant who has been validly declared in default. Such defendant
irreparably loses the right to participate in the trial. On the other hand, a defendant improvidently declared in default may
retain and exercise such right after the order of default and the subsequent judgment by default are annulled, and the case
remanded to the court of origin. The former is limited to the remedy set forth in Section 2, paragraph 3 of Rule 41 of the pre
1997 Rules of Court, and can therefore contest only the judgment by default on the designated ground that it is contrary to
evidence or law. The latter, however, has the following options: to resort to this same remedy; to interpose a petition for
certiorari seeking the nullification of the order of default, even before the promulgation of a judgment by default; or in the
event that judgment has been rendered, to have such order and judgment declared void.ch
In prohibiting appeals from interlocutory orders, the law does not intend to accord executory force to such writs, particularly
when the effect would be to cause irreparable damage. If, in the course of trial, a judge proceeds without or in excess of
jurisdiction, this rule prohibiting an appeal does not leave the aggrieved party without any remedy. In a case like this, a
special civil action of certiorari is the plain, speedy and adequate remedy.

Herein respondent controverts the judgment by default, not on the ground that it is unsubstantiated by evidence or that it is
contrary to law, but on the ground that it is intrinsically void for having been rendered pursuant to a patently invalid order of
default.

44. DE GUIA V. DE Under the pre-1997 Rules of Civil Plaintiffs Mariano De Guia, et a. filed with a complaint for partition against defendants Ciriaco, Leon, Victorina and Pablo De
GUIA [GR 135384] 04 Procedure, a notice of pretrial must Guia. They alleged that the real properties therein described were inherited by plaintiffs and defendants from their
APR 2001 be served separately on the counsel predecessors-in-interest, and that the latter unjustly refused to have the properties subdivided among them. Shortly after
YUMUL and the client. If served only on the defendants filed their traverse, an amended complaint was admitted by the lower court, in which plaintiff Tomasa De Guia
counsel, the notice must expressly was impleaded as one of the defendants for the reason that she had become an unwilling co-plaintiff.
direct the counsel to inform the
client of the date, the time and the It is further shown in the record that the branch Clerk of Court issued a Notice setting the case for pre-trial conference. Copies
place of the pretrial conference. The of said notices were sent by registered mail to parties and their counsel. It turned out that both defendants and counsel failed
absence of such notice renders the to attend the pre-trial conference. Hence, upon plaintiffs’ motion, defendants were declared as in default and plaintiffs were
proceedings void, and the judgment allowed to present their evidence ex-parte.
rendered therein cannot acquire
finality and may be attacked directly Defendants explained that they received the Notice of pre-trial only in the afternoon, giving them no chance to appear for such
or collaterally. proceeding in the morning of that day. The Motion was opposed by plaintiffs who pointed out that per Postal Delivery Receipt,
defendants’ counsel actually received his copy of the Notice. Citing Section 2, Rule 13 of the Rules of Court, plaintiffs further
urged that counsel’s receipt of the said notice was sufficient to bind defendants who received said notice on the next day.
Finally, they faulted defendants for failing to support their Motion for Reconsideration with an affidavit of merit showing among
others that they had a meritorious defense.

Plaintiffs’ motion for reconsideration was denied and judgment was rendered ordering the partition of the controverted parcels
of land.

ISSUE/S:

WON the respondent court erred in reversing the trial court’s Decision notwithstanding private respondents’ violations of Rule
15, Sections 4 and 5.

HELD:

NO. Petitioners allege that, to their detriment, the appellate court disregarded established procedural precepts in resolving the
case, and that it did so for three reasons. First, respondents’ Manifestation and Motion to Lift the Order of Default, filed with
the trial court, was merely pro forma because the former lacked the requisite notice of hearing. Second, it also lacked an
affidavit of merit. Third, respondents’ Appeal Brief did not contain a certificate of non-forum shopping.

Granting that respondents’ Manifestation and Motion to Lift the Order of Default was pro forma, this issue has become moot,
not only because the trial court had denied such Motion, but also because what was appealed was the judgment rendered by
the lower court. For the same reason, we must also reject petitioners’ insistence that an affidavit of merit was absent. In a ny
case, there was no need to attach an affidavit of merit to the Motion, as the defenses of respondents had been set out in their
Answer.

WHEREFORE, the Petition is DENIED and the assailed Decision and Resolution AFFIRMED.

45. LUNA V. Liberal interpretation or construction Facts:


MIRAFUENTE [AM MTJ- of the law or rules, however, is not a Judge Eduardo H. Mirafuente is charged with Grave Misconduct and Conduct Prejudicial to the Best Interest of the Service,
05-1610] 26 SEP 2005 free commodity that may be availed Violation of the Rules on Summary Procedure in Special Cases and Gross Ignorance of the Law by Dr. Luna arising from
ZAPANTA of in all instances under the cloak of respondent's act of giving due course to the belatedly filed an unverified answer of the defendants in a complaint for unlawful
rendering justice. Liberality in the detainer.
interpretation and application of
Rules applies only in proper cases Defendants filed an unverified answer to the complaint, seven (7) days beyond the reglementary period of ten (10) days from
and under justifiable causes and the service of the summons on them. Dr. Luna's counsel filled a Motion for Judgment, invoking Section 6 of the Revised Rule
circumstances. While it is true that on Summary Procedure, to which motion the defendants did not file any opposition. Respondent denied the motion.
litigation is not a game
of echnicalities, it is equally true The administrative complaint against respondent, Dr. Luna
that every case must be prosecuted asserting that as the defendants' answer was unverified and belatedly filed, respondent should have motu proprio or on
in accordance with the prescribed motion of the plaintiffs rendered judgment as warranted by the facts alleged in the complaint, following Section 6 of the
procedure to insure an orderly and
speedy administration of justice. Revised Rule on Summary Procedure.

In his Comment, respondent explains that his admission of the defendants' unverified, belatedly filed answer was premised on
"the spirit of justice and fair play, which underlie[s] every court litigation and serves as the bedrock to preserve the trust and
faith of parties litigants in the judicial system;" that the admission was proper because the delay was negligible, it involving
only four (4) days as June 13 to 15, 2003
were non-working holidays.The delay was also excusable as defendants acted pro se, without the benefit of legal assistance,
and not dilatory.

ISSUE:
W/N the judge should be reprimanded for violation of the Summary Rules of Procedure

HELD:
Yes. Sections 5 and 6 of the 1991 Revised Rule on Summary Procedure provide:
Sec. 5. Answer. — Within ten (10) days from service of summons, the
defendant shall file his answer to the complaint and serve a copy thereof on the
plaintiff. . . .
Sec. 6. Effect of failure to answer. — Should the defendant fail to
answer the complaint within the period above provided, the court, motu proprio , or on motion of the plaintiff, shall
render judgment as may be warranted by the facts alleged in the complaint and limited to what is prayed for therein:
Provided, however, That the court may in its discretion reduce the amount of damages and attorney's fees claimed for being
excessive or otherwise unconscionable.

The word "shall" underscores their mandatory character. Giving the provisions a directory application would subvert the
nature of the Rule and defeat its objective of expediting the adjudication of the suits covered thereby. To admit a late answer
is to put a premium on dilatory maneuvers — the very mischief that the Rule seeks to redress.

Liberal interpretation or construction of the law or rules, however, is not a free commodity that may be availed of in all
instances under the cloak of rendering justice. Liberality in the interpretation and application of Rules applies only in proper
cases and under justifiable causes and circumstances. While it is true that litigation is not a game of echnicalities, it is equally
true that every case must be prosecuted in accordance with the prescribed procedure to insure an orderly and speedy
administration of justice.

Respondent's act, albeit a disregard of procedural rules, does not, however, constitute grave misconduct.Neither does it
constitute gross ignorance of the law.

After considering the appreciation by respondent of the fact that the defendants filed a belated and unverified answer without
the assistance of counsel, and the lack of showing of malice, corrupt motives or the like on his part, this Court finds that, as in
the above-cited Ruperto case, the penalty may be as it is hereby mitigated to severe reprimand.

Judge Eduardo H. Mirafuente is hereby SEVERELY REPRIMANDED

46. PONCIANO V. A compulsory counterclaim is any Facts:


PARENTELA [GR claim for money or other relief which Private respondents Ildefonso and Leonora Clamosa filed a complaint for a sum of money and damages with the RTC of
133284] 09 MAY 2000 a defending party may have against Trece Martires City, against petitioners Claro and Gloria Ponciano for unpaid cost of labor and materials incurred by them in
ABAD an opposing party, which at the time repairing petitioner’s house in San Roque, Cavite. The case was docketed as Civil Case No. TM-601. Petitioners filed a
of suit arises out of, or is motion to dismiss the complaint for failure to state a cause of action, but the same was denied by the trial court.
necessarily connected with, the
same transaction or occurrence that Petitioners filed their answer with compulsory counterclaim, claiming that they have paid the total contract price agreed upon;
is the subject matter of plaintiff's that despite this, the work of private respondents was defective; and that private respondents abandoned the renovation
complaint. It is compulsory in the before it was completed. Petitioners asserted that they are entitled to be paid P250,000 to complete the renovation, and
sense that if it is within the damages.
jurisdiction of the court, and does
not require for its adjudication the Upon motion of private respondents, the trial court ordered that petitioners counterclaim be stricken off from the record for
presence of third parties over whom failure to comply with Administrative Circular No. 04-94, which requires an affidavit of non-forum shopping for all initiatory
the court cannot acquire jurisdiction, pleadings in all courts. Petitioners filed a motion for reconsideration arguing, among others, that since their counterclaim is
it must be set up therein, and will be compulsory in nature, it is not an initiatory pleading and therefore, does not fall within the scope of Administrative Circular No.
barred in the future if not set up. 04-94. However, the trial court denied petitioners’ motion for reconsideration.

Petitioners questioned the trial court’s orders by means of a special civil action for certiorari under Rule 65.

Petitioners filed an "Answer with Amended Compulsory Counterclaim," wherein the amendment consisted of the addition of a
certification under oath. Initially, the trial court admitted the "Answer with Amended Compulsory Counterclaim".However, after
the filing of a motion for reconsideration by private respondents, the court reconsidered its action and expunged the amended
compulsory counterclaim from the records.

Petitioners filed the present special civil action for certiorari under Rule 65, assailing the trial court’s orders denying admission
of their amended compulsory counterclaim. They maintain that this Court did not rule in its decision in G.R. No. 127701 that
the dismissal of petitioners’ compulsory counterclaim in for non-compliance with Administrative Circular No. 04-94 was with
prejudice. Consequently, petitioners assert that they should be permitted to re-file their compulsory counterclaim provided that
they comply with such circular.

Issue:
Whether or not an answer which asserts a compulsory counterclaim must include a certificate of non-forum shopping, and if
so, whether or not the dismissal of such compulsory counterclaim by the trial court due to the absence of such certification
has the effect of a dismissal with prejudice so as to bar the party from re-filing such compulsory counterclaim.

Held:

No. Administrative Circular No. 04-94 was issued by this Court in order to prevent the undesirable practice of forum-shopping,
which exists when, as a result of an adverse opinion in one forum, a party seeks a favorable opinion (other than by appeal or
certiorari) in another, or when he institutes two or more actions or proceedings grounded on the same cause, on the chance
that one or the other court would make a favorable disposition.
A compulsory counterclaim is any claim for money or other relief which a defending party may have against an opposing
party, which at the time of suit arises out of, or is necessarily connected with, the same transaction or occurrence that is the
subject matter of plaintiff’s complaint. It is compulsory in the sense that if it is within the jurisdiction of the court, and does not
require for its adjudication the presence of third parties over whom the court cannot acquire jurisdiction, it must be set up
therein, and will be barred in the future if not set up.
In the case at bar, there is no doubt that the counterclaims pleaded by petitioners in their answers are compulsory in nature.
The filing of a separate action by petitioners would only result in the presentation of the same evidence as in Civil Case No.
TM-601. Proceeding from our ruling in Santo Tomas University Hospital, petitioners need not file a certification of non-forum
shopping since their claims are not initiatory in character, and therefore, are not covered by the provisions of Administrative
Circular No. 04-94.

47. SPS. BARRAZA A party may file a Motion to FACTS: On Oct. 3, 1978, private respondent filed a Complaint for damages based on defendants’ (petitioners herein) use of
V. CAMPOS, JR. [GR L- Dismiss rather than an Answer plaintiff’s (now private respondent) trade name and style of “Gathalian- The House of Native Lechon and Restaurant”, with
50437] 28 FEB 1983 when given an extension by the prayer for preliminary injunction.
BRIONES Court.
Section 4 of Rule 16 allows the After service of summons, petitioners as defendants therein filed an “Urgent Ex-Parte Motion” for extension of time of 15 days
defendant to file his answer not within which to file an Answer which the Court granted. Instead of filing the Answer within the extended period of 15 days,
only within the original fifteen (15) defendants filed through their counsel, Atty. Esmeraldo Gathalian, a “Motion to Dismiss Complaint together with Prayer for
days period but also within "a Preliminary Injunction” which was 1 day before the expiration of the period as extended by the court.
different period (as) fixed by the
court. " Private respondent’s argument that although a motion to dismiss interrupts the running of the period within which to file an
answer, this refers to the original period of 15 days within which to file the representative pleading and not to the extension of
time within which to file the answer, is without merit. There is nothing in the Rules which provide, directly or indirectly, that the
interruption of the running of the period within which to file an answer when a motion to dismiss the complaint is filed and
pending before the court, refers only to the original period of 15 days and not to the extension of time to file the answer as
granted by the court. It may be true that under Sec. 4 Rule 16 of the Rules of Court, if the motion to dismiss is denied or if the
termination thereof is deferred, the movant shall file his answer within the time prescribed by Rule 11, computed from the time
he received notice of the denial or deferment, unless the court provides a different period.

ISSUE: W/N a motion to dismiss must be filed within the time for pleading, that is, within the time to answer?

HELD: Yes, a motion to dismiss must be so filed. Sec. 1 Rule 16 of the Rules of Court provides that a motion to dismiss must
be filed within the time for pleading, that is, within the time to answer including the extension of time granted to file such
answer. Applying this to the case at bar, Respondent Judge granted Petitioners an extension of 15 days to file their answer.
Instead of filing the answer, petitioners filed a Motion to Dismiss the Complaint, 1 day before the expiration of the period as
extended by the court. This clearly is allowed under the Rules of Court.

AMENDED AND SUPPLEMENTAL PLEADINGS (RULE 10, SECS. 1 TO 8)


48. SPS. DIONISIO
V. LINSANGAN [GR An amendment is deemed Facts: Gorgonio M. Cruz owned agricultural lands in San Rafael, Bulacan, that his tenant, Romualdo San
178159] 02 MAR 2011 proper when it is done to allege Mateo cultivated. Upon Romualdo's death, his widow, Emiliana, got Cruz's permission to stay on the
CASTRO facts which occurred prior to property provided she would vacate it upon demand. Later, spouses Dionisio bought the property from
the filing of the original Cruz. Subsequently, the Dionisios found out that Emiliana had left the property and that it was already
pleadings, but which for some Wilfredo Linsangan who occupied it under the strength of a "Kasunduan ng Bilihan ng Karapatan" The
reason, such as oversight, Dionisios wrote to Wilfredo demanding that he vacate the land but the latter declined, prompting the
inadvertence or subsequent Dionisios to file an eviction suit.
discovery, were not alleged
therein. Sec. 3 Rule 10 of the Wilfredo filed an answer with counterclaims in which he declared that he had been a tenant of the land. At
Amended Rules on Civil the pre-trial, the Dionisios orally asked leave to amend their complaint.
Procedure provides that
substantial amendments may Despite initial misgivings over the amended complaint, Wilfredo asked for time to respond to it. The
be made but only upon leave of Dionisios filed their amended complaint on August 5, 2003; Wilfredo maintained his original answer.
court. The amendment makes
it mandatory for the courts to The MTC issued a pre-trial order. After which it rendered judgment, ordering Wilfredo to vacate the land
deny leave if the motion was and remove his house from it. On appeal, the RTC affirmed the MTC decision, holding that the case was
made with intent to delay, one for forcible entry. On review, however, the CA reversed the decisions of the lower courts, ordering the
confer jurisdiction on the court, dismissal of the Dionisios' action. The CA held that, by amending their complaint, the Dionisios effectively
or the pleading stated no cause changed their cause of action from unlawful detainer to recovery of possession which fell outside the
of action from the beginning jurisdiction of the MTC. Further, since the amendment introduced a new cause of action, its filing marked
which could be amended. the passage of the one year limit from demand required in ejectment suits. More, since jurisdiction over
actions for possession depended on the assessed value of the property and since such assessed value
was not alleged, the CA cannot determine what court has jurisdiction over the action.

Issue: Whether or not the Dionisios' amendment of their complaint effectively changed their cause of
action from one of ejectment to one of recovery of possession.

Ruling: No. An amended complaint that changes the plaintiff's cause of action is technically a new
complaint. Consequently, the action is deemed filed on the date of the filing of such amended pleading,
not on the date of the filing of its original version. Thus, the statute of limitation resumes its run until it is
arrested by the filing of the amended pleading.

The Court acknowledges, however, that an amendment which does not alter the cause of action but
merely supplements or amplifies the facts previously alleged, does not affect the reckoning date of filing
based on the original complaint. The cause of action, unchanged, is not barred by the statute of
limitations that expired after the filing of the original complaint.

Based on the allegations of the amended complaint, the Dionisios allowed Emiliana, tenant Romualdo's
widow, to stay on the land for the meantime and leave when asked to do so. But, without the knowledge
or consent of the Dionisios, she sold her "right of tenancy" to Wilfredo. When the Dionisios visited the land
and found Wilfredo there, they demanded that he leave the land. They did so in writing but he refused to
leave. The Dionisios filed their eviction suit within the year.

To determine if an amendment introduces a different cause of action, the test is whether such
amendment now requires the defendant to answer for a liability or obligation which is completely different
from that stated in the original complaint. Here, both the original and the amended complaint have
identical allegations, and required Wilfredo to defend his possession based on the allegation that he had
stayed on the land after Emiliana left out of the owners mere tolerance and that the latter had demanded
that he leave. It did not introduce a new cause of action.

49. WALLEM No, the claim was not timely filed. FACTS:
PHILIPPINES Under Section 3 (6) of the COGSA, On March 25, 1992, Continental Enterprises, Ltd. loaded on board the vessel M/V "Hui Yang," at Bedi Bunder, India, a
SHIPPING, INC. V. S.R. notice of loss or damages must be shipment of Indian Soya Bean Meal, for transportation and delivery to Manila, with plaintiff [herein respondent] as
FARMS, INC. [GR filed within three days of delivery. consignee/notify party. The said shipment is said to weigh 1,100 metric tons and covered by Bill of Lading No. BEDI 4
161849] 09 JUL 2010 Admittedly, respondent did not dated March 25, 1992 (Exhibit A; also Exhibit I). The vessel is owned and operated by defendant Conti-Feed, with
DE GALA comply with this provision. defendant [herein petitioner] Wallem as its ship agent.
Under the same provision, however, The subject cargo is part of the entire shipment of Indian Soya Bean Meal/India Rapeseed Meal loaded in bulk on board
a failure to file a notice of claim the said vessel for delivery to several consignees. Among the consignees were San Miguel Corporation and Vitarich
within three days will not bar Corporation, including the herein plaintiff (Exhibit A; Exhibits 1 to 6; TSN, p. 13, June 28, 1996).
recovery if a suit is nonetheless On April 11, 1992, the said vessel, M/V "Hui Yang" arrived at the port of Manila, Pier 7 South Harbor. Thereafter, the
filed within one year from delivery of shipment was discharged and transferred into the custody of the receiving barges, the NorthFront-333 and NorthFront-
the goods or from the date when 444. The offloading of the shipment went on until April 15, 1992 and was handled by [Ocean Terminal Services, Inc.]
the goods should have been OTSI using its own manpower and equipment and without the participation of the crew members of the vessel. All
delivered. throughout the entire period of unloading operation, good and fair weather condition prevailed.
At the instance of the plaintiff, a cargo check of the subject shipment was made by one Lorenzo Bituin of Erne Maritime
and Allied Services, Co. Inc., who noted a shortage in the shipment which was placed at 80.467 metric tons based on
draft survey made on the NorthFront-33 and NorthFront-444 showing that the quantity of cargo unloaded from the vessel
was only 1019.53 metric tons. Thus, per the bill of lading, there was an estimated shortage of 80.467.
Upon discovery thereof, the vessel chief officer was immediately notified of the said short shipment by the cargo
surveyor, who accordingly issued the corresponding Certificate of Discharge dated April 15, 1992 (Exhibit D). The survey
conducted and the resultant findings thereon are embodied in the Report of Superintendence dated April 21, 1992
(Exhibits C to C-2) and in the Barge Survey Report both submitted by Lorenzo Bituin (Exhibits C-3 and C-4). As testified
to by Lorenzo Bituin, this alleged shortage of 80.467 metric tons was arrived at using the draft survey method which calls
for the measurement of the light and loaded condition of the barge in relation to the weight of the water supposedly
displaced.
Petitioner then filed a Complaint for damages against Conti-Feed & Maritime Pvt. Ltd., a foreign corporation doing
business in the Philippines and the owner of M/V "Hui Yang"; RCS Shipping Agencies, Inc., the ship agent of Conti-
Feed; Ocean Terminal Services, Inc. (OTSI), the arrastre operator at Anchorage No. 7, South Harbor, Manila; and Cargo
Trade, the customs broker.
petitioner denied the allegations of respondent claiming, among others, that it is not accountable nor responsible for any
alleged shortage sustained by the shipment while in the possession of its co-defendants; the alleged shortage was due
to negligent or faulty loading or unloading of the cargo by the stevedores/shipper/consignee; the shortage, if any, was
due to pre-shipment damage, inherent nature, vice or defect of the cargo for which herein petitioner is not liable;
respondent’s claim is already barred by laches and/or prescription.

ISSUE: Whether or not the claim against petitioner was timely filed

HELD:
Under Section 3 (6) of the COGSA, notice of loss or damages must be filed within three days of delivery. Admittedly,
respondent did not comply with this provision.
Under the same provision, however, a failure to file a notice of claim within three days will not bar recovery if a suit is
nonetheless filed within one year from delivery of the goods or from the date when the goods should have been
delivered.
There is no dispute that the vessel carrying the shipment arrived at the Port of Manila on April 11, 1992 and that the
cargo was completely discharged therefrom on April 15, 1992. However, respondent erred in arguing that the complaint
for damages, insofar as the petitioner is concerned, was filed on March 11, 1993.

In the instant case, the Court is not persuaded by respondent’s claim that the complaint against petitioner was timely
filed. Respondent argues that the suit for damages was filed on March 11, 1993, which is within one year from the time
the vessel carrying the subject cargo arrived at the Port of Manila on April 11, 1993, or from the time the shipment was
completely discharged from the vessel on April 15, 1992. 1awph!l

50. PHILIPPINE
PORTS AUTHORITY V. The clear import of such amendment Facts: In 2000, after the expiration of the lease contract of Veterans Shipping Corporation over the Marine Slip Way in the
GOTHONG [GR in Section 3, Rule 10 is that under the North Harbor, petitioner William Gothong & Aboitiz, Inc (WG&A) requested PPA for it to be allowed to lease and operate the
158401] 28 JAN 2008 new rules, "the amendment may said facility. Thereafter, the PPA surrendered possession of the Marine Slip Way in favor of the petitioner. However, believing
ESPIRITU (now) substantially alter the cause of that the said lease already expired on June 30, 2001, PPA subsequently sent a letter to petitioner WG&A directing the latter to
action or defense." vacate the contested premises and to turnover the improvements made. In response, WG&A wrote PPA to reconsider its
This should only be true, however, decision to eject the former which was denied by the PPA. Thereafter WG&A commenced an Injunction suit before the RTC of
when despite a substantial change or Manila claiming that the PPA unjustly, illegally and prematurely terminated the lease contract. It likewise prayed for the
alteration in the cause of action or issuance of a TRO to arrest the evacuation.
defense, the amendments sought to In 2001, WG&A amended its complaint for the first time. The complaint was still denominated as one for Injunction with prayer
be made shall serve the higher for TRO. In the said amended pleading, it said that PPA is already estopped from denying that the correct period of lease is
interests of substantial justice, and "until such time that the North Harbor Modernization Project has been bidded out to and operations turned over to the winning
prevent delay and equally promote bidder. The TRO was denied by the trial court. Petitioner later moved for the reconsideration and thereafter, filed a Motion to
the laudable objective of the rules Admit Attached Second Amended Complaint. This time, however, the complaint was already captioned as one for Injunction
which is to secure a "just, speedy and with Prayer for TRO and/or Writ of Preliminary Injunction and damages and/or for Reformation of Contract. Also, it included as
inexpensive disposition of every its fourth cause of action and additional relief in its prayer, the reformation of the contract as it failed to express or embody the
action and proceeding." true intent of the contracting parties.
The respondent judge issued an Order denying the Admission of the Second Amended Complaint. WG&A then filed a petition
for certiorari with the CA seeking the nullification of the RTC orders. The CA granted respondent's petition, thereby setting
aside the RTC orders and directing the RTC to admit respondent's second amended complaint pursuant to Section 3, Rule 10
of the 1997 Rules of Civil Procedure. Petitioner moved for reconsideration but the same was denied.
Issue: Whether the CA erred in ruling that the RTC committed grave abuse of discretion when it denied the admission of the
second amended complaint
Ruling: NO. The CA did not err in finding that the RTC committed grave abuse of discretion in issuing the Order dated March
22, 2002 denying the admission of respondent's second amended complaint.
The RTC applied the old Section 3, Rule 10 of the Rules of Court instead of the provisions of the 1997 Rules of Civil
Procedure, amending Section 3, Rule 10, to wit:

SECTION 3. Amendments by leave of court. Except as provided in the next preceding section, substantial
amendments may be made only upon leave of court. But such leave may be refused if it appears to the court that the
motion was made with intent to delay. Orders of the court upon the matters provided in this section shall be made upon
motion filed in court, and after notice to the adverse party, and an opportunity to be heard.

The Court has emphasized the import of Section 3, Rule 10 of the 1997 Rules of Civil Procedure in Valenzuela v. Court of
Appeals, thus:

Interestingly, Section 3, Rule 10 of the 1997 Rules of Civil Procedure amended the former rule in such manner that the
phrase "or that the cause of action or defense is substantially altered" was stricken-off and not retained in the new
rules. The clear import of such amendment in Section 3, Rule 10 is that under the new rules, "the amendment may
(now) substantially alter the cause of action or defense." This should only be true, however, when despite a substantial
change or alteration in the cause of action or defense, the amendments sought to be made shall serve the higher
interests of substantial justice, and prevent delay and equally promote the laudable objective of the rules which is to
secure a "just, speedy and inexpensive disposition of every action and proceeding."

The application of the old Rules by the RTC almost five years after its amendment by the 1997 Rules of Civil Procedure
patently constitutes grave abuse of discretion .

51. SPS. LAMBINO A supplemental pleading is meant to Facts: Petitioners, Orlando and Carmelita Lambino secured a housing loan from BPI Family Bank. They executed a Mortgage
V. PRESIDING JUDGE supply deficiencies in aid of the Loan Agreement (MLA) over their property as security. Petitioners failed to pay the monthly amortizations, and so BPI sought
[GR 169551] 24 JAN original pleading and not to dispense to foreclose on the property. Petitioners filed a complaint for annulment of the MLA and the extrajudicial foreclosure sale with a
2007 with or substitute the latter. A prayer for a TRO before the RTC. They alleged that private respondent only released P555,047.19. The RTC issued the TRO.
LEE supplemental complaint must be The Court eventually suspended pretrial. Soon, there were additional charges imposed on petitioners’ account. Petitioners
consistent with, and in aid of, the objected to these charges. Petitioners filed a motion to admit their supplemental complaint wherein they alleged that
cause of action set forth in the respondent made unauthorized deductions and advance interest charges and unilaterally increased interest rates without their
original complaint. A new and consent. The RTC and CA denied the admission. Hence, this petition. Petitioners contend that they came to know of the
independent cause of action cannot escalating interests, penalties, liquidated damages, and attorney’s fees charged by the private respondent only after the
be set up by such complaint. The complaint was filed. Hence, the failure to allege such in their original complaint.
supplemental complaint must be
based on matters arising subsequent Issue: Whether the supplemental complaint should be admitted
to the original complaint related to the
claim or defense presented therein, Held: No. Section 6 of Rule 10 reads:
and founded on the same cause of Matters subject of supplemental pleadings. — Upon motion of a party, the court may, upon reasonable notice and upon such
action. terms as are just, permit him to serve a supplemental pleading setting forth transactions, occurrences or events which have
happened since the date of the pleading sought to be supplemented. If the court deems it advisable that the adverse party
As a general rule, leave will be should plead thereto, it shall so order, specifying the time therefor.
granted to file a supplemental
complaint which alleges any material The rule is a useful device which enables the court to award complete relief in one action and to avoid the cost of delay and
fact which happened or came within waste of separate action. Thus, a supplemental pleading is meant to supply deficiencies in aid of the original pleading and not
plaintiff's knowledge since the original to dispense with or substitute the latter. A supplemental complaint must be consistent with, and in aid of, the cause of action
complaint was filed, such being the set forth in the original complaint. A new and independent cause of action cannot be set up by such complaint. The
office of a supplemental complaint. supplemental complaint must be based on matters arising subsequent to the original complaint related to the claim or defense
The purpose of the rule is that the presented therein, and founded on the same cause of action. However, although the facts occur before the commencement of
entire controversy might be settled in the suit if a party does not learn of their existence until after he has filed his pleading, he may file a supplemental pleading. As
one action a general rule, leave will be granted to file a supplemental complaint which alleges any material fact which happened or came
within plaintiff's knowledge since the original complaint was filed, such being the office of a supplemental complaint. The
purpose of the rule is that the entire controversy might be settled in one action; to avoid unnecessary litigation; prevent delay,
unnecessary repetition of effort; unwarranted expense of litigants; to broaden the scope of the issues in an action owing to the
light thrown on it by facts, events and occurrences which have accrued after the filing of the original pleading; to bring into
record the facts enlarging or charging the kind of relief to which plaintiff is entitled.

Before they filed their original complaint, petitioners were already aware of the deductions made on the proceeds of the loan,
for interest charges, MRI premium, and fire insurance premium in the total amount of P44,952.88. And because petitioners had
alleged all these charges in the petition for extrajudicial foreclosure sale, it behooved petitioners to have incorporated in their
original complaint as a cause of action the alleged "illegal/unauthorized and unconscionable charges for MRI, escalating
interest charges, liquidated damages, attorney's fees, and foreclosure expenses. They should have sought to nullify such
charges in the original complaint, but they did not. They are thus proscribed from incorporating the same via a supplemental
complaint.

52. ALPINE Considering that respondent has the Facts: A complaint for replevin was filed with RTC by Estrella Corpuz, respondent, against Alpine Lending Investors (Alpine),
LENDING INVESTORS right to amend her complaint, it is the petitioner. The complaint alleges that Zenaida was respondent's former neighbor. Pretending to help respondent in securing a
V. CORPUZ [GR correlative duty of the trial court to Garage Franchise from the Land Transportation Office (LTO), Zenaida took from her the original registration papers of her
157107] 24 NOV 2006 accept the amended complaint; vehicle, a Toyota Tamaraw FX with Plate No. UMR 660. Thereafter, Zenaida disappeared with the vehicle. She was informed
LIMIN otherwise, mandamus would lie by the LTO that Zenaida mortgaged her vehicle with petitioner Alpine. Forthwith, respondent informed Alpine about the
against it. It has always been the spurious mortgage and demanded the release of her vehicle. Alpine promised to comply with her request on condition that
policy of this Court to be liberal in Zenaida should first be charged criminally. Respondent then caused the filing with the MeTC complaints for falsification of
allowing amendments to pleadings in private document and estafa against Zenaida. Eventually, a warrant of arrest was issued against her. Respondent informed
order that the real controversies Alpine about these developments, but the latter still refused to turn over the vehicle to her. Instead of filing an answer to
between or among the parties may be respondent's complaint, Alpine submitted to the RTC a motion to dismiss on the ground that it is not a juridical person, hence,
presented and cases be decided on not a proper party in the case. The RTC denied Alpine's motion to dismiss. The RTC then directed respondent to file her
the merits without delay. amended complaint within ten (10) days. However, respondent filed her Amended Complaint with an accompanying Motion to
Admit Amended Complaint two (2) days late. Nonetheless, the RTC admitted the amended complaint. Alpine filed a Motion to
Expunge respondent's motion to admit amended complaint on the ground that the latter motion was not accompanied by a
notice of hearing. The RTC denied Alpine's motion to expunge for lack of merit.

Issue: W/N the trial court erred in admitting respondent's amended complaint.

Held: No. What petitioner Alpine filed was a motion to dismiss, not an answer. Settled is the rule that a motion to dismiss is not
a responsive pleading for purposes of Section 2, Rule 10. As no responsive pleading had been filed, respondent could amend
her complaint as a matter of right. Considering that respondent has the right to amend her complaint, it is the correlative duty of
the trial court to accept the amended complaint; otherwise, mandamus would lie against it. In fact, respondent should not have
filed a motion to admit her amended complaint. It has always been the policy of this Court to be liberal in allowing amendments
to pleadings in order that the real controversies between or among the parties may be presented and cases be decided on the
merits without delay.

53. AZOLLA When issues not raised by the


FARMS V. CA [GR pleadings are tried by express or FACTS: Petitioner Francis R. Yuseco, Jr., is the Chairman, President and Chief Operating Officer of petitioner Azolla
138085] 11 NOV 2004 implied consent of the parties, they Farms International Philippines (Azolla Farms), a corporation duly organized under existing laws and engaged in the
NAKAGAWA shall be treated in all respects, as development, exploitation, production, manufacturing, promotion, marketing, and sale of natural, organic minerals,
if they had been raised in the including its by-products, with the ultimate objective of utilizing said products for the promotion of food production.
pleadings. Such amendment of the In 1982, Azolla Farms undertook to participate in the National Azolla Production Program wherein it will purchase all
pleadings as may be necessary to the Azolla produced by the Azolla beneficiaries in the amount not exceeding the peso value of all the inputs provided
cause them to conform to the to them. The project also involves the then Ministry of Agriculture, the Kilusang Kabuhayan at Kaunlaran, and the
evidence and to raise these issues Kiwanis. To finance its participation, petitioners applied for a loan with Credit Manila, Inc., which the latter endorsed
may be made upon motion of any to its sister company, respondent Savings Bank of Manila (Savings Bank). The Board of Directors of Azolla Farms,
party at any time, even after meanwhile, passed a board resolution on August 31, 1982, authorizing Yuseco to borrow from Savings Bank in an
judgment; but failure so to amend amount not exceeding P2,200,000.00.
does not affect the result of the The loan having been approved, Yuseco executed a promissory note on September 13, 1982, promising to pay
trial of these issues. If evidence is Savings Bank the sum of P1,400,000.00 on or before September 13, 1983. The net proceeds of P1,225,443.31 was
objected to at the trial on the released to FNCB Finance, the mortgagee of a 548 - square meter lot with residential house owned by Yuseco. With
ground that it is not within the the release of the proceeds, FNCB Finance released the mortgage, and in turn, the property was mortgaged to
issues made by the pleadings, the Savings Bank as collateral for the loan. Yuseco and Francisco Bargas also executed an assignment of their shares of
court may allow the pleadings to stock in Azolla Farms as additional security. Yuseco then executed two other promissory notes on September 27,
be amended and shall do so freely 1982 and January 4, 1983, both for the amount of P300,000.00.
when the presentation of the However, the Azolla Farms project collapsed. Blaming Savings Bank, petitioners Yuseco and Azolla Farms filed on
merits of the action will be October 3, 1983 with the Regional Trial Court of Manila a complaint for damages. In essence, their complaint alleges
subserved thereby and the that Savings Bank unjustifiably refused to promptly release the remaining P300,000.00 which impaired the timetable
objecting party fails to satisfy the of the project and inevitably affected the viability of the project resulting in its collapse, and resulted in their failure to
court that the admission of such pay off the loan. Thus, petitioners pray for P1,000,000.00 as actual damages, among others.
evidence would prejudice him in Respondent Savings Bank filed its Answer denying the allegations in the complaint. It contends that there was
maintaining his action or defense evidence that Yuseco was using the loan proceeds for expenses totally unrelated to the project and they decided to
upon the merits. withhold the remaining amount until Yuseco gave the assurance that the diversion of the funds will be stopped.
Respondent bank believed that the 90-day interval between the two tranches could not have impaired the operation of
the project, and petitioners' subsequent receipt of the proceeds confirmed their agreement to the terms of the loan.

ISSUE: WON THE COMPLAINT SHOULD BE AMENDED?

HELD: YES. The amendment of the complaint was made pursuant to Section 5, Rule 10 of the Rules of Court,
governing amendment of pleadings to conform to evidence, to wit:

SEC. 5. Amendment to conform to or authorize presentation of evidence . When issues not raised by the pleadings
are tried by express or implied consent of the parties, they shall be treated in all respects, as if they had been raised
in the pleadings. Such amendment of the pleadings as may be necessary to cause them to conform to the evidence
and to raise these issues may be made upon motion of any party at any time, even after judgment; but failure so to
amend does not affect the result of the trial of these issues. If evidence is objected to at the trial on the ground that it
is not within the issues made by the pleadings, the court may allow the pleadings to be amended and shall do so
freely when the presentation of the merits of the action will be subserved thereby and the objecting party fails to
satisfy the court that the admission of such evidence would prejudice him in maintaining his action or defense upon
the merits. The court may grant a continuance to enable the objecting party to meet such evidence.

The trial court granted the motion and admitted the Amended Complaint. The Court of Appeals, however, ruled that
the trial court should not have admitted the Amended Complaint because it altered petitioners' cause of action.
Apparently, the Court of Appeals treated petitioners' amendment of the complaint as one involving amendments after
the case is set for hearing under Section 3, Rule 10 of the Rules of Court, which is not however applicable to the
present case.

In Mercader v. Development Bank of the Phils., the Court explained that the foregoing provision envisions two
scenarios - - first, when evidence is introduced on an issue not alleged in the pleadings and no objection was
interjected and second, when evidence is offered on an issue not alleged in the pleadings but this time an objection
was interpolated. In cases where an objection is made, the court may nevertheless admit the evidence where the
adverse party fails to satisfy the court that the admission of the evidence would prejudice him in maintaining his
defense upon the merits, and the court may grant him a continuance to enable him to meet the new situation created
by the evidence.

As can be gleaned from the records, it was petitioners' belief that respondent's evidence justified the amendment of
their complaint. The trial court agreed thereto and admitted the amended complaint. On this score, it should be noted
that courts are given the discretion to allow amendments of pleadings to conform to the evidence presented during
the trial. Thus, in Bank of America, NT and SA v. American Realty Corporation, the Court stated:

There have been instances where the Court has held that even without the necessary amendment, the amount proved
at the trial may be validly awarded, as in Tuazon v. Bolanos (95 Phil. 106), where we said that if the facts shown
entitled plaintiff to relief other than that asked for, no amendment to the complaint was necessary, especially where
defendant had himself raised the point on which recovery was based. The appellate court could treat the pleading as
amended to conform to the evidence although the pleadings were actually not amended. Amendment is also
unnecessary when only clerical error or non substantial matters are involved, as we held in Bank of the Philippine
Islands v. Laguna (48 Phil. 5). In Co Tiamco v. Diaz (75 Phil. 672), we stressed that the rule on amendment need not be
applied rigidly, particularly where no surprise or prejudice is caused the objecting party. And in the recent case of
National Power Corporation v. Court of Appeals (113 SCRA 556), we held that where there is a variance in the
defendant's pleadings and the evidence adduced by it at the trial, the Court may treat the pleading as amended to
conform with the evidence.

Verily, the trial court cannot be faulted for admitting the amended complaint as it had the discretion to do so.
54. SWAGMAN unless the plaintiff has a valid and Facts:
HOTELS AND subsisting cause of action at the time Sometime in 1996 and 1997, petitioner Swagman Hotels and Travel, Inc., through Atty. Leonor L. Infante and Rodney David
TRAVEL, INC. V. CA his action is commenced, the defect Hegerty, its president and vice-president, respectively, obtained from private respondent Neal B. Christian loans evidenced by
[GR 161135] 08 APR cannot be cured or remedied by the three promissory notes dated 7 August 1996, 14 March 1997, and 14 July 1997. Each of the promissory notes is in the amount
2005 acquisition or accrual of one while the of US$50,000 payable after three years from its date with an interest of 15% per annum payable every three months. In a letter
NATO action is pending, and a supplemental dated 16 December 1998, Christian informed the petitioner corporation that he was terminating the loans and demanded from
complaint or an amendment setting the latter payment in the total amount of US$150,000 plus unpaid interests in the total amount of US$13,500.
up such after accrued cause of action
is not permissible. On 2 February 1999, private respondent Christian filed with the Regional Trial Court of Baguio City, Branch 59, a complaint for
a sum of money and damages against the petitioner corporation, Hegerty, and Atty. Infante. The complaint alleged as follows:
On 7 August 1996, 14 March 1997, and 14 July 1997, the petitioner, as well as its president and vice-president obtained loans
from him in the total amount of US$150,000 payable after three years, with an interest of 15% per annum payable quarterly or
every three months. For a while, they paid an interest of 15% per annum every three months in accordance with the three
promissory notes. However, starting January 1998 until December 1998, they paid him only an interest of 6% per annum,
instead of 15% per annum, in violation of the terms of the three promissory notes. Thus, Christian prayed that the trial court
order them to pay him jointly and solidarily the amount of US$150,000 representing the total amount of the loans; US$13,500
representing unpaid interests from January 1998 until December 1998; P100,000 for moral damages; P50,000 for attorney's
fees; and the cost of the suit.
Issues:
May a complaint that lacks a cause of action at the time it was filed be cured by the accrual of a cause of action during the
pendency of the case?
Ruling:
No. Jurisprudence states that unless the plaintiff has a valid and subsisting cause of action at the time his action is
commenced, the defect cannot be cured or remedied by the acquisition or accrual of one while the action is pending, and a
supplemental complaint or an amendment setting up such after accrued cause of action is not permissible.
It is a rule of law to which there is, perhaps, no exception, either at law or in equity, that to recover at all there must be some
cause of action at the commencement of the suit. As observed by counsel for appellees, there are reasons of public policy why
there should be no needless haste in bringing up litigation, and why people who are in no default and against whom there is
yet no cause of action should not be summoned before the public tribunals to answer complaints which are groundless. We
say groundless because if the action is immature, it should not be entertained, and an action prematurely brought is a
groundless suit.
It is true that an amended complaint and the answer thereto take the place of the originals which are thereby regarded as
abandoned (Reynes v. CompañÃa General de Tabacos [1912], 21 Phil. 416; Ruyman and Farris v. Director of Lands [1916],
34 Phil., 428) and that "the complaint and answer having been superseded by the amended complaint and answer thereto, and
the answer to the original complaint not having been presented in evidence as an exhibit, the trial court was not authorized to
take it into account." (Bastida v. Menzi & Co. [1933], 58 Phil., 188.) But in none of these cases or in any other case have we
held that if a right of action did not exist when the original complaint was filed, one could be created by filing an amended
complaint. In some jurisdictions in the United States what was termed an "imperfect cause of action" could be perfected by
suitable amendment (Brown v. Galena Mining & Smelting Co., 32 Kan., 528; Hooper v. City of Atlanta, 26 Ga. App., 221) and
this is virtually permitted in Banzon and Rosauro v. Sellner ([1933], 58 Phil., 453); Asiatic Potroleum [sic] Co. v. Veloso ([1935],
62 Phil., 683); and recently in Ramos v. Gibbon (38 Off. Gaz., 241). That, however, which is no cause of action whatsoever
cannot by amendment or supplemental pleading be converted into a cause of action: Nihil de re accrescit ei qui nihil in re
quando jus accresceret habet.
We are therefore of the opinion, and so hold, that unless the plaintiff has a valid and subsisting cause of action at the tim e his
action is commenced, the defect cannot be cured or remedied by the acquisition or accrual of one while the action is pending,
and a supplemental complaint or an amendment setting up such after-accrued cause of action is not permissible. (Emphasis
ours).
Hence, contrary to the holding of the trial court and the Court of Appeals, the defect of lack of cause of action at the
commencement of this suit cannot be cured by the accrual of a cause of action during the pendency of this case arising from
the alleged maturity of two of the promissory notes on 7 August 1999 and 14 March 2000.

55. YOUNG V. As its very name denotes, a FACTS:


SPS. SY [GR 157745 & supplemental pleading only serves to G.R. No. 157955 (Re: Supplemental Complaint)
157955] 26 SEP 2014 bolster or add something to the On July 20, 2000, the petitioner filed with the RTC a Motion to Admit Supplemental Complaint, attaching the Supplemental
PEREZ primary pleading. A supplement Complaint wherein petitioner invoked her right, as co-owner, to exercise the legal redemption. The RTC denied the Motion in
exists side by side with the original. It an Order dated December 28, 2000. Petitioner, on July 16, 2001, filed a Petition for Certiorari and Mandamus under Rule 65 of
does not replace that which it the Rules of Court, docketed as CA-G.R. SP No. 65629, and raised the following grounds:
supplements. Moreover, a THE HONORABLE RESPONDENT COURT ACTED WITHOUT OR IN EXCESS OF JURISDICTION OR WITH GRAVE
supplemental pleading assumes that ABUSE OF DISCRETION IN ISSUING THE ORDERS DATED 28 DECEMBER 2000 AND 06 APRIL 2001 SINCE:
the original pleading is to stand and A.
that the issues joined with the original THE RELIEFS IN THE SUPPLEMENTAL COMPLAINT MERELY DEVELOP OR EXTEND THE ORIGINAL CAUSES OF
pleading remained an issue to be ACTION. PLAINTIFF'S CAUSE OF ACTION FOR LEGAL REDEMPTION ARISES DIRECTLY FROM AND IS A NATURAL
tried in the action. It is but a EXTENSION OR CONSEQUENCE OF HER RIGHTS AS CO-OWNER OF THE PROPERTY SUBJECT OF THE CASE.
continuation of the complaint. Its B.
usual office is to set up new facts THE SUPERVENING EVENT WHICH IS THE CONSOLIDATION OF TITLE TO THE SUBJECT PROPERTY IN THE NAME
which justify, enlarge or change the OF MANUEL SY, OCCURRED AFTER 21 JUNE 2000; SUCH DATE IS PLAINLY SUBSEQUENT TO THE FILING OF THE
kind of relief with respect to the same COMPLAINT ON 02 MAY 2000.5
subject matter as the controversy On November 18, 2002, the CA promulgated its Decision denying the Petition for Certiorari and Mandamus and held that the
referred to in the original complaint. cause of action of the petitioner in the Supplemental Complaint is entirely different from the original complaint; that the
The purpose of the supplemental Supplemental Complaint did not merely supply its deficiencies; and that, at any rate, in the event the trial court issues an
pleading is to bring into the records adverse ruling, the petitioner can still appeal the same, hence, the petition under Rule 65 is not proper.
new facts which will enlarge or Hence, the present Petition for Review on Certiorari under Rule 45, raising the following issues:
change the kind of relief to which the A.
plaintiff is entitled; hence, any WHETHER OR NOT THE RTC ACTED WITHOUT OR IN EXCESS OF JURISDICTION OR WITH GRAVE ABUSE OF
supplemental facts which further DISCRETION AMOUNTING TO LOSS OF JURISDICTION IN ISSUING THE ORDERS DATED 28 DECEMBER 2000 AND 06
develop the original right of action, or APRIL 2001.
extend to vary the relief, are available 1.
by way of supplemental complaint WHETHER OR NOT THE RELIEFS IN THE SUPPLEMENTAL COMPLAINT MERELY DEVELOP OR EXTEND THE
even though they themselves ORIGINAL CAUSES OF ACTION.
constitute a right of action. 2.
WHETHER OR NOT THE SUPERVENING EVENT WHICH IS THE CONSOLIDATION OF TITLE TO THE SUBJECT
A matter stated in a supplemental PROPERTY IN THE NAME OF MANUEL SY, OCCURRED AFTER 21 JUNE 2000 OR SUBSEQUENT TO THE FILING OF
complaint should have some relation THE COMPLAINT ON 02 MAY 2000.
to the cause of action set forth in the B.
original pleading, the fact that the WHETHER OR NOT THE COURT OF APPEALS GRAVELY ERRED IN HOLDING THAT NO GRAVE ABUSE OF
supplemental pleading technically DISCRETION WAS COMMITTED BY THE RTC AND THAT THERE WAS NO NEED TO FILE A "PETITION" TO EXERCISE
states a new cause of action should THE RIGHT OF LEGAL REDEMPTION.
not be a bar to its allowance but only C.
a factor to be considered by the court WHETHER OR NOT THE INSTANT PETITION IS MOOT AND ACADEMIC.
in the exercise of its discretion; and of D.
course, a broad definition of "cause of WHETHER OR NOT PETITIONER COMMITTED FORUM-SHOPPING.6
action" should be applied here as G.R. No. 157745 (Re: Non-Suit)
elsewhere. I. Appeal to the CA
While the Petition for Certiorari and Mandamus (re: Supplemental Complaint) was pending in the CA, trial in the RTC
Forum shopping consists of filing continued. On August 29, 2001, a day before the hearing slated for August 30, 2001, the petitioner filed a Motion to Cancel
multiple suits involving the same Hearing, alleging that she was indisposed. On the day of the hearing, respondents, through counsel, objected to the
parties for the same cause of action, postponement and moved for the dismissal of the case for non-suit. The RTC sustained the objection and issued the assailed
either simultaneously or successively, August 30, 2001 Order dismissing the Complaint. This Order reads in full:
for the purpose of obtaining a ORDER
favorable judgment. Atty. Raul S. Sison and his client arrived on time. When the case was called for hearing, the Court found attached to the
There is forum shopping where there records a last minute Motion to Cancel Hearing from Atty. Perpetuo M. Lotilla, Jr. The Court invited the attention of Atty. Sison
exist: (a) identity of parties, or at least on the said motion. Atty. Sison vehemently objected to the postponement on the following grounds:
such parties as represent the same 1) the motion is in violation of the three-day notice rule;
interests in both actions; (b) identity 2) the ground stated in the motion is too shallow to be appreciated because it merely states that a witness is indisposed
of rights asserted and relief prayed without stating the indisposition and there is no Medical Certificate attached to the motion;
for, the relief being founded on the 3) the instant motion for postponement is one of the several postponements filed by Atty. Lotilla and this is confirmed by the
same facts; and (c) the identity of the records of this case showing that last minute postponements and other postponements were filed by Atty. Lotilla;
two preceding particulars is such that 4) that damages are being suffered by defendants in paying the legal services of their counsel and that defendants are unduly
any judgment rendered in the deprived of the possession and enjoyment of the subject property.
pending case, regardless of which The Court is constrained to sustain the objection to the Motion for Postponement by Atty. Sison. The Court has also been quite
party is successful would amount to liberal with the Motions for Postponement filed by Atty. Lotilla by granting the same. The Court holds that somehow the
res judicata. practice of filing several postponements must be discouraged.
Atty. Sison therefore moved for the dismissal of the case for non-suit. The Court finds merit on the Motion to Dismiss.
WHEREFORE, the Motion to Dismiss is granted and this case is ordered DISMISSED without costs.
SO ORDERED.7
On January 4, 2002, the RTC denied the petitioner's Motion for Reconsideration. The dispositive portion of this Order states:
WHEREFORE, the Motion for Reconsideration is DENIED. The resolution on the pending incident of execution pendente lite is
now considered moot and academic.8
On January 16, 2002, the RTC issued an Order correcting the January 4, 2002 Order due to a typographical error. This Order
reads in full:
ORDER
Finding merit on the Motion, the same is granted. The Court is sure that only typographical error was committed.
The dispositive portion of the Order should therefore read as follows:
"WHEREFORE, the Motion for Reconsideration is DENIED. The resolution on the pending incident of Motion for Writ of
Possession, pendente lite, is now considered moot and academic.
SO ORDERED.9
On January 31, 2002, the petitioner filed a Notice of Appeal questioning the foregoing RTC Orders. The case was eventually
docketed as C.A.-G.R. CV No. 74045. In said appeal, the petitioner assigned the following errors:
A.
THE TRIAL COURT GRAVELY ERRED IN ISSUING THE ORDERS DATED 30 AUGUST 2001, 04 JANUARY 2002 AND 16
JANUARY 2002, SINCE THERE WAS NO FACTUAL OR LEGAL BASIS TO DISMISS THE COMPLAINT FOR NON-SUIT.
B.
THE TRIAL COURT GRAVELY ERRED IN NOT HOLDING THAT PLAINTIFF-APPELLANT HAD A JUST AND VALID
GROUND TO MOVE FOR THE CANCELLATION OF THE HEARING SET ON 30 AUGUST 2001.10
The CA rendered a Decision dated March 30, 200511 in favor of the petitioner, reversing and setting aside the RTC Orders,
the dispositive portion of this Decision reads:
WHEREFORE, premises considered, the Orders, dated August 30, 2001, January 4, 2002 and January 16, 2002, issued by
Branch 32 of the Regional Trial Court of San Pablo City are hereby REVERSED and SET ASIDE. The record/case is hereby
remanded to the court of origin for further proceedings.
SO ORDERED.12
The respondents filed their Motion for Reconsideration, and based on the records before the Court, this case is still pending in
the CA.
II. Petition for Certiorari filed with the CA
On top of the foregoing appeal, the petitioner, four months after filing her Notice of Appeal to the CA, or on May 28, 2002, filed
with the CA a Petition for Certiorari under Rule 65, docketed as CA-G.R. SP No. 70610 to annul the same RTC Orders that
comprise the subject matter of the ordinary appeal. Predictably, the petitioner raised essentially the same issues:
THE HONORABLE RESPONDENT COURT ACTED WITHOUT OR IN EXCESS OF JURISDICTION OR WITH GRAVE
ABUSE OF DISCRETION AMOUNTING TO LOSS OF JURISDICTION IN ISSUING THE ORDERS DATED AUGUST 30,
2001, JANUARY 4, 2002, AND JANUARY 16, 2002, SINCE:
A.
THERE WAS NO FACTUAL OR LEGAL BASIS FOR DISMISSAL OF THE COMPLAINT ON THE GROUNDS OF NON-SUIT;
B.
PETITIONER HAD A JUST AND VALID GROUND TO MOVE FOR THE CANCELLATION OF THE HEARING SET ON
AUGUST 30, 2001.13
On November 29, 2002, ahead of the Decision dated March 30, 2005 rendered in the appealed case, the CA denied14 the
Petition for Certiorari and held that the dismissal of the case by the RTC on the ground of non prosequitur has the effect of an
adjudication upon the merits; that an order of dismissal, whether right or wrong, is a final order that may constitute an error of
judgment correctible by ordinary appeal and not by certiorari; that the petitioner actually chose the mode of ordinary appeal by
filing a Notice of Appeal on January 31, 2000; and that since the remedy of appeal was available, then the petition for
certiorari, being an extraordinary remedy, must fail.
Hence, the present Petition for Review under Rule 45, with the following issues that are likewise similar to the appealed case in
the CA:
A.
WHETHER OR NOT THE REGIONAL TRIAL COURT ACTED WITHOUT OR IN EXCESS OF JURISDICTION OR WITH
GRAVE ABUSE OF DISCRETION AMOUNTING TO LOSS OF JURISDICTION IN ISSUING THE ORDERS DATED AUGUST
30, 2001, JANUARY 4, 2002, AND JANUARY 16, 2002, DISMISSING THE COMPLAINT.
1.
WHETHER OR NOT THERE WAS FACTUAL OR LEGAL BASIS FOR DISMISSAL OF THE COMPLAINT ON THE
GROUNDS OF NON-SUIT.
2.
WHETHER OR NOT PETITIONER HAD A JUST AND VALID GROUND TO MOVE FOR THE CANCELLATION OF THE
HEARING SET ON AUGUST 30, 2001.
B.
WHETHER OR NOT THE COURT OF APPEALS GRAVELY ERRED IN HOLDING THAT NO GRAVE ABUSE OF
DISCRETION WAS COMMITTED BY THE RTC AND THAT ORDINARY APPEAL IS PETITIONER'S REMEDY FROM THE
DISMISSAL OF THE COMPLAINT BY THE RTC.15

ISSUE:
Whether or not both the Petition (re: Supplemental Complaint) and the Petition (re: Non-Suit) are meritorious.
HELD:
The Petition (re: Supplemental Complaint) is meritorious; but the Petition (re: Non-Suit) must fail.
On the denial of the Motion to Admit Supplemental Complaint:
The courts a quo held that the Supplemental Complaint constituted a substantial amendment of the original complaint; that the
relief prayed for in the former is inconsistent with the latter; and that the causes of action of both are likewise different. This is
incorrect.
Section 6, Rule 10 of the Revised Rules of Court provides:
SECTION 6. Supplemental Pleadings. - Upon motion of a party the court may, upon reasonable notice and upon such terms as
are just, permit him to serve a supplemental pleading setting forth transactions, occurrences or events which have happened
since the date of the pleading sought to be supplemented. The adverse party may plead thereto within ten (10) days from
notice of the order admitting the supplemental pleading.
As its very name denotes, a supplemental pleading only serves to bolster or add something to the primary pleading. A
supplement exists side by side with the original. It does not replace that which it supplements. Moreover, a supplemental
pleading assumes that the original pleading is to stand and that the issues joined with the original pleading remained an issue
to be tried in the action. It is but a continuation of the complaint. Its usual office is to set up new facts which justify, enlarge or
change the kind of relief with respect to the same subject matter as the controversy referred to in the original complaint.
The purpose of the supplemental pleading is to bring into the records new facts which will enlarge or change the kind of relief
to which the plaintiff is entitled; hence, any supplemental facts which further develop the original right of action, or extend to
vary the relief, are available by way of supplemental complaint even though they themselves constitute a right of action.
In Leobrera v. Court of Appeals, the Court ruled that when the cause of action stated in the supplemental complaint is different
from the causes of action mentioned in the original complaint, the court should not admit the supplemental complaint; the
parties may file supplemental pleadings only to supply deficiencies in aid of an original pleading, but not to introduce new and
independent causes of action. However, in Planters Development Bank v. LZK Holdings and Development Co., the Court held
that a broad definition of causes of action should be applied: while a matter stated in a supplemental complaint should have
some relation to the cause of action set forth in the original pleading, the fact that the supplemental pleading technically states
a new cause of action should not be a bar to its allowance but only a factor to be considered by the court in the exercise of its
discretion; and of course, a broad definition of "cause of action" should be applied here as elsewhere
In this case, the consolidation of title over the subject property in the name of respondent Manuel Sy and the issue as to
whether it precluded petitioner as alleged co-owner from exercising the right of legal redemption, are new matters that
occurred after the filing of the original complaint. The relief prayed for in the Supplemental Complaint, which is the exercise of
the right of legal redemption accorded to co-owners of property, is germane to and intertwined with the cause of action in the
Complaint for the nullification of the "Second Supplemental to the Extrajudicial Partition" on the ground that it lacked the
approval of a guardianship court.
The petitioner's right to redeem the property is dependent on the nullification of the partition which is the subject of the original
complaint. Unless the partition is nullified or declared without any force or effect, the petitioner will not be considered a co-
owner of the property and, consequently, she will be unable to exercise any right of legal redemption under Article 1620 of the
Civil Code granted to co-owners of property.
The right of legal redemption as co-owner is conferred by law and is merely a natural consequence of co-ownership. Hence,
the petitioner's cause of action for legal redemption as embodied in her Supplemental Complaint stems directly from and is an
extension of her rights as co-owner of the property subject of the Complaint.
Furthermore, the evidence required to prove petitioner's right of legal redemption in the Supplemental Complaint will be exactly
the same evidence required to prove the nullification of the partition in the Complaint.
If a separate action is filed for the subject covered by the Supplemental Complaint, there will be multiplicity of suits. Should a
separate complaint be filed before the nullification of the partition, the same would be dismissed for being premature pending
the resolution of the Complaint for nullification.
After all, the respondents have the right to file a supplemental answer to the Supplemental Complaint, conformably with
Section 7, Rule 11 of the Rules of Court which reads:
SEC. 7. Answer to supplemental complaint. - A supplemental complaint may be answered within ten (10) days from notice of
the order admitting the same, unless a different period is fixed by the court. The answer to the complaint shall serve as the
answer to the supplemental complaint if no new or supplemental answer is filed.
In affirming the RTC's denial of the admission of the Supplemental Complaint, the CA rationalized that "[i]n the event that the
lower court rules in favor of petitioner, then there is no need for her to file a petition to exercise the right of redemption. On the
other hand, should the trial court issue[ ] an adverse ruling then petitioner can still appeal the same. The petition for certiorari is
therefore not proper."
This, too, is incorrect.
As the petitioner correctly pointed out, even if the trial court decides in her favor, the redemption period would have lapsed and
would not form a part of the decision since it was not prayed for, much less alleged in the original complaint. In such a case,
the respondents could oppose the exercise of the right to redeem since it would not have been included in the decision over
the original complaint. And should the trial court issue an adverse ruling, the petitioner can only appeal what is included in the
ruling which is limited to the denial of the prayer for the nullification of the partition. Naturally, such a decision would not
concern any right of redemption.
Besides, as in Planters Development Bank, the admission of the petitioner's Supplemental Complaint will better serve the ends
of justice. The Rules of Court were designed to facilitate the administration of justice to the rival claims of the parties in a just,
speedy and inexpensive manner.
Thus, the courts a quo erred in denying the admission of petitioner's Supplemental Complaint and the Petition (G.R. No.
157955) should be granted.
On the alleged Forum Shopping:
This Court is now concerned with the question of whether the petitioner has engaged in forum shopping in appealing the RTC
Orders which dismissed her complaint for non-suit and in filing a Petition for Certiorari under Rule 65 with the CA involving the
same RTC Orders.
Forum shopping consists of filing multiple suits involving the same parties for the same cause of action, either simultaneously
or successively, for the purpose of obtaining a favorable judgment.
There is forum shopping where there exist: (a) identity of parties, or at least such parties as represent the same interests in
both actions; (b) identity of rights asserted and relief prayed for, the relief being founded on the same facts; and (c) the identity
of the two preceding particulars is such that any judgment rendered in the pending case, regardless of which party is
successful would amount to res judicata.
Ineluctably, the petitioner, by filing an ordinary appeal and a petition for certiorari with the CA, engaged in forum shopping.
When the petitioner commenced the appeal, only four months had elapsed prior to her filing with the CA the Petition for
Certiorari under Rule 65 and which eventually came up to this Court by way of the instant Petition (re: Non-Suit).
The elements of litis pendentia are present between the two suits. As the CA, through its Thirteenth Division, correctly noted,
both suits are founded on exactly the same facts and refer to the same subject matter—the RTC Orders which dismissed Civil
Case No. SP-5703 (2000) for failure to prosecute. In both cases, the petitioner is seeking the reversal of the RTC orders. The
parties, the rights asserted, the issues professed, and the reliefs prayed for, are all the same. It is evident that the judgment of
one forum may amount to res judicata in the other.
But it is the proposition of the petitioner that between these two cases, the one that is "proper" is the petition for certiorari filed
with the CA, since the RTC, according to her, acted with grave abuse of discretion; and that her appeal in the CA "has proven
to be not a speedy remedy" and had only been instituted as a "precautionary measure." As proof of the averment that the
appeal was not speedy enough, she points out the fact that while the CA had just promulgated a Decision on March 30, 2005
with respect to the appealed case, that case, however, is still pending to this day in the CA by virtue of a motion for
reconsideration recently filed by the respondents, whereas, in the proceedings that led to the present Petition (re: Non-Suit),
the CA had rendered a Decision dated November 29, 2000 – over four years ahead of its counterpart. From these premises,
she proceeds to cite jurisprudence invoking the exceptional instances where a party may directly resort to the extraordinary
remedy of certiorari, because the appeal, in those cases, is not speedy enough.29
This is completely unacceptable.
The Court begins with the unassailable premise that the RTC orders dismissing the case for failure to prosecute are final
orders, because such orders of dismissal operate as a judgment on the merits. This principle is now an express provision in
Section 3, Rule 17 of the Rules of Court, to wit:
Section 3. Dismissal due to fault of plaintiff. – If, for no justifiable cause, the plaintiff fails to appear on the date of the
presentation of his evidence in chief on the complaint, or to prosecute his action for an unreasonable length of time, or to
comply with these Rules or any order of the court, the complaint may be dismissed upon motion of the defendant or upon the
court's own motion, without prejudice to the right of the defendant to prosecute his counterclaim in the same or in a separate
action. This dismissal shall have the effect of an adjudication upon the merits, unless otherwise declared by the court.
(emphasis supplied)
It is firmly established, and with very few exceptions, that the remedy against such final order is appeal and not certiorari.31
The general rule is that a writ of certiorari will not issue where the remedy of appeal is available to the aggrieved party. The
remedies of appeal in the ordinary course of law and that of certiorari under Rule 65 are mutually exclusive and not alternative
or cumulative. Hence, the special civil action of certiorari under Rule 65 cannot be a substitute for an appeal where the latter
remedy is available.
While indeed there are exceptions to the foregoing rule, and assuming further that the case of the petitioner falls under any of
those exceptions which allows her to elect Rule 65, the jurisprudence which she calls upon does not sanction the successive
or cumulative filing of both an appeal and a special civil action of certiorari. Quite the opposite, these cases set down the
exceptional circumstances where certiorari can be directly invoked in lieu of appeal.
The remedies of appeal and certiorari under Rule 65 are mutually exclusive and not alternative or cumulative.33 This is a firm
judicial policy. The petitioner cannot hedge her case by wagering two or more appeals, and, in the event that the ordinary
appeal lags significantly behind the others, she cannot post facto validate this circumstance as a demonstration that the
ordinary appeal had not been speedy or adequate enough, in order to justify the recourse to Rule 65. This practice, if adopted,
would sanction the filing of multiple suits in multiple fora, where each one, as the petitioner couches it, becomes a
"precautionary measure" for the rest, thereby increasing the chances of a favorable decision. This is the very evil that the
proscription on forum shopping seeks to put right. In Guaranteed Hotels, Inc. v. Baltao, the Court stated that the grave evil
sought to be avoided by the rule against forum shopping is the rendition by two competent tribunals of two separate and
contradictory decisions. Unscrupulous party litigants, taking advantage of a variety of competent tribunals, may repeatedly try
their luck in several different fora until a favorable result is reached. To avoid the resultant confusion, the Court adheres strictly
to the rules against forum shopping, and any violation of these rules results in the dismissal of the case.
Thus, the CA correctly dismissed the petition for certiorari and the petition for review (G.R. No. 157745) filed with this Court
must be denied for lack of merit.
WHEREFORE, the Petition for Review in G.R. No. 157745 is DENIED for lack of merit.
The Petition for Review in G.R. No. 157955 is GRANTED. The Decisions and Resolutions of the Court of Appeals in CA-G.R.
SP No. 65629 are REVERSED AND SET ASIDE. The Regional Trial Court, San Pablo City, Branch 32, is DIRECTED to
ADMIT the petitioner's Supplemental Complaint dated July 20, 2000.
No costs.
SO ORDERED.

56. PHILIPPINE When evidence is presented by one Facts:


NATIONAL BANK V. party, with the expressed or implied Respondent Spouses Enrique Manalo and Rosalinda Jacinto (Spouses Manalo) applied for an All-Purpose Credit
SPS. MANALO [GR consent of the adverse party, as to Facility for P1,000,000.00 with Philippine National Bank (PNB) to finance the construction of their house. After PNB granted
174433] 24 FEB 2014 issues not alleged in the pleadings, their application, they executed a Real Estate Mortgage in favor of PNB over their property covered by Transfer Certificate of
PURIFICACION judgment may be rendered validly as Title No. S-23191 as security for the loan. The credit facility was renewed and increased several times over the years. On
regards those issues, which shall be September 20, 1996, the credit facility was again renewed for P7,000,000.00. As a consequence, the parties executed a
considered as if they have been Supplement to and Amendment of Existing Real Estate Mortgage whereby the property covered by TCT No. 171859 was
raised in the pleadings. added as security for the loan. The additional security was registered in the names of respondents Arnold, Arnel, Anthony, and
Arma, all surnamed Manalo, who were their children.
Section 5, Rule 10 of the Rules of
While it was agreed upon that the Spouses Manalo would make monthly payments on the interest, PNB claimed that
Court is applicable in two situations.
their last recorded payment was made on December, 1997.
The first is when evidence is
introduced on an issue not alleged in After the Spouses Manalo still failed to settle their unpaid account despite the two demand letters, PNB foreclose the
the pleadings and no objection is mortgage. During the foreclosure sale, PNB was the highest bidder for P15,127,000.00 of the mortgaged properties of the
interposed by the adverse party. The Spouses Manalo.
second is when evidence is offered
on an issue not alleged in the After more than a year after the Certificate of Sale had been issued to PNB, the Spouses Manalo instituted this action
pleadings but an objection is raised for the nullification of the foreclosure proceedings and damages. They alleged that they had obtained a loan for P1,000,000.00
against the offer. from a certain Benito Tan upon arrangements made by Antoninus Yuvienco, then the General Manager of PNB's Bangkal
Branch where they had transacted; that they had been made to understand and had been assured that the P1,000,000.00
would be used to update their account, and that their loan would be restructured and converted into a long-term loan; that they
had been surprised to learn, therefore, that had been declared in default of their obligations, and that the mortgage on their
property had been foreclosed and their property had been sold; and that PNB did not comply with Section 3 of Act No. 3135,
as amended.
PNB and Antoninus Yuvienco countered that the P1,000,000.00 loan obtained by the Spouses Manalo from Benito
Tan had been credited to their account; that they did not make any assurances on the restructuring and conversion of the
Spouses Manalo's loan into a long-term one; that PNB's right to foreclose the mortgage had been clear especially because the
Spouses Manalo had not assailed the validity of the loans and of the mortgage; and that the Spouses Manalo did not allege
having fully paid their indebtedness.
RTC - rendered decision in favor of PNB.

CA - affirmed the decision of the RTC insofar as it upheld the validity of the foreclosure proceedings initiated by PNB, but
modified the Spouses Manalo's liability for interest.

PNB - argues that:


1. By passing upon the issue of the validity of the interest rates, and in nullifying the rates imposed on the Spouses
Manalo, the CA decided the case in a manner not in accord with Section 15, Rule 44 of the Rules of Court, which
states that only questions of law or fact raised in the trial court could be assigned as errors on appeal;
2. To allow the Spouses Manalo to raise an issue for the first time on appeal would "offend the basic rules of fair play,
justice and due process;"
3. That the resolution of the CA was limited to the issues agreed upon by the parties during pre-trial; that the CA erred in
passing upon the validity of the interest rates inasmuch as the Spouses Manalo did not present evidence thereon; and
that the Judicial Affidavit of Enrique Manalo, on which the CA relied for its finding, was not offered to prove the
invalidity of the interest rates and was, therefore, inadmissible for that purpose.
Issue:
Whether the RTC erred in accepting the evidence raised by the Private Respondent for the first time during trial. - NO.

Held:
Contrary to PNB's argument, the validity of the interest rates and of the increases, and on the lack of mutuality
between the parties were not raised by the Spouses Manalo's for the first time on appeal. Rather, the issues were impliedly
raised during the trial itself, and PNB's lack of vigilance in voicing out a timely objection made that possible.
It appears that Enrique Manalo's Judicial Affidavit introduced the issues of the validity of the interest rates and
the increases, and the lack of mutuality between the parties. PNB cross-examined Enrique Manalo upon his Judicial
Affidavit. There is no showing that PNB raised any objection in the course of the cross examination. Consequently, the RTC
rightly passed upon such issues in deciding the case, and its having done so was in total accord with Section 5, Rule
10 of the Rules of Court, which states:
Section 5. Amendment to conform to or authorize presentation of evidence. — When issues not
raised by the pleadings are tried with the express or implied consent of the parties, they shall be
treated in all respects as if they had been raised in the pleadings. Such amendment of the pleadings as
may be necessary to cause them to conform to the evidence and to raise these issues may be made upon
motion of any party at any time, even after judgment; but failure to amend does not affect the result of the
trial of these issues. If evidence is objected to at the trial on the ground that it is not within the issues made
by the pleadings, the court may allow the pleadings to be amended and shall do so with liberality if the
presentation of the merits of the action and the ends of substantial justice will be subserved thereby. The
court may grant a continuance to enable the amendment to be made. CIDcHA
In Bernardo Sr. v. Court of Appeals, we held that:
It is settled that even if the complaint be defective, but the parties go to trial thereon, and the plaintiff,
without objection, introduces sufficient evidence to constitute the particular cause of action which it intended
to allege in the original complaint, and the defendant voluntarily produces witnesses to meet the cause of
action thus established, an issue is joined as fully and as effectively as if it had been previously joined by the
most perfect pleadings. Likewise, when issues not raised by the pleadings are tried by express or
implied consent of the parties, they shall be treated in all respects as if they had been raised in the
pleadings.
The RTC did not need to direct the amendment of the complaint by the Spouses Manalo. Section 5, Rule 10 of the
Rules of Court specifically declares that the "failure to amend does not affect the result of the trial of these issues."
According to Talisay-Silay Milling Co., Inc. v. Asociacion de Agricultores de Talisay-Silay, Inc.:
The failure of a party to amend a pleading to conform to the evidence adduced during trial
does not preclude an adjudication by the court on the basis of such evidence which may embody
new issues not raised in the pleadings, or serve as a basis for a higher award of damages. Although
the pleading may not have been amended to conform to the evidence submitted during trial, judgment may
nonetheless be rendered, not simply on the basis of the issues alleged but also on the basis of issues
discussed and the assertions of fact proved in the course of trial. The court may treat the pleading as if it had
been amended to conform to the evidence, although it had not been actually so amended. Former Chief
Justice Moran put the matter in this way:
When evidence is presented by one party, with the expressed or implied consent of
the adverse party, as to issues not alleged in the pleadings, judgment may be rendered
validly as regards those issues, which shall be considered as if they have been raised in the
pleadings. There is implied, consent to the evidence thus presented when the adverse party fails to
object thereto." (Emphasis supplied)
Clearly, a court may rule and render judgment on the basis of the evidence before it even though the
relevant pleading had not been previously amended, so long as no surprise or prejudice is thereby caused to
the adverse party. Put a little differently, so long as the basic requirements of fair play had been met, as
where litigants were given full opportunity to support their respective contentions and to object to or refute
each other's evidence, the court may validly treat the pleadings as if they had been amended to conform to
the evidence and proceed to adjudicate on the basis of all the evidence before it.
Section 5, Rule 10 of the Rules of Court is applicable in two situations. The first is when evidence is introduced on an
issue not alleged in the pleadings and no objection is interposed by the adverse party. The second is when evidence is offered
on an issue not alleged in the pleadings but an objection is raised against the offer. This case comes under the first situation.
Enrique Manalo's Judicial Affidavit would introduce the very issues that PNB is now assailing. The question of whether the
evidence on such issues was admissible to prove the nullity of the interest rates is an entirely different matter. The RTC
accorded credence to PNB's evidence showing that the Spouses Manalo had been paying the interest imposed upon them
without protest. On the other hand, the CA's nullification of the interest rates was based on the credit agreements that the
Spouses Manalo and PNB had themselves submitted.
Based on the foregoing, the validity of the interest rates and their increases, and the lack of mutuality between the
parties were issues validly raised in the RTC, giving the Spouses Manalo every right to raise them in their appeal to the CA.
PNB's contention was based on its wrong appreciation of what transpired during the trial. It is also interesting to note that PNB
did not itself assail the RTC's ruling on the issues obviously because the RTC had decided in its favor. In fact, PNB did not
even submit its appellee's brief despite notice from the CA.

57. CHING V. CA Pleadings superseded or amended FACTS: Petitioner Ching was charged before the RTC of Makati with 4 counts of estafa in relation to the Trust Receipts Law.
[GR 110844] 27 APR disappear from the record, lose their Ching filed an Omnibus Motion to Strike Out Information, or in the Alternative to Require Public Prosecutor to Conduct
2000 status as pleadings and cease to be Preliminary Investigation, and to Suspend in the Meantime Further Proceedings in these Cases.
SABIO judicial admission.
Acting on the omnibus motion, the RTC required the prosecutor’s office to conduct a PI and suspended further proceedings in
the criminal cases. Ching, together with Philippine Blooming Mills Co., Inc., filed a case before the RTC of Manila for
declaration of nullity of documents and for damages. Ching then filed a petition before the RTC of Makati for the suspension of
the criminal proceedings on the ground of prejudicial question in a civil action.

As a result, Ching moved to reconsider the order to which the prosecution filed an opposition which was denied. Ching brought
before the CA a petition for certiorari and prohibition, which sought to declare the nullity of the aforementioned orders. The CA
denied the petition.

ISSUE: Whether or not the denial of the CA of Ching’s petition is proper.

HELD: YES. Under the ROC, pleadings superseded or amended disappear from the record, lose their status as pleadings and
cease to be judicial admission. While they may nonetheless be utilized against the pleader as extrajudicial admissions, they
must, in order to have such effect, be formally offered in evidence. If not offered in evidence, the admission contained therein
will not be considered.

Consequently, the original complaint, having been amended, lost its character as a judicial admission, which would have
required no proof, and became merely an extrajudicial admission, the admissibility of which, as evidence, required its formal
offer.

In the instant case, the original complaint is deemed superseded by the amended complaint. Corollarily, the judicial admissions
in the original complaint are considered abandoned.

NOTE: Although the granting of leave to file amended pleadings is a matter peculiarly within the sound discretion of the trial
court and such discretion would not normally be disturbed on appeal, it also well to mention that this rule is relaxed when
evident abuse thereof is apparent.
CASE TITLE DOCTRINE FACTS-ISSUE/s-HELD

Cause of Action

1.Juana Complex The question of whether the Facts: Juana Complex I Homeowners Association, Inc. (JCHA), together with individual residents of Juana Complex I and
Homeowners vs. Fil- complaint states a cause of action other neighboring subdivisions, instituted a complaint for damages, in its own behalf and as a class suit representing the
Estate Land 152272 is determined by its averments regular commuters and motorists of Juana Complex I and neighboring subdivisions who were deprived of the use of La Paz
and 152397 05 March regarding the acts committed by Road, against Fil-Estate Land, Inc. (Fil-Estate), Fil-estate Ecocentrum Corporation (FEEC), La Paz Housing & Development
the defendant. Thus, it must Corporation (La Paz), and Warbird Security Agency and their respective officers.
2012- Purificacion
contain a concise statement of the
ultimate or essential facts JCHA, et al., were regular commuters and motorists who constantly travelled towards the direction of Manila and Calamba;
constituting the plaintiff's cause of that they used the entry and exit toll gates of South Luzon Expressway (SLEX) by passing through right-of-way public road
action. To be taken into account known as La Paz Road; that they had been using La Paz Road for more than ten (10) years. In August 1998, Fil-estate
are only the material allegations in excavated, broke and deliberately ruined La Paz Road that led to SLEX so JCHA, et al., would not be able to pass through the
the complaint; extraneous facts said road. La Paz Road was restored by the residents to make it passable but Fil-estate excavated the road again. JCHA
and circumstances or other reported the matter to the Municipal Government and the Office of the Municipal Engineer but the latter failed to repair the road
matters aliunde are not to make it passable and safe to motorists and pedestrians. JCHA, et al. alleged that the act of Fil-estate in excavating La Paz
considered. Road caused damage, prejudice, inconvenience, annoyance, and loss of precious hours to them, to the commuters and
motorists because traffic was re-routed to narrow streets that caused terrible traffic congestion and hazard; and that its
The test of sufficiency of facts
permanent closure would not only prejudice their right to free and unhampered use of the property but would also cause great
alleged in the complaint as
damage and irreparable injury.
constituting a cause of action is
whether or not admitting the facts
alleged, the court could render a JCHA, et al., also prayed for the immediate issuance of a Temporary Restraining Order (TRO) or a writ of preliminary
valid verdict in accordance with injunction (WPI) to enjoin Fil-Estate, et al., from stopping and intimidating them in their use of La Paz Road.
the prayer of said complaint.
Stated differently, if the Fil-Estate, et al., filed a motion to dismiss arguing that the complaint failed to state a cause of action and that it was improperly
allegations in the complaint filed as a class suit.
furnish sufficient basis by which
the complaint can be maintained, RTC issued an Order granting the WPI and required JCHA, et al., to post a bond.
the same should not be dismissed
regardless of the defense that Fil-Estate, et al., filed a motion for reconsideration arguing, among others, that JCHA, et al., failed to satisfy the requirements
may be asserted by the for the issuance of a WPI.
defendant.
RTC denied both the motion to dismiss and the motion for reconsideration filed by Fil-Estate, et al.
The necessary elements for the
maintenance of a class suit are: Fil-Estate, et al., filed a petition for certiorari and prohibition before the CA. They contended:
1. That the complaint failed to state a cause of action; and
1. the subject matter of 2. That it was improperly filed as a class suit.
controversy is one of
common or general The CA ruled that:
interest to many persons; 1. The complaint sufficiently stated a cause of action when JCHA, et al., alleged in their complaint that they had been
2. the parties affected are so using La Paz Road for more than ten (10) years and that their right was violated when Fil-Estate closed and excavated
numerous that it is the road.
impracticable to bring 2. The complaint was properly filed as a class suit as it was shown that the case was of common interest and that the
them all to court; and individuals sought to be represented were so numerous that it was impractical to include all of them as parties.
3. the parties bringing the
class suit are sufficiently JCHA, et al. concurred with the CA that the complaint sufficiently stated a cause of action.
numerous or
representative of the Fil-Estate, et al.,:
class and can fully protect 1. Insisted that the complaint did not sufficiently contain the ultimate facts to show a cause of action. They aver the bare
the interests of all allegation that one is entitled to something is an allegation of a conclusion which adds nothing to the pleading; and
concerned. 2. Argued that the complaint was improperly filed as a class suit for it failed to show that JCHA, et al., and the commuters
and motorists they are representing have a well-defined community of interest over La Paz Road. They claim that the
excavation of La Paz Road would not necessarily give rise to a common right or cause of action for JCHA, et al.,
against them since each of them has a separate and distinct purpose and each may be affected differently than the
others.

Issues:
1. Whether or not the complaint states a cause of action. - YES
2. Whether the complaint has been properly filed as a class suit. - YES

Held:
1. Section 2, Rule 2 of the Rules of Court defines a cause of action as an act or omission by which a party violates the
right of another. A complaint states a cause of action when it contains three (3) essential elements of a cause of
action, namely:
1. the legal right of the plaintiff,
2. the correlative obligation of the defendant, and
3. the act or omission of the defendant in violation of said legal right.

The question of whether the complaint states a cause of action is determined by its averments regarding the acts
committed by the defendant. Thus, it must contain a concise statement of the ultimate or essential facts constituting
the plaintiff's cause of action. To be taken into account are only the material allegations in the complaint; extraneous
facts and circumstances or other matters aliunde are not considered.
The test of sufficiency of facts alleged in the complaint as constituting a cause of action is whether or not admitting the
facts alleged, the court could render a valid verdict in accordance with the prayer of said complaint. Stated differently,
if the allegations in the complaint furnish sufficient basis by which the complaint can be maintained, the same should
not be dismissed regardless of the defense that may be asserted by the defendant.
In the present case, the Court finds the allegations in the complaint sufficient to establish a cause of action. First,
JCHA, et al.'s averments in the complaint show a demandable right over La Paz Road. These are: (1) their right to use
the road on the basis of their allegation that they had been using the road for more than 10 years; and (2) an
easement of a right of way has been constituted over the said roads. There is no other road as wide as La Paz Road
existing in the vicinity and it is the shortest, convenient and safe route towards SLEX Halang that the commuters and
motorists may use. Second, there is an alleged violation of such right committed by Fil-Estate, et al., when they
excavated the road and prevented the commuters and motorists from using the same. Third, JCHA, et al.,
consequently suffered injury and that a valid judgment could have been rendered in accordance with the relief sought
therein.
2. Section 12, Rule 3 of the Rules of Court defines a class suit, as follows:
Sec. 12. Class suit. — When the subject matter of the controversy is one of common or general
interest to many persons so numerous that it is impracticable to join all as parties, a number of them
which the court finds to be sufficiently numerous and representative as to fully protect the interests of
all concerned may sue or defend for the benefit of all. Any party in interest shall have the right to
intervene to protect his individual interest.
The necessary elements for the maintenance of a class suit are:
1. the subject matter of controversy is one of common or general interest to many persons;
2. the parties affected are so numerous that it is impracticable to bring them all to court; and
3. the parties bringing the class suit are sufficiently numerous or representative of the class and can fully protect
the interests of all concerned.
In this case, the suit is clearly one that benefits all commuters and motorists who use La Paz Road. As succinctly
stated by the CA:
The subject matter of the instant case, i.e., the closure and excavation of the La Paz Road, is initially
shown to be of common or general interest to many persons. The records reveal that numerous
individuals have filed manifestations with the lower court, conveying their intention to join private
respondents in the suit and claiming that they are similarly situated with private respondents for they
were also prejudiced by the acts of petitioners in closing and excavating the La Paz Road. Moreover,
the individuals sought to be represented by private respondents in the suit are so numerous that it is
impracticable to join them all as parties and be named individually as plaintiffs in the complaint. These
individuals claim to be residents of various barangays in Biñan, Laguna and other barangays in San
Pedro, Laguna.

2. Samson vs. Sps. In determining whether an FACTS:


Gabor 182970 23 July initiatory pleading states a cause Respondent Sps. Gabor are the registered owners of a parcel of land in Rizal. In 1985, they executed a Deed of Assignment
2014- Sabio of action, the test is as follows: in transferring an undivided portion of the land in favor of petitioner Samson as attorney’s fees in payment for the services
admitting the truth of the facts rendered by him to the spouses.
alleged, can the court render a
valid judgement in accordance 2 years later, petitioner Samson executed a Deed of Assignment transferring the same undivided portion of land in favor of
with the prayer? Ramos. Upon learning of the sale, respondent Sps. Gabor filed an action for legal redemption with the RTC of Rizal. Petitioner
Samson and Ramos executed an Agreement of Rescission revoking the transfer of the undivided portion. The RTC dismissed
To be taken into account are only the suit for legal redemption, however, on appeal, the CA reversed the decision and upheld the respondent Sps. Gabor’s right
the material allegations in the of legal redemption.
complaint; extraneous facts and
circumstances are not During the pendency of the appealed case of respondent Sps. Gabor with the CA, petitioner Samson filed an action for
considered. Partition of Real Property and Damages against respondent Sps. Gabor with the RTC of Rizal, which dismissed the same on
the ground that the finality of the appealed CA case effectively barred the action for partition. On appeal of petitioner Samson’s
case, the CA upheld the RTC’s decision.
In 2006, petitioner Samson filed a Complaint before the RTC of Pasig for Recovery of Property or its Value against respondent
Sps. Gabor, claiming that he had been paying his ⅓ share of realty taxes covering the subject portion of land. In 2005, he
discovered that his payment was rejected and that respondent Sps. Gabor had mortgaged the entire property in favor of
respondent Tanay Rural Bank.

The RTC of Pasig dismissed petitioner Samson’s complaint on the grounds of improper venue, res judicata, and that the
complaint states no cause of action. With regard to the lack of a cause of action, the RTC held that petitioner Samson’s
complaint states no cause of action against respondent Bank as it does not allege any details as to the liability or any violation
of petitioner Samson’s rights. On appeal, the CA upheld the RTC’s decision and dismissed the same.

ISSUE:
Whether or not petitioner Samson’s complaint states a cause of action.

HELD:
NO. Aside from the fact that respondent Sps. Gabor had mortgaged the property to respondent Bank, there is no other
allegation of an act or omission on the part of respondent Bank in violation of a right of petitioner Samson. Nowhere in the
complaint was it alleged that respondent Bank had knowledge nor could have known with the exercise of due diligence that
respondent Sps. Gabor had acted illegally, in order to commit a wrong against the petitioner.

A cause of action is a formal statement of the operative facts that give rise to a remedial right. The question of whether the
complaint states a cause of action is determined by its averments regarding the acts committed by the respondent. Thus, it
must contain a concise statement of the ultimate or essential facts constituting the plaintiff’s cause of action. Failure to make a
sufficient allegation of a cause of action in the complaint warrants its dismissal.

Under Sec. 2, Rule 2 of the ROC, the essential elements for a cause of action to exist are:
1. A right in favor of the plaintiff by whatever means and under whatever law it arises or is created;
2. An obligation on the part of the named defendant to respect or not violate such right; and
3. Act or omission on the part of such defendant in violation of the right of the plaintiff in constituting a breach of the
obligation of the defendant to the plaintiff for which the latter may maintain an action for recovery of damages.

It is only upon the occurrence of the last element that a cause of action arises, giving the plaintiff the right to maintain an action
in court for recovery of damages or other appropriate relief. In determining whether an initiatory pleading states a cause of
action, the test is as follows: in admitting the truth of the facts alleged, can the court render a valid judgement in accordance
with the prayer? To be taken into account are only the material allegations in the complaint; extraneous facts and
circumstances are not considered.

Petitioner Samson should have specified the details of his cause of action against respondent Bank.

3. ERNESTO C. DEL This rule proscribes a party from FACTS:


ROSARIO and DAVAO dividing a single or indivisible Pet. Davao Timber Corporation (DATICOR) and Res. Private Development Corporation of the Philippines (PDCP) entered
TIMBER cause of action into several parts into a loan agreement under which PDCP extended to DATICOR a foreign currency loan and a peso loan with the total
CORPORATION vs. or claims and instituting two or amount of approximately P4.4 million.
Far East Bank and more actions based on it. The loans were secured by real estate mortgages over 6 parcels of land — 1 situated in Manila which was registered in
Trust Company Because the plaintiff cannot divide the name of Pet. Del Rosario, and 5 in Mati, Davao Oriental — and chattel mortgages over pieces of machinery and
150134 31 October the grounds for recovery, he is equipment.
2007- Someros mandated to set forth in his first Petitioners paid a total of P3 million to PDCP, which the latter applied to interest, service fees and penalty charges. This
action every ground for relief left petitioners, by PDCP's computation, with an outstanding balance on the principal of more than P10 million.
which he claims to exist and upon
(1) Civil Case No. 82-8088 CFI Manila: (complaint dismissed)
which he relies; he cannot be
petitioners had filed a complaint against PDCP before CFI of Manila for violation of the Usury Law, annulment of
permitted to rely upon them by
contract and damages but it was dismissed by the CFI. HAEDCT
piecemeal in successive actions
to recover for the same wrong or On appeal, IAC set aside the CFI's order of dismissal.
injury. G.R. No. 73198: (affirmed the IAC)
PDCP appealed the IAC's decision to this Court.
In the interim, PDCP assigned a portion of its receivables from petitioners to its co-respondent Far East Bank and Trust
Company (FEBTC) under a Deed of Assignment for a consideration of P5,435,000.
FEBTC, as assignee of the receivables, and petitioners later executed a MOA whereby petitioners agreed and pay
FEBTC P6.4 million as full settlement of the receivables.
this Court affirming in toto the decision of the IAC. It determined that after deducting the P3 million earlier paid by
petitioners to PDCP, their remaining balance on the principal loan was only P1.4 million.
(2) Civil Case No. 94-1610 under RTC Makati Br.132: (ruled against PDCP; Dismissal for FEBTC for lack of cause of
action)
Petitioners thus filed on April 25, 1994 a Complaint for sum of money against PDCP and FEBTC before the RTC of
Makati, mainly to recover the excess payment which they computed to be P5.3 million— P4.335 million from PDCP, and
P965,000 from FEBTC.
On May 31, 1995, RTC Makati Br.132 rendered a decision against PDCP to pay petitioners the sum of P4.035 million, to
bear interest at 12% per annum from April 25, 1994 until fully paid; to execute a release or cancellation of the mortgages on
the five parcels of land in Mati, Davao Oriental and on the pieces of machinery and equipment and to return the
corresponding titles to petitioners; and to pay the costs of the suit.
The case against respondent FEBTC, was dismissed for lack of cause of action, ratiocinating that the MOA between
petitioners and FEBTC was not subject to this Court's Decision in G.R. No. 73198, FEBTC not being a party.
CA-G.R. CV No. 50591: (upheld the decision in G.R. No. 73198)
petitioners and respondent PDCP appealed to CA. It holds that petitioners' outstanding obligation, which this Court had
determined in G.R. No. 73198 to be P1.4 million, could not be increased or decreased by any act of the creditor PDCP.
The CA held that when PDCP assigned its receivables, the amount payable to it by DATICOR was the same amount
payable to assignee FEBTC, irrespective of any stipulation that PDCP and FEBTC might have provided in the Deed of
Assignment, DATICOR not having been a party thereto, hence, not bound by its terms.
Noting, however, that DATICOR claimed in its complaint only the amount of P965,000 from FEBTC, the CA held that it
could not grant a relief different from or in excess of that prayed for.
Finally, the CA held that the claim of PDCP against DATICOR for the payment of P1.4 million had no basis, DATICOR's
obligation having already been paid in full, overpaid in fact, when it paid assignee FEBTC the amount of P6.4 million.
Accordingly, the CA upheld the decision of the RTC in Civil Case No. 94-1610. And it ordered FEBTC to pay
petitioners the amount of P965,000 with legal interest from the date of the promulgation of its judgment
FEBTC's motion for reconsideration of the CA Decision was denied, and so was its subsequent appeal to this Court.
(3) Civil Case No. 00-540 RTC Makati Br. 143: (ruled as splitting of cause of action and Res judicata)
petitioners filed a Complaint against FEBTC to recover the balance of the excess payment of P4.335 million.
In its Answer, FEBTC denied responsibility, it submitting that nowhere in the dispositive portion of the CA Decision in CA-
G.R. CV No. 50591 was it held liable to return the whole amount of P5.435 million representing the consideration for the
assignment to it of the receivables, and since petitioners failed to claim the said whole amount in their original complaint in
Civil Case No. 94-1610 as they were merely claiming the amount of P965,000 from it, they were barred from claiming it.
FEBTC later filed a Third Party Complaint against PDCP praying that the latter be made to pay the P965,000 and the
interests adjudged by the CA in favor of petitioners, as well as the P4.335 million and interests that petitioners were claiming
from it. It posited that PDCP should be held liable because it received a consideration of P5.435 million when it assigned the
receivables.
PDCP contended that since petitioners were not seeking the recovery of the amount of P965,000, the same cannot be
recovered via the third party complaint and that the final and executory decision in CA-G.R. CV No. 50591 had held that
DATICOR has no cause of action against it for the refund of any part of the excess payment, FEBTC can no longer re-litigate
the same issue.
Moreover, PDCP contended that it was not privy to the MOA which explicitly excluded the receivables from the effect of
the SC decision, and that the amount of P6.4 million paid by petitioners to FEBTC was clearly intended as consideration for
the release and cancellation of the lien on the Otis property.
In its reply, FEBTC pointed out that PDCP cannot deny that it benefited from the assignment of its rights over the
receivables from petitioners. It added that the third party claim being founded on a valid and justified cause, PDCP's
counterclaims lacked factual and legal basis.
Petitioners thereafter filed a Motion for Summary Judgment to which FEBTC filed its opposition. The motion was denied.
On July 10, 2001, the trial court issued the assailed Decision dismissing petitioners' complaint on the ground of res
judicata and splitting of cause of action. It recalled that petitioners had filed Civil Case No. 94-1610 to recover the alleged
overpayment both from PDCP and FEBTC and to secure the cancellation and release of their mortgages on real properties,
machinery and equipment; that when said case was appealed, the CA, in its Decision, ordered PDCP to release and cancel
the mortgages and FEBTC to pay P965,000 with interest, which Decision became final and executory; and that a Notice of
Satisfaction of Judgment between petitioners and FEBTC was in fact submitted hence, the issue between them was finally
settled under the doctrine of res judicata.
Respecting the third party claim of FEBTC, the trial court held that FEBTC's payment of the amount of P1,224,906.67
(P965,000 plus interest) to petitioners was in compliance with the final judgment of the CA, hence, it could not entertain such
claim because the Complaint filed by petitioners merely sought to recover from FEBTC the alleged overpayment of P4.335
million and attorney's fees of P200,000.
Petitioners' motion for reconsideration of the July 10, 2001 decision of the trial court was denied.
Hence, the present petition.
ISSUE: whether their complaint is dismissible for splitting of cause of action
HELD: Yes.
This Court finds well-taken then the pronouncement of the court a quo that to allow the re-litigation of an issue that was
finally settled as between petitioners and FEBTC in the prior case is to allow the splitting of a cause of action, a ground for
dismissal under Section 4 of Rule 2 of the Rules of Court reading:
SEC. 4. Splitting of a single cause of action; effect of. — If two or more suits are instituted on the basis of the
same cause of action, the filing of one or a judgment upon the merits in any one is available as a
ground for the dismissal of the others.(Emphasis and underscoring supplied)
This rule proscribes a party from dividing a single or indivisible cause of action into several parts or claims and instituting
two or more actions based on it. Because the plaintiff cannot divide the grounds for recovery, he is mandated to set forth in
his first action every ground for relief which he claims to exist and upon which he relies; he cannot be permitted to rely upon
them by piecemeal in successive actions to recover for the same wrong or injury.
Clearly then, the judgment in Civil Case No. 94-1610 operated as a bar to Civil Case No. 00-540, following the above-
quoted Section 4, Rule 2 of the Rules of Court.
A final word. Petitioners are sternly reminded that both the rules on res judicata and splitting of causes of action are
based on the salutary public policy against unnecessary multiplicity of suits — interest reipublicae ut sit finis litium. Re-
litigation of matters already settled by a court's final judgment merely burdens the courts and the taxpayers, creates
uneasiness and confusion, and wastes valuable time and energy that could be devoted to worthier cases.

4. Progressive Dev’t Under Sec. 4 of the same Rule, if Facts: On 27 May 1991 petitioner leased to private respondent Westin Seafood Market, Inc., a parcel of land with a
Corp. vs. Court of two or more suits are instituted on commercial building thereon located at Araneta Center, Cubao, Quezon City, for a period of nine (9) years and three (3)
Appeals 123555 22 the basis of the same cause of months, i.e., from 2 January 1989 to 30 April 1998, with a monthly rental of approximately P600,000.00.
January 1999- Tan action, the �ling of one or a
judgment upon the merits in any Private respondent, Westin Seafood Market, Inc., failed to pay its rentals amounting to P8,608,284.66. Admittedly, non-
one is available as a ground for payment of rentals constituted breach of their contract; thus, pursuant to the express authority granted petitioner under the
the dismissal of the other or lease agreement, petitioner repossessed the leased premises. This prompted private respondent to file a complaint against
others. "Cause of action" is petitioner for forcible entry with damages before the MTC of Quezon City. This case was still pending before the MTC when
defined by Sec. 2 of Rule 2 as the private respondent instituted before the RTC of Quezon City another action for damages against petitioner, which the latter
act of omission by which a party moved to dismiss on the ground of litis pendencia and forum shopping to no avail. The same fate waited petitioner before the
violates a right of another. Court of Appeals which dismissed his special civil action for certiorari and prohibition due to the failure of petitioner to file a
motion for reconsideration of the RTC order.
Issue: W/N the lessee which instituted before the Metropolitan Trial Court an action for forcible entry with damages against its
lessor file a separate suit with the Regional Trial Court against the same lessor for moral and exemplary damages plus actual
and compensatory damages based on the same forcible entry?

Held:
No, such lessee may not file for a separate suit.

Section 4. Splitting a single cause of action; effect of. — If two or more suits are instituted on the basis of the same cause of
action, the filing of one or a judgment upon the merits in any one is available as a ground for the dismissal of the others.

Since actual and compensatory damages were already prayed for in the forcible entry case before the MeTC, it is obvious that
this cannot be relitigated in the damage suit before the RTC by reason of res adjudicata.

The claims for damages are all based on the alleged forcible takeover of the leased premises by the petitioner; thus, since
actual and compensatory damages had already been prayed for in the forcible entry case before the MeTC, it may no longer
be relitigated in the damage suit before the RTC by reason of res judicata.

The question of damages is merely secondary or incidental, so much so that the amount thereof does not affect the jurisdiction
of the court. In other words, the unlawful act of a deforciant (one who wrongfully withholds land or property) in taking
possession of a piece of land by means of force and intimidation against the rights of the party actually in possession thereof is
a delict or wrong, or a cause of action that gives rise to two (2) remedies, namely, the recovery of possession and recovery of
damages arising from the loss of possession, but only to one action. For obvious reasons, both remedies cannot be the subject
of two (2) separate and independent actions, one for recovery of possession only, and the other, for the recovery of damages.
That would inevitably lead to what is termed in law as splitting up a cause of action.

These premises obtaining, there is no question at all that private respondent's cause of action in the forcible entry case and in
the suit for damages is the alleged illegal retaking of possession of the leased premises by the lessor, petitioner herein, from
which all legal reliefs arise. Simply stated, the restoration of possession and demand for actual damages in the case before the
MeTC and the demand for damages with the RTC both arise from the same cause of action, i.e., the forcible entry by petitioner
into the leased premises.

5. Joseph vs. Bautista A cause of action is understood to Facts: Petitioner Joseph is the plaintiff in a complaint for damages presided by respondent Judge Bautista. Private
L-41423 23 February be the delict or wrongful act or respondents Patrocinio Perez (Perez), Antonio Sioson (Sioson), Jaconto Pagarigan (Pagarigan), and Lazaro Villanueva
1989 - Tañada omission committed by the (Villanueva) were the defendants in the said case. The case arose from a vehicular accident between a truck carrying cargo
defendant in violation of the and passengers owned by Patrocinio Perez, driven by Domingo Villa, and a pickup truck supposedly owned by Antonio Sioson
primary rights of the plaintiff. It is and Jacinto Pagarigan, then driven by Lazaro Villanueva. Petitioner was a passenger in the cargo truck. As a result of the
true that a single act or omission vehicular accident, petitioner sustained a bone fracture in one of his legs. Hence, he filed the complaint for damages against
can be violative of various rights Perez, as owner of the cargo truck, based on breach of contract of carriage, and against Sioson and Villanueva, as owner and
at the same time, as when the act driver, respectively, of the pickup truck, based on quasi-delict. Petitioner filed his amended complaint to implead Pagarigan
constitutes juridically a violation of and a certain Rosario Vargas as additional defendants, since he could not ascertain who the real owner of the cargo truck was
several separate and distinct legal (whether Perez or Vargas), as well as the pickup truck (whether Sioson or Jacinto). Perez thereafter impleaded in her cross-
obligations. However, where there claim Alberto Cardeno (Cardeno) as an additional alternative defendant. Villanueva, Cardeno, Sioson, and Pagarigan, through
is only one delict or wrong, there their insurer, paid petitioner’s claim for damages. As a result, petitioner released them from their liability. Afterwards,
is but a single cause of action Villanueva, Cardeno, and their insurer paid Perez’ claim for damages to her cargo truck. Consequently, Sioson, Pagarigan,
regardless of the number of rights Villanueva, and Cardeno filed a motion to be excluded as defendants because of the aforementioned payments, claiming
that may have been violated furthermore that payment to petitioner was by way of an amicable settlement. Perez then filed a counter motion to dismiss,
belonging to one person. The averring that the release of claim executed by petitioner in favor of the other respondents/defendants inured to her benefit
singleness of a cause of action since their liability to petitioner was solidary. Respondent judge then ordered the dismissal of the case.
lies in the singleness of the delict Issue:
or wrong violating the rights of Whether or not the dismissal of the case was proper.
one person. Nevertheless, if only Held: Yes.
one injury resulted from several Petitioner argued that the case was based on two causes of action: quasi-delict and breach of contract; hence, the
wrongful acts only one cause of compromise agreement on the quasi-delict is not a bar to the cause of action for breach of contract. The SC ruled that this
action arises. argument is untenable. A cause of action is understood to be the delict or wrongful act or omission committed by the defendant
in violation of the primary rights of the plaintiff. It is true that a single act or omission can be violative of various rights at the
same time, as when the act constitutes juridically a violation of several separate and distinct legal obligations. However, where
there is only one delict or wrong, there is but a single cause of action regardless of the number of rights that may have been
violated belonging to one person. The singleness of a cause of action lies in the singleness of the delict or wrong violating the
rights of one person. Nevertheless, if only one injury resulted from several wrongful acts only one cause of action arises. In this
case, petitioner sustained a single injury, which vested in him a single cause of action, although with the correlative rights of
actions against the different respondents through appropriate remedied sanctioned by law. There is no question that
respondents herein are solidarily liable to petitioner; hence, the full payment made by some of the solidary debtors and their
subsequent release from any liability to petitioner resulted in the extinguishment and release from liability of the other solidary
debtors, including respondent Perez.

6. Flores vs. Mallare- Under the present law, the totality Facts: Petitioner has appealed by certiorari from the order of Judge Mallare-Phillips of the RTC of Baguio City and Benguet
Philipps L-66620 24 rule is applied also to cases province which dismissed his complaint for lack of jurisdiction. The order appealed from, states that the first cause of action
September 1986- Ubay where two or more plaintiffs alleged in the complaint was against respondent Ignacion Binongcal for refusing to pay the amount of P11,643.00 representing
having separate causes of action the cost of truck tires which he purchased on credit from petition on various occasions; and the second cause of action was
against a defendant join in a against respondent Fernando Calion for allegedly refusing to pay the amount of P10,212.00 representing the cost of truck tires
single complaint, as well as to which he purchased on credit from petitioner on various occasions. The counsel for respondent Binongcal filed a Motion to
cases where a plaintiff has Dismiss on the ground of lack of jurisdiction since the amount of the demand against said respondent is less than P20,000.00
separate causes of action against which is the jurisdictional amount in order for RTC to exercise original jurisdiction of the case under Section 19(8) of BP 129. It
two or more defendants joined in was further averred in said motion that although another person, Calion, was allegedly indebted to petitioner in the amount of
a single complaint. However, the P10,212.00, his obligation was separate and distinct from that of the other respondent. At the hearing, counsel for respondent
causes of action in favor of the Calion joined in moving for the dismissal of the complaint on the ground of lack of jurisdiction. The trial court dismissed the
two or more plaintiffs or against complaint for lack of jurisdiction.
the two or more defendants
should arise out of the same Issue: Whether or not the totality rule will apply in this case and cause the two collection complaint to be joined
transaction or series of
transactions and there should be Held:
a common question of law or fact, The Supreme Court held in this case that under the present law, the totality rule is applied also to cases where two or more
as provided in Section 6 of Rule plaintiffs having separate causes of action against a defendant join in a single complaint, as well as to cases where a plaintiff
3. has separate causes of action against two or more defendants joined in a single complaint. However, the causes of action in
favor of the two or more plaintiffs or against the two or more defendants should arise out of the same transaction or series of
transactions and there should be a common question of law or fact, as provided in Section 6 of Rule 3.

Under the present law, the two cases above cited (assuming they do not fall under the Labor Code) would be under the
jurisdiction of the regional trial court. Similarly, in the above cited cases of Brillo vs. Buklatan and Gacula vs. Martinez (supra),
if the separate claims against the several defendants arose out of the same transaction or series of transactions and there is a
common question of law or fact, they would now be under the jurisdiction of the regional trial court.

In cases of permissive joinder of parties, whether as plaintiffs or as defendants, under Section 6 of Rule 3, the total of all the
claims shall now furnish the jurisdictional test. Needless to state also, if instead of joining or being joined in one complaint
separate actions are filed by or against the parties, the amount demanded in each complaint shall furnish the jurisdictional
test.

In the case at bar, the lower court correctly held that the jurisdictional test is subject to the rules on joinder of parties pursuant
to Section 5 of Rule 2 and Section 6 of Rule 3 of the Rules of Court and that, after a careful scrutiny of the complaint, it
appears that there is a misjoinder of parties for the reason that the claims against respondents Binongcal and Calion are
separate and distinct and neither of which falls within its jurisdiction.

7. Lourdes Suites vs. In a case with an action for small Petitioner, Lourdes Suites, is the owner of a hotel in Makati City. It executed two (2) contracts with respondent, Noemi Binarao,
Binaro 204729 06 claims is where decisions for room accommodations for two (2) groups of students in the amount of PHP4,262,010.00. According to the records,
August 2014- Yumul rendered become final and respondent was able to pay the total amount of the contract price. However, petitioner alleges that there was an unpaid
unappealable, hence, the decision balance of Php47,810.00 resulting from damages to the furniture, lost key, and excess guests. Thus, petitioner sent a demand
dismissing the same is letter to respondent for the unsettled amount. Respondent failed to pay prompting the petitioner to file a statement of claim for
necessarily with prejudice. collection of sum of money plus damages.
Respondent alleged the petitioner billed the charges twice and hence, was not obliged to pay. Petitioner impugned the validity
of the Response, stating “it was not made in the form of an Answer as required by law(Sec 1. Rule 11 of the Revised Rules of
Court).

MeTC ruled that petitioner failed to prove the existence of the unpaid balance, hence, dismissed the complaint with prejudice
for lack of cause of action. Petitioner then filed a petition for certiorari. Arguing that a dismissal based on the ground that the
complaint states no cause of action is not a valid ground for dismissal, moreover a dismissal with prejudice.

Issue/s:
WON the dismissal of the ground of lack of cause of action is proper.

Held:
YES.Failure to state a cause of action is different from lack of cause of action. The former is the insufficiency of the pleading
and is a ground for dismissal. While the latter, is a situation where the evidence does not prove the cause of action alleged in
the pleading.

Wherefore, the petition was denied.

Parties to Civil Action

8. Pacana-Contreras Pursuant to Section 9, Rule 3 of Facts:


vs. Rovila Water the Rules of Court, parties may be Petitioners Rebecca Pacaña-Contreras and Rosalie Pacaña, children of Lourdes Teves Pacaña and Luciano Pacaña, filed
Supply 168979 2 added by order of the court on against Rovila Inc., Earl, Lilia, Dalla and Marisa for accounting and damages. The petitioners claimed that their family has long
December 2013- motion of the party or on its own been known in the community to be engaged in the water supply business; they operated the "Rovila Water Supply" from their
initiative at any stage of the family residence and were engaged in the distribution of water to customers in Cebu City.
Zapanta
action. If the plaintiff refuses to The petitioners alleged that Lilia was a former trusted employee in the family business who hid business records and burned
implead an indispensable party and ransacked the family files. Lilia also allegedly posted security guards and barred the members of the Pacaña family from
despite the order of the court, operating their business. She then claimed ownership over the family business through a corporation named "Rovila Water
then the court may dismiss the Supply, Inc." (Rovila Inc.) Upon inquiry with the Securities and Exchange Commission(SEC), the petitioners claimed that
complaint for the plaintiff's failure Rovila Inc. was surreptitiously formed with the respondents as the majority stockholders. The respondents did so by conspiring
to comply with a lawful court with one another and forming the respondent corporation to takeover and illegally usurp the family business' registered name.
order. The operative act that Respondents allegedly used the name of Lourdes as one of the incorporators and made it appear in the SEC documents that
would lead to the dismissal of the the family business was operated in a place other than the Pacaña residence. Thereafter, the respondents used the Pacaña
case would be the refusal to family's receipts and the deliveries and sales were made to appear as those of the respondent Rovila Inc. Using this scheme,
comply with the directive of the the respondents fraudulently appropriated the collections and payments.
court for the joinder of an The petitioners fled the complaint in their own names although Rosalie was authorized by Lourdes through a sworn declaration
indispensable party to the case. and special power of attorney (SPA). The respondents filed motion to dismiss on the ground that the RTC had no jurisdiction
over an intra-corporate controversy. The RTC denied the motion.
Lourdes died and the petitioners amended their complaint, with leave of court, to reflect this development. They still attached to
their amended complaint the sworn declaration with SPA, but the caption of the amended complaint remained the same.
Luciano also died.
The respondents filed their Answer. The petitioners' sister, Lagrimas Pacaña-Gonzales, filed a motion for leave to intervene
and her answer-in- intervention was granted by the trial court. At the pre-trial, the respondents manifested to the RTC that a
substitution of the parties was necessary in light of the deaths of Lourdes and Luciano. They further stated that they would
seek the dismissal of the complaint because the petitioners are not the real parties in interest to prosecute the case. The pre-
trial pushed through and the RTC directed the respondents to put their manifestation in writing. The RTC issued a pre-trial
order where one of the issues submitted was whether the complaint should be dismissed for failure to comply with Section 2,
Rule 3 of the Rules of Court which requires that every action must be prosecuted in the name of the real party in interest.
Respondents again filed a motion to dismiss, that the petitioners are not the real parties in interest to institute and prosecute
the case and that they have no valid cause of action against the respondents.
RTC: denied the respondent’s motion to dismiss. It ruled that, a motion to dismiss based on the grounds invoked by the
respondents may only be filed within the time for, but before, the filing of their answer to the amended complaint. the rule on
substitution of parties only applies when the parties to the case die, which is not what happened in the present case.
Respondents filed a motion for certiorari.
CA: RTC committed grave abuse of discretion as the petitioners filed the complaint and the amended complaint as attorneys-
in-fact of their parents. They are not the real parties in interest and cannot bring an action in their own names; thus, the
complaint should be dismissed. Neither are the petitioners suing as heirs of their deceased parents. Pursuant to jurisprudence,
the petitioners should first be declared as heirs before they can be considered as the real parties in interest.
Issue: W/N the other heirs of the spouses Pacaña should be impleaded by the court.

Ruling:
Yes. In Carandang v. Heirs of de Guzman, et al., the Court clarified that "[a] real party in interest is the party who stands to be
benefited or injured by the judgment of the suit, or the party entitled to the avails of the suit. On the other hand, an
indispensable party is a party in interest without whom no final determination can be had of an action, in contrast to a
necessary party, which is one who is not indispensable but who ought to be joined as a party if complete relief is to be
accorded as to those already parties, or for a complete determination or settlement of the claim subject of the action. . . . If a
suit is not brought in the name of or against the real party in interest, a motion to dismiss may be filed on the ground that the
complaint states no cause of action. However, the dismissal on this ground entails an examination of whether the parties
presently pleaded are interested in the outcome of the litigation, and not whether all persons interested in such outcome are
actually pleaded. The latter query is relevant in discussions concerning indispensable and necessary parties, but not in
discussions concerning real parties in interest. Both indispensable and necessary parties are considered as real parties in
interest, since both classes of parties stand to be benefited or injured by the judgment of the suit."
At the inception of the case, both the spouses Pacaña were not impleaded as parties-plaintiffs. The Court notes that they are
indispensable parties to the case as the alleged owners of Rovila Water Supply. Without their inclusion as parties, there can be
no final determination of the present case. They possess such an interest in the controversy that a final decree would
necessarily affect their rights, so that the courts cannot proceed without their presence. Their interest in the subject matter of
the suit and in the relief sought is inextricably intertwined with that of the other parties.

The Court is of the view that the proper remedy in the present case is to implead the indispensable parties especially when
their non- inclusion is merely a technical defect. To do so would serve proper administration of justice and prevent further delay
and multiplicity of suits. Pursuant to Section 9, Rule 3 of the Rules of Court, parties may be added by order of the court on
motion of the party or on its own initiative at any stage of the action. If the plaintiff refuses to implead an indispensable party
despite the order of the court, then the court may dismiss the complaint for the plaintiff's failure to comply with a lawful court
order. The operative act that would lead to the dismissal of the case would be the refusal to comply with the directive of the
court for the joinder of an indispensable party to the case.

The deceased Pacañas can no longer be included in the complaint as indispensable parties because of their death during the
pendency of the case. Upon their death, however, their ownership and rights over their properties were transmitted to their
heirs. Therefore, to obviate further delay in the proceedings of the present case and given the Court's authority to order the
inclusion of an indispensable party at any stage of the proceedings, the heirs of the spouses Pacaña, except the petitioners
who are already parties to the case and Lagrimas Pacaña-Gonzalez who intervened in the case, are hereby ordered
impleaded as parties-plaintiffs.

9. Golangco vs. Section 35 (l), Chapter 12, Title III Facts:


Fung 157952 8 of Book IV of Executive Order No.
September 2009- Abad 292 (The Administrative Code of C.A.-G.R. SP No. 66616 was a special civil action for certiorari commenced by the petitioner to assail the order issued by the
1987), which mandates the OSG RTC, Branch 53, in Manila in Criminal Case No. 95-145703 entitled People v. Jone B. Fung, whereby the RTC declared the
to represent "the Government in Prosecution to have terminated the presentation of further evidence and required the Prosecution to file a written offer of
the Supreme Court and the Court evidence within 20 days, furnishing a copy of the offer to the accused who in turn had to comment on the offer within 15 days
of Appeals in all criminal from receipt.
proceedings; represent the
Government and its officers in the Criminal Case No. 95-145703, a prosecution for libel initiated by the petitioner as the complainant against the respondent, was
Supreme Court, the Court of commenced in 1995. Allegedly, the respondent had issued an office memorandum dated maliciously imputing against the
Appeals, and all other courts or petitioner the commission of bribery and had sent copies of the memorandum to the petitioner's superiors in the POEA and to
tribunals in all civil actions and other public officers and personalities not connected with the POEA, causing damage and prejudice to the petitioner.
special proceedings in which the
Government or any officer thereof
in his official capacity is a party". After almost 6 years, the Prosecution had presented only two witnesses in Criminal Case No. 95-145703. On February 16,
2001, the Prosecution requested that a subpoena ad testificandum be issued to and served on Atty. Oscar Ramos, Resident
Ombudsman of the POEA, to compel him to testify in the criminal case on February 20, 2001. The hearing of February 20,
Although the petition for certiorari 2001 was, however, reset to May 23, 2001 due to the unavailability of Atty. Ramos.
bore the conformity of the public
prosecutor, that conformity alone
did not suffice. The authority of On May 23, 2001, the Prosecution still failed to present Atty. Ramos as its witness because no subpoena had been issued to
the City Prosecutor or his and served on him for the purpose. Consequently, the RTC judge issued an order terminating the Prosecution's presentation
assistant to appear for and of evidence.
represent the People of the
Philippines was confined only to The petitioner assailed on certiorari in the CA the order, claiming that the RTC judge thereby committed grave abuse of
the proceedings in the trial court. discretion for not issuing the subpoena to require Atty. Ramos to appear and testify in the hearing. He contended that his prior
request for the subpoena for the hearing should have been treated as a continuing request for the subpoena considering that
the Rules of Court did not require a party to apply for a subpoena again should it not be served for the first time.

CA - any request for a subpoena to a witness must indicate the date and time when the witness must appear in court to give
his or her testimony. The court cannot be tasked to guess whether or not petitioner still intends to present the witness at the
next hearing. An intention to still present the witness necessarily requires another request for a subpoena.
Hence, this petition.

Issue/s:
Whether the Court of Appeals correctly ruled on the petition for certiorari of the petitioner.

Held:
Yes. We find no reversible error on the part of the Court of Appeals.

The petitioner did not join the People of the Philippines as a party in his action for certiorari in the Court of Appeals. He thereby
ignored that the People of the Philippines were indispensable parties due to his objective being to set aside the trial court's
order dated May 23, 2001 that concerned the public aspect of Criminal Case No. 95-145703. The omission was fatal and
already enough cause for the summary rejection of his petition for certiorari.

The petitioner did not also obtain the consent of the Office of the Solicitor General (OSG) to his petition for certiorari. At the
very least, he should have furnished a copy of the petition for certiorari to the OSG prior to the filing thereof, but even that he
did not do. Thereby, he violated Section 35 (l), Chapter 12, Title III of Book IV of Executive Order No. 292 (The Administrative
Code of 1987), which mandates the OSG to represent "the Government in the Supreme Court and the Court of Appeals in all
criminal proceedings; represent the Government and its officers in the Supreme Court, the Court of Appeals, and all other
courts or tribunals in all civil actions and special proceedings in which the Government or any officer thereof in his official
capacity is a party".

Although the petition for certiorari bore the conformity of the public prosecutor, that conformity alone did not suffice. The
authority of the City Prosecutor or his assistant to appear for and represent the People of the Philippines was confined only to
the proceedings in the trial court.

Even on the merits, the petition for review fails.

Yet, the warnings of the trial court had gone unheeded. Instead, the Prosecution would deflect the responsibility for the delays
to the failure of the trial court to issue the subpoena to its proposed witness and to cause the subpoena to be served.

We find that the trial judge did not act capriciously, arbitrarily or whimsically in issuing the assailed order. Thus, the CA
properly dismissed the petition for certiorari. The petitioner now needs to be reminded that certiorari is an extraordinary remedy
to correct a grave abuse of discretion amounting to lack or excess of jurisdiction when an appeal, or any plain, speedy and
adequate remedy in the ordinary course of law is not available. In this regard, grave abuse of discretion implies a capricious
and whimsical exercise of judgment that is equivalent to lack of jurisdiction whenever the power is exercised in an arbitrary or
despotic manner by reason of passion, prejudice or personal aversion amounting to an evasion of a positive duty or to a virtual
refusal to perform the duty enjoined, or to act at all in contemplation of law

10. Equitable PCI Bank


Inc. vs. Heirs of Tiu, The AREM was executed by Facts:
GR No. 178529, Antonio, with the marital consent To secure loans in the aggregate amount of ₱7 Million obtained by one Gabriel Ching from herein petitioner Equitable PCI
September 4, 2009 - De of Matilde. Since the mortgaged Bank, Inc. (now known as Banco de Oro-EPCI, Inc.),Antonio C. Tiu (Antonio), of which herein respondents allege to be heirs,
property is presumed conjugal, executed on July 6, 1994 a Real Estate Mortgage (REM) in favor of petitioner covering a lot located in Tacloban City. Before
Gala
she is obliged principally under the words "With my Marital Consent" appearing in the REM is a signature attributed to Antonio’s wife Matilde.
the AREM. It is thus she, following On October 5, 1998, Antonio executed an Amendment to the Real Esate Mortgage(AREM) increasing the amount secured by
Art. 1397 of the Civil Code vis a the mortgage to ₱26 Million, also bearing a signature attributed to his wife Matilde above the words "With my Marital Consent."
vis Sec. 2 of Rule 3 of the Rules The property mortgaged was covered by TCT No. T-1381 of the Tacloban Register of Deeds which, the AREM states, was
of Court, who is the real party in "registered in the name of the Mortgagor."
interest, hence, the action must Antonio died on December 26, 1999.
be prosecuted in her name as she The loan obligation having remained unsettled, petitioner filed in November 2003 before the Regional Trial Court (RTC) of
stands to be benefited or injured Tacloban City a "Petition for Sale" dated November 4, 2003, for the extrajudicial foreclosure of the AREM and the sale at public
in the action. auction of the lot covered thereby. Acting on the petition, the RTC Clerk of Court and Ex-Oficio Sheriff scheduled the public
auction on December 17, 2003.
A day before the scheduled auction sale or on December 16, 2003, the herein respondents, Heirs of Antonio C. Tiu, namely
Arlene T. Fu, Michael U. Tiu, Andrew U. Tiu, Edgar U. Tiu, and Erwin U. Tiu, filed a Complaint/Petition before the RTC of
Tacloban against petitioner and the Clerk of Court-Ex Oficio Sheriff, docketed as Civil Case No. 2003-12-205 for annulment of
the AREM, injunction with prayer for issuance of writ of preliminary injunction and/or temporary restraining order and damages,
alleging, among other things, that
x x x the said AREM is without force and effect, the same having been executed without the valid consent of the wife of
mortgagor Antonio C. Tiu who at the time of the execution of the said instrument was already suffering from advance[d]
Alzheimer’s Disease and, henceforth, incapable of giving consent, more so writing and signing her name[.]
The RTC issued a temporary restraining order, and subsequently, a writ of preliminary injunction.
Petitioner filed a Motion to Dismiss,raising the following grounds:

1. THE PLAINTIFFS/PETITIONERS NOT BEING THE REAL PARTIES-IN-INTEREST, THEIR COMPLAINT STATES
NO CAUSE OF ACTION;

Tacloban RTC: Denied the Motion to Dismiss. From the facts of the case, herein plaintiffs/petitioners are so situated that they
will either be benefited or injured in subject action. They are therefore real parties in interest, as they will be damnified and
injured or their inheritance rights and interest on the subject property protected and preserved in this action. As they are real
parties in interest, they therefore have a cause of action against herein defendant.
CA - affirmed RTC’s decision.
ISSUE:
Whether the complaint filed by respondents-children of Antonio, without impleading Matilde who must also be Antonio’s heir
and who, along with Antonio, was principally obliged under the AREM sought to be annulled, is dismissible for lack of cause of
action
HELD:
The pertinent provision of the Civil Code on annulment of contracts reads:
Art. 1397. The action for the annulment of contracts may be instituted by all who are thereby obliged principally or subsidiarily.
However, persons who are capable cannot allege the incapacity of those with whom they contracted; nor can those who
exerted intimidation, violence, or undue influence, or employed fraud, or caused mistake base their action upon these flaws of
the contract.
Upon the other hand, the pertinent provisions of Rule 3 of the Rules of Court (Parties to Civil Actions) read:
SEC. 2 Parties in interest. ─ A real party in interest is the party who stands to be benefited or injured by the judgment in the
suit, or the party entitled to the avails of the suit. Unless otherwise authorized by law or these Rules, every action must be
prosecuted or defended in the name of the real party in interest. (Emphasis and underscoring supplied)
SEC. 3. Representatives as parties. ─ Where the action is allowed to be prosecuted or defended by a representative or
someone acting in a fiduciary capacity, the beneficiary shall be included in the title of the case and shall be deemed to be the
real party in interest. A representative may be a trustee of an express trust, a guardian, an executor or administrator, or a party
authorized by law or these Rules. An agent acting in his own name and for the benefit of an undisclosed principal may sue or
be sued without joining the principal except when the contract involves things belonging to the principal. (Emphasis and
underscoring supplied)
The AREM was executed by Antonio, with the marital consent of Matilde. Since the mortgaged property is presumed conjugal,
she is obliged principally under the AREM. It is thus she, following Art. 1397 of the Civil Code vis a vis Sec. 2 of Rule 3 of the
Rules of Court, who is the real party in interest, hence, the action must be prosecuted in her name as she stands to be
benefited or injured in the action.
Assuming that Matilde is indeed incapacitated, it is her legal guardian who should file the action on her behalf. Not only is there
no allegation in the complaint, however, that respondents have been legally designated as guardians to file the action on her
behalf. The name of Matilde, who is deemed the real party in interest, has not been included in the title of the case, in violation
of Sec. 3 of Rule 3 of the Rules of Court.
WHEREFORE, the petition is GRANTED.

11. White vs. City of Standing or locus standi is the


Manila 122846 20 ability of a party to demonstrate to Facts: In 1992, City Mayor Alfredo Lim (Mayor Lim) signed into law the Ordinance entitled, "An Ordinance Prohibiting Short-
January 2009- Espiritu the court sufficient connection to Time Admission, Short-Time Admission Rates, and Wash-Up Rate Schemes in Hotels, Motels, Inns, Lodging Houses, Pension
and harm from the law or action Houses, and Similar Establishments in the City of Manila". Thereafter, the Malate Tourist and Development Corporation
challenged to support that party's (MTDC) filed a complaint for declaratory relief with prayer for a writ of preliminary injunction and/or temporary restraining order
participation in the case. (TRO) with the RTC of Manila impleading as defendant, herein respondent City of Manila (the City) represented by Mayor Lim.
MTDC prayed that the Ordinance, insofar as it includes motels and inns as among its prohibited establishments, be declared
The general rules on standing invalid and unconstitutional. MTDC claimed that as owner and operator of the Victoria Court in Malate, Manila it was
admit of several exceptions such authorized by P.D. No. 259 to admit customers on a short time basis as well as to charge customers wash up rates for stays of
as the overbreadth doctrine, only three hours. In 1992, petitioners White Light Corporation (WLC), Titanium Corporation (TC) and Sta. Mesa Tourist and
taxpayer suits, third party Development Corporation (STDC) filed a motion to intervene and to admit attached complaint-in-intervention on the ground
standing and, especially in the that the Ordinance directly affects their business interests as operators of drive-in-hotels and motels in Manila. During the pre-
Philippines, the doctrine of trial conference, the WLC, TC and STDC agreed to submit the case for decision without trial as the case involved a purely
transcendental importance. legal question. In 1993, the RTC rendered a decision declaring the Ordinance null and void.

Issue: Whether or not petitioners have legal standing

Ruling: YES. Petitioners allege that as owners of establishments offering "wash-up" rates, their business is being unlawfully
interfered with by the Ordinance. However, petitioners also allege that the equal protection rights of their clients are also being
interfered with. Thus, the crux of the matter is whether or not these establishments have the requisite standing to plead for
protection of their patrons' equal protection rights. For this particular set of facts, the concept of third party standing as an
exception and the overbreadth doctrine are appropriate. Herein, it is clear that the business interests of the petitioners are
likewise injured by the Ordinance. They rely on the patronage of their customers for their continued viability which appears to
be threatened by the enforcement of the Ordinance. In this case, the petitioners claim that the Ordinance makes a sweeping
intrusion into the right to liberty of their clients. We can see that based on the allegations in the petition, the Ordinance suffers
from overbreadth. We thus recognize that the petitioners have a right to assert the constitutional rights of their clients to
patronize their establishments for a "wash-rate" time frame.

12. Rayo vs. A real party-in-interest is one with FACTS: Midas Diversified Export Corp., thru its president, Mr. Samuel Lee, obtained six (6) loans from private respondent
Metrobank 165142 10 "a present substantial interest" Metropolitan Bank and Trust Company amounting to ₱588,870,000 as evidenced by promissory notes. To secure the payment
December 2007- which means such interest of a of an ₱8,000,000 loan, Louisville Realty & Development Corporation, thru its president, Mr. Lee, executed in favor of
Nakagawa party in the subject matter of the Metrobank, a real estate mortgage over three parcels of land situated at No. 40 Timog Ave., Brgy. Laging Handa, Quezon City,
action as will entitle him, under with all the buildings and improvements thereon. The properties are covered by Transfer Certificates of Title issued by the
the substantive law, to recover if Registry of Deeds of Quezon City.
the evidence is sufficient, or that
he has the legal title to demand. When the debtor-mortgagor failed to pay, Metrobank extra-judicially foreclosed the real estate mortgage in accordance with
Act No. 3135, as amended. Thereafter, in a public auction, Metrobank was the highest bidder. A Certificate of Sale dated 4
December 11, 2000 was duly registered with the Registry of Deeds of Quezon City on December 13, 2000. When Louisville
refused to turn over the real properties, on March 17, 2001, Metrobank filed before the Regional Trial Court (RTC), Branch
223, Quezon City, an ex parte petition for the issuance of a writ of possession.

ISSUE: WHETHER OR NOT THE PETITIONER HAS THE LEGAL PERSONALITY TO SEEK THE ANNULMENT OF
JUDGMENT IN [THE] SUBJECT LRC CASE

HELD: NO. Petitioner insists that contrary to the ruling of the Court of Appeals, he has the legal personality to institute the
annulment of judgment case against Metrobank, considering that the March 25, 2002 deed of assignment he entered into with
Louisville and Winston Linwy L. Chua makes him a co-assignee over the subject real properties.

For its part, Metrobank claims that it was not a party to the deed of assignment among Louisville, Chua and petitioner, hence, it
has no privity of contract with petitioner Rayo. Moreover, Metrobank points out that the real properties had already been
extrajudicially foreclosed when petitioner and his assignors executed the deed of assignment.

Under Section 2, Rule 3 of the Rules of Court, every action must be prosecuted or defended in the name of the real party-in-
interest, or one "who stands to be benefited or injured by the judgment in the suit." A real party-in-interest is one with "a
16

present substantial interest" which means such interest of a party in the subject matter of the action as will entitle him, under
the substantive law, to recover if the evidence is sufficient, or that he has the legal title to demand.
17

Now, is petitioner Rayo a real party-in-interest? Initially, we recognized herein petitioner as the co-assignee of the subject real
properties as shown in the March 25, 2002 deed of assignment. However, while petitioner would be injured by the judgment in
this suit, we find that petitioner has no present substantial interest to institute the annulment of judgment proceedings and
nullify the order granting the writ of possession.

13. Sps. Oco vs.


Limbaring Basic in procedural law is the rule FACTS
GR 161298 that every action must be
31 January 2006 prosecuted or defended in the Sabas Limbaring subdivided his Lot 2325-D into two lots denominated as Lot Nos. 2325-D-1 and 2325-D-2. He then executed
name of the real party in interest. in favor of Jennifer Limbaring, a Deed of Sale for Lot 2325-D-2 for P60,000; and, in favor of Sarah Jane Limbaring, another
Nato
In the present case, the Deed for Lot 2325-D-1 for P14,440. Accordingly, TCT No. 5268 was cancelled and TCT Nos. T-21921 and T-21920 were
respondent, who was not a party issued in the names of Jennifer and Sarah Jane, respectively. Sensing some irregularities in the transaction, Percita Oco, the
to the contracts being sued upon, daughter of Sabas Limbaring, left Puerto Princesa City and went to Ozamis City. She then filed a case of perjury and
was not able to prove material falsification of documents against the respondent, her uncle who was the father of Jennifer and Sarah Jane. During the pre-
interest in the litigation. For his litigation conference the parties agreed that the two parcels of land should be reconveyed to Percita, who was to pay
failure to do so, the trial court respondent all the expenses that had been and would be incurred to transfer the titles to her name. Pursuant to their
cannot be faulted for dismissing agreement, respondent facilitated the transfer of the titles to her from the names of his daughters.
the action to rescind the
contracts. His status as trustor On April 6, 1999, respondent filed against Spouses Anthony and Percita Oco a Complaint for the rescission of the sales
remained a bare allegation, as he contracts, with recovery of possession and ownership of the two parcels of land. Among others, he claimed
had failed to rebut the legal
presumption: that there is 1) that he was the actual buyer of the lots, but the vendees whose names appeared on the Deeds were his daughters;
absence of a trust when the
purchase price in a deed of sale is 2) that he initially refused to reconvey the properties because he had paid for them with his hard-earned money, which was
paid by a parent in favor of a partly used by Sabas Limbaring for medical expenses;
child. Here, the prima facie
presumption is "that there is a gift 3) that Percita had prepared the two Deeds of Sale, which his daughters signed despite receiving no consideration as stated in
in favor of the child." Any the Deeds;
allegation to the contrary must be
proven by clear and satisfactory 4) that because she refused to pay the P25,000, the Limbaring clan held a meeting on October 26, 1996, during which it was
evidence, a burden that was not agreed that P1,000 per month would be given to respondent from the rentals of Sabas Limbaring’s house; and
discharged by the plaintiff.
5) that the agreement was not implemented, because Percita had failed to cooperate.

ISSUE

Whether respondent, who was the plaintiff in the trial court, was a real party in interest in the suit to rescind the Deeds of
Reconveyance

RULING

To show material interest, respondent argues that a trust was created when he purchased the properties from Sabas
Limbaring in favor of his daughters. As trustor, he allegedly stands to be benefited or injured by any decision in the case.
Respondent has presented only bare assertions that a trust was created. Noting the need to prove the existence of a trust, this
Court has held thus:

"As a rule, the burden of proving the existence of a trust is on the party asserting its existence, and such proof must be clear
and satisfactorily show the existence of the trust and its elements. While implied trusts may be proved by oral evidence, the
evidence must be trustworthy and received by the courts with extreme caution, and should not be made to rest on loose,
equivocal or indefinite declarations. Trustworthy evidence is required because oral evidence can easily be fabricated."

Under the last sentence of Article 1448, respondent’s alleged acts -- paying the price of the subject properties and, in the titles,
naming his children as owners -- raise the presumption that a gift was affected in their favor. Respondent failed to rebut this
presumption. Absent any clear proof that a trust was created, he cannot be deemed a real party in interest.That he should be
deemed a trustor on the basis merely of having paid the purchase price is plainly contradicted by the presumption based on
Article 1448 of the Civil Code ``that there is a gift in favor of the child," not a trust in favor of the parent.

14.Domingo vs. Judicial power is the power to FACTS:


Carague 161065 15 hear and decide cases pending Assailed in this Petition for Certiorari is the legality of Resolution No. 2002-05 of the Commission on Audit (COA) providing for
April 2005- Perez between parties who have the Organizational Restructuring Plan. The above-named petitioners basically alleged therein that this Plan is intrinsically void for
right to sue in courts of law and want of an enabling law authorizing COA to undertake the same and providing for the necessary standards, conditions,
equity. restrictions, limitations, guidelines, and parameters. Petitioners further alleged that in initiating such Organizational
The principle of locus standi of a Restructuring Plan without legal authority, COA committed grave abuse of discretion amounting to lack or excess of
litigant. He who is directly affected jurisdiction.
and whose interest is immediate
and substantial has the standing At this point, it is pertinent to state that the COA is a quasi-judicial body and that its decision, order or ruling may be brought to
to sue. Thus, a party must show a the Supreme Court on certiorari by the aggrieved party.
personal stake in the outcome of
the case or an injury to himself Petitioners claim that they were unceremoniously divested of their designations/ranks upon implementation of the COA
that can be redressed by a Organizational Restructuring Plan without just cause and without due process, in violation of Civil Service Law. Moreover, they
favorable decision in order to were deprived of their respective Representation and Transportation Allowances (RATA), thus causing them undue financial
warrant an invocation of the prejudice.
court's jurisdiction and justify the
exercise of judicial power on his Petitioners now invoke this Court's judicial power to strike down the COA Organizational Restructuring Plan for being
behalf. unconstitutional or illegal.

ISSUE:
Whether petitioners have the legal standing to institute the instant petition.

HELD:
NO.

Petitioners invoke our ruling in Chavez v. Public Estates Authority, Agan, Jr. v. Philippine International Air Terminals Co., Inc.,
and Information Technology Foundation of the Philippines v. Commission on Elections that where the subject matter of a case
is a matter of public concern and imbued with public interest, then this fact alone gives them legal standing to institute the
instant petition. Petitioners contend that the COA Organizational Restructuring Plan is not just a mere reorganization but a
revamp or overhaul of the COA, with a "spillover effect" upon its audit performance. This will have an impact upon the rest of
the government bodies subject to its audit supervision, thus, should be treated as a matter of transcendental importance.
Consequently, petitioners' legal standing should be recognized and upheld.

Respondents, through the Office of the Solicitor General (OSG), counter that petitioners have no legal standing to file the
present petition since they have not shown "a personal stake in the outcome of the case" or an actual or potential injury that
can be redressed by our favorable decision. Petitioners themselves admitted that "they do not seek any affirmative relief nor
impute any improper or improvident act against the said respondents" and "are not motivated by any desire to seek affirmative
relief from COA or from respondents that would redound to their personal benefit or gain." It is clear then that petitioners failed
to show any "present substantial interest" in the outcome of this case, citing Kilosbayan v. Morato. Nor may petitioners claim
that as taxpayers, they have legal standing since nowhere in their petition do they claim that public funds are being spent in
violation of law or that there is a misapplication of the taxpayers' money, as we ruled in Dumlao v. Comelec.

Petitioners' reliance upon our rulings in Chavez, Agan, Jr., and Information Technology Foundation is flawed.

Petitioners do not have legal standing because they have not shown any direct and personal interest in the COA
Organizational Restructuring Plan.

There is no indication that they have sustained or are in imminent danger of sustaining some direct injury as a result if its
implementation. They also admitted that they do not seek affirmative relief that would redound to their personal benefit or gain.

15.Heirs of Bertuldo The purpose behind the rule on Facts: On May 21, 1991, private respondents Custodio, Rufo, Tomas and Honorio, all surnamed Balane, filed a complaint for
Hinog vs. substitution is the protection of the "Recovery of Ownership and Possession, Removal of Construction and Damages" against Bertuldo Hinog (Bertuldo for
Melicor 140954 12 right of every party to due brevity).
process. It is to ensure that the
April 2005- Purificacion deceased party would continue to
be properly represented in the suit On July 2, 1991, Bertuldo filed his Answer.
through the duly appointed legal
representative of his estate. Non- After the pre-trial, trial on the merits ensued. On November 18, 1997, private respondents rested their case. Thereupon,
compliance with the rule on Bertuldo started his direct examination. However, on June 24, 1998, Bertuldo died without completing his evidence.
substitution would render the
proceedings and judgment of the
trial court infirm because the court Atty. Sulpicio A. Tinampay withdrew as counsel for Bertuldo as his services were terminated by petitioner Bertuldo Hinog III.
acquires no jurisdiction over the Atty. Veronico G. Petalcorin then entered his appearance as new counsel for Bertuldo.
persons of the legal
representatives or of the heirs on Atty. Petalcorin filed a motion to expunge the complaint from the record and nullify all court proceedings on the ground that
whom the trial and the judgment private respondents failed to specify in the complaint the amount of damages claimed so as to pay the correct docket fees; and
would be binding. Thus, proper that under Manchester Development Corporation vs. Court of Appeals, non-payment of the correct docket fee is jurisdictional.
substitution of heirs must be
effected for the trial court to Private respondents opposed the motion to expunge on the following grounds:
acquire jurisdiction over their (a) said motion was filed more than seven years from the institution of the case;
persons and to obviate any (b) Atty. Petalcorin has not complied with Section 16, Rule 3 of the Rules of Court which provides that the death of the
future claim by any heir that he original defendant requires a substitution of parties before a lawyer can have legal personality to represent a litigant
was not apprised of the and the motion to expunge does not mention of any specific party whom he is representing;
litigation. (c) collectible fees due the court can be charged as lien on the judgment; and
(d) considering the lapse of time, the motion is merely a dilatory scheme employed by petitioners.
Therefore, before said
compliance, Atty. Petalcorin The trial court ordered the complaint to be expunged from the records and the nullification of all court proceedings taken for
had no standing in the court a failure to pay the correct docket fees.
quo when he filed his
pleadings. Upon payment of deficiency docket fee, private respondents filed a manifestation with prayer to reinstate the case. The trial
court reinstated the case.

Petitioners filed their supplemental pleading which the trial court denied on the ground that the Deed of Absolute Sale is a new
matter which was never mentioned in the original answer. The trial court also noted that no formal substitution of the parties
was made because of the failure of defendant's counsel to give the names and addresses of the legal representatives
of Bertuldo, so much so that the supposed heirs of Bertuldo are not specified in any pleading in the case.

Noting that there has been no substitution of parties following the death of Bertuldo, the trial court directed Atty.
Petalcorin to comply with the provisions of Section 16, Rule 3 of the Rules of Court.
On November 19, 1999, Atty. Petalcorin complied with the directive of the trial court to submit the names and
addresses of the heirs of Bertuldo.
Issue: Whether or not the counsel complied with the requirement of the Rules of Court as provided for in Rule 3, Section 16;
effect of non-compliance.

Held: No formal substitution of the parties was effected within thirty days from date of death of Bertuldo, as required by Section
16, Rule 3 of the Rules of Court. Needless to stress, the purpose behind the rule on substitution is the protection of the right of
every party to due process. It is to ensure that the deceased party would continue to be properly represented in the suit
through the duly appointed legal representative of his estate. Non-compliance with the rule on substitution would render the
proceedings and judgment of the trial court infirm because the court acquires no jurisdiction over the persons of the legal
representatives or of the heirs on whom the trial and the judgment would be binding. Thus, proper substitution of heirs must
be effected for the trial court to acquire jurisdiction over their persons and to obviate any future claim by any heir that he
was not apprised of the litigation against Bertuldo or that he did not authorize Atty. Petalcorin to represent him.

The list of names and addresses of the heirs was submitted sixteen months after the death of Bertuldo and only when the trial
court directed Atty. Petalcorin to comply with the provisions of Section 16, Rule 3 of the Rules of Court. Strictly speaking
therefore, before said compliance, Atty. Petalcorin had no standing in the court a quo when he filed his pleadings. Be
that as it may, the matter has been duly corrected by the Order of the trial court dated October 15, 1999.

16. COMM. DOMINGO Sec. 7, Rule 3 of the ROC FACTS:


v. SCHEER [GR requires indispensable parties to Scheer is a German national who was given permanent status to reside in the Philippines. The Bureau of Immigration and
154745] 29 JAN 2004 be joined as plaintiffs or Deportation (BID) received information that respondent Scheer was wanted by the German Police and a warrant of arrest had
Sabio defendants. The joinder of been issued against him. The BOC issued a Summary Deportation Order against respondent Scheer, which they issued
indispensable parties is relying on the correspondence from the German Vice-Consul on its speculation that it was unlikely that the German Embassy
mandatory. Without the presence will issue a new passport to respondent Scheer.
of indispensable parties to the
suit, the judgement of the court When respondent Scheer was informed of the deportation order, he aired his side to then BID Commissioner Verceles. The
cannot attain real finality. Commissioner allowed respondent Scheer to remain in the Philippines, giving the latter time to secure clearance and a new
passport from the German Embassy. Respondent Scheer filed a Motion for Reconsideration, however, the BOC did not
However, non-joinder of resolve the respondent’s motion. Respondent Scheer was neither arrested nor deported.
indispensable parties is not a
ground for the dismissal of an Respondent Scheer informed Commissioner Verceles that his passport had been renewed following the dismissal of the
action. Parties may be added by physical injuries case against him in Germany. He reiterated his request for the cancellation of the Summary Deportation Order
the order of the court on motion of and the restoration of his permanent resident status. In 1996, the German Embassy issued a regular passport to respondent
the party or on its own initiative. Scheer, which will expire in 2006. Respondent Scheer remained in the Philippines and maintained his business in Palawan.

Petitioner Immigration Commissioner Domingo assumed office and she wrote to the German Embassy and inquired if
respondent Scheer is still wanted by the German Police. To which they replied in the negative. In 2002, Marine operatives and
BID agents apprehended respondent Scheer in his residence on the orders of petitioner Commissioner. He was brought to the
BID Manila Office and was held in custody while awaiting his deportation.

Respondent Scheer’s counsel filed the with CA a petition for certiorari, prohibition, and mandamus with prayer for TRO and
writ of preliminary injunction. Respondent Scheer asserted that no speedy remedy was open to him in the ordinary course of
law and his urgent motion for reconsideration of the Summary Deportation Order of the BOC had not been resolved despite
the lapse of more than 6 years. He also claimed that he was a fully documented alien, a permanent resident and a law-abiding
citizen.

The CA issued an order restraining petitioner Commissioner from deporting respondent Scheer on a bond of Php 100,000.00.
The BOC issued a resolution denying respondent Scheer’s urgent motion for reconsideration and motion for bail/recognizance.
The CA then rendered a decision in favor of respondent Scheer granting his petition for certiorari and prohibition, permanently
enjoining petitioner Commissioner from deporting respondent Scheer.
ISSUE:
Whether or not the non-joinder of an indispensable party is a ground for the dismissal of the petition.

HELD:
NO. The BOC is an indispensable party. Sec. 7, Rule 3 of the ROC requires indispensable parties to be joined as plaintiffs or
defendants. The joinder of indispensable parties is mandatory. Without the presence of indispensable parties to the suit, the
judgement of the court cannot attain real finality. Strangers to a case are not bound by the judgement rendered by the court.
The absence of an indispensable party renders all subsequent actions of the court null and void. Lack of authority to act, not
only of the absent party, but also as to those present. The responsibility of impleading all the indispensable parties rests on the
petitioner.

Non-joinder of indispensable parties is not a ground for the dismissal of an action. Parties may be added by the order of the
court on motion of the party or on its own initiative. If the petitioner refuses to implead an indispensable party despite the order
of the court, the latter may dismiss the petition for the petitioner’s failure to comply.

17. Iron and Steel Rule 3, Section 1 of the Rules of FACTS: Petitioner Iron and Steel Authority ("ISA") was created by P.D. No. 272 for a term of 5 years in order to develop and
Authority vs. Court of Court specifies who may be promote the iron and steel industry in the Philippines. When ISA's original term expired its term was extended for another 10
Appeals 102976 25 parties to a civil action. Under this years by Executive Order No. 555.
October 1995-Someros provision, it will be seen that The National Steel Corporation ("NSC") then a wholly owned subsidiary of the National Development Corporation which is
those who can be parties to a civil itself an entity wholly owned by the National Government, embarked on an expansion program which is the construction of an
action may be broadly integrated steel mill in Iligan City.
categorized into two (2) groups: Then a Proclamation No. 2239 was issued withdrawing from sale or settlement a large tract of public land located in Iligan City,
(a) those who are recognized as and reserving that land for the use and immediate occupancy of NSC.
persons under the law whether Since certain portions of the public land subject matter of Proclamation No. 2239 were occupied by a non-operational chemical
natural, i.e., biological persons, on fertilizer plant and related facilities owned by private respondent Maria Cristina Fertilizer Corporation ("MCFC"), Letter of
the one hand, or juridical persons Instruction (LOI) No. 1277, was issued directing the NSC to:
such as corporations, on the other (1) "negotiate with the owners of MCFC, for and on behalf of the Government, for the compensation of MCFC's present
hand; and (b) entities authorized occupancy rights on the subject land."
by law to institute actions. (2) that if they fail to reach an agreement within a period of 60 days, petitioner ISA was to exercise its power of eminent
The Republic itself is a body domain under P.D. No. 272 and to initiate expropriation proceedings in respect of occupancy rights of private respondent
corporate and juridical person MCFC relating to the subject public land as well as the plant itself and related facilities and to code the same to the NSC.
vested with the full panoply of Negotiations between NSC and private respondent MCFC did fail. Accordingly, petitioner ISA commenced eminent domain
powers and attributes which are proceedings against private respondent MCFC in the RTC Br. 1, of Iligan City, praying that it (ISA) be placed in possession of
compendiously described as the property involved. The PNB, as mortgagee of the plant facilities and improvements involved in the expropriation
"legal personality." proceedings, was also impleaded as party-defendant.
On 17 September 1983 a writ of possession was issued by the trial court in favor of ISA. ISA in turn placed NSC in possession
It is common knowledge that and control of the land occupied by MCFC's fertilizer plant installation.
other agencies or While the trial was on-going, the statutory existence of petitioner ISA expired. MCFC then filed a motion to dismiss, contending
instrumentalities of the that no valid judgment could be rendered against ISA which had ceased to be a juridical person. Petitioner ISA filed its
Government of the Republic are opposition to this motion.
cast in corporate form, that is to trial court granted MCFC's motion to dismiss. The dismissal was anchored on the provision of the Rules of Court stating that
say, are incorporated agencies or "only natural or juridical persons or entities authorized by law may be parties in a civil case." The trial court also referred to
instrumentalities, sometimes with non-compliance by petitioner ISA with the requirements of Section 16, Rule 3 of the Rules of Court.
and at other times without capital Petitioner ISA moved for reconsideration. In the alternative, petitioner ISA urged that the Republic of the Philippines, being the
stock, and accordingly vested with real party-in-interest, should be allowed to be substituted for petitioner ISA. In this connection, ISA referred to a letter from the
a juridical personality distinct from Office of the President which especially directed the Solicitor General to continue the expropriation case.
the personality of the Republic. The trial court denied the motion for reconsideration.
Petitioner went on appeal to the CA which affirmed the order of dismissal of the trial court.
In this Petition for Review, the Solicitor General argues that since ISA initiated and prosecuted the action for expropriation in its
capacity as agent of the Republic of the Philippines, the Republic, as principal of ISA, is entitled to be substituted and to be
made a party-plaintiff after the agent ISA's term had expired.
Private respondent MCFC, argues that the failure of Congress to enact a law further extending the term of ISA evinced a "clear
legislative intent to terminate the juridical existence of ISA," and that the authorization issued by the Office of the President to
the Solicitor General for continued prosecution of the expropriation suit could not prevail over such negative intent. It is also
contended that the exercise of the eminent domain by ISA or the Republic is improper, since that power would be exercised
"not on behalf of the National Government but for the benefit of NSC."
ISSUE:whether or not the Republic of the Philippines is entitled to be substituted for ISA in view of the expiration of ISA's
term.
HELD: YES,
Rule 3, Section 1 of the Rules of Court, specifies who may be parties to a civil action:
"SECTION 1. Who May Be Parties. — Only natural or juridical persons or entities authorized by law may be
parties in a civil action."
Under the above quoted provision, it will be seen that those who can be parties to a civil action may be broadly
categorized into two (2) groups:
(a) those who are recognized as persons under the law whether natural, i.e., biological persons, on
the one hand, or juridical persons such as corporations, on the other hand; and
(b) entities authorized by law to institute actions.
Examination of the statute which created petitioner ISA shows that ISA falls under category (b) above. P.D. No. 272, as
already noted, contains express authorization to ISA to commence expropriation proceedings.
It should also be noted that the enabling statute of ISA expressly authorized it to enter into certain kinds of contracts "for and in
behalf of the Government"
Clearly, ISA was vested with some of the powers or attributes normally associated with juridical personality.
The ISA in fact appears to the Court to be a non-incorporated agency or instrumentality of the Republic of the Philippines. It is
common knowledge that other agencies or instrumentalities of the Government of the Republic are cast in corporate form, that
they, are incorporated agencies or instrumentalities, sometimes with and at other times without capital stock, and accordingly
vested with a juridical personality distinct from the personality of the Republic.
Since, in the instant case, ISA is a non-incorporated agency or instrumentality of the Republic, its powers, duties, functions,
assets and liabilities are properly regarded as folded back into the Government of the Republic of the Philippines and hence
assumed once again by the Republic, no special statutory provision having been shown to have mandated succession thereto
by some other entity or agency of the Republic.
The general rule is that an action must be prosecuted and defended in the name of the real party in interest. (Rule 3, Section
2) Petitioner ISA was, at the commencement of the expropriation proceedings, a real party in interest, having been explicitly
authorized by its enabling statute to institute expropriation proceedings. The Rules of Court at the same time expressly
recognize the role of representative parties:
"SECTION 3. Representative Parties. — A trustee of an expressed trust, a guardian, an executor or
administrator, or a party authorized by statute may sue or be sued without joining the party for whose benefit
the action is presented or defended; but the court may, at any stage of the proceedings, order such
beneficiary to be made a party. . . ." (Emphasis supplied)
It is also relevant to note that the non-joinder of the Republic which occurred upon the expiration of ISA's statutory term, was
not a ground for dismissal of such proceedings since a party may be dropped or added by order of the court, on motion of any
party or on the court's own initiative at any stage of the action and on such terms as are just. In the instant case, the Republic
has precisely moved to take over the proceedings as party-plaintiff.

18.Air Transportation - Not all government entities, FACTS:


Office vs. Sps. whether corporate or non-
Ramos 159402 23 corporate, are immune from suits. Spouses David and Elisea Ramos (respondents) discovered that a portion of their land registered under Transfer Certificate of
February 2011- Tan Immunity from suits is determined Title No. T-58894 of the Baguio City land records with an area of 985 square meters, more or less, was being used as part of
by the character of the objects for the runway and running shoulder of the Loakan Airport being operated by petitioner Air Transportation Office (ATO). On
which the entity was organized. August 11, 1995, the respondents agreed after negotiations to convey the affected portion by deed of sale to the ATO in
consideration of the amount of P778,150.00. However, the ATO failed to pay despite repeated verbal and written demands.
- Suits against State agencies
with relation to matters in which Thus, on April 29, 1998, the respondents filed an action for collection against the ATO and some of its officials in the RTC. In
they have assumed to act in their answer, the ATO and its co-defendants invoked as an affirmative defense the issuance of Proclamation No. 1358,
private or non-governmental whereby President Marcos had reserved certain parcels of land that included the respondents' affected portion for use of the
capacity, and various suits Loakan Airport. They asserted that the RTC had no jurisdiction to entertain the action without the State's consent considering
against certain corporations that the deed of sale had been entered into in the performance of governmental functions.
created by the state for public
purposes, but to engage in The RTC held in favor of the spouses and ordered ATO to pay : (1) The amount of P778,150.00 being the value of the parcel
matters partaking more of the of land appropriated by the defendant ATO as embodied in the Deed of Sale, plus an annual interest of 12% from August 11,
nature of ordinary business rather 1995, the date of the Deed of Sale until fully paid; (2) The amount of P150,000.00 by way of moral damages and P150,000.00
than functions of a governmental as exemplary damages; (3) the amount of P50,000.00 by way of attorney's fees plus P15,000.00 representing the 10, more or
or political character, are not less, court appearances of plaintiff's counsel; (4) The costs of this suit.
regarded as suits against the
state. On appeal, the CA affirmed the RTC decision with modification deleting the awarded cost and reducing the moral and
exemplary damage to P30,000.00 each, and attorneys fees lowered to P10,000.00
The latter is true, although the
state may own stock or property ISSUE: whether or not the ATO could be sued without the State's consent.
of such a corporation for by
engaging in business operations HELD:
through a corporation, the state
divests itself so far of its Yes, it may be sued without its consent.
sovereign character, and by
implication consents to suits The CA thereby correctly appreciated the juridical character of the ATO as an agency of the Government not performing a
against the corporation. purely governmental or sovereign function, but was instead involved in the management and maintenance of the Loakan
Airport, an activity that was not the exclusive prerogative of the State in its sovereign capacity. Hence, the ATO had no claim to
the State's immunity from suit. We uphold the CA's aforequoted holding. We further observe the doctrine of sovereign
immunity cannot be successfully invoked to defeat a valid claim for compensation arising from the taking without just
compensation and without the proper expropriation proceedings being first resorted to of the plaintiffs' property.
19.Department of No suit may be filed against the Facts:
Health vs. Phil State unless it consents expressly AO No. 27 series of 1998 was issued by the former DOH Secretary, setting the guidelines and procedure for accreditation of
Pharmawealth, or impliedly. the doctrine (of State government suppliers of pharmaceutical products for sale and distribution to the public, such accreditation to be valid for three
Inc. 182358 20 immunity) extends to acts of years subject to annual review. This was amended by AO 10 series of 2000, and was again amended by AO 66 Series of
public officials done in the 2000. The DOH then issued Memorandum No. 171-C, providing for a list and category of sanctions to be imposed on
February 2013- Tanada
discharge and performance of accredited government suppliers of pharmaceutical products in case of adverse findings regarding their products or violations
their duties. committed by them during their accreditation.

In line with this, the DOH former Undersecretary issued Memorandum No. 209 series of 2000, inviting representatives of 24
accredited drug companies, including herein respondent PPI, for a meeting. During the meeting, the 24 drug companies were
informed of the report made by the BFAD, detailing their violations or adverse findings. The BFAD found that PPI’s products
being sold to the public were unfit for human consumption. The drug companies were directed to submit within 10 days their
respective explanations. PPI, instead of submitting its written explanation, belatedly sent a letter to the DOH Undersecretary,
informing her that PPI has referred the matter to its lawyers, but there was no indication when the reply would be submitted,
nor did it seek for an extension, much less offer any explanation for its belated reply.

In a letter-reply, the DOH Undersecretary informed PPI that its accreditation has been suspended for 2 years pursuant to AO
10 and Memorandum No. 171-C. PPI questioned the suspension of its accreditation in a letter addressed to the
Undersecretary, claiming that Section VII of AO 10 is illegal and null and void for being violative of RA No. 3720. It further
claimed that DOH violated its to due process as it was not given the benefit of notice and hearing.

Thereafter, PPI filed before the RTC a complaint against the DOH for the declaration as null and void of certain DOH
administrative issuances, claiming that these were in contravention of Sec. 26 (d), RA No. 3720, and for payment of damages.
It also impleaded the DOH Undersecretary and former Secretary. The current DOH Secretary was also impleaded in an
amended and supplemental complaint. The DOH moved to dismiss the case on the ground, among others, that the case,
which was against the State, violates the doctrine of State immunity from suit. The RTC dismissed the complaint, declaring that
it was one instituted against the State, in which case the principle of state immunity from suit is applicable.

The CA reversed the RTC ruling and remanded the case to the RTC. It ruled that there was a sufficient cause of action – that
due to the DOH’s acts which were beyond the scope of their authority, PPI’s accreditation was suspended. It also ruled that by
filing a motion to dismiss, the DOH hypothetically admitted the allegations, stating that petitioners were being sued in their
individual and personal capacities, negating their claim that the complaint is an unauthorized suit against the State.

Issue:
Whether or not the DOH may be impleaded in the case at bar.

Held: No.
The SC held that the invocation by the DOH that it was a suit against the State was valid. No suit may be filed against the
State unless it consents expressly or impliedly. Here, the DOH did not consent, expressly or impliedly, to the suit. Furthermore,
the complaint sought to hold the DOH solidarily and jointly liable with the other defendants for damages which constitutes a
charge or financial liability against the State, requiring an appropriation from the national treasure which is the situation sought
to be avoided by the doctrine of state immunity.

It must also be stressed that the doctrine extends to acts of public officials done in the discharge and performance of their
duties. In this case, it was clear that the acts complained of against officials of the DOH were done in the performance and
discharge of their official functions or in their official capacities, and not in their personal or individual capacities. They were
being charged with the issuance and the implementation of the assailed issuances. As this was a suit against the State, the
DOH cannot be impleaded.

20. Gochan vs. Facts: Felix Gochan & Sons Realty Corporation (Gochan Realty) is registered in SEC with Felix Gochan Sr. & 5 others as
Young 131889 12 The above-quoted rules, while incorporators.The daughter of Felix Gochan Sr. (& the mother of respondents), Alice, inherited 50 shares of stock in Gochan
March 2001-Ubay permitting an executor or Realty. When Alice died, she left the 50 shares to her husband John Young, Sr.The RTC adjudicated 6/14 of these shares to
administrator to represent or to the children of Alice.
bring suits on behalf of the
deceased, do not prohibit the Having earned dividends, these stocks numbered 179. John Young Sr. requested Gochan Realty to partition the shares of his
heirs from representing the late wife by cancelling the stock certificates in his name and issuing new stock certificates in the names of the children.
deceased. These rules are easily Petitioner Gochan Realty refused, citing as reason, the right of first refusal granted to the remaining stockholders by the
applicable to cases in which an Articles of Incorporation. John Young, Sr. died and left the shares to the respondents.
administrator has already been
appointed. But no rule Respondents Cecilia Gochan Uy and Miguel Uy filed a complaint for issuance of shares of stock to the rightful owners,
categorically addresses the nullification of shares of stock, reconveyance of property impressed with trust, accounting, removal of officers and directors
situation in which special and damages against Petitioner Gochan Realty.
proceedings for the settlement of
an estate have already been Petitioners Gochan et al filed a motion to dismiss the complaint alleging that: (1) the SEC has no jurisdiction over the nature of
instituted, yet no administrator the action; (2) the respondents were not the real parties-in-interest and had no capacity to sue; and (3) respondents’ causes of
has been appointed. In such action were barred by the Statute of Limitations.
instances, the heirs cannot be
expected to wait for the SEC Hearing Officer granted the motion to dismiss
appointment of an administrator;
then wait further to see if the Respondents filed a motion for a reconsideration but it was denied for being pro-forma.
administrator appointed would
care enough to file a suit to Respondents appealed to the SEC en banc, contending that the SEC has jurisdiction.
protect the rights and the interests
of the deceased; and in the Petitioners contend that the appeal was 97 days late and beyond the 30-day period for appeals.
meantime do nothing while the
rights and the properties of the The SEC en banc ruled for the petitioners and holding that the respondents’ motion for reconsideration did not interrupt the 30-
decedent are violated or day period for appeal because said motion was pro-forma.
dissipated.
CA ruled that the SEC had no jurisdiction as far as the heirs of Alice Gochan were concerned, because they were not yet
stockholders. But it upheld the capacity of Respondents Cecilia Gochan Uy and Miguel Uy. It also upheld that the intestate
Estate of John Young Sr. was an indispensable party. Moreover, it declared that respondents' Motion for Reconsideration
before the SEC was not pro forma; thus, its filing tolled the appeal period.

Issue: Whether or not the intestate estate of John D. Young Sr. is an indispensable party in the SEC case considering that the
individual heirs shares are still in the decedent stockholders name.
Held: Petitioners contend that the Intestate Estate of John D. Young Sr. is not an indispensable party, as there is no showing
that it stands to be benefited or injured by any court judgment.

It would be useful to point out at this juncture that one of the causes of action stated in the Complaint filed with the SEC refers
to the registration, in the name of the other heirs of Alice Gochan Young, of 6/14th of the shares still registered under the name
of John D. Young Sr. Since all the shares that belonged to Alice are still in his name, no final determination can be had without
his estate being impleaded in the suit. His estate is thus an indispensable party with respect to the cause of action dealing with
the registration of the shares in the names of the heirs of Alice.

Petitioners further claim that the Estate of John Young Sr. was not properly represented. They claim that when the estate is
under administration, suits for the recovery or protection of the property or rights of the deceased may be brought only by the
administrator or executor as approved by the court. The rules relative to this matter do not, however, make any such
categorical and confining statement.

Section 3 of Rule 3 of the Rules of Court, which is cited by petitioner in support of their position, reads:

Sec. 3. Representatives as parties. - Where the action is allowed to be prosecuted or defended by a representative or
someone acting in a fiduciary capacity, the beneficiary shall be included in the title of the case and shall be deemed to be the
real party in interest. A representative may be a trustee of an express trust, a guardian, an executor or administrator, or a party
authorized by law or these Rules. An agent acting in his own name and for the benefit of an undisclosed principal may sue or
be sued without joining the principal except when the contract involves things belonging to the principal.

Section 2 of Rule 87 of the same Rules, which also deals with administrators, states:

Sec. 2. Executor or administrator may bring or defend actions which survive. - For the recovery or protection of the property or
rights of the deceased, an executor or administrator may bring or defend, in the right of the deceased, actions for causes which
survive.

The above-quoted rules, while permitting an executor or administrator to represent or to bring suits on behalf of the deceased,
do not prohibit the heirs from representing the deceased. These rules are easily applicable to cases in which an administrator
has already been appointed. But no rule categorically addresses the situation in which special proceedings for the settlement
of an estate have already been instituted, yet no administrator has been appointed. In such instances, the heirs cannot be
expected to wait for the appointment of an administrator; then wait further to see if the administrator appointed would care
enough to file a suit to protect the rights and the interests of the deceased; and in the meantime do nothing while the rights and
the properties of the decedent are violated or dissipated.

The Rules are to be interpreted liberally in order to promote their objective of securing a just, speedy and inexpensive
disposition of every action and proceeding. They cannot be interpreted in such a way as to unnecessarily put undue hardships
on litigants. For the protection of the interests of the decedent, this Court has in previous instances recognized the heirs as
proper representatives of the decedent, even when there is already an administrator appointed by the court. When no
administrator has been appointed, as in this case, there is all the more reason to recognize the heirs as the proper
representatives of the deceased. Since the Rules do not specifically prohibit them from representing the deceased, and since
no administrator had as yet been appointed at the time of the institution of the Complaint with the SEC, we see nothing wrong
with the fact that it was the heirs of John D. Young Sr. who represented his estate in the case filed before the SEC.

21.Chua vs. Neither misjoinder nor non-joinder A complaint for damages was filed by Christine Chua, petitioner, before the RTC of Caloocan with her brother, Jonathan, as
Torres 151900 30 of parties is ground for dismissal co-plaintiff. Jonathan Chua issued a check for the purchase of diesel oil in Caltex Service center which was owned by Torres,
August 2005- Yumul of an action. Parties may be respondent. The check bounced and Beltran, other respondent, sent a demand letter to Christine which was ignored by the
dropped or added by order of the latter on the ground that she was not the one who issued the check. Beltran then instituted a criminal action for violation of BP
court on motion of any party or on 22.
its own initiative at any stage of
the action and on such terms as Significantly, while Jonathan Chua was named as a plaintiff to the suit, it was explicitly qualified in the second paragraph of the
are just. complaint that he was being "impleaded herein as a necessary party-plaintiff". No allegation of any damage or injury was
sustained by Jonathan neither did he sign any verification or certification against forum-shopping wherein petition Christine
identified herself as “the principal plaintiff”.

RTC ordered the dismissal of the complaint9 on the ground that Jonathan Chua had not executed a certification against forum-
shopping. The RTC stressed that Section 5, Rule 7 of the Rules of Civil Procedure, the rule requiring thecertification, makes no
distinction whether the plaintiff required to execute the certification is a principal party, a nominal party or a necessary party.

Issue/s:
WON the absence of the signature in the required verification and certification against non-forum shopping of a party misjoined
is a valid ground for dismissal in the complaint.

Held:
NO. Since the misjoined party plaintiff receives no recognition from the court as either an indispensable or necessary party-
plaintiff, it then follows that whatever action or inaction the misjoined party may take on the verification or certification against
forum-shopping is inconsequential.

Neither misjoinder nor non-joinder of parties is ground for dismissal of an action. Any claim against a misjoined party may be
severed and proceeded with separately.

Wherefore, the petition is denied.

22.Lotte Phils. vs. dela An indispensable party is a party Facts:


Cruz 166302 28 July in interest without whom no final Lotte Phils., Inc. (Lotte) is a domestic corporation Dela Crux et.al are among those who were hired and assigned to the
2005- Zapanta determination can be had of an confectionery facility operated by private respondent.
action, and who shall be joined
either as plaintiffs or defendants. On December 14, 1995 — and yearly thereafter until the year 2000 — 7J Maintenance and Janitorial Services ("7J") entered
The joinder of indispensable into a contract with private respondent to provide manpower for needed maintenance, utility, janitorial and other services to the
parties is mandatory. The latter. In compliance with the terms and conditions of the service contract, and to accommodate the needs of private
presence of indispensable parties respondent for personnel/workers to do and perform "piece works," petitioners, among others, were hired and assigned to
is necessary to vest the court with private respondent as repackers or sealers.
jurisdiction, which is "the authority
to hear and determine a cause, However, private respondent dispensed with their services allegedly due to the expiration/ termination of the service contract
the right to act in a case". Thus, by respondent with 7J. They were either told "hwag muna kayong pumasok at tatawagan na lang kung may gawa"; or were
without the presence of asked to wait "pag magrereport sila sa trabaho." Unfortunately, petitioners were never called back to work again.
indispensable parties to a suit or
proceeding, judgment of a court Aggrieved, petitioners lodged a labor complaint against both private respondent Lotte and 7J, for illegal dismissal,
cannot attain real finality. The regularization, payment of corresponding backwages and related employment benefits, 13th month pay, service incentive
absence of an indispensable party leave, moral and exemplary damages and attorney's fees based on total judgment award.
renders all subsequent actions of
the court null and void for want of Labor Arbiter Cresencio G. Ramos, Jr., rendered judgment declaring 7J as employer of respondents, found 7J guilty of illegal
authority to act, not only as to the dismissal and ordered to reinstate respondents, pay P2,374,710.00 as backwages, P713,648.00 as 13th month pay and
absent parties but even as to P117,000.00 as service incentive leave pay.
those present.
Respondents appealed to the NLRC praying that Lotte be declared as their direct employer because 7J is merely a labor-only
contractor. NLRC found no reason to disturb the findings of the labor arbiter and affirmed its ruling that 7J is the employer of
respondents and solely liable for their claims. Respondents' motion for reconsideration was denied by the NLRC.

They filed a petition for certiorari in the Court of Appeals against the NLRC and Lotte, insisting that their employer is Lotte and
not 7J.

Lotte, denied that respondents were its employees. It prayed that the petition be dismissed for failure to implead 7J who is a
party interested in sustaining the proceedings in court.

Court of Appeals reversed and set aside the rulings of the Labor Arbiter and the NLRC, declaring Lotte as the real employer of
respondents and that 7J who engaged in labor-only contracting was merely the agent of Lotte. Respondents who performed
activities directly related to Lotte's business were its regular employees. As such, they must be accorded security of tenure and
their services terminated only on "just" and "authorized" causes.
Lotte's motion for reconsideration was denied

Issue: w/n the presence of indispensable parties is necessary to vest the court with jurisdiction

Held:
Yes.. An indispensable party is a party in interest without whom no final determination can be had of an action, and who shall
be joined either as plaintiffs or defendants. The joinder of indispensable parties is mandatory.

The presence of indispensable parties is necessary to vest the court with jurisdiction, which is "the authority to hear and
determine a cause, the right to act in a case". Thus, without the presence of indispensable parties to a suit or proceeding,
judgment of a court cannot attain real finality. The absence of an indispensable party renders all subsequent actions of the
court null and void for want of authority to act, not only as to the absent parties but even as to those present.

In the case at bar, 7J is an indispensable party. It is a party in interest because it will be affected by the outcome of the case.
The Labor Arbiter and the NLRC found 7J to be solely liable as the employer of respondents. The Court of Appeals however
rendered Lotte jointly and severally liable with 7J who was not impleaded by holding that the former is the real employer of
respondents. Plainly, its decision directly affected 7J.

In Domingo v. Scheer, we held that the non-joinder of indispensable parties is not a ground for the dismissal of an action and
the remedy is to implead the non-party claimed to be indispensable. Parties may be added by order of the court on motion of
the party or on its own initiative at any stage of the action and/or such times as are just. If the petitioner refuses to implead an
indispensable party despite the order of the court, the latter may dismiss the complaint/petition for the petitioner/plaintiff's
failure to comply therefor.

Although 7J was a co-party in the case before the Labor Arbiter and the NLRC, respondents failed to include it in their petition
for certiorari in the Court of Appeals. Hence, the Court of Appeals did not acquire jurisdiction over 7J. No final ruling on this
matter can be had without impleading 7J, whose inclusion is necessary for the effective and complete resolution of the case
and in order to accord all parties with due process and fair play.

23.National Power An indispensable party is one who Facts:


Corporation vs. has an interest in the controversy On March 28, 2003 petitioner National Power Corporation received a notice of franchise tax delinquency from the respondent
Provincial or subject matter and in whose Provincial Government of Bataan (the Province) for P45.9 million covering the years 2001, 2002, and 2003. The Province
Government of absence there cannot be a based its assessment on the NPC's sale of electricity that it generated from two power plants in Bataan. Rather than pay the
determination between the parties tax or reject it, the NPC chose to reserve its right to contest the computation pending the decision of the Supreme Court in
Bataan 180654 21
already before the court which is National Power Corporation v. City of Cabanatuan, a case where the issue of the NPC's exemption from the payment of local
April 2014- Abad effective, complete or equitable. franchise tax was then pending.
Here, since the subject properties
belong to PSALM Corp. and The Province again sent notices of tax due to the NPC, calling its attention to the Court's Decision in National Power
TRANSCO, they are certainly Corporation v. City of Cabanatuan that held the NPC liable for the payment of local franchise tax. The NPC replied, however,
indispensable parties to the case that it had ceased to be liable for the payment of that tax after Congress enacted R.A. 9136, also known as the Electric Power
that must be necessarily included Industry Reform Act (EPIRA) that took effect on June 26, 2001. The new law relieved the NPC of the function of generating
before it may properly go forward. and supplying electricity beginning that year. Consequently, the Province has no right to further assess it for the 2001, 2002,
For this reason, the proceedings and 2003 local franchise tax.
below that held the NPC liable for
the local franchise tax is a nullity. Ignoring the NPC's view, the Province issued a "Warrant of Levy" on 14 real properties that it used to own in Limay, Bataan. In
It did not matter where the RTC March 2004 the Province caused their sale at public auction with itself as the winning bidder.
Decision was appealed, whether The NPC filed with the RTC of Mariveles, Bataan, a petition for declaration of nullity of the foreclosure sale with prayer for
before the CA or the CTA. preliminary mandatory injunction against the Province, the provincial treasurer, and the Sangguniang Panlalawigan.

The NPC alleged that the law provided that power generation is not a public utility operation requiring a franchise, hence, not
taxable. What remains subject to such tax is the business of transmission and distribution of electricity since these required a
national franchise. NPC had ceased by operation of the EPIRA in 2001 to engage in power transmission, given that all its
facilities for this function, including its nationwide franchise, had been transferred to the National Transmission Corporation
(TRANSCO).

NPC asked the RTC to issue a preliminary injunction, enjoining the transfer of title and the sale of the foreclosed lands to
Bataan and, after trial, to make the injunction permanent, declare NPC exempt from the local franchise tax and annul the
foreclosure sale.

RTC - dismissed the NPC's petition, stating that the franchise tax was not based on ownership of property but on the NPC's
exercise of the privilege of doing business within Bataan. Further, the NPC presented no evidence that it had ceased to
operate its power plants in that jurisdiction.
The NPC appealed the RTC Decision to the CA but the Province moved to dismiss the same for lack of jurisdiction of that
court over the subject matter of the case. The Province pointed out that, it was essentially a local tax case questioning the
validity of the Province's imposition of the local franchise tax. Any appeal from the action should, therefore, be lodged with the
CTA.
CA - granted the Province's motion and dismissed the petition on the ground cited.

Issue/s:
Whether or not the CA erred in failing to rule that the NPC no longer owned or operated the business subject to local franchise
tax and that the Province cannot execute on former NPC properties that had been taken from it and transferred to other
government corporations

Held:
The NPC failed to present evidence that it no longer owned or operated the business subject to local franchise tax and that
the properties the Province levied on did not belong to it. But proving these things did not require the presentation of evidence
in this case since these events took place by operation of law, particularly the EPIRA.

The above created the TRANSCO and transferred to it the NPC's electrical transmission function with effect on June 26, 2001.
The NPC, therefore, ceased to operate that business in Bataan by operation of law. Since the local franchise tax is imposed on
the privilege of operating a franchise, not a tax on the ownership of the transmission facilities, it is clear that such tax is not a
liability of the NPC.

Section 49 created the PSALM Corp. and transferred to it all of the NPC's "generation assets" which would include the Bataan
Thermal Plant. Clearly, the NPC had ceased running its former power transmission and distribution business in Bataan by
operation of law. It is, therefore, not the proper party subject to the local franchise tax for operating that business.
Consequently, such tax is collectible solely from PSALM Corp.

An indispensable party is one who has an interest in the controversy or subject matter and in whose absence there cannot be
a determination between the parties already before the court which is effective, complete or equitable. Here, since the subject
properties belong to PSALM Corp. and TRANSCO, they are certainly indispensable parties to the case that must be
necessarily included before it may properly go forward. For this reason, the proceedings below that held the NPC liable for the
local franchise tax is a nullity. It did not matter where the RTC Decision was appealed, whether before the CA or the CTA.

24.Bulawan vs. FACTS:


Aquende 182819 22 The general rule with reference to On 1 March 1995, Bulawan filed a complaint for annulment of title, reconveyance and damages against Lourdes Yap (Yap)
June 2011- De gala the making of parties in a civil and the Register of Deeds before the trial court docketed as Civil Case No. 9040. Bulawan claimed that she is the owner of Lot
action requires, of course, the No. 1634-B of Psd-153847 covered by Transfer Certificate of Title (TCT) No. 13733 having bought the property from its
joinder of all necessary parties owners, brothers Santos and Francisco Yaptengco (Yaptengco brothers), who claimed to have inherited the property from Yap
where possible, and the joinder of Chin Cun. Bulawan alleged that Yap claimed ownership of the same property and caused the issuance of TCT No. 40292 in
all indispensable parties under Yap’s name.
any and all conditions, their In her Answer, Yap clarified that she asserts ownership of Lot No. 1634-A of Psd-187165, which she claimed is the controlling
presence being a sine qua non for subdivision survey for Lot No. 1634. Yap also mentioned that, in Civil Case No. 5064, the trial court already declared that Psd-
the exercise of judicial power. It is 153847 was simulated by the Yaptengco brothers and that their claim on Lot No. 1634-B was void. The trial court likewise
precisely "when an indispensable adjudged Yap Chin Cun as the rightful owner of Lot No. 1634-B. Yap also stated that Lot No. 1634-B was sold by Yap Chin
party is not before the court (that) Cun to the Aquende family.
the action should be dismissed." On 26 November 1996, the trial court ruled in favor of Bulawan.
The absence of an indispensable Yap appealed. On 20 July 2001, the Court of Appeals dismissed Yap’s appeal.
party renders all subsequent On 7 February 2002, the trial court’s 26 November 2006 Decision became final and executory per entry of judgment dated 20
actions of the court null and void July 2001. On 19 July 2002, the trial court issued a writ of execution.
for want of authority to act, not In a letter dated 24 July 2002, the Register of Deeds informed Aquende of the trial court’s writ of execution and required
only as to the absent parties but Aquende to produce TCT No. 40067 so that a memorandum of the lien may be annotated on the title. On 25 July 2002,
even as to those present. Aquende wrote a letter to the Register of Deeds questioning the trial court’s writ of execution against his property. Aquende
- a person who was not alleged that he was unaware of any litigation involving his property having received no summons or notice thereof, nor was he
impleaded in the complaint cannot aware of any adverse claim as no notice of lis pendens was inscribed on the title.
be bound by the decision On 2 August 2002, Aquende filed a Third Party Claim against the writ of execution because it affected his property and, not
rendered therein, for no man shall being a party in Civil Case No. 9040, he argued that he is not bound by the trial court’s 26 November 1996 Decision.
be affected by a proceeding in In a letter dated 5 August 2002, the Clerk of Court said that a Third Party Claim was not the proper remedy because the
which he is a stranger. sheriff did not levy upon or seize Aquende’s property. Moreover, the property was not in the sheriff’s possession and it was not
about to be sold by virtue of the writ of execution.
Aquende then filed a Notice of Appearance with Third Party Motion and prayed for the partial annulment of the trial court’s 26
November 1996 Decision, specifically the portion which ordered the cancellation of Psd-187165 as well as any other certificate
of title issued pursuant to Psd-187165. Aquende also filed a Supplemental Motion where he reiterated that he was not a party
in Civil Case No. 9040 and that since the action was in personam or quasi in rem, only the parties in the case are bound by the
decision.
In its 19 February 2003 Order, the trial court denied Aquende’s motions.
Aquende filed a petition for annulment of judgment before the Court of Appeals on the ground that Aquende is an
indispensable party and the trial court did not acquire jurisdiction over his person because he was not impleaded as a party in
the case. Aquende also pointed out that the trial court went beyond the jurisdiction conferred by the allegations on the
complaint because Bulawan did not pray for the cancellation of Psd-187165 and TCT No. 40067.
The Court of Appeals ruled in favor of Aquende.

ISSUE:
Whether or not Aquende is an indispensable party in the case

HELD:
Bulawan argues that Aquende was not an indispensable party in Civil Case No. 9040 because the lot Aquende claims
ownership of is different from the subject matter of the case. Bulawan clarifies that she claims ownership of Lot No. 1634-B of
Psd-153847, while Aquende claims ownership of Lot No. 1634-B of Psd-187165. Bulawan argues that even if Aquende will be
affected by the trial court’s 26 November 1996 Decision, this will not make him an indispensable party.
Contrary to Bulawan’s argument, it appears that Aquende’s Lot No. 1634-B of Psd-187165 and Bulawan’s Lot No. 1634-B of
Psd-153847 actually refer to the same Lot No. 1634-B originally owned by Yap Chin Cun. Both Aquende and Bulawan trace
their ownership of the property to Yap Chin Cun. Aquende maintains that he purchased the property from Yap Chin Cun, while
Bulawan claims to have purchased the property from the Yaptengco brothers, who alleged that they inherited the property from
Yap Chin Cun. However, as the Court of Appeals declared, the title of the Yaptengco brothers over Lot No. 1634-B of Psd-
153847 had already been cancelled and they were forever enjoined not to disturb the right of ownership and possession of
Yap Chin Cun.
Section 7, Rule 3 of the Rules of Court defines indispensable parties as parties in interest without whom no final determination
can be had of an action. An indispensable party is one whose interest will be affected by the court’s action in the litigation. As
such, they must be joined either as plaintiffs or as defendants. In Arcelona v. Court of Appeals, we said:
The general rule with reference to the making of parties in a civil action requires, of course, the joinder of all necessary parties
where possible, and the joinder of all indispensable parties under any and all conditions, their presence being a sine qua non
for the exercise of judicial power. It is precisely "when an indispensable party is not before the court (that) the action should be
dismissed." The absence of an indispensable party renders all subsequent actions of the court null and void for want of
authority to act, not only as to the absent parties but even as to those presentwhi1
During the proceedings before the trial court, the answers of Yap and the Register of Deeds should have prompted the trial
court to inquire further whether there were other indispensable parties who were not impleaded. The trial court should have
taken the initiative to implead Aquende as defendant or to order Bulawan to do so as mandated under Section 11, Rule 3 of
the Rules of Court. The burden to implead or to order the impleading of indispensable parties is placed on Bulawan and on the
trial court, respectively.
However, even if Aquende were not an indispensable party, he could still file a petition for annulment of judgment. We have
consistently held that a person need not be a party to the judgment sought to be annulled. What is essential is that he can
prove his allegation that the judgment was obtained by the use of fraud and collusion and that he would be adversely affected
thereby.
We agree with the Court of Appeals that Bulawan obtained a favorable judgment from the trial court by the use of fraud.
Bulawan prevented Aquende from presenting his case before the trial court and from protecting his title over his property. We
also agree with the Court of Appeals that the 26 November 1996 Decision adversely affected Aquende as he was deprived of
his property without due process.
Moreover, a person who was not impleaded in the complaint cannot be bound by the decision rendered therein, for no man
shall be affected by a proceeding in which he is a stranger. In National Housing Authority v. Evangelista, we said:
In this case, it is undisputed that respondent was never made a party to Civil Case No. Q-91-10071. It is basic that no man
shall be affected by any proceeding to which he is a stranger, and strangers to a case are not bound by judgment rendered by
the court. Yet, the assailed paragraph 3 of the trial court’s decision decreed that "(A)ny transfers, assignment, sale or
mortgage of whatever nature of the parcel of land subject of this case made by defendant Luisito Sarte or his/her agents or
assigns before or during the pendency of the instant case are hereby declared null and void, together with any transfer
certificates of title issued in connection with the aforesaid transactions by the Register of Deeds of Quezon City who is likewise
ordered to cancel or cause the cancellation of such TCTs." Respondent is adversely affected by such judgment, as he was the
subsequent purchaser of the subject property from Sarte, and title was already transferred to him. It will be the height of
inequity to allow respondent’s title to be nullified without being given the opportunity to present any evidence in support of his
ostensible ownership of the property. Much more, it is tantamount to a violation of the constitutional guarantee that no person
shall be deprived of property without due process of law. Clearly, the trial court’s judgment is void insofar as paragraph 3 of its
dispositive portion is concerned.
Likewise, Aquende was never made a party in Civil Case No. 9040. Yet, the trial court ordered the cancellation of Psd-187165
and any other certificate of title issued pursuant to Psd-187165, including Aquende’s TCT No. 40067. Aquende was adversely
affected by such judgment as his title was cancelled without giving him the opportunity to present his evidence to prove his
ownership of the property.
WHEREFORE, we DENY the petition.

25.Office of the City An indispensable party is one Facts: Respondents, Mario D. Ebio, claim that they are the absolute owners of a parcel of land in Parañaque City that was
Mayor of Paranaque whose interest in the controversy subject to the construction of an access road along Cut-cut Creek located in the said barangay. The city gov't advised all the
vs. Ebio 178411 23 is such that a final decree would affected residents to vacate the said area, respondents immediately registered their opposition thereto. However, several
June 2010- Lee necessarily affect his/her right, so officials from the barangay and the city planning office proceeded to cut 8 coconut trees planted on the said lot so respondents
that the court cannot proceed filed letter-complaints before the Regional Director of the Bureau of Lands, the Department of Interior and Local Government
without their presence. In and the Office of the Vice Mayor. Thus, the Sangguniang Barangay of Vitalez held a meeting along with the respondents 2
contrast, a necessary party is one times but no definite agreement was reached by and among the parties. So the City Administrator Noli Aldip sent a letter to the
whose presence in the respondents ordering them to vacate the area within the next thirty (30) days, or be physically evicted from the said property.
proceedings is necessary to Threatened of being evicted, respondents went to the RTC.
adjudicate the whole controversy
but whose interest is separable RTC- Respondents applied for a writ of preliminary injunction against petitioners and during the course of the proceedings,
such that a final decree can be respondents admitted before the trial court that they have a pending application for the issuance of a sales patent before the
made in their absence without Department of Environment and Natural Resources (DENR). Thus the RTC denied their petition for lack of merit and reasoned
affecting them. that respondents were not able to prove successfully that they have an established right to the property since they have not
instituted an action for confirmation of title and their application for sales patent has not yet been granted. Additionally, they
failed to implead the Republic of the Philippines, which is an indispensable party.

CA- After the respondents’ MR was denied by RTC, they elevated the case to the CA which in return ruled in favor of the
respondents stating that the respondents were able to acquire ownership over the property by extraordinary acquisitive
prescription. Thus the petitioners elevated the case to the SC

Issue: Whether the State is an indispensable party to respondents’ action for prohibitory injunction

Held: No. An indispensable party is one whose interest in the controversy is such that a final decree would necessarily affect
his/her right, so that the court cannot proceed without their presence. In contrast, a necessary party is one whose presence in
the proceedings is necessary to adjudicate the whole controversy but whose interest is separable such that a final decree can
be made in their absence without affecting them. In the instant case, the action for prohibition seeks to enjoin the city
government of Parañaque from proceeding with its implementation of the road construction project. The State is neither a
necessary nor an indispensable party to an action where no positive act shall be required from it or where no obligation shall
be imposed upon it, such as in the case at bar. Neither would it be an indispensable party if none of its properties shall be
divested nor any of its rights infringed.

26. Oposa v. Factoran The subject matter of the


101083 30 July 1993 complaint is of common and Facts: This petition bears upon the right of Filipinos to a balanced and healthful ecology which the petitioners dramatically
Espiritu general interest not just to associate with the twin concepts of "inter-generational responsibility" and "inter-generational justice." Specifically, whether the
several, but to all citizens of the said petitioners have a cause of action to "prevent the misappropriation or impairment" of Philippine rainforests and "arrest the
Philippines. We likewise declare unabated hemorrhage of the country's vital life support systems and continued rape of Mother Earth.
that the plaintiffs therein are
numerous and representative In 1990, the principal plaintiffs therein, now the principal petitioners, are all minors duly represented and joined by their
enough to ensure the full respective parents. Impleaded as an additional plaintiff is the Philippine Ecological Network, Inc. (PENI), a domestic, non-stock
protection of all concerned and non-profit corporation organized for the purpose of engaging in concerted action geared for the protection of our
interests. environment and natural resources. The original defendant was Fulgencio Factoran, Jr., then Secretary of the DENR. His
substitution in this petition by the new Secretary, Angel Alcala, was subsequently ordered upon proper motion by the
Hence, all the requisites for the petitioners. The complaint was instituted as a taxpayers' class suit and alleges that the plaintiffs "are all citizens of the Republic
filing of a valid class suit under of the Philippines, taxpayers, and entitled to the full benefit, use and enjoyment of the natural resource treasure that is the
Section 12, Rule 3 of the Revised country's virgin tropical forests." The same was filed for themselves and others who are equally concerned about the
Rules of Court are present. preservation of said resource but are "so numerous that it is impracticable to bring them all before the Court." The minors
further asseverate that they "represent their generation as well as generations yet unborn."

It is prayed for that judgment be rendered: “ordering defendant, his agents, representatives and other persons acting in his
behalf to — (1) Cancel all existing timber license agreements in the country; (2) Cease and desist from receiving, accepting,
processing, renewing or approving new timber license agreements.”

Plaintiffs further assert that the adverse and detrimental consequences of continued and deforestation are so capable of
unquestionable demonstration that the same may be submitted as a matter of judicial notice. Thereafter, the original
defendant, Secretary Factoran, Jr., filed a Motion to Dismiss the complaint based on two (2) grounds, namely: (1) the plaintiffs
have no cause of action against him and (2) the issue raised by the plaintiffs is a political question which properly pertains to
the legislative or executive branches of Government. The motion was granted.

Issue: Whether or not this is a class suit

Ruling: YES. The subject matter of the complaint is of common and general interest not just to several, but to all citizens of the
Philippines. Consequently, since the parties are so numerous, it becomes impracticable, if not totally impossible, to bring all of
them before the court. We likewise declare that the plaintiffs therein are numerous and representative enough to ensure the full
protection of all concerned interests. Hence, all the requisites for the filing of a valid class suit under Section 12, Rule 3 of the
Revised Rules of Court are present both in the said civil case and in the instant petition, the latter being but an incident to the
former. This case, however, has a special and novel element. Petitioners minors assert that they represent their generation as
well as generations yet unborn. We find no difficulty in ruling that they can, for themselves, for others of their generation and
for the succeeding generations, file a class suit. Their personality to sue in behalf of the succeeding generations can only be
based on the concept of intergenerational responsibility insofar as the right to a balanced and healthful ecology is concerned.
Such a right, as hereinafter expounded, considers the "rhythm and harmony of nature."

27. Newsweek v. IAC Defamatory matter which does FACTS: It appears that on March 5, 1981, private respondents, incorporated associations of sugarcane planters in Negros
L-63559 30 MAY 1986 not reveal the Identity of the Occidental claiming to have 8,500 members and several individual sugar planters, filed Civil Case No. 15812 in their own
Nakagawa person upon whom the imputation behalf and/or as a class suit in behalf of all sugarcane planters in the province of Negros Occidental, against petitioner and two
is cast, affords no ground of of petitioners' non-resident correspondents/reporters Fred Bruning and Barry Came.
action unless it be shown that the
readers of the libel could have The complaint alleged that petitioner and the other defendants committed libel against them by the publication of the article
Identified the personality of the "An Island of Fear" in the February 23, 1981 issue of petitioner's weekly news magazine Newsweek. The article supposedly
individual defamed. portrayed the island province of Negros Occidental as a place dominated by big landowners or sugarcane planters who not
only exploited the impoverished and underpaid sugarcane workers/laborers, but also brutalized and killed them with imprunity.

Complainants therein alleged that said article, taken as a whole, showed a deliberate and malicious use of falsehood, slanted
presentation and/or misrepresentation of facts intended to put them (sugarcane planters) in bad light, expose them to public
ridicule, discredit and humiliation here in the Philippines and abroad, and make them objects of hatred, contempt and hostility
of their agricultural workers and of the public in general. They prayed that defendants be ordered to pay them PlM as actual
and compensatory damages, and such amounts for moral, exemplary and corrective damages as the court may determine,
plus expenses of litigation, attorney's fees and costs of suit. A photo copy of the article was attached to the complaint.

ISSUE: 1. WON THE COMPLAINT FAILED TO STATE A CAUSE OF ACTION

2. IS THERE A CLASS SUIT?

HELD: 1. YES. In the case of Corpus vs. Cuaderno, Sr. this Court ruled that "in order to maintain a libel suit, it is essential that
the victim be identifiable, although it is not necessary that he be named. In an earlier case, this Court declared that"
defamatory matter which does not reveal the Identity of the person upon whom the imputation is cast, affords no ground of
action unless it be shown that the readers of the libel could have Identified the personality of the individual defamed."

2. NO. This principle has been recognized to be of vital importance, especially where a group or class of persons, as in the
case at bar, claim to have been defamed, for it is evident that the larger the collectivity, the more difficult it is for the individual
member to prove that the defamatory remarks apply to him.

In the case of Uy Tioco vs. Yang Shu Wen, this Court held as follows:

Defamatory remarks directed at a class or group of persons in general language only, are not actionable by individuals
composing the class or group unless the statements are sweeping; and it is very probable that even then no action would lie
where the body is composed of so large a number of persons that common sense would tell those to whom the publication
was made that there was room for persons connected with the body to pursue an upright and law abiding course and that it
would be unreasonable and absurd to condemn all because of the actions of a part.

It is evident from the above ruling that where the defamation is alleged to have been directed at a group or class, it is essential
that the statement must be so sweeping or all-embracing as to apply to every individual in that group or class, or sufficiently
specific so that each individual in the class or group can prove that the defamatory statement specifically pointed to him, so
that he can bring the action separately, if need be. We have here a case where each of the plaintiffs has a separate and
distinct reputation in the community. They do not have common or general interest in the subject matter of the controversy.

28. Mathay v. "Rule 3 SECTION 12. Class suit. Facts: The complaint in this case was a class suit, under Section 12, Rule 3, of the Rules of Court, contained six causes of
Consolidated Bank L- — When the subject matter of the action. Under the first cause of action, plaintiffs appellants alleged that they were, on or before March 28, 1962, stockholders in
23136 26 Aug 1974 controversy is one of common or the Consolidated Mines, Inc. (hereinafter referred to as CMI), a corporation duly organized and existing under Philippine laws;
Lee general interest to many persons, that the stockholders of the CMI, including the plaintiffs appellants, passed, at a regular stockholders' meeting, a Resolution
and the parties are so numerous providing: (a) that the Consolidated Bank & Trust Co. be organized with an authorized capital of P20,000,000.00. The
that it is impracticable to bring President and members of the Board of Directors of the CMI, who are the individuals-defendants-appellees in the instant case,
them all before the court, one or constituted themselves as the Interim Board of Organizers; that said Board sent out to the CMI stockholders, including the
more may sue or defend for the plaintiffs-appellants, circular letters with "Pre-Incorporation Agreement to Subscribe" forms that provided that the payment of
benefit of all. But in such case the the subscription should be made in cash from time to time or by the application of the special dividend declared by the CMI.
court shall make sure that the The Board of Organizers caused the execution of the Articles or Incorporation of the proposed Bank indicating an 50 original
parties actually before it are subscription of 50,000 shares worth P5,000,000 subscribed and paid only by six of the individuals-defendants-appellees,
sufficiently numerous and namely, Antonio P. Madrigal, Jose P. Madrigal Simon R. Paterno, Fermin Z. Caram, Jr., Claudio Teehankee, and Wilfredo C.
representative so that all interests Tecson, thereby excluding the plaintiffs-appellants and the other CMI subscribing stockholders who had already subscribed;
concerned are fully protected. Any that the execution of said Articles of Incorporation was "in violation of law and in breach of trust and contractual agreement as
party in interest shall have a right a means to gain control of Defendant Bank by Defendant Individuals and persons or entities chosen by them and for their
to intervene in protection of his personal profit or gain in disregard of the rights of Plaintiffs and other CMI Subscribing Stockholders;" The Defendants-
individual interest." appellees, except Fermin Z. Caram, Jr., filed a motion to dismiss on the grounds that (a) plaintiffs appellants had no legal
The necessary elements for the standing or capacity to institute the alleged class suit; (b) that the complaint did not state a sufficient and valid cause of action;
maintenance of a class suit are and (c) that plaintiffs-appellants' complaint against the increase of the number of directors did not likewise state a cause of
accordingly: (1) that the subject action. In its order, the trial court granted the motion to dismiss, holding, among other things, that the class suit could not be
matter of the controversy be one maintained because of the absence of a showing in the complaint that the plaintiffs-appellants were sufficiently numerous and
of common or general interest to representative, and that the complaint failed to state a cause of action.
many persons, and (2) that such
persons be so numerous as to Issue: Whether the instant action could be maintained as a class suit
make it impracticable to bring
them all to the court.Whether the Held: No. The governing statutory provision for the maintenance of a class suit is Section 12 of Rule 3 of the Rules of Court:
suit is or is not a class suit
depends upon the existence of a "SECTION 12. Class suit. — When the subject matter of the controversy is one of common or general interest to many
subject matter of common persons, and the parties are so numerous that it is impracticable to bring them all before the court, one or more may sue or
interest, and the existence of a defend for the benefit of all. But in such case the court shall make sure that the parties actually before it are sufficiently
class and the number of persons numerous and representative so that all interests concerned are fully protected. Any party in interest shall have a right to
in the alleged class intervene in protection of his individual interest."

The necessary elements for the maintenance of a class suit are accordingly: (1) that the subject matter of the controversy be
one of common or general interest to many persons, and (2) that such persons be so numerous as to make it impracticable to
bring them all to the court. Whether the suit is or is not a class suit depends upon the existence of a subject matter of common
interest, and the existence of a class and the number of persons in the alleged class

In the given case, the interest that appellants, plaintiffs and intervenors, and the CMI stockholders had in the subject matter of
this suit — the portion of stocks offering of the Bank left unsubscribed by CMI stockholders who failed to exercise their right to
subscribe on or before January 15, 1963 — was several, not common or general in the sense required by the statute. Each
one of the appellants and the CMI stockholders had determinable interest; each one had a right, if any, only to his respective
portion of the stocks. No one of them had any right to, or any interest in, the stock to which another was entitled. Even if it be
assumed, for the sake of argument, that the appellants and the CMI stockholders suffered wrongs that had been committed by
similar means and even pursuant to a single plan of the Interim Board of Organizers of the Bank, the wrong suffered by each
of them would constitute a wrong separate from those suffered by the other stockholders, and those wrongs alone would not
create that common or general interest in the subject matter of the controversy as would entitle any one of them to bring a
class suit on behalf of the others. Anent this point it has been said that: "Separate wrongs to separate persons, although
committed by similar means and even pursuant to a single plan, do not alone create a 'common' or 'general' interest in those
who are wronged so as to entitle them to maintain a representative action."

29. Chiang Kai Shek


School v. CA 58028 18 As the school itself may be sued FACTS:
April 1989 in its own name, there is no need
Nato to apply Rule 3, Section 15, under Petitioner Chiang Kai Shek College is a private educational institution that offers elementary to college education to the
which the persons joined in an public. Individual petitioner Carmelita Espino is the Vice–President of the school. Respondent had been employed as a grade
association without any juridical school teacher of the school from July 1970 until 31 May 2003. The manner of her severance from employment is the matter
personality may be sued with at hand.
such association. Besides, it has
been shown that the individual Respondent was accused of leaking a copy of a special quiz given to Grade 5 students of HEKASI (HEKASI 5). HEKASI
members of the board of trustees stands for Heograpiya, Kasaysayan at Sibika (Geography, History and Civics). Petitioners came to know about the leakage
are not liable, having been from one of the teachers of HEKASI 5, Aileen Benabese (Ms. Benabese). Ms. Benabese narrated that after giving a special
appointed only after the private quiz, she borrowed the book of one of her students, Aileen Regine M. Anduyan (Aileen), for the purpose of making an answer
respondent's dismissal. key. When she opened Aileen’s book, a piece of paper fell. Said paper turned out to be a copy of the same quiz she had just
given and the same already contained answers.

Ms. Benabese informed the school’s Assistant Supervisor Mrs. Gloria Caneda (Mrs. Caneda) about the incident. Mrs. Caneda
conferred with Assistant Supervisor Encarnacion Koo (Mrs. Koo), who was in charge of the HEKASI area, and Supervisor
Luningning Tibi (Ms. Tibi). Mrs. Koo confronted respondent, who had initially denied leaking the test paper but later on
admitted that she gave the test paper to Mrs. Teresita Anduyan (Mrs. Anduyan), her co–teacher and the mother of
Aileen. Respondent and Mrs. Anduyan were both directed to submit their written statement on the incident.

Respondent explained that she was busy checking the writing workbook when somebody handed her the special quiz for
HEKASI 5, thus:

Yesterday morning, before the bell rings, I was busy checking the writing workbook when somebody handed me the special
quiz for Hekasi 5. I placed them on the table and continued with what I’m doing. Mrs. Anduyan got one paper and read
it. When I finished checking the books I got the papers and went upstairs forgetting about the paper Mrs. Anduyan got.

Then, this morning (July 30), Mrs. Koo confronted me about the two answered test papers of Aileen Anduyan, I knew one of
them was the paper Mrs. Anduyan borrowed from me. I admitted it to Mrs. Koo and I was so sorry and apolog[e]tic for any
carelessness and for what happened.

ISSUE:

1.Whether or not a school that has not been incorporated may be sued by reason alone of its long continued existence and
recognition by the government.

2.Whether or not a complaint filed against persons associated under a common name will justify a judgment against the
association itself and not its individual members.

RULING:

1. As a school, the petitioner was governed by Act No. 2706 as amended by C.A. No. 180, which provided as follows: Unless
exempted for special reasons by the Secretary of Public Instruction, any private school or college recognized by the
government shall be incorporated under the provisions of Act No. 1459 known as the Corporation Law, within 90 days after the
date of recognition, and shall file with the Secretary of Public Instruction a copy of its incorporation papers and by-laws. Having
been recognized by the government, it was under obligation to incorporate under the Corporation Law within 90 days from
such recognition. It appears that it had not done so at the time the complaint was filed notwithstanding that it had been in
existence even earlier than 1932. The petitioner cannot now invoke its own non-compliance with the law to immunize it from
the private respondent’s complaint. There should also be no question that having contracted with the private respondent every
year for thirty-two years and thus represented itself as possessed of juridical personality to do so, the petitioner is now
estopped from denying such personality to defeat her claim against it. According to Article 1431 of the Civil Code, “through
estoppel an admission or representation is rendered conclusive upon the person making it and cannot be denied or disproved
as against the person relying on it.” As the school itself may be sued in its own name, there is no need to apply Rule 3, Section
15, under which the persons joined in an association without any juridical personality may be sued with such association.
Besides, it has been shown that the individual members of the board of trustees are not liable, having been appointed only
after the private respondent’s dismissal.

2. There should also be no question that having contracted with the private respondent every year for thirty-two years and thus
represented itself as possessed of juridical personality to do so, the petitioner is now estopped from denying such personality
to defeat her claim against it. According to Article 1431 of the Civil Code, "through estoppel an admission or representation is
rendered conclusive upon the person making it and cannot be denied or disproved as against the person relying on it."

As the school itself may be sued in its own name, there is no need to apply Rule 3, Section 15, under which the persons joined
in an association without any juridical personality may be sued with such association. Besides, it has been shown that the
individual members of the board of trustees are not liable, having been appointed only after the private respondent's dismissal.

It is clear now that a charitable institution is covered by the labor laws although the question was still unsettled when this case
arose in 1968.At any rate, there was no law even then exempting such institutions from the operation of the labor laws
(although they were exempted by the Constitution from ad valorem taxes).Hence, even assuming that the petitioner was a
charitable institution as it claims, the private respondent was nonetheless still entitled to the protection of the Termination Pay
Law, which was then in force.

While it may be that the petitioner was engaged in charitable works, it would not necessarily follow that those in its employ
were as generously motivated. Obviously, most of them would not have the means for such charity. The private respondent
herself was only a humble school teacher receiving a meager salary of P180.00 per month.

At that, it has not been established that the petitioner is a charitable institution, considering especially that it charges tuition
fees and collects book rentals from its students. While this alone may not indicate that it is profit-making, it does weaken its
claim that it is a non-profit entity.

30. Sarsaba v. Fe Vda. The trial court’s jurisdiction Facts:


De Te 175910 30 Jul subsists despite the death of a
2009 party. The NLRC rendered a Decision regarding the illegal dismissal of Patricio Sereno. Teodoro Gasing was ordered by the NLRC
perez to pay Sereno’s money claims. The Writ of Execution was returned unsatisfied so LA Sancho issued an Alias Writ of Execution
to satisfy the judgment award. Sheriff Lavarez, Sereno, and Atty. Sarsaba levied a Fuso Truck with license plate no. LBR-514
that was in the possession of Gasing and was later sold at public auction to Sereno.Fe Vda. de Te, represented by her
attorney-in-fact, Faustino Castañeda filed for recovery of motor vehicle, damages with prayer for the delivery of the truck
pendente lite against Sarsaba, Sereno, Lavarez and NLRC. Sarsaba and the NLRC filed a Motion to Dismiss that was later
denied by the RTC while Lavarez filed an Answer with Compulsory Counterclaim and Third-Party Complaint.On Oct. 17, 2005,
Sarsaba filed an Omnibus Motion to Dismiss the case for lack of jurisdiction over one of the principal defendants and to
discharge Vda de. Te’s attorney-in-fact for lack of legal personality to sue.

Fe Vda. de Te died on Apr. 12, 2005. Her lawyer filed an Opposition with the contention that failure to serve summons on
Sereno was not a ground for dismissing the complaint because the other defendants have already submitted their respective
responsive pleadings. Moreover, Vda. de Te’s death did not render functus officio her right to sue as her attorney-in-fact had
long testified on the complaint and had submitted documentary exhibits in support of the complaint.The Omnibus Motion to
Dismiss was denied by the RTC.

Issues:

1) W/N the death of one of the principal defendants (Sereno) warrants a dismissal of the case as regards the other
defendants;
2) W/N the death of Vda de. Te warrants the outright dismissal of the case.

Held:

1. NO

The Court’s failure to acquire jurisdiction over one’s person is a defense which is personal to the person claiming it. Sarsaba
cannot invoke such ground, on behalf of Sereno, so as to reap the benefit of having the case dismissed against all the
defendants. At the same time, Sarsaba only raised the issue of lack of jurisdiction over the person of Sereno in his Omnibus
Motion to Dismiss. He was unable to raise the same in his Motion to Dismiss or in his Answer. Thus having failed to invoke this
ground at the proper time he cannot raise it now for the first time on appeal.Failure to serve summons on Sereno's person will
not be a cause for the dismissal of the complaint against the other defendants, considering that they have been served with
copies of the summons and complaints and have long submitted their respective responsive pleadings.Only the case against
Sereno will be dismissed and the same may be filed as a claim against the estate of Sereno but the case with respect to the
three others will proceed.

2. NO

When a party to a pending action dies and the claim is not extinguished, the Rules of Court require a substitution of the
deceased. The rule on substitution by heirs is not a matter of jurisdiction, but a requirement of due process.Moreover, non-
compliance with the Rules results in the denial of the right to due process for the heirs who, though not duly notified of the
proceedings, would be substantially affected by the decision rendered therein. Thus, it is only when there is a denial of due
process, as when the deceased is not represented by any legal representative or heir, that the court nullifies the trial
proceedings and the resulting judgment therein.

The proper remedy in this case is the Substitution of Heirs.


SEC. 16, RULE 3 provides for the substitution of the plaintiff who dies pending hearing of the case by his/her legal heirs. As to
whether or not the heirs will still continue to engage the services of the Attorney-in-fact is another matter, which lies within the
sole discretion of the heirs

31. Judge Sumalijag v. “Section 16. Death of a party; duty Facts: Josefa D. Maglasang was the sister of Menendez Maglasang Literato ("Menendez"). They were two (2) of the six (6)
Sps Diosdidit 149787 of counsel. — Whenever a party heirs who inherited equal parts of a 6.3906-hectare property (Lot 1220) passed on to them by their parents Cristito and Inecita
18 Jun 2008 to a pending action dies, and the Diano Maglasang. Lot 1220-D was partitioned to Josefa, while Lot 1220-E was given to Menendez.
Lee claim is not thereby extinguished, Case No. 1239
it shall be the duty of his counsel Josefa filed with the RTC a complaint for the nullity of the deed of sale of real property purportedly executed between her as
to inform the court within thirty vendor and the spouses Diosdidit and Menendez Literato (the "respondent spouses" ) as vendees. The complaint alleged that
(30) days after such death of the this deed of sale of Lot 1220-D is spurious. The respondent spouses filed an answer with counterclaim denying that the deed
fact thereof, and to give the name of sale was falsified. They impleaded the petitioner with Josefa as counterclaim defendant on the allegation that the petitioner,
and address of his legal at the instance of Josefa, occupied Lot 1220-D and Lot 1220-E without their (the respondent spouses') authority
representative or representatives. Case No. 1281
Failure of counsel to comply with Menendez filed a complaint with the RTC for the declaration of the inexistence of lease contract, recovery of possession of
this duty shall be a ground for land, and damages against the petitioner and Josefa after the RTC dismissed the respondent spouses' counterclaim in Civil
disciplinary action. Case No. 1239.

The heirs of the deceased may be Josefa died on May 3, 1999 during the pendency of Civil Case Nos. B-1239 and B-1281. Atty. Puray filed with the RTC a
allowed to be substituted for the notice of death and substitution of party, praying that Josefa — in his capacity as plaintiff and third party counterclaim
deceased, without requiring the defendant — be substituted by the petitioner. The submission alleged that prior to Josefa's death, she executed a Quitclaim
appointment of an executor or Deed over Lot 1220-D in favor of Remismundo D. Maglasang who in turn sold this property to the petitioner. Menendez,
administrator and the court may through counsel, objected to the proposed substitution, alleging that Atty. Puray filed the notice of death and substitution of
appoint a guardian ad litem for the party beyond the thirty-day period provided under Section 16, Rule 3 of the 1997 Rules of Civil Procedure, as amended. She
minor heirs. recommended instead that Josefa be substituted by the latter's full-blood sister, Michaeles Maglasang Rodrigo ("Michaeles").
The RTC denied Atty. Puray's motion for substitution and instead ordered the appearance of Michaeles as representative of
The court shall forthwith order the deceased Josefa and also denied the petitioner’s MR.
said legal representative or CA - The petitioner went to the CA on a petition for certiorari to question the above interlocutory orders. The CA dismissed the
representatives to appear and be petition for lack of merit. The appellate court similarly denied the petitioner's motion for reconsideration.
substituted within a period of thirty
(30) days from notice. Issue: Whether Josefa may be substituted by Atty. Puray

If no legal representative is Held: No. The rule on substitution in case of death of a party is governed by Section 16, Rule 3 of the 1997 Rules of Civil
named by the counsel for the Procedure, as amended, which provides:
deceased party, or if the one so
named shall fail to appear within Section 16. Death of a party; duty of counsel. — Whenever a party to a pending action dies, and the claim is not thereby
the specified period, the court extinguished, it shall be the duty of his counsel to inform the court within thirty (30) days after such death of the fact thereof,
may order the opposing party, and to give the name and address of his legal representative or representatives. Failure of counsel to comply with this duty
within a specified time, to procure shall be a ground for disciplinary action.
the appointment of an executor or
administrator for the estate of the The heirs of the deceased may be allowed to be substituted for the deceased, without requiring the appointment of an executor
deceased, and the latter shall or administrator and the court may appoint a guardian ad litem for the minor heirs.
immediately appear for and on
behalf of the deceased. The court
charges in procuring such The court shall forthwith order said legal representative or representatives to appear and be substituted within a period of thirty
appointment, if defrayed by the (30) days from notice.
opposing party, may be recovered
as costs.” If no legal representative is named by the counsel for the deceased party, or if the one so named shall fail to appear within the
specified period, the court may order the opposing party, within a specified time, to procure the appointment of an executor or
Application: administrator for the estate of the deceased, and the latter shall immediately appear for and on behalf of the deceased. The
a. Survival of Pending court charges in procuring such appointment, if defrayed by the opposing party, may be recovered as costs.
action
In the causes of action which Application of the Governing Rule
survive, the wrong complained [of] a. Survival of the pending action
affects primarily and principally A question preliminary to the application of the above provision is whether Civil Case Nos. B-1239 and B-1281 are actions that
property and property rights, the survive the death of Josefa. In the case of Bonilla v. Barcena, we held that the question as to whether an action survives or not
injuries to the person being depends on the nature of the action and the damage sued for. In the causes of action which survive, the wrong complained [of]
merely incidental, while in the affects primarily and principally property and property rights, the injuries to the person being merely incidental, while in the
causes of action which do not causes of action which do not survive, the injury complained of is to the person, the property and rights of property affected
survive, the injury complained of being incidental. Since the question involved in these cases relate to property and property rights, then we are dealing with
is to the person, the property and actions that survive so that Section 16, Rule 3 must necessarily apply.
rights of property affected being
incidental.
b. Duty of Counsel Under b. Duty of Counsel under the Rule.
the Rule The duty of counsel is to inform the court within thirty (30) days after the death of his client of the fact of death, and to give the
The duty of counsel is to inform name and address of the deceased's legal representative or representatives. Incidentally, this is the only representation that
the court within thirty (30) days counsel can undertake after the death of a client as the fact of death terminated any further lawyer-client relationship. In the
after the death of his client of the present case, it is undisputed that the counsel for Josefa did in fact notify the lower court, although belatedly, of the fact of her
fact of death, and to give the death. This notification, although filed late, effectively informed the lower court of the death of litigant Josefa Maglasang so as
name and address of the to free her counsel of any liability for failure to make a report of death under Section 16, Rule 3 of the Rules of Court.
deceased's legal representative or
representatives.
c. Heirs as legal The reporting issue that goes into the core of this case is whether counsel properly gave the court the name and address of
representatives the legal representative of the deceased that Section 16, Rule 3 specifies. We rule that he did not. The "legal representatives"
The heirs of the deceased may be that the provision speaks of, refer to those authorized by law — the administrator, executor or guardian who, under the rule on
allowed to be substituted for the settlement of estate of deceased persons, is constituted to take over the estate of the deceased. Section 16, Rule 3 likewise
deceased, without requiring the expressly provides that "the heirs of the deceased may be allowed to be substituted for the deceased, without requiring the
appointment of an executor or appointment of an executor or administrator . . .". Significantly, the person — now the present petitioner — that counsel gave
administrator as substitute was not one of those mentioned under Section 16, Rule 3. Rather, he is a counterclaim co-defendant of the
deceased whose proferred * justification for the requested substitution is the transfer to him of the interests of the deceased in
the litigation prior to her death.

c. The Heirs as legal representatives


In determining who the appropriate legal representative/s should be in the absence of an executor or administrator, the second
paragraph of the Section 16, Rule 3 of the 1997 Rules of Court, as amended, is clear — the heirs of the deceased may be
allowed to be substituted for the deceased, without requiring the appointment of an executor or administrator. Josefa's death
certificate shows that she was single at the time of her death. The records do not show that she left a will. Therefore, as
correctly held by the CA, in applying Section 16, Rule 3, her heirs are her surviving sisters (Michaelis, Maria, Zosima, and
Consolacion) and the children of her deceased sister, Lourdes (Manuel, Cesar, Huros and Regulo) who should be her legal
representatives. Menendez, although also a sister, should be excluded for being one of the adverse parties in the cases before
the RTC.

32. Dela Cruz v. WHEN A PARTY TO A PENDING Facts:


Joaquin 162788 28 July ACTION DIES AND THE CLAIM The case originated from a Complaint for the recovery of possession and ownership, the cancellation of title, and damages,
2005 IS NOT EXTINGUISHED, filed by Pedro Joaquin against petitioner spouses Felipe and Julita Dela Cruz in the Regional Trial Court of Baloc, Sto.
Purificacion SUBSTITUTION OF THE Domingo, Nueva Ecija. Respondent Joaquin alleged that he had obtained a loan from them in the amount of P9,000 on June
DECEASED IS REQUIRED; 29, 1974, payable after five (5) years; that is, on June 29, 1979. To secure the payment of the obligation, he supposedly
PURPOSE. — When a party to a executed a Deed of Sale in favor of the petitioner spouses. The Deed was for a parcel of land in Pinagpanaan, Talavera,
pending action dies and the claim Nueva Ecija. The parties also executed another document entitled "Kasunduan."
is not extinguished, the Rules of
Court require a substitution of the Respondent Joaquin - the Kasunduan showed the Deed of Sale to be actually an equitable mortgage
deceased. The procedure is
specifically governed by Section Spouses De la Cruz - this document was merely an accommodation to allow the repurchase of the property until June 29,
16 of Rule 3, . . . . The rule on the 1979, a right that he failed to exercise.
substitution of parties was crafted
to protect every party's right to RTC - issued a Decision in his (respondent Joaquin’s) favor. The trial court declared that the parties had entered into a sale
due process. The estate of the with a right of repurchase. It further held that respondent Joaquin had made a valid tender of payment on two separate
deceased party will continue to be occasions to exercise his right of repurchase. Accordingly, petitioner spouses were required to reconvey the property upon his
properly represented in the suit payment.
through the duly appointed legal
representative. Moreover, no CA - sustained RTC ruling. denied reconsideration and ordered a substitution by legal representatives, in view of respondent's
adjudication can be made against death on December 24, 1988.
the successor of the deceased if
the fundamental right to a day in
court is denied. Petitioner spouses - RTC's Decision was invalid for lack of jurisdiction. They claim that respondent Joaquin died during the
pendency of the case. There being no substitution by the heirs, the trial court allegedly lacked jurisdiction over the litigation.
A FORMAL SUBSTITUTION BY
HEIRS IS NOT NECESSARY Issue: Whether the decision of the trial court should be nullified on the ground that there is a violation of due process on the
WHEN THEY THEMSELVES issue of substitution. - No.
VOLUNTARILY APPEAR,
PARTICIPATE IN THE CASE, Ruling:
AND PRESENT EVIDENCE IN
DEFENSE OF THE DECEASED. Rule on Substitution
— The Court has nullified not only When a party to a pending action dies and the claim is not extinguished, the Rules of Court require a substitution of
trial proceedings conducted the deceased. The procedure is specifically governed by Section 16 of Rule 3.
without the appearance of the
legal representatives of the The rule on the substitution of parties was crafted to protect every party's right to due process. The estate of the
deceased, but also the resulting deceased party will continue to be properly represented in the suit through the duly appointed legal representative. Moreover,
judgments. In those instances, the no adjudication can be made against the successor of the deceased if the fundamental right to a day in court is denied.
courts acquired no jurisdiction
over the persons of the legal The Court has nullified not only trial proceedings conducted without the appearance of the legal representatives of the
deceased, but also the resulting judgments. In those instances, the courts acquired no jurisdiction over the persons of the legal
representatives or the heirs upon
representatives or the heirs upon whom no judgment was binding.
whom no judgment was binding.
This general rule notwithstanding, This general rule notwithstanding, a formal substitution by heirs is not necessary when they themselves
a formal substitution by heirs is voluntarily appear, participate in the case, and present evidence in defense of the deceased. These actions negate any
not necessary when they claim that the right to due process was violated.
themselves voluntarily appear,
participate in the case, and Strictly speaking, the rule on the substitution by heirs is not a matter of jurisdiction, but a requirement of due
present evidence in defense of process. Thus, when due process is not violated, as when the right of the representative or heir is recognized and
the deceased. These actions protected, noncompliance or belated formal compliance with the Rules cannot affect the validity of a promulgated
negate any claim that the right to decision. Mere failure to substitute for a deceased plaintiff is not a sufficient ground to nullify a trial court's decision.
due process was violated. The alleging party must prove that there was an undeniable violation of due process. TECIaH

RULE ON SUBSTITUTION BY Substitution in the Instant Case


HEIRS IS NOT A MATTER OF The records of the present case contain a "Motion for Substitution of Party Plaintiff" dated February 15, 2002, filed
JURISDICTION BUT A before the CA.
REQUIREMENT OF DUE
PROCESS. — Strictly speaking, Evidently, the heirs of Pedro Joaquin voluntarily appeared and participated in the case. We stress that the
the rule on the substitution by appellate court had ordered his legal representatives to appear and substitute for him. The substitution even on appeal had
heirs is not a matter of jurisdiction, been ordered correctly. In all proceedings, the legal representatives must appear to protect the interests of the deceased. After
but a requirement of due process. the rendition of judgment, further proceedings may be held, such as a motion for reconsideration or a new trial, an appeal, or
Thus, when due process is not an execution.
violated, as when the right of
the representative or heir is Considering the foregoing circumstances, the Motion for Substitution may be deemed to have been granted; and
recognized and protected, the heirs, to have substituted for the deceased, Pedro Joaquin. There being no violation of due process, the issue of
noncompliance or belated substitution cannot be upheld as a ground to nullify the trial court's Decision.
formal compliance with the
Rules cannot affect the validity
of a promulgated decision.
Mere failure to substitute for a
deceased plaintiff is not a
sufficient ground to nullify a
trial court's decision. The
alleging party must prove that
there was an undeniable violation
of due process.
The Rules require the legal
representatives of a dead litigant
to be substituted as parties to a
litigation. This requirement is
necessitated by due process.
Thus, when the rights of the legal
representatives of a decedent are
actually recognized and
protected, noncompliance or
belated formal compliance with
the Rules cannot affect the
validity of the promulgated
decision. After all, due process
had thereby been satisfied.

33. O. Ventanilla It is the party’s duty to inform FACTS:


Enterprises v. Tan the court of its counsel’s Petitioner Ventanilla leased out two of its properties to Alfredo Tan and respondent Adelina Tan. Due to the failure
180325 20 Feb 2013 demise, and failure to apprise of the Tans to comply with the terms of the lease, petitioner Ventanilla filed a complaint against them for
Sabio the court of such fact shall be cancellation and termination of contract of lease with the RTC of Cabanatuan. The RTC rendered a decision in
considered negligence on the favor of petitioner Ventanilla.
part of said party.
Respondent Tan appealed from the RTC decision. However, petitioner Ventanilla filed a motion for execution
pending appeal and the same was granted by the trial court. Several properties and bank accounts of respondent
Alfredo Tan were levied upon. The Tans decided to pay the amounts ordered by the RTC and the court issued
orders lifting and cancelling the Notice of Levy on respondent Adelina Tan’s properties and also on several bank
accounts in the name of the Tans.
The appeal filed by Alfredo Tan was dismissed by the CA, but the appeal filed by respondent Adelina Tan
proceeded. Respondent Adelina Tan then filed with trial court a Motion for Execution, praying that the excess
amounts she previously paid as exemplary damages, attorney’s fees and liquidated damages be refunded to her.
This was granted by the RTC.

Petitioner filed a motion praying that the appeal of Adelina Tan be dismissed or the appeal be reopened to allow
petitioner to file an appeal brief. Petitioner argued that its counsel died during the pendency of the case in the CA,
hence, any notice sent to him must be deemed ineffective; that the parties have arrived at a settlement of the cae,
as shown by the fact respondent already paid the petitioner, and thus, the appeal should have been deemed
mooted.

ISSUE:
Whether or not petitioner Ventanilla’s contention is correct.

HELD:
NO. The court strikes down the argument that the CA decision did not attain finality because petitioner’s counsel,
who died while the case was pending before the CA, was unable to receive the copy thereof. The CA was correct
in ruling that there is no extraordinary circumstance in this case that would merit a recall of the entry judgement to
reopen the case. It is the party’s duty to inform the court of its counsel’s demise, and failure to apprise the court of
such fact shall be considered negligence on the part of said party.

Thus, for failure of petitioner to notify the CA of the death of its counsel of record and have said counsel
substituted, then service of the CA Decision at the place or law office designated by its counsel of record as his
address, is sufficient notice. The case then became final and executory when no motion for reconsideration or
appeal was filed within the reglementary period therefor.

34. Torres v. Rodellas Section 16. Death of party; Facts: Respondent Balligi V. Rodellas (Balligi) occupied a property in Occidental Mindoro in 1967, built a
177836 04 Sept 2009 duty of counsel. — Whenever residential house thereon and subsequently applied for a Miscellaneous Sales Application (MSA) with the DENR
Lee a party to a pending action for the property. She and her family left for Manila in 1989 and eventually went to Saudi Arabia. In the same ear,
dies, and the claim is not Edwino moved into Balligi’s house, occupying the portion vacated by Balligi’s sister, claiming that Balligi sold him
thereby extinguished, it shall the house and property. Edwino also filed an MSA in his name for the same property with the DENR. Balligi’s MSA
be the duty of his counsel to application was then rejected in favor of Edwino’s application. When Balligi’s son Eugenio returned and learned of
inform the court within thirty Edwino’s claim he filed a Protest with the Community Environment and Natural Resources Office (CENRO) against
(30) days after such death of Edwino’s MSA on the ground that the Affidavit of Relinquishment was forged, as Balligi was in Saudi Arabia at the
the fact thereof, and to give the time.
name and address of his legal
representative or Issue: Whether CA erred in dismissing the Petition for review of Edwino’s legal hairs
representatives. Failure of
counsel to comply with his duty Held: No. Sec. 16, Rule 3 of the Revised Rules of Court provides:
shall be a ground for Section 16. Death of party; duty of counsel. — Whenever a party to a pending action dies, and the claim is not
disciplinary action. thereby extinguished, it shall be the duty of his counsel to inform the court within thirty (30) days after such death of
the fact thereof, and to give the name and address of his legal representative or representatives. Failure of counsel
The heirs of the deceased may to comply with his duty shall be a ground for disciplinary action.
be allowed to be substituted The heirs of the deceased may be allowed to be substituted for the deceased, without requiring the appointment of
for the deceased, without an executor or administrator and the court may appoint a guardian ad litem for the minor heirs.
requiring the appointment of The court shall forthwith order said legal representative or representatives to appear and be substituted within a
an executor or administrator period of thirty (30) days from notice.
and the court may appoint a If no legal representative is named by the counsel for the deceased party, or if the one so named shall fail to
guardian ad litem for the minor appear within the specified period, the court may order the opposing party, within a specified time, to procure the
heirs. appointment of an executor or administrator for the estate of the deceased and the latter shall immediately appear
for and on behalf of the deceased. The court charges in procuring such appointment, if defrayed by the opposing
The court shall forthwith order party, may be recovered as costs. Clear from the aforequoted provision that a deceased party may be substituted
said legal representative or by his heirs, but it must be emphasized that substitution may only be allowed in actions that survive the death of a
representatives to appear and party thereto.
be substituted within a period
of thirty (30) days from notice. In the case at bar, both parties accuse the other of unlawfully depriving them of their respective rights to acquire
the subject property, together with the house built thereon, by means of an MSA grant from the State. Evidently,
If no legal representative is what are primarily and principally affected herein are the property and property rights of the parties, and any
named by the counsel for the injuries to their persons (i.e., damages) are only incidental. Such property and property rights survived Edwino's
deceased party, or if the one death and may pass on by succession to his heirs. Therefore, the heirs must be allowed to continue any litigation to
so named shall fail to appear protect said property or property rights and to substitute themselves for the deceased party in accordance with
within the specified period, the appropriate rules.
court may order the opposing
party, within a specified time, Since Atty. Restor filed the Motion for Reconsideration within the reglementary period and no longer requested for
to procure the appointment of suspension/extension of time to do so, the Office of the President need not suspend the running of said
an executor or administrator reglementary period but it could have deferred any action on said Motion until a substitution had been effected and
for the estate of the deceased it had ascertained that the substituted heirs chose to retain Atty. Restor's services as legal counsel. Conspicuously,
and the latter shall immediately the Office of the President completely failed to act on the information that Edwino had died so as to effect proper
appear for and on behalf of the substitution by the latter's heirs, as set forth in Section 16, Rule 3 of the Revised Rules of Court. The only action
deceased. The court charges the Office of the President took as regards said information was to deny the Motion for Reconsideration filed by
in procuring such appointment, Atty. Restor for his lack of personality, given his client's death. This we find totally contrary to equity and fair play
if defrayed by the opposing since Edwino's heirs were, in effect, deprived of their right to seek reconsideration or appeal of the adverse
party, may be recovered as decision of the Office of the President which was itself partly responsible for their non-substitution. We emphasize
costs. Clear from the that the purpose behind Section 16, Rule 3 of the Revised Rules of Procedure is the protection of the right to due
aforequoted provision that a process of every party to a litigation who may be affected by the intervening death. The deceased litigant is himself
deceased party may be or herself protected, as he/she continues to be properly represented in the suit through the duly appointed legal
substituted by his heirs, but it representative of his estate. The spirit behind the general rule requiring a formal substitution of heirs is "not really
must be emphasized that because substitution of heirs is a jurisdictional requirement, but because non-compliance therewith results in the
substitution may only be undeniable violation of the right to due process of those who, though not duly notified of the proceedings, are
allowed in actions that survive substantially affected by the decision rendered therein".
the death of a party thereto.
The spirit behind the general
rule requiring a formal
substitution of heirs is "not
really because substitution of
heirs is a jurisdictional
requirement, but because non-
compliance therewith results in
the undeniable violation of the
right to due process of those
who, though not duly notified
of the proceedings, are
substantially affected by the
decision rendered therein".

35.Dagadag v. The established rule is that a FACTS: Petitioner former mayor of the municipality of Tanudan, Province of Kalinga. Michael Tongnawa and
Tongnawa 161166-67 real party in interest is one Antonio Gammod, respondents, are the municipal engineer and municipal planning and development coordinator,
03 Feb 2005 who would be benefited or respectively, of the said municipality.
Someros injured by the judgment, or one petitioner, former mayor of Tanudan, sent respondents a memorandum ordering them to explain within 72 hours
entitled to the avails of the why they should not be administratively sanctioned for acts unbecoming of public servants and failure to perform
suit. their duties. Respondents submitted to petitioner their respective explanations.
The word "interest," as petitioner issued EO No. 95-002 creating a Municipal Grievance Committee to investigate the charges against
contemplated by the Rules, respondents. Guilbert Dangpason, former vice-mayor of Tanudan, was designated Chairman.
means material interest or an After investigation, the Committee found respondents liable for insubordination, non-performance of duties and
interest in issue and to be absences without official leaves (AWOL).
affected by the judgment, as petitioner issued an order suspending respondents from their respective positions for 2 months.
distinguished from mere Respondents then appealed to the Civil Service Commission (CSC) contending that their right to due process has
interest in the question been violated that there was actually no investigation conducted on petitioner's charges, are "devoid of credibility." .
involved or a mere incidental during the pendency of appeal, petitioner issued an order dropping them from the roll of employees by reason of
interest. their unauthorized absences. Again, they appealed to the CSC.
Stated differently, the CA-G.R. SP No. 54511:
rule refers to a real or present CSC issued Resolution No. 974229 affirming petitioner's order of suspension. They moved for a reconsideration
substantial interest as but was denied prompting them to file with the CA a petition for review.
distinguished from a mere CA-G.R. SP 57315:
expectancy, or a future, Meanwhile, CSC issued Resolution No. 991136 affirming petitioner's order dropping respondents from the roll.
contingent, subordinate, or When their motion for reconsideration was denied, respondents filed with CA a petition for review.
consequential interest. As a CA in its joint Decision in CA-G.R. SP Nos. 54511 and 57315, granted respondents' petitions for review, reversing
general rule, one who has no the CSC challenged Resolutions and reinstating them to their respective positions and ordering the payment of
right or interest to protect their corresponding backwages.
cannot invoke the jurisdiction Petitioner filed a joint motion for reconsideration but was denied by the Court of Appeals.
of the court as party-plaintiff in Hence, the instant petition.
an action. ISSUE: WON CSC and the mayor of Tanudan are real parties in interest in and, therefore, can contest the assailed
joint Decision of the Court of Appeals?
HELD:
CSC: YES,
Mayor Tanduan: yes a real party in interest but has lost his legal personality to interpose the instant petition.
Section 2, Rule 3 of the 1997 Rules of Civil Procedure, as amended, provides:
"SEC. 2. Parties in interest. — A real party in interest is the party who stands to be benefited or
injured by the judgment in the suit, or the party entitled to the avails of the suit. Unless otherwise
authorized by law or these Rules, every action must be prosecuted or defended in the name of the
real party in interest." CSDcTA
The established rule is that a real party in interest is one who would be benefited or injured by the judgment, or one
entitled to the avails of the suit. The word "interest," as contemplated by the Rules, means material interest or an
interest in issue and to be affected by the judgment, as distinguished from mere interest in the question involved or
a mere incidental interest. Stated differently, the rule refers to a real or present substantial interest as distinguished
from a mere expectancy, or a future, contingent, subordinate, or consequential interest. As a general rule, one who
has no right or interest to protect cannot invoke the jurisdiction of the court as party-plaintiff in an action.
CSC: is the party adversely affected by the questioned Decision of the CA because it has been mandated by the
Constitution to preserve and safeguard the integrity of our civil service system. Thus, any transgression by herein
respondents of the CSC rules and regulations will adversely affect its integrity. Significantly, it has not challenged
the assailed Decision.
As regards the mayor of Tanudan:
there are two (2) reasons why he may interpose such appeal.
(1)is rooted in his power to appoint officials and employees of his municipality.Similarly, where a municipal mayor
orders the suspension or dismissal of a municipal employee on grounds he believes to be proper, but his order is
reversed or nullified by the CSC or the Court of Appeals (as in this case), he has the right to contest such adverse
ruling. His right to appeal flows from the fact that his power to appoint carries with it the power to remove. Being
chief executive of the municipality, he possesses this disciplinary power over appointive municipal officials and
employees.|

(2)salaries of the respondents, being municipal officials, are drawn from the municipal funds. Obviously, the mayor
has real and substantial interest in the outcome of the administrative cases against respondents.|||
However, petitioner, at the time he filed with this Court the instant petition assailing the Appellate Court Decision,
was no longer the mayor of Tanudan.
Section 17, Rule 3 of the 1997 Rules of Civil Procedure, as amended, is relevant, thus:
"Sec. 17. Death or separation of a party who is a public officer. — When a public officer is a party
in an action in his official capacity and during its pendency dies, resigns or otherwise ceases to
hold office, the action may be continued and maintained by or against his successor if, within thirty
(30) days after the successor takes office or such time as may be granted by the court, it is
satisfactorily shown to the court by any party that there is a substantial need for continuing or
maintaining it and that the successor adopts or continues or threatens to adopt or continue the
action of his predecessor. Before a substitution is made, the party or officer to be affected, unless
expressly assenting thereto, shall be given reasonable notice of the application therefor and
accorded an opportunity to be heard." (emphasis ours)
Interpreting the above rule, in Miranda vs. Carreon, Heirs of Mayor Nemencio Galvez vs. Court of Appeals, and
Roque, et al. vs. Delgado, et al., we held that where the petitioner (a public officer) ceases to be mayor, the appeal
and/or action he initiated may be continued and maintained by his successor if there is substantial need to do so. If
the successor failed to pursue the appeal and/or action, the same should be dismissed.
Records show that upon petitioner's cessation from public office, his successor did not file any manifestation to the
effect that he is continuing and maintaining this appeal.
We thus agree with the respondents that the petitioner has lost his legal personality to interpose the instant petition.

36.Re: Query of Mr. - The provisions of the Rules FACTS:


Roger Prioreschi in Re of Court indicate that only a
Exemption from Legal natural party litigant may be Mr. Roger c. Prioreschi, Administrator of the Good Shepherd Foundation, Inc., wrote to the Chief Justice after the
and Filing Fees of the regarded as an indigent Hon. Court Administrator Jose Perez denied Good Shepherd exemption from legal and filing fees and claimed that
Good Shepherd litigant. it cannot include foundations or Associations that work with and for the most indigent persons. Mr. Prioreschi
Foundation, Inc. AM claims that Good Shepherd is a foundation that reaches out to the “poorest of the poor, to the newly born and
NO 09-6-9-SC 19 Aug - The Good Shepherd abandoned babies, to children who never saw the smile of their mother, to old people who cannot afford a few
2006 Foundation, Inc., being a pesos to pay for "common prescriptions", to broken families who returned to a normal life.
Tan corporation invested by the
State with a juridical ISSUES:
personality separate and
distinct from that of its Whether or not the Courts can grant to the Foundation who works for indigent and underprivileged people, the
members, is a juridical same option granted to indigent people?
person. Among others, it has
the power to acquire and HELD:
possess property of all kinds
as well as incur obligations No. the Courts cannot grant to foundations like the Good Shepherd Foundation, Inc. the same exemption from
and bring civil or criminal payment of fees granted to indigent litigants even if the foundations are working for indigent and underprivileged
actions, in conformity with the people.
laws and regulations of their
organization. As a juridical Rule 3, Sec. 21. Indigent party. — A party may be authorized to litigate his action, claim or defense as an indigent
person, therefore, it cannot if the court, upon an ex parte application and hearing, is satisfied that the party is one who has no money or
be accorded the exemption property sufficient and available for food, shelter and basic necessities for himself and his family. Such authority
from legal and filing fees shall include an exemption from payment of docket and other lawful fees, and of transcripts of stenographic notes
granted to indigent litigants. which the court may order to be furnished him. The amount of the docket and other lawful fees which the indigent
was exempted from paying shall be a lien on any judgment rendered in the case favorable to the indigent, unless
the court otherwise provides.

Any adverse party may contest the grant of such authority at any time before judgment is rendered by the trial
court. If the court should determine after hearing that the party declared as an indigent is in fact a person with
sufficient income or property, the proper docket and other lawful fees shall be assessed and collected by the clerk
of court. If payment is not made within the time fixed by the court, execution shall issue for the payment thereof,
without prejudice to such other sanctions as the court may impose. (22a)

The clear intent and precise language of the aforequoted provisions of the Rules of Court indicate that only a
natural party litigant may be regarded as an indigent litigant. The Good Shepherd Foundation, Inc., being a
corporation invested by the State with a juridical personality separate and distinct from that of its members, 4 is a
juridical person. Among others, it has the power to acquire and possess property of all kinds as well as incur
obligations and bring civil or criminal actions, in conformity with the laws and regulations of their organization. 5 As
a juridical person, therefore, it cannot be accorded the exemption from legal and �ling fees granted to indigent
litigants.

That the Good Shepherd Foundation, Inc. is working for indigent and underprivileged people is of no moment.
Clearly, the Constitution has explicitly premised the free access clause on a person's poverty, a condition that only
a natural person can suffer.

37. Sps. Alhura v. City When an application to litigate as Facts:


of Naga 150135 30 Oct an indigent litigant is filed, the Petitioners filed a complaint for damages against the Naga City Government and its officers, arising from the alleged illegal
2006 court shall scrutinize the affidavits demolition of their residence and boarding house and for payment of lost income derived from fees paid by their boarders.
Tanada and supporting documents Simultaneously, petitioners filed an ex-parte Motion to Litigate as Indigent Litigants. The motion was granted, and, thus, the
submitted by the applicant to petitioners were exempted to pay the filing fees. After the pre-trial, respondents filed a Motion to Disqualify the Plaintiffs for
determine if the applicant Non-Payment of Filing Fees, alleging that petitioners were not qualified to become indigent litigants. They asserted that
complies with the income and petitioners were earning more than P3,000.00 a month from the husband’s income as a policeman and the wife’s small
property standards prescribed in businesses. The RTC then issued an Order, disqualifying petitioners as indigent litigants on the ground that they failed to
the present Section 19 of Rule substantiate their claim for exemption from payment of legal fees and to comply with the third paragraph of Sec. 18, Rule 141
141. If the trial court finds that the of the Rules of Court. Petitioners filed an MR against the Order. The RTC gave petitioners an opportunity to comply with the
applicant meets the income and requisites laid down in Sec. 18, Rule 141, for them to qualify as indigent litigants, to which petitioners obeyed. The RTC then
property requirements, the issued an Order denying petitioners’ MR.
authority to litigate as indigent Issue:
litigant is automatically granted Whether or not petitioners are qualified as indigent litigants.
and the grant is a matter of right. Held: Yes.
However, if the trial court finds Section 18, Rule 141 was then known as Section 16 of the same rule, which provides that a pauper litigant is one whose grow
that one or both requirements income does not exceed P2,000 a month or P24,000 a year for those residing in Metro Manila, and P1,500 a month or
have not been met, then it would P18,000 a year for those residing outside Metro Manila or those who do not own real property with an assessed value of not
set a hearing to enable the more than P24,000 or not more than P18,000 as the case may be. When the Rules of Court were amended by the 1997 Rules
applicant to prove that the of Civil Procedure, Rule 3, Section 22 thereof was superseded by Rule 3, Section 21. However, Rule 141, Sec. 16 was not
applicant has “no money or amended. The latter provision was amended in 2000, whereby the amounts increased (P2,000 to P4,000; P1,500 to P3,000;
property sufficient and available P24,000 and P18,000 on the real property value to P50,000), and the provision became Section 18 of the same Rule. The
for food, shelter and basic 2000 amendment on Rule did not revoke or amend Section 21, Rule 3; thus, under the 2000 amendment, there were two
necessities for himself and his existing rules on pauper litigant: Rule 3, Section 21 and Rule 141, Section 18. In 2004, Rule 141, Section 18 (which was later
family.” The adverse party in the known as Section 19 of the same rule), was again amended. It provided that an indigent litigant is one whose gross income
hearing may adduce and that of their immediate family do not exceed an amount double the monthly minimum wage of an employee and who do
countervailing evidence to not own real property with a fair market value stated in the current tax declaration of more than P300,000, and shall be exempt
disprove the evidence presented from payment of legal fees. Even with this 2004 amendment, Rule 3, Section 21 was not amended or recalled. Hence, the SC
by the applicant; after which the ruled that there was no intent to remove or expunge Rule 3, Section 21, nor was there any intention for the said provision to be
trial court shall rule on the superseded and amended by Rule 141, Section 16 (now Section 19). These two rules are still valid and enforceable rules on
application. Additionally, Section indigent litigants. Instead of declaring that Rule 3, Section 21 has been superseded and impliedly amended by Section 18 and
21, Rule 3 also provides that the later Section 19 of Rule 141, the Court finds that the two rules can and should be harmonized. When an application to litigate
grant may still be contested by the as an indigent litigant is filed, the court shall scrutinize the affidavits and supporting documents submitted by the applicant to
adverse party at any time before determine if the applicant complies with the income and property standards prescribed in the present Section 19 of Rule 141
judgment is rendered. If it is later as abovestated. If the trial court finds that the applicant meets the income and property requirements, the authority to litigate as
determined that the indigent party indigent litigant is automatically granted and the grant is a matter of right. However, if the trial court finds that one or both
is in fact a person of sufficient requirements have not been met, then it would set a hearing to enable the applicant to prove that the applicant has “no money
income or property, the proper or property sufficient and available for food, shelter and basic necessities for himself and his family.” The adverse party in the
docket and lawful fees shall be hearing may adduce countervailing evidence to disprove the evidence presented by the applicant; after which the trial court
assessed and collected. If shall rule on the application. Additionally, Section 21, Rule 3 also provides that the grant may still be contested by the adverse
payment is not made within the party at any time before judgment is rendered. If it is later determined that the indigent party is in fact a person of sufficient
time fixed by the court, execution income or property, the proper docket and lawful fees shall be assessed and collected. If payment is not made within the time
shall issue or the payment of fixed by the court, execution shall issue or the payment of prescribed fees shall be made, without prejudice to other sanctions
prescribed fees shall be made, that may be imposed. The SC ruled that Rule 141, Sec. 19 provides specific standards while Rule 3, Sec. 21 does not clearly
without prejudice to other draw the limits of the entitlement to the exemption. In this case, the trial court should have applied Rule 3, Section 21 to the
sanctions that may be imposed. application of the Alguras after their affidavits and supporting documents showed that petitioners did not satisfy the twin
requirements on gross monthly income and ownership of real property under Rule 141. Instead of disqualifying the Alguras as
indigent litigants, the trial court should have called a hearing as required by Rule 3, Section 21 to enable the petitioners to
adduce evidence to show that they didn't have property and money sufficient and available for food, shelter, and basic
necessities for them and their family. In that hearing, the respondents would have had the right to also present evidence to
refute the allegations and evidence in support of the application of the petitioners to litigate as indigent litigants. Since the SC
is not a trier of facts, it will have to remand the case to the trial court to determine whether petitioners can be considered as
indigent litigants using the standards set in Rule 3, Section 21. Recapitulating the rules on indigent litigants, therefore, if the
applicant for exemption meets the salary and property requirements under Section 19 of Rule 141, then the grant of the
application is mandatory. On the other hand, when the application does not satisfy one or both requirements, then the
application should not be denied outright; instead, the court should apply the "indigency test" under Section 21 of Rule 3 and
use its sound discretion in determining the merits of the prayer for exemption.
TITLE DOCTRINE FACTS / ISSUE / HELD

XIII. BILL OF PARTICULARS (RULE 12, SEC. 1 TO 6)


1. Guy vs. FACTS:
Guy Failure to specifically allege the fraudulent
189486 5 acts in intra-corporate controversies is Gilbert Guy is the son of spouses Francisco and Simny (one of the petitioners). He owned practically 80% of the 650,000
September indicative of a harassment or nuisance subscribed capital stock of GoodGold Realty and Development Corporation. In 1999, Francisco instructed to redistribute
2012 suit and maybe dismissed motu proprio. GoodGold’s shareholdings evenly among his children while maintaining a proportionate share for himself and his wife, Simny.
ARIBON 5 years after the distribution of shares of stock, Gilbert filed with the RTC of Manila, a complaint for the “Declaration of Nullity
of Transfer of Shares in GoodGold” against petitioners. However, Gilbert withdrew the complaint but filed another complaint
in the RTC of Mandaluyong captioned as “Intra-Corporate Controversy: For the Declaration of Nullity of Fraudulent
Transfers of Shares of Stock Certificates, Falsified General Information Sheets….. and damages with Application
for the Issuance of a Writ of Preliminary and Mandatory Injunction” against the same. He alleged that he never signed any
document which would justify and support the transfer of his shares to his siblings and that he has in no way, disposed,
alienated, assigned or sold any or part of his shares in GoodGold. The RTC dismissed the case. CA ruled that the complaint
should be heard on the merits. CA ruled in favor of Gilbert Guy. Hence, the consolidated petitions.

Issue:

W/N respondent’s case be given due course

Held:

No. The Court finds that the charges of fraud which Gilbert accuses his siblings are not supported by the required factual
allegations. In ordinary cases, the failure to allege the fraudulent act does not constitute a ground for dismissal because such
defect can be cured by a bill of particulars. However, it does not apply to intra-corporate controversies.

The case of Reyes v. RTC, states that in order to bring an intra-corporate controversy suit fall within the special commercial
court’s jurisdiction, it must show sufficient nexus showing that the corporation’s nature, structure or powers were used to
facilitate the fraudulent device or scheme. In the case at bar, no corporate power or officer was alleged to have facilitated the
transfer of Gilbert’s share. How the petitioners perpetrated the fraud, if ever they did, is an indispensable allegation which
Gilbert must have alleged with particularity in his complaint, but which he failed to. Instead of particularly describing the
fraudulent acts that he complained of, he just made a sweeping denial of the existence of stock certificates by claiming that
such were not necessary, GoodGold being a mere family corporation. Hence, the petitions were granted.

2. Salita vs. A complaint only needs to state the FACTS: Spouses Erwin Espinosa and Joselita Salita separated. Subsequently, Erwin sued for annulment on the ground of
Magtolis 106429 "ultimate facts constituting the plaintiff's Joselita's psychological incapacity.
13 June 1994 cause or causes of action." Ultimate facts The petition for annulment was filed before the RTC of Quezon City. Therein it is alleged that "sometime in 1987, petitioner
SOMEROS has been defined as "those facts which the came to realize that respondent was psychologically incapacitated to comply with the essential marital obligations of their
expected evidence will support." marriage, which incapacity existed at the time of the marriage although the same became manifest only thereafter."
Dissatisfied, Joselita moved for a bill of particulars (BOP) which the trial court granted. Subsequently, in his BOP, Edwin
We sustain the view of respondent CA that specified that —
the Bill of Particulars filed by private
respondent is sufficient to state a cause of . . . at the time of their marriage, respondent (Joselita Salita) was psychologically incapacitated to comply
action, and to require more details from with the essential marital obligations of their marriage in that she was unable to understand and accept the
private respondent would be to ask for demands made by his profession — that of a newly qualified Doctor of Medicine — upon petitioner's time
information on evidentiary matters. and efforts so that she frequently complained of his lack of attention to her even to her mother, whose
intervention caused petitioner to lose his job.
Still Joselita was not contended with BOP. She argued that the "assertion (in the BOP) is a statement of legal conclusion made
by petitioner's counsel and not an averment of 'ultimate facts,' as required by the Rules of Court, from which such a conclusion
may properly be inferred . . ."
RTC: trial court uphold the sufficiency of the BOP and directing Joselita to file her responsive pleading.
CA: Joselita was not convicted. She filed a petition for certiorari and her petition were referred to the CA for resolution but was
denied.
Petitioner insists that the allegations in the BOP constitute a legal conclusion, not an averment of facts, and fail to point out the
specific essential marital obligations shal allegedly was not able to perform, and thus render the BOP insufficient if not
irrelevant to her husband's cause of action.
Private respondent on the other hand believes that his allegations in the BOP constitute the ultimate facts that "conclusions of
law and evidentiary matters need not be stated in the complaint. The details of probative matter or particulars of evidence,
statements of law, inferences and arguments need not be stated."
ISSUE: whether the BOP is of sufficient definiteness or particularly as to enable herein petitioner to properly prepare her
responsive pleading or for trial.
HELD: YES, A complaint only needs to state the "ultimate facts constituting the plaintiff's cause or causes of action."
Ultimate facts has been defined as "those facts which the expected evidence will support." to require more details from
private respondent would be to ask for information on evidentiary matters.
On the basis of the allegations, it is evident that petitioner can already prepare her responsive pleading or for trial. Private
respondent has already alleged that "she (petitioner) was unable to understand and accept the demands made by his profession
. . . upon his time and efforts . . ." Certainly, she can respond to this. To demand for more details would indeed be asking for
information on evidentiary facts — facts necessary to prove essential or ultimate facts. For sure, the additional facts called for
by petitioner regarding her particular acts or omissions would be evidentiary, and to obtain evidentiary matters is not the
functions of a motion for bill of particulars.

3. Baritua vs. Motion for a bill of particulars; already FACTS: Dominador Mercader, a businessman engaged in the buy and sell of dry goods in Laoang, Northern Samar, boarded
Mercader 136048 moot and academic where parties had petitioners' Bus No. 142 with Plate No. 484 EU in Pasay City on March 16, 1983 for Brgy. Rawis, Laoang, Northern Samar.
23 January 2001 already filed their answer to amended However, Dominador Mercader was not able to reach his destination because the bus fell into a river while traversing the
TAN complaint prior to filing thereof. Bugko Bailey Bridge.

Section 1, Rule 12 of the Rules of Court Although Dominador survived the fall, he later died of asphyxia secondary to drowning. Respondents, heirs of deceased
provides: Dominador Mercader, filed a complaint against herein petitioners.

"SECTION 1. When applied for; purpose. RTC: RTC of Laoang, Northern Samar, after due trial, rendered a decision in favor of respondents and against petitioners,
— Before responding to a pleading, a party ordering the latter to pay compensatory, actual, moral and exemplary damages, loss of earnings of the late Dominador
may move for a more definite statement Mercader, and attorney's fees.
or for a bill of particulars of any matter
which is not averred with sufficient
definiteness or particularity to enable him CA: On appeal, the Court of Appeals affirmed all the monetary damages granted by the trial court with modification reducing
properly to prepare his responsive the amount of the deceased's lost earnings. In its decision, the appellate court held that petitioners failed to rebut the
pleading. If the pleading is a reply, the presumption that in the event a passenger died or was injured, the carrier had acted negligently. It added that petitioners
motion must be filed within ten (10) days presented no sufficient proof that they had exercised extraordinary diligence.
from service thereof. Such motion shall
point out the defects complained of, the Hence, petitioners filed this Petition for Review questioning the jurisdiction of the trial court. Invoking the Manchester ruling,
paragraphs wherein they are contained, petitioners contended, among others, that the trial court did not acquire jurisdiction over the subject matter because petitioner
and the details desired." failed to pay the correct amounts of docket and other lawful fees.

ISSUE: Whether the CA disregarded petitioners' procedural rights.

HELD:

Motion for a Bill of Particulars

Petitioners argue that the Court of Appeals erred when it passed sub silencio on the trial court's failure to rule frontally on their
plea for a bill of particulars.

We are not impressed. It must be noted that petitioners' counsel manifested in open court his desire to file a motion for a bill of
particulars. The RTC gave him ten days from March 12, 1985 within which to do so. He, however, filed the aforesaid motion
only on April 2, 1985 or eleven days past the deadline set by the trial court. Moreover, such motion was already moot and
academic because, prior to its filing, petitioners had already filed their answer and several other pleadings to the amended
Complaint. Section 1, Rule 12 of the Rules of Court provides:

"SECTION 1. When applied for; purpose. — Before responding to a pleading, a party may move for a more definite statement or
for a bill of particulars of any matter which is not averred with sufficient definiteness or particularity to enable him properly to
prepare his responsive pleading. If the pleading is a reply, the motion must be filed within ten (10) days from service thereof.
Such motion shall point out the defects complained of, the paragraphs wherein they are contained, and the details desired."
(emphasis supplied)

4. Estardante The OSG claimed that a bill of particulars Facts:


vs. People is not allowed by AO No. 7 (Rules of Petitioner was the school principal of the Ramon Torres National High School (RTNHS) in Bago City, Negros Occidental.
156851-55 18 Procedure in the Office of the Sometime in 1998, a group of concerned RTNHS teachers (private complainants) sent a letter to the Schools Division of Bago
February 2008 Ombudsman), and that, therefore, the City attaching a list of 15 irregularities allegedly committed by the petitioner, which the private complainants requested to be
TANADA Ombudsman cannot be bound by the Bill investigated. Two complaints were eventually filed by private complainants against petitioner with the Office of the
of Particulars submitted by private Ombudsman-Visayas. The Ombudsman-Visayas forwarded the complaint to the Office of the City Prosecutor. The City
complainants. The Court agrees with the Prosecutor served the petitioner with a subpoena requiring her to submit her counter-affidavit. Instead of filing a counter-
OSG. affidavit, petitioner filed before the City Prosecutor a Motion for Bill of Particulars, alleging that there were no specific criminal
charges that were stated in the subpoenas, insisting that she cannot prepare for her counter-affidavit unless the criminal
charges and the laws she violated are specified. Additionally, she also filed a Motion for Extension of Time to File Counter-
Affidavit. The City Prosecutor issued an Order attaching private complainants’ Bill of Particulars, which states that she is
charged for violation of Sections 68 and 69 of PD 1445. Thereafter, petitioner filed her counter-affidavit limiting herself only to
the charges specified in the Bill of Particulars. The City Prosecutor then referred the case back to the Ombudsman-Visayas who
found sufficient grounds to hold petitioner liable for five counts of violation of Section 3 (e) of RA No. 3019 (Anti-Graft and
Corrupt Practices Act, and filed before the RTC the corresponding Informations. Petitioner filed for a Motion for
Reinvestigation before the RTC alleging, among others, that she cannot be charged under RA No. 3019 and PD No. 1445. The
RTC denied the motion. She filed an MR over the RTC’s denial of her Motion for Reinvestigation, claiming that when the five
Informations for the violation of Section 3 (e) of RA No. 3019 were filed by the Ombudsman-Visayas, her right to due process
was violated, and that the Ombudsman-Visayas in effect went beyond the Bill of Particulars filed by the private respondents.
The RTC denied the MR.

Issue:
Whether or not the five Informations for violation of RA No. 3019, which were not included in the Bill of Particulars, may be
filed against petitioner.

Held:
Petitioner insisted that the Ombudsman-Visayas should have limited the charges filed against her to the crimes mentioned in
the Bill of Particulars, and that the filing of the Informations charging her with crimes different from those specified in the Bill
of Particulars violated her right to due process. The OSG countered this in claiming that a bill of particulars is not allowed by
AO No. 7 (Rules of Procedure in the Office of the Ombudsman), and that, therefore, the Ombudsman cannot be bound by the
Bill of Particulars submitted by private complainants. The Court agrees with the OSG. Clearly, the act of the prosecutor in
granting the petitioner’s Motion for Bill of Particulars is an act contrary to the express mandate of AO No. 7. Petitioner argued
that the granting of the Motion for Bill of Particulars by the City Prosecutor should bind the Ombudsman. The Court disagrees.
Section 31 of R.A. No. 6770 or The Ombudsman Act of 1989 expressly provides that those designated or deputized to assist the
Ombudsman shall be under his supervision and control. Indubitably, when the City Prosecutor is deputized by the Office of the
Ombudsman, he comes under the "supervision and control" of the Ombudsman which means that he is subject to the power of
the Ombudsman to direct, review, approve, reverse or modify the prosecutor's decision. In the present case, petitioner has no
valid basis for insisting that the Ombudsman-Visayas must be bound by the erroneous act of the City Prosecutor in granting
petitioner's Motion for Bill of Particulars. Laws and jurisprudence grant the Office of the Ombudsman the authority to reverse
or nullify the acts of the prosecutor pursuant to its power of control and supervision over deputized prosecutors. Hence, it was
within the prerogative of the Ombudsman-Visayas not to consider the Bill of Particulars submitted by the private
complainants. The SC, however, opines that the filing of Informations for violation of RA No. 3019 against petitioner violated
her right to due process. The SC did not find, after a thorough examination of the records, that she was served with subpoenas
and other documents apprising her of her violations. While there is no rule that the initial complaint filed against an accused
with the prosecutor's office should specifically state the particular law under which he is being charged, it is a basic elementary
rule that the complaint should specifically allege the criminal acts complained of, so as to enable the accused to prepare his
answer or counter-affidavit accurately and intelligently.

XIV. FILING AND SERVICE OF PLEADINGS, JUDGEMENTS AND OTHER PAPERS (RULE 13, SEC. 1 TO 19)
5. Bracero vs. Notice sent directly to client is not Facts: The Complaint stated that Monisit owned the 48, 632 sqm land. During Monisit’s lifetime, 5,000 square meters of the
Arcelo 212496 18 notice in law. Nevertheless, this rule land was mortgage to Rudolfo Arcelo’s grandmother, Damiana Mendoza. Mendoza’s death was followed by her son’s death, and
March 2015 admits of exceptions. Rudolfo Arcelo inherited the right over the mortgaged portion of the property. Sometime in 1982, Nestor Bracero claiming to be
UBAY In Santiago, this court considered the Arcelo’s tenant, cultivated this 5,000 sqm mortgaged portion of the property. Sometime in 1993, Monisit sued Bracero for the
filing of a motion for :consideration as recovery of the property he cultivated for his failure to share the products. Bracero countered that the land he cultivated
actual notice of the assailed Decision: belonged to Arcelo. Both complaint and counterclaim were dismissed.
The petitioners also maintain that they
should have first been furnished with a Petitioner’s counsel raises that the RTC’s Clerk of Court departed from usual procedure by sending a copy of the Decision
copy of the final decision before a writ directly to petitioner. He explains that his client is a poor farmer who lives in the remote mountain barangay of Lubo with no
of execution could be validly enforced telephone connection, and these circumstances made it easy to defeat his client’s right to appeal. Lastly, he argues that even if he
against them. Formal service of the received a copy of the motion for execution, “to require undersigned counsel to verify the existence of the decision with the RTC
judgment is indeed necessary as a rule is to unfairly burden the undersigned counsel and to unduly exonerate the clerck of court who was remiss in his duty in sending
but not, as it happens, in the case at a copy of the Decision to the undersigned counsel. “He explains that the court in Danao is 30 kilometers away from his office in
bar. The reason is that the petitioners Mandaue.
had filed a motion for reconsideration
of the decision of Judge Guadiz, which Respondent heirs contend that petitioner’s counsel did not categorically say that he was not informed by his client of the
would indicate that they were then decision on the date of receipt on May 4, 2009”. Respondnet heirs quote Santiago v. Guadiz, in that “petitioners cannot invoke
already informed of such decision. The due process on the basis of feigned ignorance as lack of formal notice cannot prevail against the fact of actual notice.
petitioners cannot now invoke due
process on the basis of a feigned Issue: Whether or not receipt of petitioner's counsel of a copy of the motion for execution amounts to effective official notice of
ignorance as the lack of formal notice the Regional Trial Court Decision
cannot prevail against the fact of actual
notice. Held: Yes. Rule 13, Section 2 of the Rules of Court states in part that “if any party has appeared by counsel, service upon him
shall be made upon his counsel or one of them, unless service upon the party himself is ordered by the court."

Notice sent directly to client is not notice in law. Nevertheless, this rule admits of exceptions.
Petitioner's counsel was furnished a copy of the motion for execution on September 11, 2009. As discussed by the Court of
Appeals, this motion categorically states that the trial court rendered its Decision on April 16, 2009, yet petitioner's counsel filed
no opposition. At that time, he did not file any motion asserting that he was not furnished a copy of the Decision. It was only on
January 8, 2010 when his client informed him of the Writ of Execution did petitioner's counsel file an Urgent Motion to Vacate
the Writ of Execution on the ground that he did not receive a copy of the Regional Trial Court Decision.

Jurisprudence reiterates that "litigants who are represented by counsel should not expect that all they need to do is sit back,
relax and await the outcome of their cases." This court has held that "equity aids the vigilant, not those who slumber on their
rights," and a party should "periodically keep in touch with his counsel, check with the court, and inquire about the status of the
case."

The explanation of petitioner's counsel that his client only finished Grade 6 and lives in a remote mountain barangay fails to
convince. Petitioner immediately informed his counsel about the Notice to Vacate on Execution on the same day he was served a
copy. This contradicts counsel's explanation implying difficulty in communicating with his client. This even raises the possibility
that his client did immediately inform him about the Regional Trial Court Decision upon receiving a copy.

Equally unconvincing and disappointing is the submission of petitioner's counsel that even if he received a copy of the motion
for execution, "to require undersigned counsel to verify the existence of the decision with the Regional Trial Court is to unfairly
burden the undersigned counsel and to unduly exonerate the clerk of court who was remiss in his duty in sending a copy of the
Decision to the undersigned counsel," and that the court in Danao is 30 kilometers away from his office in Mandaue. Counsels
have the duty to serve their clients with competence and diligence. The distance from counsel's office to the court should not be
used as an excuse by counsel from keeping himself updated with the status of the cases he is handling.

This court has held that "relief will not be granted to a party who seeks avoidance from the effects of the judgment when the loss
of the remedy at law was due to his own negligence." Petitioner, through his counsel, did not file an answer to the Complaint.
After the trial court declared petitioner in default for failure to file an answer, his counsel did not file an opposition to or motion
to lift the Order declaring him in default. After petitioner's counsel was furnished a copy of the motion for execution, he did not
immediately file an opposition to the motion or raise the ground that he was not furnished a copy of the Decision.

Petitioner Nestor Bracero, through his counsel Atty. Danilo Pilapil, had several opportunities to argue his position before the
courts but failed to take them. Petitioner should now be considered in estoppel from assailing the Regional Trial Court Order
dated February 11, 2010 denying petitioner's Urgent Motion to Vacate the Writ of Execution, affirmed by the Court of Appeals.
Also, "to frustrate the winning party's right through dilatory schemes is to frustrate all the efforts, time and expenditure of the
courts, which thereby increases the costs of litigation."

6. UP vs. Dizon Doctrine of immutability of a final Stern Builders submitted three progress billings corresponding to the work accomplished, but the UP paid only two of the
171182 23 August judgment has not been absolute, and billings prompting Stern Builders to sue the UP and its co-respondent officials to collect the unpaid billing and to recover
2012 has admitted several exceptions, various damages.
YUMUL among them: (a) the correction of
clerical errors; (b) the so-called nunc On November 28, 2001, the RTC rendered its decision ordering UP to pay Stern Builders. Then on January 16, 2002, UP filed its
pro tunc entries that cause no motion for reconsideration. The RTC denied the motion. The denial of the said motion was served upon Atty. Nolasco of the
prejudice to any party; (c) void UPLB Legal Office on May 17, 2002. Notably, Atty. Nolasco was not the counsel of record of the UP but the OLS in Diliman,
judgments; and (d) whenever Quezon City.
circumstances transpire after the
finality of the decision that render its UP filed a notice of appeal on June 3, 2002. However, the RTC denied due course to the notice of appeal for having been filed
execution unjust and inequitable. out of time.

UP entered into a General Aggrieved, UP elevated the matter to the CA. The CA sustained the RTC. Hence, this petition.
Construction Agreement with
respondent Stern Builders for the
construction of the extension building ISSUE/S:
and the renovation of the College of WON UP’s appeal dated June 3, 2002 was filed out of time.
Arts and Sciences Building in the
campus of the University of the HELD:
Philippines in Los Baños (UPLB). NO. The period of appeal did not start without effective service of decision upon counsel of record. Firstly, the service of the
denial of the motion for reconsideration upon Atty. Nolasco of the UPLB Legal Office was invalid and ineffectual because he was
admittedly not the counsel of record of the UP. The rule is that it is on the counsel and not the client that the service should be
made.

Verily, the service of the denial of the motion for reconsideration could only be validly made upon the OLS in Diliman, and no
other. The fact that Atty. Nolasco was in the employ of the UP at the UPLB Legal Office did not render the service upon him
effective. It is settled that where a party has appeared by counsel, service must be made upon such counsel.

This is clear enough from Section 2, second paragraph, of Rule 13, Rules of Court, which explicitly states that: "If any party
has appeared by counsel, service upon him shall be made upon his counsel or one of them, unless service upon the party himself
is ordered by the court. Where one counsel appears for several parties, he shall only be entitled to one copy of any paper served
upon him by the opposite side." As such, the period to appeal resumed only on June 1, 2002, the date following the service on
May 31, 2002 upon the OLS in Diliman of the copy of the decision of the RTC, not from the date when the UP was notified.
WHEREFORE, the Court GRANTS the petition for review on certiorari; REVERSES and SETS ASIDE the decision of the
Court of Appeals under review.

7. Heirs of The Petition lacks merit. The Notice of Facts:


Numeriano vs. Appeal was belatedly filed. It is basic Heirs of Numeriano Miranda, Sr., filed before the RTC a Complaint for Annulment of Titles and Specific Performance against
Miranda 179638 8 and elementary that a Notice of Appeal the heirs of Pedro Miranda, namely: Pacita and Oscar Miranda; the heir of Tranquilino Miranda, Rogelio Miranda; and the
July 2013 should be filed "within fifteen (15) days spouses respondent Pablo Miranda and Aida Lorenzo.
ZAPANTA from notice of the judgment or final
order appealed from." Under Section 3, After trial, the RTC, Branch 256, rendered a Decision upholding and sustaining the ff:
Rule 13 of the Rules of Court, pleadings o Validity of TCT Nos. 186011, 186012, and 186013;
may be filed in court either personally o Ordered Pablo Miranda to indemnify all other heirs of NUMERIANO MIRANDA the amount equivalent to 12/13 fair
or by registered mail. In the first case, market value of the co-owned residential house, erected on the lot 826-A-3 covered by TCT No. 186013 corresponding to their
the date of filing is the date of receipt. shares, and for the said heirs to divide among themselves the aforesaid amount;
In the second case, the date of mailing o Proclaim that ROGELIO MIRANDA is not the biological son or child by nature of TRANQUILINO MIRANDA, and
is the date of receipt. therefore is not entitled to inherit from the latter;
o Declare CORNELIO MIRANDA, NUMERIANO MIRANDA, JR., etc. as the lawful legal heirs of the deceased
TRANQUILINO MIRANDA and ordering them to partition among themselves Lot 826-A-1 covered by TCT No. 186011
registered in the name of TRANQUILINO MIRANDA, containing an area of 213 square meters
o Order all the abovenamed heirs to commission the survey of Lot 826-A-1 or to authorize in writing, one of them to
commission such survey, in order to avoid a chaotic situation similar to the case at bar. Should they not agree as to what
particular portion shall belong to one another, they may agree that it be allotted to one or two or several of them, who shall
indemnify the others at a price agreed upon by all of them. Should they not agree as to whom shall the property be allotted, to
sell the property to a third person at a price agreed upon by a majority of all of them, and to partition the proceeds of the sale in
accordance with No. 5 above.

Petitioners did not file any appeal hence the Decision became final and executory.

RTC issued a Writ of Execution, which was not implemented.

Respondent filed an Ex-parte Motion praying that the RTC issue a "Break-Open and Demolition Order" in order to compel the
petitioners to vacate his property. But since more than five years have elapsed from the time the Writ of Execution should have
been enforced, the RTC denied the Motion in its Order.

This prompted respondent to file with the RTC a Petition for Revival of Judgment. Petitioners opposed the revival of judgment
assailing, among others, the jurisdiction of the RTC to take cognizance of the Petition for Revival of Judgment.
RTC= Petition GRANTED. Decision is REVIVED. Hence, a notice of appeal via LBC was filed. But this was opposed for having
been final and executory. Thus, RTC subsequently denied the notice of appeal for lack of merit.
CA=Denied the Petition for Mandamus on the ground that the Notice of Appeal was filed out of time. MR= Denied.

Issue: W/N THE APPEAL WAS PERFECTED ON TIME and W/N THE LATE (ONE DAY) FILING WAS JUSTIFIED

Held:
Petitioners assert that an action to revive judgment is appealable, and that their appeal was perfected on time. They insist that
the Notice of Appeal, which they filed on the 15th day via LBC, was seasonably filed since the law does not require a specific
mode of service for filing a notice of appeal. Besides, even if their appeal was belatedly filed, it should still be given due course in
the interest of justice considering that their counsel had to brave the storm and the floods caused by typhoon "Florita" just to file
their Notice of Appeal on time. Petitioners further contend that their appeal is meritorious

The Petition lacks merit. The Notice of Appeal was belatedly filed. It is basic and elementary that a Notice of Appeal should be
filed "within fifteen (15) days from notice of the judgment or final order appealed from." Under Section 3, Rule 13 of the Rules of
Court, pleadings may be filed in court either personally or by registered mail. In the first case, the date of filing is the date of
receipt. In the second case, the date of mailing is the date of receipt.

In this case, however, the counsel for petitioners filed the Notice of Appeal via a private courier, a mode of filing not provided in
the Rules. Though not prohibited by the Rules, we cannot consider the filing of petitioners’ Notice of Appeal via LBC timely filed.
It is established jurisprudence that "the date of delivery of pleadings to a private letter-forwarding agency is not to be considered
as the date of filing thereof in court;" instead, "the date of actual receipt by the court x x x is deemed the date of filing of that
pleading."42 Records show that the Notice of Appeal was mailed on the 15th day and was received by the court on the 16th day
or one day beyond the reglementary period. Thus, the CA correctly ruled that the Notice of Appeal was filed out of time.

Neither can petitioners use typhoon "Florita" as an excuse for the belated filing of the Notice of Appeal because work in
government offices in Metro Manila was not suspended on July 13, 2006, the day petitioners’ Notice of Appeal was mailed via
LBC. And even if we, in the interest of justice, give due course to the appeal despite its late filing, the result would still be the
same. The appeal would still be denied for lack of merit. The Decision dated August 30, 1999 is already final and executory.

SC= Petition is DENIED. Decision and Resolution of CA are AFFIRMED.

8. Villalongha, Section 2, Rule 13 of the Rules of Court FACTS: A complaint for annulment of sale, damages, and attorney's fees filed by siblings Virgilio, Luzviminda, Virgincita,
et al. vs. Court of provides that "if any party has Deogracias Villalongha (Deogracias), and Alejandro Villalongha (Alejandro; collectively, plaintiffs Villalongha) against their
Appeals, et al. appeared by counsel, service upon him mother, Felipa Vda. de Villalongha (Felipa), and their siblings Aurora Villalongha-Cabarrubias (Aurora), Josefina Villalongha-
227222 20 August shall be made upon his counsel or one Daleon (Josefina), and Ramonito Villalongha (Ramonito; respondents Villalongha), together with Bolton Bridge Homeowners'
2019 of them, unless service upon the party Association, Incorporated (BBHAI), and the Register of Deeds for the City of Davao before the RTC of Davao.
ABAD himself is ordered by the court." Thus,
even if a party represented by counsel Plaintiffs Villalongha claimed that:
has been actually notified, said notice (a) they are co-owners of the properties that used to be conjugal properties of Felipa and her late husband, Mauricio Villalongha
is not considered notice in law. "The (Mauricio), who passed away in 1978;
reason is simple — the parties, (b) in a Deed of Extra-Judicial Settlement of Estate with Deed of Donation, Felipa waived her rights over her shares in the said
generally, have no formal education or lands
knowledge of the rules of procedure, (c) despite having lost all rights and interests on the said lands, Felipa subsequently sold to BBHAI the lands upon the malicious
specifically, the mechanics of an appeal instigation of respondent Aurora, resulting in the issuance of TCTs in the name of BBHAI, which is now threatening to eject the
or availment of legal remedies; thus, plaintiffs from the subject lands.
they may also be unaware of the rights
and duties of a litigant relative to the Respondents Villalongha denied the conjugal nature of the subject lands and their participation in the execution of the
receipt of a decision. It is best for the extrajudicial settlement. They averred that: (a) Felipa is the sole owner of the subject lands; (b) Felipa signed the extrajudicial
courts to deal only with one person in settlement on the representation of Luzviminda that the said document will only show the boundaries and monuments of the
the interest of orderly procedure — properties involved, without any intention to donate her properties to her children; and (c) the signatures of Aurora and Josefina
either the lawyer retained by the party appearing thereon were forged, and they did not sign any acceptance of the alleged donation.
or the party him/herself if [he/she]
does not intend to hire a lawyer. On the other hand, BBHAI claimed to be an innocent purchaser in good faith and for value.

RTC - (a) dismissed the complaint on the ground that plaintiffs Villalongha failed to establish their claim by a preponderance of
evidence; (b) declared the extrajudicial settlement null and void; and (c) adjudged Felipa as the sole owner of the subject lands.
CA - affirmed with modification the RTC ruling, thereby (a) adjudging Felipa as the exclusive and sole owner of the subject
lands; (b) declaring her sale to BBHAI as valid and binding; (c) ordering Felipa to deliver possession of the subject lands to
BBHAI; and (d) ordering plaintiffs Villalongha to pay litigation expenses and attorney's fees.

A copy of the March 22, 2013 Decision was sent to plaintiffs Villalongha's counsel, Atty. Advincula, Jr., with
registered letter, and was received by a certain Ariel Hernandez on May 8, 2013. 25 However, in a manifestation 26 dated March
11, 2014, Atty. Advincula, Jr. informed the CA that: (a) he did not receive said notice; and (b) Ariel Hernandez is not his staff or
employee, and not personally known to him or to his associate, Atty. Victorio S. Advincula, Sr.

Atty. Advincula, Jr. also filed a Motion to Withdraw as Counsel for plaintiffs Villalongha.

Virgilio received a copy of the said resolution on July 15, 2015, and filed a notice of receipt, requesting for time to engage the
services of a new counsel. On August 17, 2015, he also received notice of BBHAI's Motion for Issuance of Entry of Judgment 33
in the case. Thereafter, Atty. Abejaron filed a formal entry of appearance as counsel for herein petitioners only, and opposed
BBHAI's motion on the ground of prematurity, averring that there was no proper notice of the March 22, 2013
Decision on their former counsel, Atty. Advincula, Jr.
Respondents Villalongha countered that petitioners already had knowledge of the said Decision, yet petitioners failed to do
anything to verify if Atty. Advincula, Jr. received notice of said Decision and/or protect their remedial rights, if any.

CA - held that the March 22, 2013 Decision had become final and executory in the absence of any motion for reconsideration or
further appeal. The said Decision was thus entered in the Book of Entries of Judgments.
Petitioners filed a Manifestation/Compliance with Motion to Recall Entry of Judgment.
Respondents Villalongha and BBHAI moved for the issuance of a writ of execution before the RTC; hence, this petition with
prayer for issuance of a (TRO) enjoining, among others, the Presiding Judge of the RTC from hearing and/or giving due course
to the said motions;

ISSUE:
Whether or not the CA committed grave abuse of discretion in directing an entry of judgment in the case, and denying
petitioners' motion to recall the same, despite their claim of lack of proper service of the March 22, 2013 Decision.

HELD:
Yes. Section 2, Rule 13 of the Rules of Court provides that "if any party has appeared by counsel, service upon him shall be made
upon his counsel or one of them, unless service upon the party himself is ordered by the court." Thus, even if a party represented
by counsel has been actually notified, said notice is not considered notice in law. "The reason is simple — the parties, generally,
have no formal education or knowledge of the rules of procedure, specifically, the mechanics of an appeal or availment of legal
remedies; thus, they may also be unaware of the rights and duties of a litigant relative to the receipt of a decision. It is best for
the courts to deal only with one person in the interest of orderly procedure — either the lawyer retained by the party or the party
him/herself if [he/she] does not intend to hire a lawyer. (also see SEC. 13 Rule 13)
In the case at bar, the registry return card pertaining to Atty. Advincula, Jr.'s copy of the notice was not returned to the CA.
However, the CA concluded that the notice was received by Atty. Advincula, Jr. on the basis of the reply to tracer of William H.
Olmoguez, Postmaster of Davao City, that a certain Ariel Hernandez received the notice on May 8, 2013. 50 But in his request
manifestation, Atty. Advincula, Jr. denied having received such notice and knowing Ariel Hernandez, which was not refuted by
respondents.

It must be stressed that the mail matter must be received by the addressee or his duly authorized representative since service on
a person who was not a clerk, employee or one in charge of the attorney's office, is invalid. "[S]ervice of the court's order
upon any person other than the counsel of record is not legally effective and binding upon the party, nor may
it start the corresponding reglementary period for the subsequent procedural steps that may be taken by the
attorney." Since Ariel Hernandez was not an employee and, thus, not authorized to receive court notices in behalf of Atty.
Advincula, Jr., his alleged receipt of the notice of the March 22, 2013 Decision on May 8, 2013 is without any effect in law, and
cannot start the running of the period within which to file a motion for reconsideration or appeal.

Notably, Atty. Advincula, Jr. likewise withdrew as counsel for petitioners with the conformity of Virgilio, which was approved by
the CA.

Having been informed that the withdrawing counsel has not been duly served with notice of the March 22, 2013 Decision, and
considering further that no new counsel has entered any appearance in behalf of plaintiffs Villalongha, the CA should have
ensured that the latter were duly served notice thereof, but it did not. While it originally sent a copy of the said Decision to them
under registered letter on April 12, 2013, the same was unserved, and thus, returned to sender. Nonetheless, it bears to reiterate
that such earlier notice is not considered notice in law since plaintiffs Villalongha were then represented by counsel.

The Court finds that plaintiffs Villalongha have not been duly served with notice of the March 22, 2013 Decision; hence, the
period within which they may file a motion for reconsideration has not commenced to run. Thus, the Entry of Judgment made in
the case on the ground that the said Decision had become final and executory on May 24, 2013 or after the lapse of the fifteen
(15)-day period from the invalid receipt by Ariel Hernandez was therefore premature and inefficacious, and should be recalled
and lifted. An entry of judgment merely records the fact that a judgment, order or resolution has become final and executory; but
it is not the operative act that makes such judgment, order or resolution final and executory. In the case at bar, the Entry of
Judgment did not make the March 22, 2013 Decision final and executory considering that, notice of said Decision has not yet
been served on plaintiffs Villalongha/petitioners.

9. Aguilar vs.
Benlot, et al. In adjudging the plausibility of an FACTS: Elvira J. Benlot and Samuel L. Cuico (herein respondents) filed a Complaint before the Ombudsman against Arias
232806 21 January explanation as to why personal service (petitioner’s sister), Oralde, and Mancao the for violation of Republic Act No. 6713. The respondents were convinced that they
2019 has not been effected, a court shall resigned from their respective positions to pave the way for petitioner’s succession as punong barangay, leading to the violation
ARIBON likewise consider the importance of the of the three-term limit. On February 2015, the Ombudsman dismissed the administrative complaint when the petitioner was re-
subject matter of the case or the issues elected as Punong Barangay and Arias and Oraldee re-elected as Barangay Kagawads. They cite the applicability of the Aguinaldo
involved therein, and the [prima facie] Doctrine, also known as the doctrine of condonation. However, in September 2015, the Ombudsman reconsidered its Decision,
merit of the pleading sought to be finding that the said doctrine can no longer be applied to Petitioner and Arias. Thus, they were decided to have committed grave
expunged for violation of Section 11, misconduct.
Rule 13 of the Rules of Court
Petitioner moved for reconsideration. Ombudsman denied. CA dismissed due to the lack of explanation as to why the petition
was neither personally filed before the CA nor personally served to the parties. Petitioner filed for an MR before the CA. He
explained that his failure to personally file and serve the petition was due to an honest mistake. CA did not find the reasons
advanced by the petitioner compelling. Hence, this petition.

Issue:

W/N the CA erred in dismissing the petition and in failing to decide the case on its merit.

Held:

No. The Court finds that while the CA had good reason to find petitioner’s belated explanation unsatisfactory, the present case
merits the relaxation of the rules. It must be clarified that under Section 11, Rule 13 of the 1997 ROC, personal service and filing
is the general rule, and resort to other modes of service and filing is the exception. Henceforth, whenever personal service or
filing is practicable, it is mandatory. If personal service or filing is not practicable, resort to other modes may be had, but must be
accompanied by a written explanation as to why personal service or filing was not practicable to begin with.

In determining the plausibility of an explanation, a court shall likewise consider the importance of the subject matter of the case
or the issues involve therein, and the prima facie merit of the pleading sought to be expunged for violation of Section 11, Rule 13.
The Court rules that while the CA could not be faulted for not finding merit in the petitioner’s belated explanation, having
judicial notice of the proximity of the counsel’s offices to the CA, to the ombudsman, and with each other, the CA should have
also considered the prima facie merit of the petitioner’s case. The Court ruled that while petitioner is guilty of Grave Misconduct,
an analysis of the questions of law warrant that he benefit from the doctrine of condonation prevailing at that time. Hence,
petition granted.

10. Marinduque Under Section 11, Rule 13 of the Rules FACTS: NAPOCOR filed a complaint for expropriation against the petitioners for the construction of a Transmission Line
Mining and of Civil Procedure, personal service and Project. NAPCOR filed a motion for reconsideration on the RTC’s decision, but was denied. A supplemental decision was issued
Industrial Corp. vs. filing is the general rule, and resort to fixing the fair market value of an area of the property involved. A notice of appeal was filed with the CA.
NAPOCOR 161219 6 other modes of service and filing, the Petitioners filed a “motion to strike out or declare as not filed the notice of appeal, to declare the supplemental decision as final
October 2008 exception. and executory; and to issue the corresponding writ of execution thereon.” Petitioners argued that NAPOCOR violated Sec. 11
BRIONES Henceforth, whenever personal service Rule 13 of the Rules of Court because NAPOCOR filed and served the notice of appeal by registered mail. According to
or filing is practicable, in light of the petitioners, NAPOCOR had all the vehicles and manpower to personally serve and file the notice of appeal.
circumstances of time, place and NAPOCOR argues that the Rules allow the resort to other modes of service and filing as long as the pleading was accompanied
person, personal service or filing is by a written explanation why service or filing was not done personally. NAPOCOR maintains that is complied with the RUles
mandatory. because the notice of appeal contained an explanation why NAPOCOR resorted to service and filing by registered mail - due to
Only when personal service or filing is lack of manpower to effect personal service.
not practicable may resort to other
modes be had, which must then be ISSUE: Whether or not NAPOCOR failed to comply with Sec. 11 Rule 13 of the Rules of Court
accompanied by a written explanation
as to why personal service or filing was HELD: NAPOCOR complied. NAPOCOR’s notice of appeal sufficiently explained why the notice of appeal was served and filed
not practicable to begin with by registered mail - due to lack of manpower to effect personal service. This explanation is acceptable for it satisfactorily shows
why personal service was not practicable.
Moreover, the Court of Appeals correctly considered the importance of the issue involved in the case. Therefore, the Court of
Appeals didn’t err when it ruled that the trial court acted with grave abuse of discretion in the issuance of its Orders.
11. Quelnan vs. Under Rule 13, Section 10 of the 1997 FACTS:
VHF Philippines Rules of Civil Procedure provides for An ejectment suit, filed by the respondent against the petitioner, involves a condominium unit at Legaspi Towers 300 in Manila.
138500 16 the completeness of service, which Respondent leased to the petitioner. The Metropolitan Trial Court (MeTC) of Manila, on its finding that "summons together with
September 2005 personal service is complete upon a copy of the complaint was served to the petitioner thru his wife on 25th day of August 1992 by substituted service". The
CASTRO actual delivery. Service by ordinary petitioner "failed to file his answer within the reglementary period" the decision dated 23rd day of November 1992, rendering
mail is complete upon the expiration of the judgment for the respondent to vacate the premises, restore the possession and rental payment.
ten days after mailing unless the Court
otherwise provides. Service by PROCEDURAL HISTORY:
registered mail is complete upon actual Copy of the decision mentioned above was served on the petitioner by registered mail. However, the same was returned
receipt by the addressee or after five unclaimed on the petitioner's failure to claim the same despite the postmaster's three successive notices on 25 November 1992, 7
(5) days from the date he received the December 1992, and 11 December 1992.
postmaster's first notice, whichever
date is earlier. It means the period to The petitioner has taken no appeal. The MeTC decision became final and executory. A writ of execution, a notice of levy, and a
appeal or to file the necessary pleading notice to vacate served on petitioner's wife, who acknowledged receipt.
begins to run after five days from the
postmaster's first notice. The party is
deemed to have received and to have Petitioner filed with the Regional Trial Court (RTC) at Manila a Petition for Relief from Judgment with Prayer for Preliminary
been notified of the judgment at that Injunction and/or temporary restraining order, thereunder alleging that he was never served with a summons and was utterly
point. The burden is on the petitioner unaware of the proceedings in the ejectment suit. The RTC granted the petitioner's petition for relief and set aside the MeTC
to show that the postmaster's notice decision. Respondent sought reconsideration of the RTC decision, but its motion was denied.
never reached him and that he did not
acquire knowledge of the judgment. From there, the respondent directly went to this Court on a petition for review. This Court remanded it to the Court of Appeals
Sadly, the petitioner failed to discharge (CA). The CA granted the petition, set aside the RTC's decision, and reinstated the MeTC decision. Petitioner moved for a
his burden. The postmaster's reconsideration but denied.
certification belies the petitioner's
denial of receipt of the notice that the ISSUES:
petitioner did not claim the mail 1) WON if a party fails to claim his copy of the adverse decision which sent through registered mail, when is he deemed to have
despite the three notices to him. In this knowledge of the said decision
case, the postmaster's certification is 2) WON will the presumption of completeness of service of a registered mail matter under Rule 13, Section 10 of the 1997 Rules
the best evidence to prove that the first of Civil Procedure apply with the 60 days for filing a petition for relief from judgment under Rule 38, Section 3 of the Rules
notice was sent and delivered to the
addressee. RULING:
1) The records show that the summons' service upon the petitioner's wife effected by Section 7 of Rule 14 of the 1997 Rules of
Civil Procedure. This law provides for substituted service of summons.

With the reality that the postmaster first notified the petitioner on 25 November 1992, it follows that service of a copy of the
MeTC decision deemed complete and adequate five days from there or on 30 November 1992. Necessarily, the 60 days for filing
a petition for relief must be reckoned from such date (30 November 1992) as this was the day when the petitioner's actual receipt
is presumed. The petitioner was deemed to know the MeTC decision on 30 November 1992. The 60-day period for filing a
petition for relief thus expired on 29 January 1993. Unfortunately, it was only on 24 May 1993, or 175 days after the petitioner
deemed to have learned of the judgment, that he filed his petition for relief with the RTC. Indubitably, the petition was filed way
beyond the 60 days provided by law.

The Court has invariably held that the doctrine of finality of judgments is grounded on fundamental considerations of public
policy and sound practice that at the risk of occasional error, courts' judgments must become final at some definite date fixed by
law. The Court views with disfavor the unjustified delay in the enforcement of the final orders and decision in this case. Once a
judgment becomes final and executory, the prevailing party should not be denied the fruits of his victory by some subterfuge
devised by the losing party.

Moreover, the records are bereft of showing why the petitioner failed to claim his copy of the MeTC decision. Indeed, the
petitioner has not explained why he could not obtain a copy of said decision despite the postmaster's three notices sent to him.
The failure to claim a registered mail matter that the postmaster had duly given notice is not excusable neglect that would
warrant the reopening of a decided case.

2) YES. Under Rule 13, Section 10 of the 1997 Rules of Civil Procedure provides for the completeness of service, which personal
service is complete upon actual delivery. Service by ordinary mail is complete upon the expiration of ten days after mailing
unless the Court otherwise provides. Service by registered mail is complete upon actual receipt by the addressee or after five (5)
days from the date he received the postmaster's first notice, whichever date is earlier.

It means that the period to appeal or to file the necessary pleading begins to run after five days from the postmaster's first notice.
The party is deemed to have received and to have been notified of the judgment at that point. The burden is on the petitioner to
show that the postmaster's notice never reached him and that he did not acquire knowledge of the judgment. Sadly, the
petitioner failed to discharge his burden. The postmaster's certification belies the petitioner's denial of receipt of the notice that
the petitioner did not claim the mail despite the three notices to him. In this case, the postmaster's certification is the best
evidence to prove that the first notice was sent and delivered to the addressee.

Verily, relief will not be granted to a party who seeks to be relieved from the effects of a judgment when the loss of the remedy at
law was due to his negligence or a mistaken mode of procedure; otherwise, petitions for relief will be tantamount to reviving the
right of appeal which has already been lost. It is a well-known maxim that "equity aids the vigilant, not those who slumber on
their rights."

XV. SUMMONS (RULE 14, SEC. 1 TO 23)


 AM NO. 11-3-6-SC, NEW RULE ON SERVICE OF SUMMONS ON FOREIGN JURIDICAL ENTITIES
12. Express Padala In this jurisdiction, the general FACTS:
(Italia), S.P.A. vs. rule is that summons must be BDO Remittance, a corporation with a principal office in Italy, hired respondent Ocampo as a remittance processor. Later,
Ocampo 202505 6 served personally on the dismissed for misappropriating the sum of €24,035.60 by falsifying invoices of money payments relating to customers' money
September 2017 defendant under Section 6, Rule transfer orders.
DE GALA 14 of the Rules of Court.
However, for justifiable reasons, Accordingly, BDO Remittance filed a criminal complaint against Ocampo for the same acts before the Court of Turin, Italy.
other modes of serving summons Ocampo pleaded guilty to the offense charged. On 13 April 2005, the Honorable Court of Turin convicted and sentenced her to
may resort. When the defendant suffer imprisonment of six months and a penalty of €300.00. However, it granted her the benefit of suspending the sentence's
cannot be served personally enforcement on account of her guilty plea.
within a reasonable time after
efforts to locate him have failed, On 22 September 2008, BDO Remittance filed a petition to recognize foreign judgment with the RTC of Mandaluyong City.
the rules allow summons to be BDO Remittance prayed for recognizing the Court of Turin Decision and the cancellation or restriction of Ocampo's Philippine
served by substituted service. passport by the Department of Foreign Affairs (DFA).
Substituted service is effected by
leaving copies of the summons at On 21 November 2008, the sheriff attempted to personally serve Ocampo's summons at her local address alleged in the petition
the defendant's residence with located in San Bernardo Village, Darasa, Tanauan, Batangas. However, since the address was incomplete, the sheriff sought
some person of suitable age and barangay officials, who pointed him to the house belonging to Ocampo's father, Nicasio Ocampo. Victor P. Macahia, uncle of
discretion then residing therein, Ocampo and present occupant, informed the sheriff that Ocampo and her family were already in Italy. He was only a caretaker of
or by leaving the copies at the the house. The sheriff then proceeded to serve the summons upon Macahia. After Ocampo failed to file an answer, BDO
defendant's office or regular place Remittance filed a motion to declare Ocampo in default. The RTC granted the motion and allowed BDO Remittance to present
of business with some competent evidence ex parte.
person in charge.
PROCEDURAL HISTORY:
Thus, where the defendant On 14 September 2009, the RTC rendered a Decision in favor of BDO Remittance. It recognized as valid and binding in the
neither resides nor holds office in Philippines the Court of Turin Decision. It ordered the DFA to cancel or restrict Ocampo's Philippine passport and not renew
the address stated in the until she has served her sentence.
summons, substituted service
cannot be resorted. On 11 February 2010, Ocampo's mother, Laureana Macahia, received a copy of the RTC Decision and forwarded it to
Ocampo. Not having been represented by counsel a quo, the period of appeal lapsed.

The CA set aside the RTC Decision and revoked the order to cancel or restrict Ocampo's Philippine passport. The CA held that
since Ocampo's whereabouts were unknown, summons should have served by Section 14, Rule 14 of the Rules of Civil
Procedure. The sheriff, however, erroneously effected the substituted service of summons under Section 7 of Rule 14.

After the CA denied its motion for reconsideration, BDO Remittance filed the present petition for review. However, they
denied it.

ISSUE:
WON the service of summons was validly effected upon respondent, who lives in Italy, through substituted service

RULING:
No. In this jurisdiction, the general rule is that summons must be served personally on the defendant under Section 6, Rule 14
of the Rules of Court. However, for justifiable reasons, other modes of serving summons may resort. When the defendant
cannot be served personally within a reasonable time after efforts to locate him have failed, the rules allow summons to be
served by substituted service. Substituted service is effected by leaving copies of the summons at the defendant's residence
with some person of suitable age and discretion then residing therein, or by leaving the copies at the defendant's office or
regular place of business with some competent person in charge.

Thus, where the defendant neither resides nor holds office in the address stated in the summons, substituted service cannot be
resorted. As we explained in Keister v. Navarro: The terms "dwelling house" or "residence" are generally held to refer to the
time of service. Hence it is not sufficient "to leave the copy at the defendant's former dwelling house, residence, or place of
abode, as the case may be, after his removal from that place." They refer to the place where the person named in the summons
is living at the time when the service is made, even though he may be temporarily out of the country at the time. Similarly, the
terms "office" or "regular place of business" refers to the defendant's business's office or place at the time of service.

The summons' service is a vital and indispensable ingredient of a defendant's constitutional right to due process. As a rule, if a
defendant has not been validly summoned, the court acquires no jurisdiction over his person, and a judgment rendered
against him is void. Hence, in the instant case, not being a resident of the address where the summons was served, the
substituted service of the summons is ineffective

13. Cathay Metal The enumeration in Section 11 of Facts: Respondent Laguna West Multi-Purpose Cooperative allegedly entered into a joint venture agreement with farmer-
Corp. vs. Laguna West Rule 14 is exclusive. beneficiaries through Certificates of Land Ownership Award (CLOA) in Silang, Cavite. While respondent was negotiating with
Multi-Purpose the farmer-beneficiaries, petitioner Cathay Metal Corporation entered into Irrevocable Exclusive Right to Buy (IERB) contracts
Cooperative 172204 10 This provision of the rule does not with the same farmer-beneficiaries. Under the IERB, the farmer-beneficiaries committed themselves to sell to petitioner their
July 2014 limit service to the officers' places agricultural properties upon conversion to industrial or commercial properties or upon expiration of the period of prohibition
ESPIRITU of residence or offices. If summons from transferring title to the properties.
may not be served upon these
In 1996, respondent caused the annotation of its adverse claim on the farmer-beneficiaries' certificates of title. In 1999,
persons personally at their
petitioner and the farmer-beneficiaries executed contracts of sale of the properties. Transfer certificates of title were also issued
residences or offices, summons
in the name of petitioner in the same year.
may be served upon any of the
officers wherever they may be Respondent's Vice-President, Orlando dela Peña, sent two letters dated to petitioner, informing it of respondent's claim to the
found. properties. Petitioner did not respond. In 2000, petitioner filed a consolidated petition for cancellation of adverse claims on its
transfer certificates of title with the Regional Trial Court of Tagaytay City. It served a copy of the petition by registered mail to
respondent's alleged official address at "Barangay Mayapa, Calamba, Laguna." The petition was returned to sender because
respondent could not be found at that address. Petitioner allegedly attempted to serve the petition upon respondent
personally. However, this service failed for the same reason.
Upon petitioner's motion, the RTC issued an order declaring petitioner's substituted service, apparently by registered mail to
have been effected.
Upon learning that a case involving its adverse claim was pending, respondent, through dela Peña, filed a manifestation and
motion, alleging that respondent never received a copy of the summons and the petition. It moved for the service of the
summons and for a copy of the petition to be sent to sent to No. 160, Narra Avenue, Looc, Calamba, Laguna. Instead of
furnishing respondent with a copy of the petition, petitioner filed a motion for reconsideration arguing that the case was already
submitted for decision after all of petitioner's evidence had been admitted, and a memorandum had been filed. Therefore, it was
too late for respondent to ask the court that it be furnished with a copy of the petition.
In 2003, the RTC granted petitioner's motion for reconsideration. It ruled that service should be made to the address indicated
in its Cooperative Development Authority Certificate of Registration.
On March 21, 2003, the RTC issued a decision granting petitioner's petition for cancellation of annotations. Respondent
appealed to the CA that the trial court erred in applying the rule on substituted service, thus, it did not validly acquire
jurisdiction over the appellant.
The CA ruled that there was no valid service of summons upon respondent in accordance with Rule 14, Section 11 of the Revised
Rules of Civil Procedure. Hence, the "court acquired no jurisdiction to pronounce a judgment in the case."
Issue: Whether respondent was properly served with summons or notices of the hearing
Ruling: NO. On matters relating to procedures in court, it shall be the Rules of Procedure that will govern. Proper court
procedures shall be determined by the Rules as promulgated by this court.

Service of notices and summons on interested parties in a civil, criminal, or special proceeding is court procedure. Section 11,
Rule 14 of the Rules of Court provides the rule on service of summons upon a juridical entity. It provides that summons may be
served upon a juridical entity only through its officers.
Sec. 11. Service upon domestic private juridical entity. — When the defendant is a corporation, partnership or
association organized under the laws of the Philippines with a juridical personality, service may be made on
the president, managing partner, general manager, corporate secretary, treasurer, or in-house counsel.
Service of summons upon persons other than those officers enumerated in Section 11 is invalid. Even substantial compliance is
not sufficient service of summons. This provision of the rule does not limit service to the officers' places of residence or offices.
If summons may not be served upon these persons personally at their residences or offices, summons may be served upon any of
the officers wherever they may be found.
Hence, petitioner cannot use respondent's failure to amend its Articles of Incorporation to reflect its new address as an excuse
from sending or attempting to send to respondent copies of the petition and the summons. The Rules of Court provides that
notices should be sent to the enumerated officers. Petitioner failed to do this. No notice was ever sent to any of the enumerated
officers.
Petitioner insists that it should not be made to inquire further as to the whereabouts of respondent after the attempt to serve the
summons by registered mail to respondent's address as allegedly indicated in its Articles of Incorporation. The Rules do not
provide that it needs to do so. However, it provides for service by publication. Service by publication is available when the
whereabouts of the defendant is unknown. (Section 14, Rule 14 ROC)
In this case, petitioner s
erved summons upon respondent by registered mail and, allegedly, by personal service at the office address indicated in
respondent's Certificate of Registration. Summons was not served upon respondent's officers.

14. Spouses Rule 14, Section 6 of the 1997 Rules Facts: Respondent Ramon Ong (Ong) filed with the Regional Trial Court, La Trinidad, Benguet, a complaint for accion
Manuel vs. Ong of Civil Procedure provides: reivindicatoria. Ong charged the Spouses Manuel with having constructed improvements — through force, intimidation,
205249 15 October SEC. 6. Service in person on strategy, threats, and stealth — on a property he supposedly owned. Ong filed an "amended complaint." A summons was issued
2014 defendant. — Whenever directed to the Spouses Manuel. Ong filed with the Regional Trial Court a motion to declare the Spouses Manuel in default. Per
LEE practicable, the summons shall be the sheriff's return on summons, Sheriff Joselito Sales, along with Ong's counsel, Atty. Christopher Donaal, and a certain
served by handing a copy thereof to Federico Laureano, attempted to personally serve summons on the Spouses Manuel at their address in Lower Bacong, Loacan,
the defendant in person, or, if he Itogon, Benguet. The Spouses Manuel, however, requested that service be made at another time considering that petitioner
refuses to receive and sign for it, by Sandra Manuel's mother was then critically ill. The sheriff's return further indicates that another attempt at personal service
tendering it to him was made. After Sheriff Joselito Sales had personally explained to petitioner Sandra Manuel the content of the summons and the
Tendering summons is itself a complaint, the latter refused to sign and receive the summons and the complaint. Sheriff Joselito Sales was thus prompted to
means of personal service as it is merely tender the summons and complaint to petitioner Sandra Manuel and to advise her to file their answer within fifteen (15)
contained in Rule 14, Section 6. days. As the Spouses Manuel failed to file their answer within this period, Ong asked that they be declared in default. The
Personal service, as provided by Regional Trial Court issued an order granting Ong's motion to declare the Spouses Manuel in default. Following this, Ong moved
Rule 14, Section 6, is distinguished for the ex parte presentation of evidence, which the Regional Trial Court granted. The Spouses Manuel filed a motion to lift the
from its alternative — substituted order of default. They alleged that it is the siblings of petitioner Sandra Manuel who resided in Lower Bacong, Itogon, Benguet,
service — as provided by Rule 14, while they resided in Ambiong, La Trinidad, Benguet. Thus, summons could not have been properly served on them in the
Section 7: former address. They surmised that Ong and his companions mistook petitioner Sandra Manuel's siblings as the defendants.
They further claimed that they only subsequently received via registered mail copies of (1) a compliance and manifestation filed
SEC. 7. Substituted service. — If, by Ong and (2) the Regional Trial Court's order scheduling the ex parte presentation of evidence. Attached to the Spouses
for justifiable causes, the defendant Manuel's motion to lift order of default was their answer. The Regional Trial Court denied the Spouses Manuel's motion to lift
cannot be served within a order of default. It noted that, first, their motion was not sworn to, as required by the 1997 Rules of Civil Procedure, and, second,
reasonable time as provided in the they did not show that their failure to timely file an answer "was due to fraud, accident, mistake or excusable negligence."
preceding section, service may be
effected (a) by leaving copies of the Court of Appeals: Aggrieved, the Spouses Manuel filed a petition for certiorari before the Court of Appeals. The assailed
summons at the defendant's decision of the Court of Appeals dismissed the Spouses Manuel's Rule 65 petition for lack of merit and the resolution of the
residence with some person of Court of Appeals denied their motion for reconsideration.
suitable age and discretion then
residing therein, or (b) by leaving Issue: Whether the Spouses Manuel may be granted relief from the RTC’s order of default
the copies at defendant's office or
regular place of business with some
competent person in charge Held: No.
thereof. Jurisdiction over the persons of the Souses Manuel Acquired
We hold that jurisdiction over the persons of both defendants the Spouses Benedict and Sandra Manuel — was validly acquired.
This is so because personal service of summons, via tender to petitioner Sandra Manuel, was made by Sheriff Joselito Sales. Rule
The Spouses Manuel cannot 14, Section 6 of the 1997 Rules of Civil Procedure provides:
capitalize on the supposed variance SEC. 6. Service in person on defendant. — Whenever practicable, the summons shall be served by handing a copy thereof to the
of address. Personal service of defendant in person, or, if he refuses to receive and sign for it, by tendering it to him
summons has nothing to do with Tendering summons is itself a means of personal service as it is contained in Rule 14, Section 6. Personal service, as provided by
the location where summons is Rule 14, Section 6, is distinguished from its alternative — substituted service — as provided by Rule 14, Section 7:
served. A defendant's address is SEC. 7. Substituted service. — If, for justifiable causes, the defendant cannot be served within a reasonable time as provided in
inconsequential. Rule 14, Section 6 the preceding section, service may be effected (a) by leaving copies of the summons at the defendant's residence with some
of the 1997 Rules of Civil Procedure person of suitable age and discretion then residing therein, or (b) by leaving the copies at defendant's office or regular place of
is clear in what it requires: business with some competent person in charge thereof.
personally handing the summons to
the defendant (albeit tender is
sufficient should the defendant In this case, the sheriff's return on summons indicated that Sheriff Sales endeavored to personally hand the summons and a
refuse to receive and sign). What is copy of the complaint to the Spouses Manuel on two (2) separate occasions. He relented from doing so on the first occasion in
determinative of the validity of deference to the medical condition of petitioner Sandra Manuel's mother. On the second occasion, he was constrained to tender
personal service is, therefore, the the summons and copy of the complaint as petitioner Sandra Manuel refused to accept them. The Spouses Manuel did not deny
person of the defendant, not the the occurrence of the events narrated in the sheriff's return but claimed that no valid service of summons was made. They
locus of service. claimed that they did not reside in Lower Bacong, Loacan, Itogon, Benguet, where the service of summons was made. From this,
they surmised that the "Sandra Manuel" who was specifically identified in the sheriff's return was someone other than petitioner
Sandra Manuel. The Spouses Manuel cannot capitalize on the supposed variance of address. Personal service of summons has
nothing to do with the location where summons is served. A defendant's address is inconsequential. Rule 14, Section 6 of the
1997 Rules of Civil Procedure is clear in what it requires: personally handing the summons to the defendant (albeit tender is
sufficient should the defendant refuse to receive and sign). What is determinative of the validity of personal service is, therefore,
the person of the defendant, not the locus of service.
The Spouses Manuel are not entitled to relief from the order of default.
As valid service of summons was made on them, it was incumbent upon the Spouses Manuel, pursuant to Rule 11, Section 1 of
the 1997 Rules of Civil Procedure, to file their answer within fifteen (15) days from March 16, 2011. Having failed to do so, they
were rightly declared to be in default.

15. People’s As a general rule, personal service Facts:


General Insurance is the preferred mode of service of
Corporation vs. summons. Substituted service is the Lizaso, Guansing's employee, was driving Guansing's truck when he hit the rear portion of Yokohama's Isuzu Crosswind. The
Guansing 204759 14 exception to this general rule. For strong impact caused the Isuzu Crosswind to hit other vehicles, rendering it beyond repair. Yokohama's Isuzu Crosswind was
November 2018 the sheriff to avail of substituted insured with People's General Insurance Corporation. Yokohama filed a total loss claim under her insurance policy. Thus,
LIMIN service, there must be a detailed People's General Insurance Corporation claimed to have been subrogated to all the rights and interests of Yokohama against
enumeration of the sheriff's actions Guansing, seeking from Guansing reimbursement of the total amount paid to Yokohama less the salvage value. Despite repeated
showing that a defendant cannot be demands, Guansing failed to reimburse the amount claimed. Thus, People's General Insurance Corporation filed a Complaint for
served despite diligent and a sum of money and damages against Guansing and Lizaso. The sheriff served the summons on Guansing's brother, Reynaldo
reasonable efforts. These details are Guansing. The sheriff's return did not explain why summons was served on his brother instead of Guansing. Guansing filed a
contained in the sheriff's return. Motion to Dismiss the complaint for lack of jurisdiction over his person. He alleged that he did not personally receive the
Thus, the sheriff's return is entitled summons. People's General Insurance Corporation argued that summons was properly served since substituted service was an
to a presumption of regularity. alternative mode of service. The RTC denied to Motion to Dismiss. Guansing reiterated that the RTC had no jurisdiction over his
Courts may allow substituted person in both his Answer then the Pre-Trial Brief. The RTC ruled against Guansing. Guansing reiterated that the RTC had no
service based on what the sheriff's jurisdiction over his person in his Motion for Reconsideration then on his Appeal to the CA. The CA ruled in Guansing's favor
return contains. and held that the RTC did not acquire jurisdiction over him because summons was improperly served on his brother. Moreover,
the sheriff did not provide an explanation on why the summons was not personally served upon him. It further remanded the
Failure to serve summons means case to the RTC.
that the court did not acquire
jurisdiction over the person of the Issue:
defendant. Absent proper service of
summons, the court cannot acquire 1. Whether or not the RTC acquired jurisdiction over the person of respondent Edgardo Guansing through service of summons;
jurisdiction over the defendant and
unless there is voluntary
appearance. The filing of an answer 2. Whether or not respondent Edgardo Guansing, in filing his Answer and other subsequent pleadings, voluntarily submitted
and other subsequent pleadings is himself to the jurisdiction of the court.
tantamount to voluntary
appearance.
Ruling:

1. No. In the service of summons, personal service is the preferred mode. As a rule, summons must be served personally on a
defendant. Sheriffs, in doing substituted service, must strictly comply with the prescribed requirements and circumstances
authorized by the rules. Manotoc v. CA laid down the rules on personal service of summons: "For substituted service of
summons to be available, there must be several attempts by the sheriff to personally serve the summons within a reasonable
period of one month which eventually resulted in failure to prove impossibility of prompt service. "Several attempts" mean at
least 3 tries, preferably on at least two different dates. In addition, the sheriff must cite why such efforts were unsuccessful." In
this case, the basis for resorting to substituted service on respondent Guansing's brother is not provided for in the Sheriff's
Return. Although Rule 131, Section 3 (m) of the Rules of Court provides that there is a disputable presumption that "official duty
has been regularly performed," in this case, presumption of regularity does not apply. To enjoy the presumption of regularity, a
sheriff's return must contain: (1) detailed circumstances surrounding the sheriff's attempt to serve the summons on the
defendant; and (2) the specifics showing impossibility of service within a reasonable time. In this case, the sheriff should have
established the impossibility of prompt personal service before he resorted to substituted service. Impossibility of prompt
personal service is established by a sheriff's failure to personally serve the summons within a period of 1 month. Within this
period, he or she must have had at least 3 attempts, on 2 different dates, to personally serve the summons. Moreover, he or she
must cite in the sheriff's return why these attempts are unsuccessful.

2. Yes. Rapid City Realty Development Corporation v. Villa laid down the rules on voluntary appearance as follows: “(1) Special
appearance operates as an exception to the general rule on voluntary appearance; (2) Accordingly, objections to the jurisdiction
of the court over the person of the defendant must be explicitly made, i.e., set forth in an unequivocal manner; and (3) Failure to
do so constitutes voluntary submission to the jurisdiction of the court, especially in instances where a pleading or motion
seeking affirmative relief is filed and submitted to the court for resolution.” Respondent Guansing filed his: (1) Answer; (2) Pre-
trial Brief; (3) Urgent Ex-parte Motion for Postponement; (4) Motion for Reconsideration; and (5) Notice of Appeal. By filing
numerous pleadings, he has confirmed that notice has been effected, and that he has been adequately notified of the proceedings
for him to sufficiently defend his interests. Respondent Guansing, who actively participated in the proceedings, cannot impugn
the court's jurisdiction. A long line of cases has established that the filing of an answer, among other pleadings, is considered
voluntary appearance and vests the court with jurisdiction over the person.

16. Nation Summons is a writ by which the FACTS: Respondent filed against petitioner a Complaint for civil damages arising from estafa in relation to violations of the
Petroleum Gas, Inc. defendant is notified of the action Trust Receipts Law. After an ex parte hearing was conducted, respondent’s prayer for a writ of preliminary attachment was
vs. RCBC 183370 17 brought against him or her. Its granted and the corresponding writ was issued. Thereafter, Sheriff Leodel N. Roxas served upon petitioners a copy of the
August 2015 purpose is two-fold: to acquire summons, complaint, application for attachment, respondent’s affidavit and bond, and the order and writ of attachment.
NAKAGAWA jurisdiction over the person of the
defendant and to notify the
defendant that an action has been Petitioners filed through counsel a Special Appearance with Motion to Dismiss on November 15, 2006. They asserted that the
commenced so that he may be trial court did not acquire jurisdiction over the corporation since the summons was improperly served upon Claudia Abante
given an opportunity to be heard on (Abante), who is a mere liaison officer and not one of the corporate officers specifically enumerated in Section 11, Rule 14 of the
the claim against him. "Compliance Rules. the RTC denied petitioners’ motion to dismiss and ruled that there was valid service of summons. CA affirmed RTC
with the rules regarding the service decision.
of summons is as much an issue of
due process as of jurisdiction. The
ISSUE: WON THE TRIAL COURT ACQUIRED JURISDICTION OVER THE PERSON OF THE DEFENDANT CORPORATION
essence of due process is to be
BY SERVICE OF SUMMONS UPON ITS MERE EMPLOYEE.
found in the reasonable
opportunity to be heard and submit
any evidence one may have in RULING: YES. Service of summons on Domestic Corporation, partnership or other juridical entity is governed by Section 11,
support of his defense. It is Rule 14 of the Rules, which states:
elementary that before a person can
be deprived of his property, he
should first be informed of the SECTION 11. Service upon domestic private juridical entity. – When the defendant is a corporation, partnership or association
claim against him and the theory organized under the laws of the Philippines with a juridical personality, service may be made on the president, managing
on which such claim is premised. partner, general manager, corporate secretary, treasurer, or in-house counsel.

In this case, Abante proceeded to receive the summons and accompanying documents only after receiving instructions to do so
from Melinda Ang, an individual petitioner herein and the petitioner corporation’s corporate secretary. It is clear, therefore, that
Abante, in so receiving the summons, did so in representation of Ang who, as corporate secretary, is one of the officers
competent under the Rules of Court to receive summons on behalf of a private juridical person. Thus, while it may be true that
there was no direct, physical handing of the summons to Ang, the latter could at least be charged with having constructively
received the same, which in Our view, amounts to a valid service of summons.

17.
18. Orion Security Courts acquire jurisdiction Facts:
Corp. vs. Kaifam over the plaintiffs upon the
Enterprises Inc. filing of the complaint. On the Petitioner Orion Security Corporation is a domestic private corporation engaged in the business of providing security services.
163287 27 April 2007 other hand, jurisdiction over One of its clients is respondent Kalfam Enterprises, Inc. Respondent was not able to pay petitioner for services rendered.
NATO the defendants in a civil case is Petitioner thus filed a complaint against respondent for collection of sum of money. The sheriff tried to serve the summons and a
acquired either through the copy of the complaint on the secretary of respondent’s manager. However, respondent’s representatives allegedly refused to
service of summons upon acknowledge their receipt. The summons and the copy of the complaint were left at respondent’s office. When respondent failed
them or through their to file an Answer, petitioner filed a motion to declare respondent in default.
voluntary appearance in court
and their submission to its The trial court, however, denied the motion on the ground that there was no proper service of summons on respondent.
authority. Petitioner then filed a motion for alias summons, which the trial court granted. The process server again left the summons and a
copy of the complaint at respondents office through respondents security guard, who allegedly refused to acknowledge their
As a rule, summons should be receipt. Again, respondent failed to file an Answer. On motion of petitioner, respondent was declared in default. Thereafter,
personally served on the petitioner was allowed to adduce evidence ex parte.
defendant. It is only when
summons cannot be served Respondent filed a motion for reconsideration of the resolution declaring it in default. Respondent alleged the trial court did not
personally within a reasonable acquire jurisdiction over its person due to invalid service of summons. The trial court denied the motion for reconsideration. On
period of time that substituted appeal, the Court of Appeals held that summons was not validly served on respondent. Petitioner’s motion for reconsideration of
service may be resorted to. In the Court of Appeals decision was denied. Hence, the instant petition.
case of substituted service,
there should be a report Issue: WON there was proper service of summons
indicating that the person who
received the summons in the Held:
defendant’s behalf was one
with whom the defendant had Petitioner contends that the Court of Appeals completely brushed aside respondents voluntary appearance in the proceedings of
a relation of confidence the trial court. According to petitioner, the trial court acquired jurisdiction over respondent due to the latters voluntary
ensuring that the latter would appearance in the proceedings before the said court. Petitioner insists substituted service of summons on respondents security
actually receive the summons. guard is substantial compliance with the rule on service of summons, in view of the exceptional circumstances in the present
A party who makes a special case.
appearance in court
challenging the jurisdiction of Respondent, however, counters that the special appearance of its counsel does not constitute voluntary appearance. Respondent
said court based on the maintains that its filing of an opposition to petitioners motion to declare respondent in default and other subsequent pleadings
ground of invalid service of questioning the trial court’s jurisdiction over it does not amount to voluntary appearance.
summons is not deemed to
have submitted himself to the We find the petition without merit.
jurisdiction of the court.
Courts acquire jurisdiction over the plaintiffs upon the filing of the complaint. On the other hand, jurisdiction over the
defendants in a civil case is acquired either through the service of summons upon them or through their voluntary appearance in
court and their submission to its authority.

In this case, records show that respondents president, managing partner, general manager, corporate secretary, treasurer, or in-
house counsel never received the summons against respondent, either in person or by substituted service.

Note that in case of substituted service, there should be a report indicating that the person who received the summons in the
defendant's behalf was one with whom the defendant had a relation of confidence ensuring that the latter would actually receive
the summons. Here, petitioner failed to show that the security guard who received the summons in respondents behalf shared
such relation of confidence that respondent would surely receive the summons. Hence, we are unable to accept petitioners
contention that service on the security guard constituted substantial compliance with the requirements of substituted service.
Neither did the trial court acquire jurisdiction over respondent by the latter’s voluntary appearance in court proceedings. Note
that a party who makes a special appearance in court challenging the jurisdiction of said court based on the ground of invalid
service of summons is not deemed to have submitted himself to the jurisdiction of the court. In this case, records show that
respondent, in its special appearance, precisely questioned the jurisdiction of the trial court on the ground of invalid service of
summons. Thus, it cannot be deemed to have submitted to said courts authority.

19. Robinsons vs. DOCTRINE: FACTS:


Millares 163584 12 Summons is a writ by which the Celita Miralles, respondent, filed with the said court a complaint for sum of money against Remelita Robinson, petitioner.
December 2006 defendant is notified of the action
PEREZ brought against him or her. In a Summons was served on petitioner at her given address. However, per return of service of Sheriff Maximo Potente dated March
civil action, service of summons is 5, 2001, petitioner no longer resides at such address.
the means by which the court
acquires jurisdiction over the On July 20, 2001, the trial court issued an alias summons to be served at No. 19 Baguio St., Alabang Hills, Muntinlupa City,
person of the defendant. Any petitioner’s new address.
judgment without such service, in Again, the summons could not be served on petitioner.
the absence of a valid waiver, is null Eventually, respondent filed a motion to declare petitioner in default for her failure to file an answer seasonably despite service
and void. Where the action is in of summons.
personam and the defendant is in
the Philippines, the service of
summons may be made through Trial court granted respondent’s motion declaring petitioner in default and allowing respondent to present her evidence ex
personal or substituted service in parte.
the manner provided for in
Sections 6 and 7, Rule 14 of the A copy of the Order was sent to petitioner by registered mail at her new address.
1997 Rules of Procedure.
Personal service is generally Upon respondent’s motion, the trial court, on September 8, 2003, issued a writ of execution.
preferred over substituted service,
the latter mode of service being a On September 26, 2003, petitioner filed with the trial court a petition for relief from the judgment by default. She claimed that
method extraordinary in character. summons was improperly served upon her, thus, the trial court never acquired jurisdiction over her and that all its proceedings
For substituted service to be are void.
justified, the following
circumstances must be clearly
Trial court issued a Resolution denying the petition for relief. Petitioner filed a motion for reconsideration, but it was denied by
established:
the trial court.
(a) personal service of summons
within a reasonable time was
impossible; ISSUE:
(b) efforts were exerted to locate Whether the trial court correctly ruled that a substituted service of summons upon petitioner has been validly effected.
the party; and
(c) the summons was served upon a HELD:
person of sufficient age and YES. We agree with the trial court that summons has been properly served upon petitioner and that it has acquired jurisdiction
discretion residing at the party’s over her.
residence or upon a competent
person in charge of the party’s Petitioner contends that the service of summons upon the subdivision security guard is not in compliance with Section 7, Rule 14
office or place of business. Failure since he is not related to her or staying at her residence. Moreover, he is not duly authorized to receive summons for the
to do so would invalidate all residents of the village. Hence, the substituted service of summons is not valid and that the trial court never acquired
subsequent proceedings on jurisdiction over her person.
jurisdictional grounds.
We have ruled that the statutory requirements of substituted service must be followed strictly, faithfully, and fully and any
substituted service other than that authorized by the Rules is considered ineffective. However, we frown upon an overly strict
application of the Rules. It is the spirit, rather than the letter of the procedural rules, that governs.

In his Return, Sheriff Potente declared that he was refused entry by the security guard in Alabang Hills twice. The latter
informed him that petitioner prohibits him from allowing anybody to proceed to her residence whenever she is out. Obviously, it
was impossible for the sheriff to effect personal or substituted service of summons upon petitioner. We note that she failed to
controvert the sheriff’s declaration. Nor did she deny having received the summons through the security guard.

Considering her strict instruction to the security guard, she must bear its consequences.

Summons is a writ by which the defendant is notified of the action brought against him or her. In a civil action, service of
summons is the means by which the court acquires jurisdiction over the person of the defendant. Any judgment without such
service, in the absence of a valid waiver, is null and void. Where the action is in personam and the defendant is in the
Philippines, the service of summons may be made through personal or substituted service in the manner provided for in
Sections 6 and 7, Rule 14 of the 1997 Rules of Procedure, as amended, thus:

SEC. 6. Service in person on defendant. – Whenever practicable, the summons shall be served by handing a copy thereof to the
defendant in person, or if he refuses to receive and sign for it, by tendering it to him.

SEC. 7. Substituted service. – If, for justifiable causes, the defendant cannot be served within a reasonable time as provided in
the preceding section, service may be effected (a) by leaving copies of the summons at the defendant’s residence with some
person of suitable age and discretion then residing therein; or (b) by leaving the copies at the defendant’s office or regular place
of business with some competent person in charge thereof.

Under our procedural rules, personal service is generally preferred over substituted service, the latter mode of service being a
method extraordinary in character.
For substituted service to be justified, the following circumstances must be clearly established: (a) personal service of summons
within a reasonable time was impossible; (b) efforts were exerted to locate the party; and (c) the summons was served upon a
person of sufficient age and discretion residing at the party’s residence or upon a competent person in charge of the party’s office
or place of business. Failure to do so would invalidate all subsequent proceedings on jurisdictional grounds.9

WHEREFORE, we DENY the petition and we AFFIRM the assailed Orders of the RTC, Branch 274, Parañaque City, in Civil Case
No. 00-0372. Costs against petitioner.
SO ORDERED.

20. Green Star Facts:


The rules on service of summons
Express, Inc. vs. A Mitsubishi L-300 van which Universal Robina Corporation (URC) owned figured in a vehicular accident with petitioner Green
upon a domestic private juridical
Nissin-Universal Star Express, Inc.'s (Green Star) passenger bus, resulting in the death of the van's driver. Thus, the bus driver, petitioner Fruto
entity must be strictly complied
Robina Corp. 181517 6 Sayson, Jr., was charged with the crime of reckless imprudence resulting in homicide.
with. Otherwise, the court cannot be
July 2015
said to have acquired jurisdiction
PURIFICACION Thereafter, Green Star sent a demand letter to respondent Nissin-Universal Robina Corporation (NURC) for the repair of its
over the person of the defendant. passenger bus. NURC denied any liability therefor and argued that the criminal case shall determine the ultimate liabilities of
the parties. Thereafter, the criminal case was dismissed without prejudice, due to insufficiency of evidence.

Section 11. Service upon domestic Sayson and Green Star then filed a complaint for damages against NURC before the RTC. Francis Tinio, one of NURC's
private juridical entity. — When employees, was the one who received the summons. NURC filed a Motion to Dismiss claiming lack of jurisdiction due to
the defendant is a corporation, improper service.
partnership or association
organized under the laws of the RTC - denied NURC’s motion to dismiss
Philippines with a juridical  ruled that there was substantial compliance because there was actual receipt of the summons by NURC.
personality, service may be made on
the president, managing
CA - granted NURC’s Petition for Certiorari
partner, general manager,
 reversed RTC ruling stating that the complaint for damages filed by Sayson and Green Star against NURC is dismissed
corporate secretary, treasurer,
for lack of jurisdiction
or in-house counsel.
Issue:
Whether or not the summons was properly served on NURC, vesting the trial court with jurisdiction. - NO.

Held:
It is a well-established rule that the rules on service of summons upon a domestic private juridical entity must be strictly
complied with. Otherwise, the court cannot be said to have acquired jurisdiction over the person of the defendant.
NURC maintains that the RTC did not acquire jurisdiction over it as the summons was received by its cost accountant,
Francis Tinio. It argues that under Section 11, Rule 14 of the 1997 Rules of Court, which provides the rule on service of summons
upon a juridical entity, in cases where the defendant is a domestic corporation like NURC, summons may be served only through
its officers. Thus:
Section 11. Service upon domestic private juridical entity. — When the defendant is a corporation, partnership or
association organized under the laws of the Philippines with a juridical personality, service may be made on the
president, managing partner, general manager, corporate secretary, treasurer, or in-house counsel.
Here, Tinio, a member of NURC's accounting staff, received the summons on January 22, 2004. Green Star claims that it
was received upon instruction of Junadette Avedillo, the general manager of the corporation. Such fact, however, does not appear
in the Sheriff's Return. The Return did not even state whether Avedillo was present at the time the summons was received by
Tinio, the supposed assistant manager. Green Star further avers that the sheriff tendered the summons, but Avedillo simply
refused to sign and receive the same. She then allegedly instructed Tinio to just receive it in her behalf. However, Green Star
never presented said sheriff as witness during the hearing of NURC's motion to dismiss to attest to said claim. And while the
sheriff executed an affidavit which appears to support such allegation, the same was likewise not presented as evidence. It was
only when the case was already before the CA that said affidavit first surfaced. Since the service of summons was made on
a cost accountant, which is not one of the designated persons under Section 11 of Rule 14, the trial court did not
validly acquire jurisdiction over NURC, although the corporation may have actually received the summons. To rule
otherwise will be an outright circumvention of the rules, aggravating further the delay in the administration of justice.
|

21. Santos vs. It should be noted that the present FACTS: Respondent PNOC filed a complaint for a sum of money against petitioner Santos, Jr. in the RTC of Pasig. Personal
PNOC Exploration rule expressly states that the service service of summons to Santos failed because he could not be located in his last known address despite earnest efforts to do so.
Corp. 170943 23 of summons via publication in a Subsequently, on PNOC’s motion, the trial court allowed service of summons by publication.
September 2008 newspaper of general circulation,
SABIO when the defendant’s whereabouts PNOC caused the publication of the summons in Remate, a newspaper of general circulation. Thereafter, PNOC submitted the
is unknown, applies to any action, affidavit of publication of the advertising manager of Remate and an affidavit of service of PNOC’s employee to the effect that he
whether it be in rem or in sent a copy of the summons by registered mail to Santos’ last known address.
personam.
When Santos failed to file his answer within the prescribed period, PNOC moved that the case be set for the reception of its
With regard to the affidavit of evidence ex parte. After such, Santos filed an Omnibus Motion for Reconsideration and to Admit Attached Answer. He sought
service, under Sec. 19, Rule 14 of reconsideration, alleging that the affidavit of service submitted by PNOC failed to comply with Sec. 19, Rule 14 of the ROC as it
the ROC, the rules do not require was not executed by the clerk of court.
that the affidavit of complementary
service be executed by the clerk of The RTC denied Santos’ motion for reconsideration, On appeal, the CA also dismissed Santos’ petition and motion for
court. reconsideration.

ISSUE: Whether or not the service of summons was properly complied with by PNOC.

HELD: YES. Sec. 14, Rule 14 of the ROC provides that “in any action where the defendant xxx whenever his whereabouts are
unknown and cannot be ascertained by diligent inquiry, service may, by leave of court, be effected upon him by publication in a
newspaper of general circulation.”

Since Santos could not be personally served with summons despite diligent efforts to locate him, PNOC sought and was granted
leave of court to effect service of summons upon him by publication in a newspaper of general circulation. Thus, Santos was
properly served with summons by publication.

It should be noted that the present rule expressly states that the service of summons via publication in a newspaper of general
circulation, when the defendant’s whereabouts is unknown, applies to any action, whether it be in rem or in personam. With
regard to the affidavit of service, under Sec. 19, Rule 14 of the ROC, the rules do not require that the affidavit of complementary
service be executed by the clerk of court.

22. Montefalcon vs. For Residents Temporarily Outside


Vasquez 165016 17 of the Philippines, Sec 16, Rule 14 FACTS: Petitioner filed a complaint for acknowledgment and support against respondent Ronnie Vasquez before the RTC of
June 2008 provides that service, may, by leave Naga City. Respondent in this case was an overseas seafarer by profession. Sheriff tried to serve the summons and complaint on
ARIBON of court be also effected outside of respondent in Camarines Sur. As Vasquez was in Manila, Vasquez’s mother returned the documents to the clerk of court who
the Philippines. It should be noted informed the court of the non-service of summons. Petitioners filed a motion to declare Vasquez in default. Court denied for lack
that it uses the words "may" and of proper service of summons. In 2000, upon petitioner’s motion, the court issued an alias summons in Metro Manila. The first
"also," thus,it is not mandatory. failed, the second was served again to Vasquez’s caretaker, Raquel Bejer, who is of sound discretion. This was reflected in second
Other methods of service of sheriff’s return. Despite substituted service of summons, the trial court declared Vasquez in default. In 2001, the court granted
summons allowed under the Rules petitioner’s prayers and ordered Vasquez to give support to his illegitimate son. In the same year, Vasquez filed a notice of
may also be availed of appeal. The appeal was granted by the court. Vasquez argued that the trial court erred in trying and deciding the case as it never
acquired jurisdiction over his person. Appellate court ruled that the summons was defective as there was no explanation of
impossibility of personal serve and an attempt to effect personal service. Petitioners filed an MR. Appellate court denied. Hence,
this petition.
ISSUE: W/N respondent was validly served with summons

HELD: Yes. The Court agrees that the substituted service was valid. As an overseas seafarer, Vasquez was a Filipino resident
temporarily out of the country. Hence, service of summons on him is governed by Rule 14, Section 16 of the Rules of Court,
which states that service upon a person who ordinarily resides in the Philippines but who is temporarily out of it, may, by leave
of court, be also effected out of the Philippines. Since the said provision uses the words “may” and “also,” it is not mandatory.
Other methods of service of summons allowed under the Rules may also be availed of by the serving officer on the defendant.
Furthermore, substituted service of summons was justified because the impossibility of prompt personal service was shown by
the facts that the sheriff not only attempted to personally serve the summons in a barrio in Camarines Sur, it also ascertained the
whereabouts of Vasquez when the first service failed. Finding out that he was residing in Manila, a Taguig City-based sheriff was
commissioned to serve the summons in his residence in Taguig. There was no undue haste in effecting substituted service. Thus,
respondent was validly served with summons.

23. Perkin Elmer When the case instituted is an FACTS: Petitioner is a foreign corporation duly organized and existing under the laws of Singapore and engaged in the business
Singapore Pte Ltd. vs. action in rem or quasi in rem, of manufacturing, producing, selling or distributing various laboratory/analytical instruments but not considered "doing
Dakila Trading Corp. Philippine courts already have business" in the Philippines. Herein respondent Dakila Trading Corporation is a Filipino corporation and engaged in the
172242 14 August 2007 jurisdiction to hear and decide the business of selling and leasing out laboratory instrumentation and process control instrumentation, and trading of laboratory
SOMEROS case because, in actions in rem and chemicals and supplies.
quasi in rem, jurisdiction over the Respondent entered into a Distribution Agreement on 1 June 1990 with Perkin-Elmer Instruments Asia Pte Ltd. (PEIA),
person of the defendant is not a By virtue of the said agreement, PEIA appointed the respondent as the sole distributor with right of purchase and sell of its
prerequisite to confer jurisdiction products in the Philippines. PEIA, on the other hand, shall give respondent a commission for the sale of its products in the
on the court, provided that the Philippines.
court acquires jurisdiction over the Under the Distribution Agreement, respondent shall order the products of PEIA, either from PEIA itself or from Perkin-Elmer
res. Thus, in such instance, Instruments (Philippines) Corporation (PEIP), an affiliate of PEIA. PEIP is a corporation duly organized and existing
extraterritorial service of summons under Philippine laws, and involved in the business of wholesale trading of all kinds of scientific, biotechnological, and analytical
can be made upon the defendant. instruments and appliances. PEIA allegedly owned 99% of the shares of PEIP.
The said extraterritorial service of On 2 August 1997, however, PEIA unilaterally terminated the Distribution Agreement.
summons is not for the purpose of Civil Case No. MC99-605.
vesting the court with jurisdiction, respondent filed before the RTC of Mandaluyong City, Branch 212, a Complaint for Collection of Sum of Money and Damages
but for complying with the with Prayer for Issuance of a Writ of Attachment against PEIA and PEIP.
requirements of fair play or due RTC denied respondent's prayer then the respondent moved for reconsideration but it was denied.
process, so that the defendant will Respondent then filed Ex-Parte Motions for Issuance of Summons and for Leave of Court to Deputize Respondent's General
be informed of the pendency of the Manager, Richard A. Tee, to Serve Summons Outside of the Philippines, which the RTC granted.
action against him and the Thus, an Alias Summons, was issued by the RTC to PEIA. But the said Alias Summons was served and received by
possibility that property in the Perkinelmer Asia, a Singaporean based sole proprietorship, owned by the petitioner and, allegedly, a separate and distinct
Philippines belonging to him or in entity from PEIA.
which he has an interest may be PEIP moved to dismiss the Complaint filed by respondent on the ground that it states no cause of action. Perkinelmer Asia,
subjected to a judgment in favor of informed respondent and RTC, of the wrongful service of summons upon Perkinelmer Asia.
the plaintiff, and he can thereby respondent filed an Ex-Parte Motion to Admit Amended Complaint, to which it claimed that PEIA had become a sole
take steps to protect his interest if proprietorship owned by the petitioner, and subsequently changed its name to Perkinelmer Asia. Hence, in its Amended
he is so minded. Complaint respondent sought to change the name of PEIA to that of the petitioner.
On the other hand, when the RTC admitted the Amended Complaint. Respondent then filed another Motion for the Issuance of Summons and for Leave of
defendant or respondent does not Court to Deputize Respondent's General Manager, Richard A. Tee, to Serve Summons Outside the Philippines.
reside and is not found in the RTC deputized respondent's General Manager to serve summons on petitioner in Singapore.
Philippines, and the action RTC denied the Motion to Dismiss filed by PEIP, compelling the latter to file its Answer to the Amended Complaint.
involved is in personam, Philippine Petitioner subsequently filed with the RTC a Special Appearance and Motion to Dismiss respondent's Amended Complaint.
courts cannot try any case against RTC: denied petitioner's Motion to Dismiss.
him because of the impossibility of Petitioner moved for the reconsideration but it was denied.
acquiring jurisdiction over his petitioner filed a Petition for Certiorari under Rule 65 of the 1997 Revised Rules of Civil Procedure with application for TRO
person unless he voluntarily and/or preliminary injunction before the CA alleging that the RTC committed grave abuse of discretion amounting to lack or
appears in court excess of jurisdiction in refusing to dismiss the Amended Complaint.
CA: never issued any temporary restraining order or writ of injunction. It affirmed the RTC Orders.
ISSUE: WON there is a proper service of summons and acquisition of jurisdiction by the RTC over the person of the petitioner.
HELD: NO, this Court finds that the petitioner did not submit itself voluntarily to the authority of the court a quo; and in the
absence of valid service of summons, the RTC utterly failed to acquire jurisdiction over the person of the petitioner.
The proper service of summons differs depending on the nature of the civil case instituted by the plaintiff or petitioner: whether
it is in personam, in rem, or quasi in rem. Actions in personam, are those actions brought against a person on the basis of his
personal liability; actions in rem are actions against the thing itself instead of against the person; and actions are quasi in rem,
where an individual is named as defendant and the purpose of the proceeding is to subject his or her interest in a property to the
obligation or loan burdening the property.
Under Section 15, Rule 14 of the 1997 Revised Rules of Civil Procedure, there are only four instances wherein a defendant who is
a non-resident and is not found in the country may be served with summons by extraterritorial service, to wit:
(1) when the action affects the personal status of the plaintiff;
(2) when the action relates to, or the subject of which is property, within the Philippines, in which the defendant claims a lien or
an interest, actual or contingent;
(3) when the relief demanded in such action consists, wholly or in part, in excluding the defendant from any interest in property
located in the Philippines; and
(4) when the defendant non-resident's property has been attached within the Philippines.
In these instances, service of summons may be effected by:
(a) personal service out of the country, with leave of court;
(b) publication, also with leave of court; or
(c) any other manner the court may deem sufficient.
Undoubtedly, extraterritorial service of summons applies only where the action is in rem or quasi in rem, but not if an action is
in personam.
In the case at bar, this Court sustains the contention of the petitioner that there can never be a valid extraterritorial service of
summons upon it, because the case before the court a quo involving collection of a sum of money and damages is, indeed, an
action in personam, as it deals with the personal liability of the petitioner to the respondent by reason of the alleged unilateral
termination by the former of the Distribution Agreement. Even the CA, in its Decision upheld the nature of the instant case as an
action in personam.
Thus, being an action in personam, personal service of summons within the Philippines is necessary in order for the RTC to
validly acquire jurisdiction over the person of the petitioner, and this is not possible in the present case because the petitioner is
a non-resident and is not found within the Philippines.
It is worthy to note that what is required under the aforesaid provision of the Revised Rules of Civil Procedure is not a mere
allegation of the existence of personal property belonging to the non-resident defendant within the Philippines but, more
precisely, that the non-resident defendant's personal property located within the Philippines must have been actually attached.
Evidently, petitioner's alleged personal property within the Philippines, in the form of shares of stock in PEIP, had not been
attached; hence, Civil Case No. MC99-605, for collection of sum of money and damages, remains an action in personam. As a
result, the extraterritorial service of summons was not validly effected by the RTC against the petitioner, and the RTC thus failed
to acquire jurisdiction over the person of the petitioner. The RTC is therefore bereft of any authority to act upon the Complaint
filed before it by the respondent insofar as the petitioner is concerned.
Moreover, even though the petitioner raised other grounds in its Motion to Dismiss aside from lack of jurisdiction over its
person, the same is not tantamount to its voluntary appearance or submission to the authority of the court a quo.
Most telling is Section 20, Rule 14 of the Rules of Court, which expressly provides:
SEC. 20. Voluntary appearance. — The defendant's voluntary appearance in the action shall be equivalent to
service of summons. The inclusion in a motion to dismiss of other grounds aside from lack of jurisdiction
over the person of the defendant shall not be deemed a voluntary appearance. (Emphasis supplied.)

24. NM Rothschild FACTS:


& Sons vs. Lepanto Undoubtedly, extraterritorial
Consolidated Mining service of summons applies Respondent Lepanto Consolidated Mining Company led with the Regional Trial Court (RTC) of Makati City a Complaint against
Company 175799 28 only where the action is in rem petitioner NM Rothschild & Sons (Australia) Limited praying for a judgment declaring the loan and hedging contracts between
November 2011 or quasi in rem, but not if an the parties void for being contrary to Article 2018 of the Civil Code of the Philippines and for damages. Upon respondent's
TAN action is in personam. (plaintiff's) motion, the trial court authorized respondent's counsel to personally bring the summons and Complaint to the
Philippine Consulate General in Sydney, Australia for the latter office to effect service of summons on petitioner (defendant).
When the case instituted is an
action in rem or quasi in rem, October 20, 2005, petitioner filed a Special Appearance with Motion to Dismiss 5 praying for the dismissal of the Complaint on
Philippine courts already have the following grounds: (a) the court has not acquired jurisdiction over the person of petitioner due to the defective and improper
jurisdiction to hear and decide service of summons; (b) the Complaint failed to state a cause of action and respondent does not have any against petitioner; (c)
the case because, in actions in the action is barred by estoppel; and (d) respondent did not come to court with clean hands.
rem and quasi in rem,
jurisdiction over the person of RTC: According to the trial court, there was a proper service of summons through the Department of Foreign Affairs (DFA) on
the defendant is not a account of the fact that the defendant has neither applied for a license to do business in the Philippines, nor filed with the
prerequisite to confer Securities and Exchange Commission (SEC) a Written Power of Attorney designating some person on whom summons and other
jurisdiction on the court, legal processes maybe served. The trial court also held that the Complaint sufficiently stated a cause of action.
provided that the court
acquires jurisdiction over the CA: The Court of Appeals ruled that since the denial of a Motion to Dismiss is an interlocutory order, it cannot be the subject of
res. Thus, in such instance, a Petition for Certiorari, and may only be reviewed in the ordinary course of law by an appeal from the judgment after trial.
extraterritorial service of summons
can be made upon the defendant. Petitioner insists that (a) an order denying a motion to dismiss may be the proper subject of a petition for certiorari; and (b) the
The said extraterritorial service of trial court committed grave abuse of discretion in not finding that it had not validly acquired jurisdiction over petitioner and that
summons is not for the purpose of the plaintiff had no cause of action.
vesting the court with jurisdiction,
but for complying with the ISSUE: Whether or not there was proper service of summons.
requirements of fair play or due
process, so that the defendant will HELD: No, the summons in this case was not properly served.
be informed of the pendency of the
action against him and the When the defendant or respondent does not reside and is not found in the Philippines, and the action involved is in personam,
possibility that property in the Philippine courts cannot try any case against him because of the impossibility of acquiring jurisdiction over his person unless he
Philippines belonging to him or in voluntarily appears in court.
which he has an interest may be
subjected to a judgment in favor of
the plaintiff, and he can thereby The service of summons to petitioner through the DFA by the conveyance of the summons to the Philippine Consulate General
take steps to protect his interest if in Sydney, Australia was clearly made not through the above-quoted Section 12, but pursuant to Section 15 of the same rule
he is so minded. On the other hand,
when the defendant or which provides:
respondent does not reside
and is not found in the Sec. 15. Extraterritorial service. – When the defendant does not reside and is not found in the Philippines, and the action
Philippines, and the action affects the personal status of the plaintiff or relates to, or the subject of which is property within the Philippines, in which the
involved is in personam, defendant has or claims a lien or interest, actual or contingent, or in which the relief demanded consists, wholly or in part, in
Philippine courts cannot try excluding the defendant from any interest therein, or the property of the defendant has been attached within the Philippines,
any case against him because service may, by leave of court, be effected out of the Philippines by personal service as under section 6; or by publication in a
of the impossibility of newspaper of general circulation in such places and for such time as the court may order, in which case a copy of the
acquiring jurisdiction over his summons and order of the court shall be sent by registered mail to the last known address of the defendant, or in any other
person unless he voluntarily manner the court may deem sufficient. Any order granting such leave shall specify a reasonable time, which shall not be less
appears in court. than sixty (60) days after notice, within which the defendant must answer.

Undoubtedly, extraterritorial service of summons applies only where the action is in rem or quasi in rem, but
not if an action is in personam.

When the case instituted is an action in rem or quasi in rem, Philippine courts already have jurisdiction to
hear and decide the case because, in actions in rem and quasi in rem, jurisdiction over the person of the
defendant is not a prerequisite to confer jurisdiction on the court, provided that the court acquires
jurisdiction over the res. Thus, in such instance, extraterritorial service of summons can be made upon the defendant. The
said extraterritorial service of summons is not for the purpose of vesting the court with jurisdiction, but for complying with the
requirements of fair play or due process, so that the defendant will be informed of the pendency of the action against him and
the possibility that property in the Philippines belonging to him or in which he has an interest may be subjected to a judgment in
favor of the plaintiff, and he can thereby take steps to protect his interest if he is so minded. On the other hand, when the
defendant or respondent does not reside and is not found in the Philippines, and the action involved is in
personam, Philippine courts cannot try any case against him because of the impossibility of acquiring
jurisdiction over his person unless he voluntarily appears in court.

25. De Pedro vs. The sheriff's return must contain a Facts:


Romasan narration of the circumstances The case originated from separate complaints filed by respondent Romasan Development Corporation (Romasan) before the
Development showing efforts to personally serve Antipolo City RTC for nullification of free patent and original certificates of title, filed against several defendants, one of which
Corporation 194751 26 summons to the defendants or was petitioner Aurora De Pedro (De Pedro). Respondent alleged in its complaints that it was the owner and possessor of a parcel
November 2014 respondents and the impossibility of land in Antipolo City. It narrated that its representative, Mr. Rodrigo Ko, discovered that De Pedro put up fences on a portion
TANADA of personal service of summons. A of the subject property. Mr. Ko confronted De Pedro on this, but she was able to show title and documents evidencing her
sheriff's return enjoys the ownership. Upon checking with the CENRO-DENR, it was later discovered that the DENR issued free patents covering portion
presumption of regularity in its of respondent’s property to several recipients (including De Pedro). Based on these free patents, the Register of Deeds issued
issuance if it contains (1) the details titles covering portions of respondent’s property. Respondent further alleged that the government could not legally issue the free
of the circumstances surrounding patents since at the time of their issuance, the land was already released for disposition to private individuals. Attempts to
the sheriff's attempt to serve the personally serve summons on De Pedro failed. Respondent then filed a motion to serve summons and the complaint by
summons personally upon the publication. The RTC granted the motion; hence, the summons and the complaint were published in People’s Balita on its April
defendants or respondents; and (2) 24, May 1, and May 8, 1998 issues. Respondent moved to declare all defendants, including De Pedro, in default for failure to file
the particulars showing the their answers, and to be allowed to present evidence ex parte. The RTC granted the motions. In its decision on Jan. 7, 2000, the
impossibility of serving the RTC declared the titles and free patents issued to all defendants (including De Pedro) as a nullity. The RTC noted that the title
summons within reasonable time. and free patent issued to De Pedro were void, since the property was already “segregated from the mass of public domain” that
It does not enjoy the presumption can be disposed by the government. De Pedro filed before the RTC a motion for new trial alleging that her counsel received
of regularity if the return was notice of the Jan. 7, 2000 decision only on March 16, 2000. She argued that because of improper and defective service of
merely pro forma. Failure to state summons, the RTC did not acquire jurisdiction over her. Citing the officer's return, De Pedro pointed out that summons was not
the facts and circumstances that personally served upon her "for the reason that according to the messenger of Post Office of Pasig there is no person in the said
rendered service of summons given address.” She also alleged that the case should have been dismissed on the ground of litis pendentia. The RTC denied De
impossible renders service of Pedro’s motion for new trial. It ruled that summons were validly served upon De Pedro through publication, in accordance with
summons and the return the Rules of Court, and that the period to file the motion for new trial has already lapsed. De Pedro filed a petition for certiorari
ineffective. In that case, no before the CA, which dismissed the petition and affirmed the denial of her motion for new trial. The CA held that De Pedro
substituted service or service by belatedly filed her motion for new trial and that she failed to allege any ground that would justify the grant of a new trial. She
publication can be valid. then elevated the case to the SC, but was also denied for her failure to pay the Special Allowance for the Judiciary and s heriff’s
fees. De Pedro then filed before the SC a petition for annulment of the Jan. 7, 2000 judgment on the grounds of lack of
jurisdiction, litis pendentia, and for having been dispossessed of her property without due process. The CA denied De Pedro’s
petition for annulment of judgment, ruling that since she already availed of the remedy of new trial, and raised the case before
the CA via petition for certiorari, she can no longer file a petition for annulment of judgment.
Issue:
Whether or not summons was properly served upon De Pedro.
Held: No.
De Pedro argued that the complaint was an action in personal, which required personal service of summons upon her for the
court to acquire jurisdiction over her person. Jurisdiction over the parties is required regardless of the type of action – whether it
is in personam, in rem, or quasi in rem. Courts need not acquire jurisdiction over parties on this basis in in rem and quasi in rem
actions. Actions in rem or quasi in rem are not directed against the person based on his or her personal liability. However, to
satisfy the requirements of due process, jurisdiction over the parties in in rem and quasi in rem actions is required. Violation of
due process rights is a jurisdictional defect. An action for annulment of certificate of title is quasi in rem. Regardless of the
nature of the action, proper service of summons is imperative. Personal service of summons is the preferred mode of service of
summons. If the defendant or respondent refuses to receive the summons, it shall be tendered to him or her. Other modes of
serving summons may be done when justified. Service of summons through other modes will not be effective without showing
serious attempts to serve summons through personal service. Thus, the rules allow summons to be served by substituted service
only for justifiable causes and if the defendant or respondent cannot be served within reasonable time. Substituted service is
effected "(a) by leaving copies of the summons at the defendant's residence with some person of suitable age and discretion then
residing therein, or (b) by leaving the copies at defendant's office or regular place of business with some competent person in
charge thereof." Service of summons by publication in a newspaper of general circulation is allowed when the defendant or
respondent is designated as an unknown owner or if his or her whereabouts are "unknown and cannot be ascertained by diligent
inquiry." It may only be effected after unsuccessful attempts to serve the summons personally, and after diligent inquiry as to the
defendant's or respondent's whereabouts. A look into the content of the sheriff's return will determine if the circumstances
warranted the deviation from the rule preferring personal service of summons over other modes of service. The sheriff's return
must contain a narration of the circumstances showing efforts to personally serve summons to the defendants or respondents
and the impossibility of personal service of summons. A sheriff's return enjoys the presumption of regularity in its issuance if it
contains (1) the details of the circumstances surrounding the sheriff's attempt to serve the summons personally upon the
defendants or respondents; and (2) the particulars showing the impossibility of serving the summons within reasonable time. It
does not enjoy the presumption of regularity if the return was merely pro forma. Failure to state the facts and circumstances that
rendered service of summons impossible renders service of summons and the return ineffective. In that case, no substituted
service or service by publication can be valid. This return shows no detail of the sheriff's efforts to serve the summons personally
upon petitioner. The summons was unserved only because the post office messenger stated that there was no "Aurora N. De
Pedro" in the service address. The return did not show that the sheriff attempted to locate petitioner's whereabouts. Moreover, it
cannot be concluded based on the return that personal service was rendered impossible under the circumstances or that service
could no longer be made within reasonable time. The lack of any demonstration of effort on the part of the sheriff to serve the
summons personally upon petitioner is a deviation from this court's previous rulings that personal service is the preferred mode
of service, and that the sheriff must narrate in his or her return the efforts made to effect personal service. Thus, the sheriff's
return in this case was defective. No substituted service or service by publication will be allowed based on such defective return.

26. Reicon vs. Facts: Reicon is the owner of a parcel of land and the one-storey building erected thereon located at the comer of Aurora
Diamond 204796 4 The underlying rationale behind Boulevard and Araneta Avenue, Sta. Mesa, Quezon City. Reicon and respondent Diamond Dragon Realty and Management, Inc.
February 2015 this rule is that a certiorari entered into a Contract of Lease, whereby Reicon leased the subject property to Diamond for a period of twenty (20) years for a
UBAY proceeding is, by nature, an original monthly rental of ₱75,000.00. However, Diamond sublet portions of the subject property to Jollibee Foods Corporation and
and independent action, and, Maybunga U.K. Enterprises.
therefore not considered as part of
Starting June 2006, Diamond failed to pay the monthly rentals due and the checks it had issued were all dishonored. Thus,
the trial that had resulted in the
Reicon demand payment of the accrued rentals and terminated the Contract. Thereafter, it entered into separate contracts with
rendition of the judgment or order
Jollibee and Maybunga over the portions of the subject property.
complained of. Hence, at the
preliminary point of serving the Diamond filed a complaint for breach of contract with damages against Reicon, Jollibee, Maybunga, Andrew, and a certain Mary
certiorari petition, as in other Palangdao, defendants herein before the Regional Trial Court alleging that the Contract did not provide for its unilateral
initiatory pleadings, it cannot be termination by either of the parties. It also alleged that the act of defendants in entering into separate contracts, despite the
said that an appearance for existence of their Contract, constitutes unlawful interference and prayed that the separate contracts of lease entered with
respondent has been made by his Jollibee and Maybunga, be declared invalid and illegal.
counsel. Consequently, the
Reicon filed a motion to dismiss the complaint on the following grounds:
requirement under Section 2, Rule
13 of the Rules, which provides that (a) lack of jurisdiction over its person;
if any party has appeared by
counsel, service upon him shall be (b) lack of legal capacity to sue as a juridical person on the part of Diamond; and
made upon his counsel, should not ( c) lack of cause of action.
apply.
Also, Jollibee filed a separate motion to dismiss.
Thus, the CA erred when it The Regional Trial Court denied Reicon's and Jollibee's motion to dismiss. Motion for reconsideration was also denied.
dismissed Reicon's certiorari
petition outright for non- On appeal, the Court of Appeals dismissed Reicon's certiorari petition based on the following grounds:
compliance with Section 3, Rule 46 (a) non-compliance with the requirements of proof of service of the petition on Diamond pursuant to Section 3, Rule 46 of the
of the Rules as well as the rule on Rules, and
service upon a party through
counsel under Section 2, Rule 13 of (b) non-compliance with the rule on service upon a party through counsel under Section 2, Rule 13 of the Rules.
the Rules. The service of said Reicon’s motion for reconsideration was also denied.
pleading upon the person of the
respondent, and not upon his Hence, the petition.
counsel, is what the rule properly Issue: Whether or not Reicon's certiorari petition before the CA was properly served upon the person of Diamond.
requires, as in this case.

Held: Yes. Reicon's certiorari petition before the CA was properly served upon the person of Diamond.
Sections 3 and 4, Rule 46 of the Rules, which covers cases originally Filed before the CA, provide as follows:
SEC. 3. Contents and filing of petition; effect of non-compliance with requirements. - The petition shall contain the full names
and actual addresses of all the petitioners and respondents, a concise statement of the matters involved, the factual background
of the case, and the grounds relied upon for the relief prayed for.
The failure of the petitioner to comply with any of the foregoing requirements shall be sufficient ground for the dismissal of the
petition.
SEC. 4. Jurisdiction over the person of respondent, how acquired. - The court shall acquire jurisdiction over the person of the
respondent by the service on him of its order or resolution indicating its initial action on the petition or by his voluntary
submission to such jurisdiction.
In this case, the Court notes that Diamond declared the aforesaid address as its business address in its complaint before the
RTC, and that there is dearth of evidence to show that it had since changed its address or had moved out. Hence, Reicon cannot
be faulted for adopting the said address in serving a copy of its certiorari petition to Diamond in light of the requirement under
Sections 3 and 4, Rule 46 of the Rules as above-cited, which merely entails service of the petition upon the respondent itself, not
upon his counsel.
Certiorari proceeding is, by nature, an original and independent action, and, therefore not considered as part of the trial that had
resulted in the rendition of the judgment or order complained of. Hence, at the preliminary point of serving the certiorari
petition, as in other initiatory pleadings, it cannot be said that an appearance for respondent has been made by his counsel.
Consequently, the requirement under Section 2, Rule 13 of the Rules, which provides that if any party has appeared by counsel,
service upon him shall be made upon his counsel, should not apply.
In ordinary civil cases, a conditional appearance to object to a trial court's jurisdiction over the person of the defendant may be
made when said party specifically objects to the service of summons, which is an issuance directed by the court, not the
complainant. If the defendant, however, enters a special appearance but grounds the same on the service of the complainant's
initiatory pleading to him, then that would not be considered as an objection to the court's jurisdiction over his person. It must
be underscored that the service of the initiatory pleading has nothing to do with how courts acquire jurisdiction over the person
of the defendant in an ordinary civil action. Rather, it is the propriety of the trial court's service of summons - same as the CA's
service of its resolution indicating its initial action on the certiorari petition - which remains material to the matter of the court's
acquisition jurisdiction over the defendant's/respondents' person.

27. Rapid Realty Jurisdiction over the defendant in a Rapid City Realty and Development Corporation (petitioner) filed a complaint for declaration of nullity of subdivision plans,
and Development civil case is acquired either by the mandamus, and damages against several defendants including Spouses Orlando and Lourdes Villa (respondents).
Corp. vs. Villa 184197 coercive power of legal processes
11 February 2010 exerted over his person, or his After one failed attempt at personal service of summons, Zapanta , court process server, resorted to substituted service by
YUMUL voluntary appearance in court. As a serving summons upon respondents’ househelp who did not acknowledge receipt thereof and refused to divulge their names.
general proposition, one who seeks
an affirmative relief is deemed to Despite substituted service, respondents failed to file their Answer, prompting petitioner to file a "Motion to Declare
have submitted to the jurisdiction Defendants[-herein respondents] in Default" which the trial court granted.
of the court. It is by reason of this
rule that we have had occasion to
declare that the filing of motions to More than eight months thereafter, respondents filed a Motion to Lift Order of Default. Alleging they had only received the
admit answer, for additional time pertinent papers recently and denied the existence of the househelp who refused to sign and acknowledge receipt of the
to file answer, for reconsideration summons. In any event, they contended that assuming that the allegation were true, the helpers had no authority to receive the
of a default judgment, and to lift documents.
order of default with motion for
reconsideration, is considered Trial court set aside the Order of Default but respondents still failed to file their answer within the time required. They were once
voluntary submission to the court’s again declared in default so respondents filed an Omnibus Motion for reconsideration of the second order declaring them in
jurisdiction. default and to vacate proceedings, this time claiming that the trial court did not acquire jurisdiction over their persons due to
invalid service of summons.
This, however, is tempered by the
concept of conditional appearance,
such that a party who makes a
special appearance to challenge, ISSUE/S:
among others, the court’s WON there was an invalid service of summons.
jurisdiction over his person cannot
be considered to have submitted to
HELD:
its authority.
NO. It is settled that if there is no valid service of summons, the court can still acquire jurisdiction over the person of the
defendant by virtue of the latter’s voluntary appearance. Thus Section 20 of Rule 14 of the Rules of Court provides:

Sec. 20. Voluntary appearance. – The defendant’s voluntary appearance in the action shall be equivalent to service of summons.
The inclusion in a motion to dismiss of other grounds aside from lack of jurisdiction over the person shall not be deemed a
voluntary appearance.

Respondents did not, in said motion, allege that their filing thereof was a special appearance for the purpose only to question the
jurisdiction over their persons. Clearly, they had acquiesced to the jurisdiction of the court.

WHEREFORE, the petition is GRANTED. The assailed Court of Appeals Decision is REVERSED and SET ASIDE.

28. PCIB vs. Preliminarily, jurisdiction over the FACTS:


Spouses Wilson Dy defendant in a civil case is acquired Sps. Amadeo are indebted to petitioner PCIB, as sureties for Streamline Cotton Dev. Corp. The promissory notes became due
Hong Pi, et. al 171137 5 either by the coercive power of legal and demandable, but Sps. Amadeo failed to pay their outstanding obligations despite repeated demands, their obligation is at
June 2009 processes exerted over his person, P10, 671,726.61.
ZAPANTA or his voluntary appearance in
court. As a general proposition, one Petitioner subsequently discovered that Sps. Amadeo sold 3 or nearly all of their real properties to respondents, Sps. Dy and Sps.
who seeks an affirmative relief is Chuyaco, and immediately transferred the titles covering the parcels of land in favor of the latter. The consideration for these
deemed to have submitted to the sales was grossly insufficient or inadequate.
jurisdiction of the court.
Believing that the transfers were done in fraud of creditors, petitioner filed an action for rescission and damages, for the
annulment of the Deeds of Absolute Sale and cancel the TCTs issued in the name of Sps. Dy and Sps. Chuyaco, and, issue new
ones under the name of Sps. Amadeo.

Upon service of summons on the Sps. Amadeo, the latter filed a Motion to Dismiss on the ground that the Complaint violated the
explicit terms of SC Circular No. 04-94, as the Verification was executed by petitioner’s legal counsel.

Petitioner filed an Ex Parte Motion for Leave to Serve Summons by Publication on Sps. Dy and Chuyaco. However, this was
denied in on the ground that summons by publication cannot be availed of in an action in personam.Petitioner opposed and
argued that it had already filed a motion for the service of summons by publication, but the trial court had yet to act on it. This
Motion was submitted for resolution.

The motion for inhibition was adopted by their counsel, Clarissa Castro, through a Motion to Adopt Motion for Inhibition and
Manifestation, which was filed and noted by the trial court in an Order. However, the RTC (a) denied the motion for inhibition
for lack of merit, (b) ruled that Sps. Dy and Chuyaco have voluntarily submitted themselves to the jurisdiction of the trial court,
and (c) gave them 15 days from receipt of the Order within which to file their respective answers.
Unsatisfied, respondent Sps. Dy and Chuyaco filed a Petition for Certiorari under Rule 65 before the CA, alleging that the public
respondent committed grave abuse of discretion when he considered the Motion to Inhibit (without submitting to the
jurisdiction of the Honorable Court) which they had filed to question his impartiality and competence due to the delay in
resolving the Motion to Dismiss based on lack of jurisdiction, as voluntary appearance, and wherein he required the respondents
to file their Answer within the required period. The CA granted the petition in this wise:
The old provision under Section 23, Rule 14 of the Revised Rules of Court provided that: Section 23. What is equivalent to
service. The defendants voluntary appearance in the action shall be equivalent to service.

Under Section 20, Rule 14 of the 1997 Rules of Civil Procedure, the provision now reads as follows: Sec. 20. Voluntary
Appearance. The defendant's voluntary appearance in the action shall be equivalent to service of summons. The inclusion in a
motion to dismiss of other grounds aside from lack of jurisdiction over the person of the defendant shall not be deemed a
voluntary appearance.

Petitioner’s motion for reconsideration was denied by the appellate court. Hence this appeal.

ISSUES: W/N there has been voluntary appearance on the part of respondent Spouses Dy and Chuyaco as to confer the trial
court with jurisdiction over their persons.

HELD:

Preliminarily, jurisdiction over the defendant in a civil case is acquired either by the coercive power of legal processes exerted
over his person, or his voluntary appearance in court. As a general proposition, one who seeks an affirmative relief is deemed to
have submitted to the jurisdiction of the court. It is by reason of this rule that we have had occasion to declare that the filing of
motions to admit answer, for additional time to file answer, for reconsideration of a default judgment, and to lift order of default
with motion for reconsideration, is considered voluntary submission to the courts jurisdiction. This, however, is tempered by the
concept of conditional appearance, such that a party who makes a special appearance to challenge, among others, the courts
jurisdiction over his person cannot be considered to have submitted to its authority.

It is readily apparent that respondents have acquiesced to the jurisdiction of the trial court, when they filed their Motion to
Dismiss for Failure to Prosecute. Significantly, the motion did not categorically and expressly raise the jurisdiction of the court
over their persons as an issue. It merely (a) reminded the court of its purportedly conflicting Orders in respect of summons by
publication, (b) alleged that because petitioner has not lifted a finger to pursue this case against movants-defendants, the case
may be dismissed for failure to prosecute, and (c) prayed additionally for the deletion of the Notice of Lis Pendens indicated at
the back of the transfer certificates of title covering the subject properties. We note, that the motion failed to qualify the capacity
in which respondents were appearing and seeking recourse.
Doubts on the issue of voluntary appearance dissipate when the respondents motion for inhibition is considered. This motion
seeks a sole relief: inhibition of Judge Inoturan from further hearing the case. Evidently, by seeking affirmative relief other than
dismissal of the case, respondents manifested their voluntary submission to the courts jurisdiction. It is well-settled that the
active participation of a party in the proceedings is tantamount to an invocation of the courts jurisdiction and a willingness to
abide by the resolution of the case, and will bar said party from later on impugning the courts jurisdiction.

To be sure, the convenient caveat in the title of the motion for inhibition does not detract from this conclusion. It would suffice
to say that the allegations in a pleading or motion are determinative of its nature; the designation or caption thereof is not
controlling. Furthermore, no amount of caveat can change the fact that respondents tellingly signed the motion to inhibit in their
own behalf and not through counsel, let alone through a counsel making a special appearance.
XVI. MOTIONS (RULE 15, SEC. 1 TO 13)
29. Sarmiento The notice requirement in a motion is FACTS:
vs. Zaratan mandatory. As a rule, a motion without a Petitioner Gliceria Sarmiento filed an ejectment case against respondent Emerita Zaratan, in the MeTC of QC. The MeTC
167471 5 February Notice of Hearing is considered pro rendered a decision in favor of petitioner.
2007 forma and does not affect the
ABAD reglementary period for the appeal or the Respondent filed her notice of appeal. Thereafter, the case was raffled to the RTC of QC.
filing of the requisite pleading.
As a general rule, notice of motion is In the Notice of Appealed Case, the RTC directed respondent to submit her memorandum in accordance with the provisions of
required where a party has a right to Section 7 (b) of Rule 40 of the Rules of Court and petitioner to file a reply memorandum within 15 days from receipt.
resist the relief sought by the motion and
principles of natural justice demand that
his right be not affected without an Respondent's counsel having received the notice on 19 May 2003, he had until 3 June 2003 within which to file the requisite
opportunity to be heard. The three-day memorandum. But on 3 June 2003, he filed a Motion for Extension of Time of five days due to his failure to finish the draft of
notice required by law is intended not for the said Memorandum. He cited as reasons for the delay of filing his illness for one week, lack of staff to do the work due to
the benefit of the movant but to avoid storm and flood compounded by the grounding of the computers because the wirings got wet. But the motion remained unacted.
surprises upon the adverse party and to On June 9 2003, respondent filed her Memorandum.
give the latter time to study and meet the
arguments of the motion. Principles of RTC - dismissed the appeal as the defendant-appellant filed the required Memorandum only on June 9, 2003 or six (6) days
natural justice demand that the right of a beyond the expiration of the aforesaid fifteen day period.
party should not be affected without
giving it an opportunity to be heard. Petitioner filed a Motion for Immediate Execution, while respondent moved for the Reconsideration. Both motions were denied
by the RTC.
Further, it has been held that a "motion
for extension of time . . . is not a litigated RTC reconsidered its previous Order by granting petitioner's motion for Immediate Execution, but denied respondent's Motion
motion where notice to the adverse party for Clarification.
is necessary to afford the latter an
opportunity to resist the application, but CA - respondent filed a Petition for Certiorari in the CA, which was granted. CA nullified and set aside the 19 June 2003 and 31
an ex parte motion made to the court in July 2003 Orders of the RTC and ordered the reinstatement of respondent's appeal. Consequently, respondent's appeal
behalf of one or the other of the parties to memorandum was admitted and the case remanded to the RTC for further proceedings.
the action, in the absence and usually
without the knowledge of the other party
Petitioner filed a motion for reconsideration but was denied for lack of merit.
or parties." It has been said that "ex parte
motions are frequently permissible in
procedural matters, and also in situations ISSUE:
and under circumstances of emergency; Whether the lack of notice of hearing in the Motion for Extension of Time to file Memorandum on Appeal is fatal, such that the
and an exception to a rule requiring filing of the motion is a worthless piece of paper
notice is sometimes made where notice
or the resulting delay might tend to HELD:
defeat the objective of the motion. It is not disputed that respondent perfected her appeal on 4 April 2003 with the filing of her Notice of Appeal and payment of
the required docket fees. However, before the expiration of time to file the Memorandum, she filed a Motion for Extension of
Time seeking an additional period of five days within which to file her Memorandum, which motion lacked the Notice of Hearing
required by Section 4, Rule 15 of the 1997 Rules of Court which provides:

 SEC. 4. Hearing of Motion. — Except for motions which the court may act upon without prejudicing the rights of the
adverse party, every written motion shall be set for hearing by the applicant.

Every written motion required to be heard and the notice of the hearing thereof shall be served in such a manner as to ensure its
receipt by the other party at least three (3) days before the date of hearing, unless the court for good cause sets the hearing on
shorter notice.

The notice requirement in a motion is mandatory. As a rule, a motion without a Notice of Hearing is
considered pro forma and does not affect the reglementary period for the appeal or the filing of the requisite
pleading.

As a general rule, notice of motion is required where a party has a right to resist the relief sought by the motion and principles of
natural justice demand that his right be not affected without an opportunity to be heard. 20 The three-day notice required by law
is intended not for the benefit of the movant but to avoid surprises upon the adverse party and to give the latter time to study
and meet the arguments of the motion. Principles of natural justice demand that the right of a party should not be affected
without giving it an opportunity to be heard.

Considering the circumstances of the present case, we believe that procedural due process was substantially complied
with.

The suspension of the Rules is warranted in this case. The motion in question does not affect the substantive rights of petitioner
as it merely seeks to extend the period to file Memorandum. The required extension was due to respondent's counsel's illness,
lack of staff to do the work due to storm and flood, compounded by the grounding of the computers. There is no claim likewise
that said motion was interposed to delay the appeal. As it appears, respondent sought extension prior to the expiration of the
time to do so and the memorandum was subsequently filed within the requested extended period.

Further, it has been held that a "motion for extension of time . . . is not a litigated motion where notice to the
adverse party is necessary to afford the latter an opportunity to resist the application, but an ex parte motion
made to the court in behalf of one or the other of the parties to the action, in the absence and usually without
the knowledge of the other party or parties." It has been said that "ex parte motions are frequently permissible in
procedural matters, and also in situations and under circumstances of emergency; and an exception to a rule requiring notice is
sometimes made where notice or the resulting delay might tend to defeat the objective of the motion.

30. Vette When the purpose of Notice and Hearing Facts:


Industrial vs. has been served, the three-day notice A complaint for specific performance and damages was filed against Vette Industrial Sales Co., Inc. Sui (Respondent) alleged
Cheng 170232- under Rule 15, Sec 3 shall deviate from that on October 2001, he executed a Deed of Assignment, where he transferred his 40,000 shares in the company in favor of
170301 5 the literal application of the rule. petitioners-assignees. To implement the Deed of Assignment, the company acknowledged in a MOA, that it owed him 6.8
December 2006 million pesos plus insurance proceeds. 48 postdated checks were issued but after the 11th check, the rest were dishonored by the
ARIBON bank. In an answer with compulsory counterclaim, petitioners alleged that the MOA was unenforceable. Sui filed a motion to set
pre-trial. Petitioners received the motion but did not attend because there was no notice from the court setting the pre-trial date.
On the rescheduled date for pre-trial, Sui and counsel failed to appear. The trial court ordered the dismissal of the case.
Respondent’s Attorney filed a manifestation and motion for consideration of the order of dismissal. Petitioners asserted that the
MR be denied because, among others, that sui did not comply with the 3 day notice rule under Rule 15, sec 4, considering that
the manifestation and MR was received only one day prior to the date of hearing of the motion for resolution, and thus the same
must be treated as a mere scrap of paper. Trial court ruled in favor of Sui. CA held that the dismissal of the case was proper but
without prejudice to filing a new action. Hence, these consolidated petitions
Issue:
W/N CA erred in not considering respondent’s manifestation and motion for reconsideration.
Held:
Yes. The Court is not persuaded with the argument imposed by the petitioner that since respondent violated the three-day notice
rule, its motion is a mere scrap of paper. Rather, the Court finds that the purpose of notice and hearing has been served in this
case. When the trial court received Sui’s Manifestation and Motion for Reconsideration, it did not immediately resolve the
motion. Instead, it allowed petitioners to file their comment and also leave to file a rejoinder if Sui files a reply. These
circumstances justify a departure from the literal application of the rule because petitioners were given the opportunity to study
and answer the arguments in the motion.

31. Boiser vs. A motion without notice of hearing is pro FACTS: Complainant Alfredo Boiser was the plaintiff in an ejectment case filed before the Municipal Trial Court of
Aguirre A.M. forma, a mere scrap of paper. It presents Himamaylan City, Negros Occidental. On July 11 2003, the Municipal Trial Court rendered a decision in favor of the
RTJ-04-1886 16 no question which the court could decide. complainant.
May 2005 The court has no reason to consider it Defendant-appellant Salvador Julleza filed a motion to release bond on the ground that the MTC of Hinigaran, Negros
BRIONES and the clerk has no right to receive it. Occidental, in its decision dated July 11, 2003, has already resolved the writ of preliminary injunction without the mentioning
The rationale behind the rule is plain: the applicant’s liability. Respondent Judge granted the motion.
unless the movant sets the time and place
of hearing, the court will be unable to ISSUE: Whether or not the Judge acted with grave abuse of discretion.
determine whether the adverse party
agrees or objects to the motion, and if he HELD: Yes, respondent judge acted with grave abuse of discretion. The Rules of Court requires that every motion must be set
objects, to hear him on his objection, for hearing by the movant, except those motions which the court may act upon without prejudicing the rights of the adverse
since the rules themselves do not fix any party. It appears that the Motion to Release Bond was defective as it didn’t have a proper notice of hearing. The date and time of
period within which he may file his reply the hearing were not specified. Neither complainant nor his counsel furnished a copy thereof. These were never controverted by
or opposition. the respondent judge. A motion without notice of hearing is pro forma, a mere scrap of paper.

32. De los The subject motion is a non-litigious FACTS:


Reyes vs. motion. While, as a general rule, all
Ramnani 169135 written motions should be set for hearing On October 11, 1977, the trial court rendered a Decision in Civil Case No. 24858 in favor of respondent Josephine Anne
18 June 2010 under Section 4 Rule 15 of the Rules of B. Ramnani. Thereafter, a writ of execution was issued by the trial court. On June 6, 1978, then Branch Sheriff Pedro T. Alarcon
CASTRO Court, excepted from this rule are non- conducted a public bidding and auction sale over the property covered by Transfer Certificate of Title (TCT) No. 480537 (subject
litigious motions or motions which may property) during which respondent was the highest bidder. Consequently, a certificate of sale was executed in her favor on even
be acted upon by the court without date. On November 17, 1978, a writ of possession was issued by the trial court. On March 8, 1990, the certificate of sale was
prejudicing the rights of the adverse annotated at the back of TCT No. 480537. Thereafter, the taxes due on the sale of the subject property were paid on September
party. 26, 2001.

Respondent is entitled to the issuance of PROCEDURAL HISTORY:


the final certificate of sale as a matter of
right and petitioner is powerless to On February 17, 2004, respondent filed a motion (subject motion) for the issuance of an order directing the sheriff to
oppose the same. The subject motion execute the final certificate of sale in her favor. Petitioner opposed on the twin grounds that the subject motion was not
falls under the class of non-litigious accompanied by a notice of hearing and that the trial court’s October 11, 1977 Decision can no longer be executed as it is barred
motions. by prescription. The trial court ruled that the prescription for the issuance of a writ of execution is not applicable in this case.
Petitioner did not exercise his right to redeem the subject property within one year from said registration. Thus, what remains to
be done is the issuance of the final certificate of sale which was, however, not promptly accomplished at that time due to the
demise of the trial court’s sheriff. The issuance of the final certificate of sale is a ministerial duty of the sheriff in order to
complete the already enforced judgment. CA denied the petition.

ISSUE:
1. Whether the subject motion was defective for lack of a notice of hearing?
2. Whether respondent is barred by prescription, laches or estoppel

RULING:
1. NO. Motion was valid. The subject motion is a non-litigious motion. While, as a general rule, all written motions
should be set for hearing under Section 4 Rule 15 of the Rules of Court, excepted from this rule are non-litigious
motions or motions which may be acted upon by the court without prejudicing the rights of the adverse party.
Respondent is entitled to the issuance of the final certificate of sale as a matter of right and petitioner is powerless to
oppose the same. The subject motion falls under the class of non-litigious motions.
2. No. Respondent is entitled to the issuance of the final certificate of sale as a matter of right. It is, likewise, not
disputed that petitioner failed to redeem the subject property within one year from the annotation of the certificate
of sale on TCT No. 480537. The expiration of the one-year redemption period foreclosed petitioner’s right to redeem
the subject property and the sale thereby became absolute. The issuance thereafter of a final certificate of sale is a
mere formality and confirmation of the title that is already vested in respondent. Thus, the trial court properly
granted the motion for issuance of the final certificate of sale.

33. Preysler The three-day notice rule is not absolute. FACTS:


vs. Manila A liberal construction of the procedural
Southcoast Dev. rules is proper where the lapse in the Preysler filed with the Municipal Trial Court (MTC) of Batangas a complaint for forcible entry against Manila Southcoast
Corp. 171872 28 literal observance of a rule of procedure Development Corporation. The subject matter of the complaint is a parcel of land with an area of 21,922 square meters located in
June 2010 has not prejudiced the adverse party and Sitio Kutad, Barangay Papaya, Nasugbu, Batangas. The disputed land, covered by Transfer Certificate of Title (TCT) No. TF-1217
DE GALA has not deprived the court of its in the name of Preysler, is also within the property covered by TCT No. T-72097 in the name of respondent company. TCT No. T-
authority. Indeed, Section 6, Rule 1 of the 72097 covers three contiguous parcels of land with an aggregate area of 86,507,778 square meters.
Rules of Court provides that the Rules MTC: Ruled in favor of Preysler and ordered respondent company to vacate the disputed land covered by TCT No. TF-1217 and
should be liberally construed in order to to return the possession.
promote their objective of securing a just,
speedy and inexpensive disposition of RTC: Reversed the MTC decision and dismissed the complaint
every action and proceeding. Rules of
procedure are tools designed to Preysler received the RTC Decision on 9 February 2004 and thereafter filed a Motion for Reconsideration, which was set for
facilitate... the attainment of justice, and hearing on 26 February 2004. Preysler sent a copy of the Motion for Reconsideration to respondent company’s counsel by
courts must avoid their strict and rigid registered mail on 23 February 2004. During the 26 February 2004 scheduled hearing of the motion, the RTC judge reset the
application which would result in hearing to 2 April 2004 because the courts calendar could not accommodate the hearing of the motion. It was only on 3 March
technicalities that tend to frustrate rather 2004, or 6 days after the scheduled hearing on 26 February 2004, that respondents counsel received a copy of petitioners
than promote substantial justice. Motion for Reconsideration. The rescheduled hearing on 2 April 2004 was again reset on 7 May 2004 because the RTC judge
was on official leave. The 7 May 2004 hearing was further reset to 6 August 2004.
After the hearing, respondent company filed its Motion to Dismiss dated 9 August 2004, claiming that non-compliance
with the three-day notice rule did not toll the running of the period of appeal, which rendered the decision final.
RTC: On October 4, 2004, denied the Motion for Reconsideration for failure to appeal within the 15 days reglementary period
and declaring the 22 January 2004 Decision as final and executory. Motion for Reconsideration was fatally flawed for failure to
observe the three-day notice rule. Preysler filed an Omnibus Motion for Reconsideration of the Order dated 4 October 2004.
RTC: In its Order dated 22 February 2005, dismissed the Omnibus Motion.
A petition for certiorari was then filed with the Court of Appeals, alleging that the RTC committed grave abuse of discretion in
dismissing the Motion for Reconsideration and Omnibus Motion for alleged failure to observe the three-day notice rule.
CA: Dismissed the petition.
The three-day notice rule under Sections 4, 5, and 6 of Rule 15 of the Rules of Court is mandatory and non-compliance therewith
is fatal and renders the motion pro forma. As found by the RTC, Preysler’s Motion for Reconsideration dated 12 February 2004
was received by respondent only on 3 March 2004, or six days after the scheduled hearing on 26 February 2004. All violations of
Sections 4, 5, and 6 of Rule 15 are deemed fatal. Thus, this petition.

ISSUE:
Whether or not the three-day notice rule is absolute

RULING:
The three-day notice rule is not absolute. A liberal construction of the procedural rules is proper where the lapse in the
literal observance of a rule of procedure has not prejudiced the adverse party and has not deprived the court of its authority.
Indeed, Section 6, Rule 1 of the Rules of Court provides that the Rules should be liberally construed in order to promote their
objective of securing a just, speedy and inexpensive disposition of every action and proceeding. Rules of procedure are tools
designed to facilitate... the attainment of justice, and courts must avoid their strict and rigid application which would result in
technicalities that tend to frustrate rather than promote substantial justice.

In Somera Vda. De Navarro v. Navarro, the Court held that there was substantial compliance of the rule on notice of
motions even if the first notice was irregular because no prejudice was caused the adverse party since the motion was not...
considered and resolved until after several postponements of which the parties were duly notified.
Likewise, in Jehan Shipping Corporation v. National Food Authority, the Court held that despite the lack of notice of hearing in a
Motion for Reconsideration, there was substantial compliance with the requirements of due process where the adverse party
actually had the opportunity to be heard and had filed pleadings in opposition to the motion. This Court has indeed held time
and again, that under Sections 4 and 5 of Rule 15 of the Rules of Court, mandatory is the requirement in a motion, which is
rendered defective by failure to comply with the requirement. As a rule, a motion without a notice of hearing is considered pro
forma and does not affect the reglementary period for the appeal or the filing of the requisite pleading. As an integral component
of the procedural due process, the three-day notice required by the Rules is not intended for the benefit of the movant. Rather,
the requirement is for the purpose of avoiding surprises that may be sprung upon the adverse party, who must be given time to
study and meet the arguments in the motion before a resolution of the court. Principles of natural justice demand that the right
of a party should not be affected without giving it an opportunity to be heard.
The test is the presence of opportunity to be heard, as well as to have time to study the motion and meaningfully oppose or
controvert the grounds upon which it is based. x x x
In this case, the Court of Appeals ruled that petitioner failed to comply with the three-day notice rule. However, the
Court of Appeals overlooked the fact that although respondent received petitioner's Motion for Reconsideration six days after
the scheduled hearing on 26
February 2004, the said hearing was reset three (3) times with due notice to the parties. Thus, it was only on 6 August 2004, or
more than five months after respondent received a copy of petitioner's Motion for Reconsideration, that the motion was heard by
the RTC. Clearly, respondent had more than sufficient time to oppose petitioner's Motion for Reconsideration. In fact,
respondent did oppose the motion when it filed its Motion to Dismiss dated 9 August 2004. In view of the circumstances of this
case, we find that there was substantial compliance with procedural due process. Instead of dismissing petitioner's Motion for
Reconsideration based merely on the alleged procedural lapses, the RTC should have resolved the motion based on the merits.
The RTC likewise erred in dismissing petitioner's Omnibus Motion for allegedly failing to comply with the three-day notice
requirement. It held that the service of the notice of hearing was one day short of the prescribed minimum three day notice.
Petitioner's Omnibus Motion which was set to be heard on 12 November 2004 was received by respondent on 9 November 2004.
We disagree. Section 4 of Rule 15 provides that "[e]very written motion required to be heard and the notice of the hearing
thereof shall be served in such a manner as to ensure its receipt by the other party at least three (3) days before the date of the
hearing, unless the court for good cause sets the hearing on shorter notice." Thus, the date of the hearing should be at least three
days after receipt of the notice of hearing by the other parties. In this case, the petitioner's Omnibus Motion was set for hearing
on 12 November 2004. Thus, to comply with the notice requirement, respondent should have received the notice of the hearing
at least three days before 12 November 2004, which is 9 November 2004. Clearly, respondent's receipt on 9 November 2004
(Tuesday) of the notice of hearing of the Omnibus Motion which was set to be heard on 12 November 2004 (Friday), was within
the required minimum three-days' notice. The ordinary motion day is Friday. Hence, the notice should be served by Tuesday at
the latest, in order that the requirement of the three days may be complied with. If notice be given by ordinary mail, it should be
actually received by Tuesday, or if not claimed from the post office, the date of the first notice of the postmaster should be at
least five (5) days before Tuesday. WHEREFORE, we GRANT the petition.

34. PNB vs.


It is a basic rule of remedial law that a Facts: Respondents Deang Marketing Corporation and Berlita Deang (DMC) filed before the RTC of Angeles a Complaint
Deang Marketing
motion for extension of time to file a against petitioner Philippine National Bank (PNB) for reformation of contract and specific performance, claiming that a
Corp. 177931 8
pleading must be filed before the dacion en pago arrangement in the Consolidation and Restructuring Agreement forged by them transformed DMC’s
December 2008
expiration of the period sought to be outstanding loan obligations into a 7-year term loan of P36.4 million. Summons was served on petitioner on April 20, 2006.
ESPIRITU
extended. On May 15, 2006, DMC filed a Motion to Declare petitioner in Default, which they set for hearing on May 24, 2006.
The court’s discretion to grant a motion Thereafter, the trial court received PNB’s Motion for Extension of Time [30 days up to June 11, 2006] to File Answer dated
for extension is conditioned upon such May 5, 2006.
motion’s timeliness, the passing of which
renders the court powerless to entertain The following day, May 16, 2006 or eight days prior to the slated hearing of DMC's Motion to Declare PNB in Default in which
or grant it. the trial court denied such motion but granted PNB's Motion for Extension of Time to File Answer. To the trial court's Order
Since the motion for extension was filed DMC filed a Motion for Reconsideration. In the meantime, PNB filed its Answer to the Complaint on May 25, 2006. The trial
after the lapse of the prescribed period, court, by Order of August 9, 2006, denied DMC’s Motion for Reconsideration of its May 16, 2006 Order denying their Motion
there was no more period to extend. to Declare PNB in default and granting the latter's Motion for Extension.
DMC subsequently assailed the trial court's Orders of May 16, 2006 and August 9, 2006 via certiorari to the Court of Appeals
which annulled the trial court's orders and remanded the same.

Issue: Whether or not the CA erred in declaring PNB in default


Ruling: NO. Petitioner had, following the reglementary 15-day period after service of summons (unless a different period is
fixed by the court), until May 5, 2006 within which to file an Answer or appropriate pleading. It filed the Motion for Extension,
however, via a private courier on May 14, 2006, which was received by the trial court on May 15, 2006 or ten days late.
Petitioner was not candid enough to aver in the Motion for Extension that the period had lapsed, as it still toyed with the idea
that it could get away with it. The allegations therein were crafted as if the said motion was timely filed. Notably, the May 16,
2006 Order expressed no inkling that the motion was filed out of time. The trial court either was deceived by or it casually
disregarded the apparent falsity foisted by petitioner. Lest this Court be similarly deceived, it is imperative to carefully
examine the facts.
In requesting for a 30-day extension or until June 11, 2006 to file an answer, petitioner apparently reckoned the date from
which the extension would start on May 12, 2006, which was not the last day of the 15-day period sought to be extended, it
being May 5, 2006. By computation, petitioner sought more than 30 days, contrary to the period of extension it purportedly
requested. The counting of the period was erroneous, even if one uses the material dates alleged by petitioner. Petitioner
clearly disregarded elementary rules and jurisprudence on the matter.
In the present case, no satisfactory reason was adduced to justify the tardiness of the Answer and no compelling reason was
given to justify its admission. The intention to delay was rather obvious. The Court thus finds petitioner's negligence
inexcusable, as the circumstances behind and the reasons for the delay are detestable.

RULE 16 - MOTION TO DISMISS [PROVISION DELETED - RELEVANT CASES TRANSFERRED TO RULE 8]

XVII. DISMISSAL OF ACTIONS [RULE 17, SEC. 1 TO 4)


35. O.B. Jovenir Section 1, Rule 17 of the 1964 Rules Facts: A complaint was filed before the Regional Trial Court (RTC) of Makati City, with private respondents Macamir Realty
Construction of Civil Procedure stated: and Development Corp. (Macamir Realty) and spouses Rosauro and Gloria Miranda as plaintiffs, and petitioners O.B. Jovenir
Development Corp. vs. Construction and Development Corp. (Jovenir Construction), Oscar B. Jovenir, and Gregorio Liongson being among the
Macamir Realty and Dismissal by the plaintiff — An defendants. The complaint, sought the annulment of certain agreements between private respondents and petitioners, as well
Court of Appeals action may be dismissed by the as damages. It was alleged that Jovenir Construction was contracted to complete the construction of private respondents
135803 plaintiff without order of court by condominium project. Private respondents subsequently sought the termination of their agreements with petitioners after it
26 March 2006 filing a notice of dismissal at any was discovered that Jovenir Construction had misrepresented itself as a legitimate contractor. Respondents likewise prayed
LEE time before service of the answer or for the issuance of a writ of preliminary injunction. A hearing on the prayer appears to have been conducted.It was also
of a motion for summary judgment. alleged in the complaint that Gloria Miranda was the principal stockholder and President of Macamir Realty while her
Unless otherwise stated in the husband Rosauro was the owner of the real properties on which the condominium project was being constructed. It was also
notice, the dismissal is without alleged in the complaint that Gloria Miranda was the principal stockholder and President of Macamir Realty while her
prejudice, except that a notice husband Rosauro was the owner of the real properties on which the condominium project was being constructed. 10 days after
operates as an adjudication upon the filing of the complaint, private respondents filed a Motion to Withdraw Complaint, alleging that during the initial hearing
the merits when filed by a plaintiff on the prayer for preliminary injunction, counsel for plaintiffs "discovered a supposed technical defect in the complaint . . .
who has once dismissed in a that . . . may be a ground for the dismissal of this case." Thus, private respondents prayed that the plaintiffs be allowed to
competent court an action based on withdraw the complaint without prejudice. Petitioners filed an opposition to the Motion to Withdraw Complaint, wherein they
or including the same claim. A class adopted Madeja's arguments as to the lack of authority on the part of the spouses Miranda to sue on behalf of Macamir Realty.
suit shall not be dismissed or However, just one day earlier, private respondents filed another complaint against the same defendants save for Madeja, and
compromised without the approval seeking the same reliefs as the first complaint. This time, a Board Resolution authorizing the spouses Miranda to file the
of the court. Complaint on behalf of Macamir Realty was attached to the complaint. This second complaint was also filed with the Makati
It is quite clear that under Section RTC. The Verification and Certification [of] Non-Forum Shopping in the second complaint was accomplished by Rosauro
1, Rule 17 of the old Rules, the Miranda. 11 days after the filing of the Motion to Withdraw Complaint and seven days after the filing of the second Complaint,
dismissal contemplated therein the Makati RTC, granted the Motion to Withdraw Complaint. The RTC noted in its Order that "an action may be dismissed by
could be accomplished by the the plaintiffs even without Order of the Court by filing a notice of dismissal at anytime before the service of the answer under
plaintiff through mere notice of Rule 17, Section 1 of the Rules of Court," and accordingly considered the complaint withdrawn without prejudice.
dismissal, and not through motion The battle then shifted to the case which had been raffled to the Makati RTC. The petitioners filed a Motion to Dismiss the
subject to approval by the Court. second complaint on the ground of forum-shopping. They pointed out that at the time of the filing of the second complaint,
Dismissal is ipso facto upon notice, the first complaint was still pending. The Makati RTC denied the Motion to Dismiss, observing that at the time the Motion to
and without prejudice unless Withdraw Complaint was filed, none of the defendants had filed any answer or any responsive pleading. Thus, it was then
otherwise stated in the notice. It is within respondents' right to cause the dismissal of the complaint without having to await action of the court on their motion.
due to these considerations that the Court of Appeals: This Order was affirmed by the Court of Appeals Special Sixth Division in its Decision after petitioners
petition should be denied. had assailed the RTC's order via a special civil action for certiorari filed with the appellate court. Hence, the present petition
As noted at the onset, the 1997 Issue: Whether the RTC erred in the denial of the motion to dismiss
Rules of Civil Procedure now Held: No. The denial of the motion to dismiss is wholly in accord with the Rules of Civil Procedure. Section 1, Rule 17 of the
requires that upon the filing of such 1964 Rules of Civil Procedure stated:
notice, the court issue an order
confirming the dismissal. The new Dismissal by the plaintiff — An action may be dismissed by the plaintiff without order of court by filing a notice of dismissal at
requirement is intended to qualify any time before service of the answer or of a motion for summary judgment. Unless otherwise stated in the notice, the
the right of a party to dismiss the dismissal is without prejudice, except that a notice operates as an adjudication upon the merits when filed by a plaintiff who
action before the adverse party files has once dismissed in a competent court an action based on or including the same claim. A class suit shall not be dismissed or
an answer or asks for summary compromised without the approval of the court.
judgment.
Indubitably, the provision ordained the dismissal of the complaint by the plaintiff as a matter of right at any time before
service of the answer. The plaintiff was accorded the right to dismiss the complaint without the necessity of alleging in the
notice of dismissal any ground nor of making any reservation. It is quite clear that under Section 1, Rule 17 of the old Rules,
the dismissal contemplated therein could be accomplished by the plaintiff through mere notice of dismissal, and not through
motion subject to approval by the Court. Dismissal is ipso facto upon notice, and without prejudice unless otherwise stated in
the notice. It is due to these considerations that the petition should be denied.
Evidently, respondents had the right to dismiss their complaint by mere notice since petitioners had not yet served their
answer on respondents. The Motion to Withdraw Complaint makes clear respondents' "desire to withdraw the complaint
without prejudice." That respondents resorted to a motion to effect what they could have instead by mere notice may be
indicative of a certain degree of ignorance of procedural rules on the part of respondents' counsel. Yet such "error," if it could
be called as such, should hardly be of fatal consequence. Petitioners posit that the "remedy" of filing a notice of dismissal is
not exclusive, respondents having the "option" of securing the court's approval to the dismissal. On the contrary, the trial
court has no discretion or option to deny the motion, since dismissal by the plaintiff under Section 1, Rule 17 is guaranteed as
a matter of right to the plaintiffs. Even if the motion cites the most ridiculous of grounds for dismissal, the trial court has no
choice but to consider the complaint as dismissed, since the plaintiff may opt for such dismissal as a matter of right, regardless
of ground.
Thus, the complaint could be properly considered as having been dismissed or withdrawn. Accordingly, when respondents
filed their new complaint relating to the same cause of action, the old complaint was no longer pending. The certification
against forum-shopping attached to the new complaint correctly asseverated that the old complaint "was withdrawn.
As noted at the onset, the 1997 Rules of Civil Procedure now requires that upon the filing of such notice, the court issue an
order confirming the dismissal. The new requirement is intended to qualify the right of a party to dismiss the action before the
adverse party files an answer or asks for summary judgment. Still, there is no cause to apply the 1997 Rules retroactively to
this case. A plaintiff's right to cause the dismissal of his complaint under the old rules was unqualified. Procedural rules may
not be given retroactive effect if vested rights would be disturbed, or if their application would not be feasible or would work
injustice. Since respondents possessed an unqualified right to cause the dismissal of their complaint without need of
confirmation by the trial court, as enunciated in the 1964 Rules, they did not err in asserting that their first complaint was
withdrawn on the day of the filing of their motion to withdraw, and the lower courts were correct in agreeing with respondents
on this point.

36. Ching vs. Cheng Rule 17 of the Rules of Civil Facts:


175507 8 October 2014 Procedure governs dismissals of Antonio Ching owned several businesses and properties, among which was Po Wing Properties. While he was unmarried, he
LIMIN actions at the instance of the had children from two women. Ramon Ching alleged that he was the only child of Antonio Ching with his common-law wife,
plaintiff. Hence, the "two-dismissal Lucina Santos. Joseph Cheng and Jaime Cheng, on the other hand, claim to be Antonio Ching's illegitimate children with his
rule" under Rule 17, Section 1 of the housemaid, Mercedes Igne. One day, Antonio Ching was murdered. Ramon Ching allegedly induced Mercedes Igne and her
Rules of Civil Procedure will not children, Joseph Cheng and Jaime Cheng (the Chengs), to sign an agreement and waiver to Antonio Ching's estate in
apply if the prior dismissal was consideration of PhP22.5 million. Mercedes Igne's children alleged that Ramon Ching never paid them. Ramon Ching
done at the instance of the allegedly executed an affidavit of settlement of estate, naming himself as the sole heir and adjudicating upon himself the
defendant. entirety of Antonio Ching's estate. The Chengs eventually filed a complaint for declaration of nullity of titles against Ramon
Ching before the RTC, impleading Po Wing Properties thereafter. Po Wing Properties filed a motion to dismiss on the ground
of lack of jurisdiction of the subject matter. The RTC granted the Motion. Upon motion of the Chengs' counsel, the Chengs and
Lucina Santos were given 15 days to file the appropriate pleading. They did not do so. Later on, the Chengs and Lucina Santos
filed a complaint for "Annulment of Agreement, Waiver, Extra-Judicial Settlement of Estate and the Certificates of Title
Issued by Virtue of Said Documents with Prayer for TRO and Writ of Preliminary Injunction" against Ramon Ching and Po
Wing Properties. However, Chengs and Lucina Santos filed a motion to dismiss their complaint in the second case, praying
that it be dismissed without prejudice. RTC granted the motion on the basis that the summons had not yet been served on
Ramon Ching and Po Wing Properties, and they had not yet filed any responsive pleading. The dismissal of the second case
was made without prejudice. Ramon Ching and Po Wing Properties filed a motion for reconsideration, arguing that the
dismissal should have been with prejudice under the "two-dismissal rule" of Rule 17, Section 1 of the Rules of Civil Procedure,
in view of the previous dismissal of the first case. During the pendency of the motion for reconsideration, the Chengs and
Lucina Santos filed a complaint for "Disinheritance and Declaration of Nullity of Agreement and Waiver, Affidavit of
Extrajudicial Agreement, Deed of Absolute Sale, and Transfer Certificates of Title with Prayer for TRO and Writ of Preliminary
Injunction" against Ramon Ching and Po Wing Properties. The RTC denied the motion for reconsideration by Ramon Ching
and Po Wing Properties. The CA upheld the denial, ruling that Ramon Ching and Po Wing Properties' reliance on the "two-
dismissal rule" was misplaced since the rule involves two motions for dismissals filed by the plaintiff only. In this case, it
found that the dismissal of the first case was upon the motion of the defendants, while the dismissal of the second case was at
the instance of the plaintiffs.

Issue:
Whether the trial court's dismissal of the second case operated as a bar to the filing of a third case, as per the "two-dismissal
rule."

Ruling:
No. As a general rule, dismissals under Section 1 of Rule 17 are without prejudice except when it is the second time that the
plaintiff caused its dismissal. The purpose of the "two-dismissal rule" is "to avoid vexatious litigation." In this case, the first
action was filed as an ordinary civil action. It was later amended to include not only new defendants but new causes of action
that should have been adjudicated in a special proceeding. A motion to dismiss was inevitably filed by the defendants on the
ground of lack of jurisdiction. The trial court granted the motion. When it allowed Atty. Mirardo Arroyo Obias a period of 15
days to file an appropriate pleading, it was merely acquiescing to a request made by the plaintiff's counsel that had no bearing
on the dismissal of the case. It does not constitute a dismissal based on an adjudication upon the merits, in accordance with
Rule 17, Section 3, i.e., a dismissal through the default of the plaintiff. This provision does not contemplate a situation where
the dismissal was due to lack of jurisdiction. The dismissal of the first case was done at the instance of the defendant under
Rule 16. Also, petitioners do not deny that the second dismissal was requested by respondents before the service of any
responsive pleadings. Accordingly, the dismissal at this instance is a matter of right that is not subject to the trial court's
discretion. For this reason, the trial court dismissed the case, without prejudice. When respondents filed the third case on
substantially the same claim, there was already one prior dismissal at the instance of the plaintiffs and one prior dismissal at
the instance of the defendants. Even assuming for the sake of argument that the failure of Atty. Mirardo Arroyo Obias to file
the appropriate pleading in the first case came under the purview of Rule 17, Section 3 of the Rules of Civil Procedure, the
dismissal in the second case is still considered as one without prejudice due to the express statement from the RTC.

However, while the dismissal of the second case was without prejudice, respondents' act of filing the third case while
petitioners' motion for reconsideration was still pending constituted forum shopping. The prudent thing that respondents
could have done was to wait until the final disposition of the second case before filing the third case. While it may be true that
the trial court already dismissed the second case when the third case was filed, it failed to take into account that a motion for
reconsideration was filed in the second case and, thus, was still pending. Nonetheless, the motion for reconsideration filed in
the second case has since been dismissed and is now the subject of a petition for certiorari. The third case filed apparently
contains the better cause of action for the plaintiffs and is now being prosecuted by a counsel they are more comfortable with.
Substantial justice will be better served if respondents do not fall victim to the labyrinth in the procedures that their travails
led them. It is for this reason that we deny the petition.

37. Cruz vs. Court The doctrine of res judicata is a FACTS: There are four (4) cases involved in this controversy. The first case that was filed between the parties is Civil Case No.
of Appeals 164797 13 rule which pervades every well- 4365 for Unlawful Detainer litigated before the Municipal Trial Court of Gapan, Nueva Ecija entitled "Josefina M. Cruz and
February 2006 regulated system of jurisprudence Ernestina M. Concepcion, plaintiffs, vs. Mariano `Boy' Bunag, Rolando Bunag, Remedios Bunag, et al., Defendants." This case
NAKAGAWA and is founded upon two grounds was decided on 6 November 1998 by the Municipal Trial Court in favor of herein petitioner Josefina M. Cruz and Ernestina M.
embodied in various maxims of the Concepcion.
common law, namely: (1) public The second case is Civil Case No. 1600 for Quieting of Title, filed before the Regional Trial Court of Gapan, Nueva Ecija,
policy and necessity, which makes Branch 36 with "Carlos L. Bunag, Elias Bunag Natividad, Mariano Bunag, Salud Bunag Clanaoc and Juliana Bunag Arevalo, as
it to the interest of the State that Plaintiffs and Josefina M. Cruz and Ernestina M. Concepcion as Heirs of Sps. Carlos Maniquis and Marina Bunag, as
there should be an end to litigation Defendants." This case was dismissed for failure to prosecute as evidenced by the Regional Trial Court Order dated 10 March
- republicae ut sit litium, and (2) 2000.
the hardship on the individual that
he should be vexed twice for the The third case is Civil Case No. 2573-02 for Injunction, with "Mariano `Boy' Bunag and Rolando Bunag as Petitioners against
same cause - nemo debet bis vexari Carlos Bunag, Elias Bunag Natividad, Mariano Bunag, Salud Bunag Clanaoc and Juliana Bunag Arevalo as Defendants." This
et eadem causa. A contrary case, which was filed before the Regional Trial Court of Gapan City, Branch 35, was dismissed on ground of res judicata. The 6
doctrine would subject the public November 2002 Order, in effect, ruled that there is a substantial identity of parties in this case and in Civil Case No. 1600, a
peace and quiet to the will and Petition for Quieting of Title.
neglect of individuals and prefer
the gratification of the litigious The fourth case is the instant controversy for Annulment of Title With Damages. Docketed as Civil Case No. 2583-02, it was
disposition on the part of suitors to lodged by herein private respondents Mariano "Bo[y]" Bunag and Rolando Bunag against herein petitioners Josefina M. Cruz
the preservation of the public and Ernestina M. Concepcion before the sala of Branch 35, Regional Trial Court of Gapan City.
tranquility and happiness. ISSUE: Does res judicata apply in the case at bar?

HELD: YES. The principle of res judicata may not be evaded by the mere expedient of including an additional party to the
first and second action. Only substantial identity is necessary to warrant the application of res judicata. The addition or
elimination of some parties does not alter the situation. There is substantial identity of parties when there is a community of
interest between a party in the first case and a party in the second case albeit the latter was not impleaded in the first case.

In the case at bar, it is apparent that from the face of the complaint for Quieting of Title, private respondent Rolando Bunag
was not a party therein as his name does not appear in the title. This, notwithstanding, his claim and that of the plaintiffs
therein, which included private respondent Mariano Bunag, are the same - to be declared the true owners of the parcel of land
covered by Original Certificate of Title (OCT) No. 22262 and Transfer Certificate of Title (TCT) No. 67161 of the Registry of
Deeds of Nueva Ecija. Private respondent Rolando Bunag and the plaintiffs are all heirs of the alleged owners of the parcel of
land covered by OCT No. 22262. Private respondent Rolando Bunag, though not a party therein, shared an identity of interest
from which flowed an identity of relief sought, namely, to declare them the true owners of the parcel of land covered by OCT
No. 22262 and TCT No. 67161. Such identity of interest is sufficient to make them privy-in-law, thereby satisfying the requisite
of substantial identity of parties.

We likewise rule that there is identity of causes of action. Hornbook is the rule that identity of causes of action does not mean
absolute identity. Otherwise, a party could easily escape the operation of res judicata by changing the form of the action or the
relief sought. The test to determine whether the causes of action are identical is to ascertain whether the same evidence will
sustain both actions, or whether there is an identity in the facts essential to the maintenance of the two actions. If the same
facts or evidence would sustain both, the two actions are considered the same, and a judgment in the first case is a bar to the
subsequent action. In Stilianopulos v. The City of Legaspi, this Court had this to say:
20 21

The underlying objectives or reliefs sought in both the quieting-of-title and the annulment-of-title cases are essentially the
same -- adjudication of the ownership of the disputed lot and nullification of one of the two certificates of title. Thus, it
becomes readily apparent that the same evidence or set of facts as those considered in the quieting-of-title case would also be
used in this Petition.

The difference in form and nature of the two actions is immaterial and is not a reason to exempt petitioner from the effects of
res judicata. The philosophy behind this rule prohibits the parties from litigating the same issue more than once. When a right
or fact has been judicially tried and determined by a court of competent jurisdiction or an opportunity for such trial has been
given, the judgment of the court, as long as it remains unreversed, should be conclusive upon the parties and those in privity
with them. Verily, there should be an end to litigation by the same parties and their privies over a subject, once it is fully and
fairly adjudicated.

We find no reason not to adhere to the doctrine of res judicata. A case for Quieting of Title had been filed for the purpose of
determining the ownership of the subject land, but same was dismissed because the plaintiffs therein failed to attend the
scheduled hearings for the presentation of their evidence. As above discussed, the dismissal was an adjudication on the merits.
They had all the opportunity to present all the evidence for their cause but they failed to do so. It is undeniable that there was
no denial of due process in this case.

38. Quintos vs.


Nicolas 210252 16 We have held that res judicata Facts:
June 2014 applied because after the
NATO parties executed a Petitioners Vilma Quintos... And respondents Pelagia Nicolas are siblings
compromise agreement that
was duly approved by the Their parents, Bienvenido and Escolastica Ibarra, were the owners of the subject property parcel of land Tarlac. By 1999, both
court, the different portions of
the owners have already been Bienvenido and Escolastica had already passed away, leaving to their ten (10) children ownership over the subject property.
ascertained.
In 2002, respondent siblings brought an action for partition against petitioners. The trial court dismissed the case for failure
of the party’s counsels, to appear despite due notice respondent siblings instead resorted to executing a Deed of Adjudication
on September 21, 2004 to transfer the property in favor of the ten (10) sibling’s respondent siblings sold their 7/10 undivided
share over the property in favor of their co-respondents the spouses Recto and Rosemarie Candelario. TCT No. 390484 was
partially canceled and TCT No. 434304 was issued in the name of the Candelarios, covering the 7/10 portion petitioners filed a
complaint for Quieting of Title and Damages against respondents. They also deny any participation in the execution of the
aforementioned Deed of Adjudication dated September 21, 2004 and the Agreement of Subdivision.

Petitioners' cause of action was already barred by estoppel when sometime in 2006, one of petitioners offered to buy the 7/10
undivided share of the respondent siblings. They point out that this is an admission on the part of petitioners that the
Property is not entirely theirs.

During pre-trial, respondents, or defendants a quo, admitted having. Filed an action for partition dismissed petitioners'
complaint. Subsequent transfer of their interest in favor of respondent spouses Candelario was then upheld by the trial court
hereby orders the partition of the subject lots between the herein plaintiffs and the defendants-spouses Candelarios Partition
should no longer be allowed since it is already barred by res Judicata, respondent siblings having already filed a case for
partition that was dismissed with finality, as admitted by respondents themselves during pre-trial.

Appellate court upheld the finding that petitioners and respondent spouses Candelario co-own the property, 30-70 in favor of
the respondent spouses.

COURT OF APPEALS ERRED WHEN IT ORDERED PARTITION IN ACCORDANCE WITH THE SUBDIVISION PLAN
MENTIONED IN ITS DECISION, IN CONTRAVENTION OF THE PROCEDURE ESTABLISHED IN RULE 69 OF THE
RULES OF CIVIL PROCEDURE.

Issues:

Whether or not the petitioners were able to prove ownership over the property;

Whether or not the respondents' counterclaim for partition is already barred by laches or res judicata; and

Whether or not the CA was correct in approving the subdivision agreement as basis for the partition of the property.

Ruling:

Petitioners, as aptly observed by the courts below, indeed, failed to substantiate their claim.

Their alleged open, continuous, exclusive, and uninterrupted possession of the subject property is belied by the fact, That
respondent siblings, in 2005, entered into a contract of lease with the avico lending investor co. Over the subject lot without
any objection from the petitioners.

The general rule is that dismissal of a case for failure to prosecute is to be regarded as an adjudication on the merits and with
prejudice to the filing of another action, and the only exception is when the order Of dismissal expressly contains a
qualification that the dismissal is without prejudice.

However, Dismissal with prejudice under rule 17, sec. 3 of the rules of court cannot defeat the right of a co-owner to ask for
partition at any time, provided that there is no actual adjudication of ownership of shares yet.

We hold that art. 494, as cited, is an exception to rule 17, sec. 3 of the rules of court to the effect that even if the order of
dismissal for failure to prosecute is silent on whether or not it is with prejudice. It shall be deemed to be without prejudice.

This is not to say, however, that the action for partition will never be barred by res judicata. There can still be res judicata in
partition cases concerning the same parties and the same subject matter once the respective shares of the co-owners have
been Determined with finality by a competent court with jurisdiction or if the court determines that partition is improper for
co-ownership does not or no longer exists.

We have held that res judicata applied because after the parties executed a compromise agreement that was duly approved by
the court, the different portions of the owners have already been ascertained.

We have held that res judicata applied because after the parties executed a compromise agreement that was duly approved by
the court, the different portions of the owners have already been ascertained with the case at bar wherein the co-ownership, as
determined by the trial court, is still subsisting 30-70 in favor of respondent spouses candelario. Consequently, there is no
legal bar preventing herein respondents from praying for the partition of the property through counterclaim.

Art. 496, as earlier cited, provides that partition shall either be by agreement of the parties or in accordance with the rules of
court.

Agreement of subdivision allegedly executed by respondent spouses candelario and petitioners cannot serve as basis for
partition, for, as stated in the pre-trial order, herein respondents admitted that the agreement was a falsity and that
petitioners never took part in preparing the same.

The "agreement" was crafted without any consultation whatsoever or any attempt to arrive at mutually acceptable terms with
petitioners. It, therefore, lacked the essential requisite of consent. Thus, to approve the agreement in spite of this fact would
be tantamount to allowing respondent spouses to divide unilaterally the property among the co-owners based on their own
whims and caprices.

39. Eloisa DOCTRINE: FACTS:


Merchandising Inc. vs. Under Section 3, Rule 17 of the Assailed in this petition for review on certiorari under Rule 45 are the Decision[1] dated March 30, 2010 and Resolution dated
Trebel International 1997 Rules of Civil Procedure, as June 15, 2010 of the Court of Appeals (CA) in CA-G.R. CV No. 89779. The CA affirmed the trial court’s dismissal of
Inc. 192716 13 June amended, the failure on the part of petitioners’ complaint on the ground of failure to prosecute.cralaw
2012 the plaintiff, without any justifiable
PEREZ cause, to comply with any order of Petitioner Eloisa Merchandising, Inc. (EMI) executed in favor of respondent Banco de Oro Universal Bank (BDO) a real estate
the court or the Rules, or to mortgage (REM) over its properties located at No. 129 Neptune St., Bel-Air Village II, Makati City, Metro Manila.
prosecute his action for an The REM was further amended to secure the principal obligation totalling Twenty-Nine Million Nine Hundred Thousand
unreasonable length of time, may Pesos (P29,900,000.00) drawn from the Credit Line Agreement of EMI and Term Loan Agreement of Trebel International,
result in the dismissal of the Inc. (Trebel). EMI likewise executed a Continuing Suretyship in favor of BDO to secure the credit accommodation extended
complaint either motu proprio or by BDO to petitioner’s affiliate, Trebel.
on motion by the defendant. The
failure of a plaintiff to prosecute the BDO initiated foreclosure proceedings by filing an application for extrajudicial foreclosure before the Office of the Ex-Officio
action without any justifiable cause Sheriff of the Regional Trial Court (RTC) of Makati City. Accordingly, respondent Engracio M. Escasinas, Jr. issued a notice
within a reasonable period of time setting the auction sale of the mortgaged property on March 7, 2002.
will give rise to the presumption
that he is no longer interested to On March 1, 2002, petitioners filed a Complaint for “annulment of Real Estate Mortgage, Injunction & Damages With Prayer
obtain from the court the relief for Issuance of a Writ of Preliminary Injunction and/or Temporary Restraining Order,” in the RTC of Makati City, Branch 59.
prayed for in his complaint; hence,
the court is authorized to order the
dismissal of the complaint on its BDO filed a motion to dismiss on the ground of lack of cause of action which can be determined from the facts alleged in the
own motion or on motion of the complaint and considering all annexes, motions and evidence on record.
defendants. The presumption is
not, by any means, conclusive On May 7, 2002, petitioners filed an amended complaint which impleaded the Register of Deeds and alleged that the
because the plaintiff, on a motion mortgaged property was sold at a public auction on March 7, 2002.
for reconsideration of the order of
dismissal, may allege and establish On July 18, 2002, petitioners filed a “Motion for Leave to File and to Admit Second Amended Complaint,” which averred that
a justifiable cause for such failure. the Register of Deeds of Makati City has consolidated the titles over the foreclosed properties and issued new titles in the
The burden to show that there are name of BDO.
compelling reasons that would
make a dismissal of the case The trial court granted the motion to admit second amended complaint and denying the motion to dismiss. BDO was directed
unjustified is on the petitioners. to file a responsive pleading.

Under Section 1, Rule 18 of the BDO filed its Answer traversing the allegations of the complaint. In the Notice of Pre-Trial the trial court set the pre-trial
1997 Rules of Civil Procedure, as conference on February 27, 2003. In compliance with the trial court’s directive, the parties submitted their respective pre-
amended, it is the duty of the trial briefs.
plaintiff, after the last pleading has
been served and filed, to promptly
On March 13, 2003, petitioners filed a “Motion to Admit Supplemental Complaint” which further alleged that BDO’s petition
move ex parte that the case be set
for issuance of a writ of possession was granted by the RTC of Makati City, Branch 143. EMI reiterated that its rights as
for pre-trial. On August 16, 2004,
surety-mortgagor were violated in the railroaded ex parte proceedings implementing the writ of possession even as EMI’s
A.M. No. 03-1-09-SC (Re:
pending motion for reconsideration was still unresolved by Branch 143.
Proposed Rule on Guidelines to be
Observed by Trial Court Judges and
Clerks of Court in the Conduct of The trial court denied the motion to admit supplemental complaint on the ground that the matters raised in the supplemental
Pre-Trial and Use of Deposition- complaint were improper as they pertain to issuances by another branch in a separate petition for writ of possession.
Discovery Measures) took effect,
which provides that: At the scheduled pre-trial conference on June 26, 2003, on motion of petitioners, they were allowed to present evidence ex
parte in view of the absence of BDO which was non-suited. In its motion for reconsideration, BDO’s counsel cited
extraordinary and non-moving traffic as reason for his failure to arrive on time for the pre-trial conference.
Within five (5) days from date of The trial court granted the said motion, reinstated the case and set the case again for pre-trial conference on September 26,
filing of the reply, the plaintiff must 2003, later moved to November 10, 2003, and finally rescheduled to January 12, 2004 by agreement of the parties.
promptly move ex parte that the
case be set for pre-trial conference. On July 16, 2003, petitioners filed a motion for reconsideration of the order denying their motion to admit supplemental
If the plaintiff fails to file said complaint; BDO filed its opposition to the said motion.
motion within the given period, the
Branch COC shall issue a notice of For failure of the petitioners to appear despite due notice at the scheduled pre-trial conference on January 12, 2004, the case
pre-trial. was ordered dismissed. In their motion for reconsideration, petitioners’ counsel claimed that his failure to attend was due to
his accidental falling on the stairs of his house in the morning of January 12, 2004, due to which he had to be attended by a
We note that when the above “hilot”. In an Order dated May 7, 2004, the trial court reconsidered the dismissal and scheduled anew the pre-trial conference
guidelines took effect, the case was on June 29, 2004, which date was subsequently reset to August 3, 2004 for lack of proof of service upon petitioners’ counsel.
already at the pre-trial stage and it
was the failure of petitioners to set Since petitioners again failed to appear on the re-scheduled pre-trial conference on August 3, 2004, the trial court issued the
the case anew for pre-trial following Order:
conference which prompted the
trial court to dismiss their
complaint. When this case was called for pre-trial conference, only counsel for the defendants appeared. There was no appearance on the
part of the plaintiffs, despite the fact that as early as June 29, 2004, they were notified for today’s hearing. The Court,
however, is in receipt of a Motion to Reset filed by counsel for the plaintiff, alleging among others, that he is to appear at the
MTC of San Jose, Batangas, which was set earlier than the hearing of this case. The Court finds the ground not meritorious
because counsel of plaintiffs in open Court on June 29, 2004 signed the notification for the hearing of this case. Counsel
could have objected to the chosen date if indeed he was not available. Likewise, the records will show that on January 12,
2004, this case was also dismissed for failure of the plaintiffs to appear for pre-trial conference. This should have served as a
warning to herein plaintiffs.

In view hereof, upon motion of the herein defendants, the above-entitled case is hereby ordered dismissed pursuant to Section
5, Rule 18 of the Rules of Court.

Petitioners moved to reconsider the above order, their counsel alleging that he had misplaced or lost his calendar book and
could not have ascertained the availability of his schedule. Stressing that he had no intention to ignore the hearing as in fact he
filed a motion to reset the same six days prior to the scheduled hearing, petitioners’ counsel pleaded for the kind indulgence of
the court.

The trial court granted petitioners’ motion for reconsideration “in the interest of justice” and reinstating the case. The trial
court, however directed petitioners to be “more circumspect in attending to this case.”

In its Order dated September 20, 2005, the trial court dismissed the case for failure of petitioners to prosecute their case.
Citing the two previous dismissals on account of petitioners’ non-appearance at the pre-trial conference, the trial court said
that “[f]rom the date of its second reconsideration of the order of dismissal on December 29, 2004 until today, plaintiffs did
not do anything to prosecute the instant case.”

Petitioners filed a motion for reconsideration in which they averred that:

1. After the reconsideration of the Order of dismissal on December 29, 2004, the plaintiffs counsel, Atty. Anselmo A.
Marqueda, on several occasion, passed by the court and diligently followed-up the hearing of this case. He was assured by an
officer of the court to just wait for the notice of hearing that they will issue in the instant case.

2. While waiting for the notice of hearing from this court, the respective counsels of the parties negotiated in earnest for an
amicable settlement of the case. During the last telephone conversation with Atty. Roy P.R. Talao, the defendant’s bank
counsel, and the undersigned agree on some proposals for settlement which are however subject to final confirmation of their
respective clients. The plaintiff believe that the parties are very close to agree and enter into an amicable settlement of this
case.

3. Apart from the reliance of the undersigned counsel on the statement of the court officer to just wait for the notice of
hearing, the undersigned counsel suffered a handicap in making a personal follow-up of this case because of his numerous
travels and lengthy sojourn in the province due to family conflict and death of a member of the family.

In its Order, the trial court denied petitioners’ motion for reconsideration.

Aggrieved, petitioners appealed to the CA arguing that the trial court erred in dismissing the case for failure to prosecute
considering that (1) the trial court has not yet resolved petitioners’ motion for reconsideration of the order denying their
motion to admit supplemental complaint; (2) petitioners are very much interested to prosecute this case to protect their
rights in the premises; (3) petitioners have valid and meritorious causes of action; (4) petitioners may not be deprived of their
day in court by the negligence of their counsel; and (5) non-suit or default judgment is not encouraged as it violates due
process.

The CA affirmed the trial court’s dismissal of the case. The CA said that petitioners cannot justify their prolonged inaction by
belatedly raising as issue the pending motion for reconsideration from the trial court’s denial of their motion to admit the
supplemental complaint, when all along they were aware that the case was at the pre-trial stage as in fact the case was twice
dismissed for their failure to attend the pre-trial conference. Under the circumstances stated, the CA held that the trial court
cannot be faulted for dismissing the case on the ground of petitioners’ failure to prosecute their action.

The CA also denied the motion for reconsideration filed by the petitioners.

Petitioners contend that the only reason for the trial court’s dismissal of the case was the failure of their counsel to move to set
the case for pre-trial. However, Section 1, Rule 18 of the 1997 Rules of Civil Procedure, as amended, imposing upon the
plaintiff the duty to promptly move to set the case for pre-trial, had been repealed and amended by A.M. No. 03-1-09-SC
which took effect on August 16, 2004. This amendment to the rule on pre-trial now imposes on the clerk of court the duty to
issue a notice of pre-trial if the plaintiff fails to file a motion to set the case for pre-trial conference.

Petitioners point out that the case was not yet ripe for pre-trial because of the unresolved pending motion for reconsideration
of the trial court’s denial of the motion to admit supplemental complaint. In any event, petitioners assert that they are very
much interested to prosecute the case as they have presented evidence in their application for the issuance of TRO and writ of
preliminary injunction, amended the complaint several times, their representatives have always been attending as notified by
their lawyers, and their counsel was following up the case but the Clerk of Court could not set the case for pre-trial because of
the pending motion. As to the prior dismissals of the case, these should not be taken as badges of failure to prosecute because
these had been set aside on meritorious grounds. The circumstances that respondent BDO itself had been declared in default
for failure to appear at the pre-trial on June 26, 2003 and has asked repeatedly for extensions of time from the court, the
ongoing negotiations with BDO for amicable settlement even at the appeal stage, and petitioners’ meritorious causes of action,
justify a liberal application of the rules so that petitioners will be given their day in court.

Respondent BDO, on the other hand, asserts that the failure of petitioners to move for the setting of the case for pre-trial
conference, coupled with their repeated violations of the Rules which prompted the trial court to dismiss their complaint
twice, are sufficient grounds for the trial court to finally dismiss the complaint. A.M. No. 03-1-09-SC did not remove
plaintiff’s obligation to set the case for pre-trial. Petitioners’ claim that they relied on a supposed assurance by a court
personnel to set the case for pre-trial is doubtful, aside from being contradictory to the admission of petitioners’ counsel that
he “suffered a handicap in making a personal follow-up of this case because of [his] numerous travels and lengthy sojourn in
the province due to family conflict and death of a member of the family.”

As to the alleged negotiations for an amicable settlement, respondent admitted there were talks during court hearings and
telephone calls but these were casual and at best, exploratory. No serious offer was made by petitioners, much less
concretized. At any rate, even if true, such talks is not a ground to tarry and delay the prosecution of the case which had been
pending with the trial court for more than three years and had not even left the pre-trial stage. If indeed petitioners were
sincere in their desire to settle, they should have promptly moved for the setting of pre-trial so that the case can be referred for
mandatory mediation proceedings.

ISSUE:
Whether or not the CA err in dismissing the case.

HELD:
No. The petition has no merit.

Under Section 3, Rule 17 of the 1997 Rules of Civil Procedure, as amended, the failure on the part of the plaintiff, without any
justifiable cause, to comply with any order of the court or the Rules, or to prosecute his action for an unreasonable length of
time, may result in the dismissal of the complaint either motu proprio or on motion by the defendant. The failure of a plaintiff
to prosecute the action without any justifiable cause within a reasonable period of time will give rise to the presumption that
he is no longer interested to obtain from the court the relief prayed for in his complaint; hence, the court is authorized to order
the dismissal of the complaint on its own motion or on motion of the defendants. The presumption is not, by any means,
conclusive because the plaintiff, on a motion for reconsideration of the order of dismissal, may allege and establish a
justifiable cause for such failure. The burden to show that there are compelling reasons that would make a dismissal of the
case unjustified is on the petitioners.

Under Section 1, Rule 18 of the 1997 Rules of Civil Procedure, as amended, it is the duty of the plaintiff, after the last pleading
has been served and filed, to promptly move ex parte that the case be set for pre-trial. On August 16, 2004, A.M. No. 03-1-09-
SC (Re: Proposed Rule on Guidelines to be Observed by Trial Court Judges and Clerks of Court in the Conduct of Pre-Trial
and Use of Deposition-Discovery Measures) took effect, which provides that:

Within five (5) days from date of filing of the reply, the plaintiff must promptly move ex parte that the case be set for pre-trial
conference. If the plaintiff fails to file said motion within the given period, the Branch COC shall issue a notice of pre-trial.

We note that when the above guidelines took effect, the case was already at the pre-trial stage and it was the failure of
petitioners to set the case anew for pre-trial conference which prompted the trial court to dismiss their complaint.

In Olave v. Mistas this Court said that even if the plaintiff fails to promptly move for pre-trial without any justifiable cause for
such delay, the extreme sanction of dismissal of the complaint might not be warranted if no substantial prejudice would be
caused to the defendant, and there are special and compelling reasons which would make the strict application of the rule
clearly unjustified. In the more recent case of Espiritu v. Lazaro, this Court affirmed the dismissal of a case for failure to
prosecute, the plaintiff having failed to take the initiative to set the case for pre-trial for almost one year from their receipt of
the Answer. Although said case was decided prior to the effectivity of A.M. No. 03-1-09-SC, the Court considered the
circumstances showing petitioners’ and their counsel’s lack of interest and laxity in prosecuting their case.

In this case, while there was no substantial prejudice caused to herein respondent, who has already consolidated the
ownership of petitioners’ properties, secured new titles in its name and successfully implemented a writ of possession issued
by another branch, there was neither patent abuse in the trial court’s dismissal of the complaint for the third time, the earlier
two dismissals having been precipitated by petitioners’ non-appearance at the pre-trial conference. Contrary to petitioners’
assertion, the trial court did not find their offered excuses as meritorious or justifiable; the trial court in the exercise of
discretion simply reinstated the case “in the interest of justice” but explicitly warned petitioners to be more circumspect in
attending to the case.

However, despite the trial court’s leniency and admonition, petitioners continued to exhibit laxity and inattention in attending
to their case. Assuming domestic problems had beset petitioners’ counsel in the interregnum, with greater reason should he
make proper coordination with the trial court to ensure his availability on the date to be chosen by the trial court for the long-
delayed conduct of a pre-trial conference. Petitioners themselves did nothing to get the case moving for nine months and set
the case anew for pre-trial even as BDO was already seeking their judicial ejectment with the implementation of the writ of
possession issued by Branch 143. Such circumstance also belies their pretense that the parties were then still negotiating for a
settlement. We have held that a party cannot blame his counsel when he himself was guilty of neglect; and that the laws aid
the vigilant, not those who slumber on their rights. Vigilantibus sed non dormientibus jura subveniunt.

We also agree with the CA that petitioners are belatedly raising as issue the unresolved motion for reconsideration of the
denial of petitioners’ motion to admit supplemental complaint. Petitioners did not even file a motion to resolve the said
pending incident which, in any event, could have been brought to the trial court’s attention had petitioners acted promptly to
have the case set anew for pre-trial conference soon after or within a reasonable time from the reinstatement of the case on
December 29, 2004.

While under the present Rules, it is now the duty of the clerk of court to set the case for pre-trial if the plaintiff fails to do so
within the prescribed period, this does not relieve the plaintiff of his own duty to prosecute the case diligently. This case had
been at the pre-trial stage for more than two years and petitioners have not shown special circumstances or compelling
reasons to convince us that the dismissal of their complaint for failure to prosecute was unjustified.cralaw

WHEREFORE, the petition for review on certiorari is DENIED. The Decision dated March 30, 2010 and Resolution dated
June 15, 2010 of the Court of Appeals in CA-G.R. CV No. 89779 are hereby AFFIRMED and UPHELD.

Costs against the petitioners.

SO ORDERED.

40. Martinez vs. Facts:


The doctrinal rule concerning the
Republic 160895 30
remedies of a party declared in Petitioner Jose R. Martinez (Martinez) filed a petition for the registration in his name of three (3) parcels of land
October 2006
default had evolved into a fairly included in the Cortes, Surigao del Sur Cadastre. The lots, individually identified as Lot No. 464-A, Lot No. 464-B, and Lot
PURIFICACION
comprehensive restatement as No. 370, Cad No. 597, collectively comprised around 3,700 square meters. Martinez alleged that he had purchased the lots in
1952 from his uncle, whose predecessors-in-interest were traceable up to the 1870s. It was claimed that Martinez had
offered in Lina v. Court of Appeals: remained in continuous possession of the lots; that the lots had remained unencumbered; and that they became private
property through prescription pursuant to Section 48(b) of Commonwealth Act No. 141. Martinez further claimed that he had
a. The defendant in default
been constrained to initiate the proceedings because the Director of the Land Management Services had failed to do so despite
may, at any time after discovery
the completion of the cadastral survey of Cortes, Surigao del Sur.
thereof and before judgment, file a
motion, under oath, to set aside the The trial court set the case for hearing and directed the publication of the corresponding Notice of Hearing in the
order of default on the ground that Official Gazette. The OSG, in behalf of the Republic of the Philippines, opposed the petition on the grounds that appellee's
his failure to answer was due to possession was not in accordance with Section 48(b) of Commonwealth Act No. 141; that his muniments of title were
fraud, accident, mistake or insufficient to prove bona-fide acquisition and possession of the subject parcels; and that the properties formed part of the
excusable neglect, and that he has public domain and thus not susceptible to private appropriation.
meritorious defenses; (Sec 3, Rule
Despite the opposition filed by the OSG, the RTC issued an order of general default, even against the Republic of the
18)
Philippines, on 29 March 2000. This ensued when during the hearing of even date, no party appeared before the Court to
b. If the judgment has already
oppose Martinez's petition.
been rendered when the defendant
discovered the default, but before The OSG filed a Notice of Appeal which was approved by the RTC. However, after the records had been transmitted to
the same has become final and the Court of Appeals, the RTC received a letter from the Land Registration Authority (LRA) stating that only Lot Nos. 464-A
executory, he may file a motion for and 464-B were referred to in the Notice of Hearing published in the Official Gazette; and that Lot No. 370, Cad No. 597 had
new trial under Section 1(a) of Rule been deliberately omitted due to the lack of an approved survey plan for that property. Accordingly, the LRA manifested that
37; this lot should not have been adjudicated to Martinez for lack of jurisdiction. This letter was referred by the RTC to the Court
c. If the defendant discovered of Appeals for appropriate action.
the default after the judgment has
become final and executory, he may The Court of Appeals promulgated the assailed Decision, reversing the RTC and instead ordering the dismissal of the
file a petition for relief under petition for registration. In light of the opposition filed by the OSG, the appellate court found the evidence presented by
Section 2 of Rule 38; and Martinez as insufficient to support the registration of the subject lots.
d. He may also appeal from No motion for reconsideration appears to have been filed with the Court of Appeals by Martinez, who instead directly
the judgment rendered against him assailed its Decision before this Court through the present petition.
as contrary to the evidence or to the
law, even if no petition to set aside Petitioner Martinez - OSG no longer had personality to oppose the petition, or appeal its allowance by the RTC, following
the order of default has been the order of general default.
presented by him. (Sec. 2, Rule 41) Respondent (RP/OSG) -
A defendant party declared in 1. duly opposed Martinez's application for registration before the RTC;
default retains the right to appeal 2. jurisprudence and the Rules of Court acknowledge that a party in default is not precluded from
from the judgment by default on the appealing the unfavorable judgment;
ground that the plaintiff failed to 3. RTC had no jurisdiction over Lot No. 370 since its technical description was not published in the Official Gazette; and
prove the material allegations of the 4. as found by the Court of Appeals the evidence presented by Martinez is insufficient for registering the lots in his
complaint, or that the decision is name.
contrary to law, even without need
of the prior filing of a motion to set
aside the order of default. Issue:
Whether an order of general default issued by a trial court in a land registration case bars the Republic of the Philippines,
through the Office of the Solicitor General, from interposing an appeal from the trial court's subsequent decision in favor of
the applicant. - NO.

Held:
By 1997, the doctrinal rule concerning the remedies of a party declared in default had evolved into a fairly
comprehensive restatement as offered in Lina v. Court of Appeals:
a) The defendant in default may, at any time after discovery thereof and before judgment, file a motion, under oath, to
set aside the order of default on the ground that his failure to answer was due to fraud, accident, mistake or excusable neglect,
and that he has meritorious defenses; (Sec 3, Rule 18)
b) If the judgment has already been rendered when the defendant discovered the default, but before the same has
become final and executory, he may file a motion for new trial under Section 1(a) of Rule 37;
c) If the defendant discovered the default after the judgment has become final and executory, he may file a petition for
relief under Section 2 of Rule 38; and
d) He may also appeal from the judgment rendered against him as contrary to the evidence or to the law, even if no
petition to set aside the order of default has been presented by him. (Sec. 2, Rule 41)
|In Rural Bank of Sta. Catalina v. Land Bank of the Philippines, the Court, through Justice Callejo, Sr., again
provided a comprehensive restatement of the remedies of the defending party declared in default, which we adopt for
purposes of this decision:

It bears stressing that a defending party declared in default loses his standing in court and his right to
adduce evidence and to present his defense. He, however, has the right to appeal from the
judgment by default and assail said judgment on the ground, inter alia, that the amount of the
judgment is excessive or is different in kind from that prayed for, or that the plaintiff failed to
prove the material allegations of his complaint, or that the decision is contrary to law. Such party
declared in default is proscribed from seeking a modification or reversal of the assailed decision on the basis of the
evidence submitted by him in the Court of Appeals, for if it were otherwise, he would thereby be allowed to regain
his right to adduce evidence, a right which he lost in the trial court when he was declared in default, and which he
failed to have vacated. In this case, the petitioner sought the modification of the decision of the trial court based on
the evidence submitted by it only in the Court of Appeals.
If it cannot be made any clearer, we hold that a defendant party declared in default retains the right to
appeal from the judgment by default on the ground that the plaintiff failed to prove the material allegations
of the complaint, or that the decision is contrary to law, even without need of the prior filing of a motion to
set aside the order of default. We reaffirm that the Lim Toco doctrine, denying such right to appeal unless the order of
default has been set aside, was no longer controlling in this jurisdiction upon the effectivity of the 1964 Rules of Court, and up
to this day.

41. Ko vs. PNB In every action, the plaintiff is duty- FACTS: Ko filed an action for annulment of mortgage in the RTC of Laoag. During the course of the proceedings, Ko and
169131-32 28 January bound to prosecute the same with their counsel failed to attend a scheduled trial. Upon motion of PNB, the complaint was dismissed.
2006 utmost diligence and with
SABIO reasonable dispatch to enable him Ko filed a motion for reconsideration claiming that they have been continuously pursuing negotiations with PNB, PNB
to obtain the relief prayed for, and countered that from the time the complaint was filed, a period of three years had elapsed but Ko failed to prosecute their case.
at the same time, minimize the
clogging of the court dockets. The trial court denied the motion for reconsideration.

ISSUE: Whether or not the dismissal of the case due to Ko’s failure to appear at the scheduled hearing is proper.
HELD: YES. Ko erred in filing a petition for review on certiorari under Rule 45 of the ROC instead of filing an appeal with the
CA. Sec. 3, Rule 17 of the ROC provides “If, for no justifiable cause, the plaintiff fails to appear on the date of the presentation
xxx the complaint may be dismissed upon the motion of the defendant or upon the court’s own motion xxx this dismissal shall
have the effect of an adjudication upon the merits, unless otherwise declared by the court.”

The dismissal was proper because in every action, the plaintiff is duty-bound to prosecute the same with utmost diligence and
with reasonable dispatch to enable him to obtain the relief prayed for, and at the same time, minimize the clogging of the
court dockets.

In the case at bar, three years have since lapsed from the filing of the complaint. Ko’s failure to prosecute their case and
proceed with the trial during the span of three years leads to no other conclusion than that Ko did not have any interest in
seeing their case terminated at the earliest possible time.

42. Laurel vs. To constitute failure to prosecute,


Ardelon 202967 5 his non-appearance must be Facts:
August 2015 equated with unwillingness to
ARIBON proceed with the trial as when both Petitioner Alicia Y. Laurel filed a Complaint for recovery of possession and ownership and/or quieting of title against
plaintiff and counsel made no respondent Ferdinand M. Vardeleon concerning a 20,306-square meter island in Caticlan, Malay, Aklan. Respondent denied
appearance at all, or with the the material allegations in the complaint, claiming that he bought the island on April 9, 1973 from Avelina Casimero, and that
assumption that plaintiff has petitioner was guilty of laches in filing her claim. The trial court denied respondent's motion to correct the Pre-Trial Order.
already lost interest in prosecuting Respondent filed a motion for reconsideration but the trial court did not act on the motion. Petitioner's counsel moved to reset
his action the scheduled September 7, 2005 hearing to October 12, 2005 or any available date. Petitioner moved to reconsider, but the
court failed to act on the same. October 12, 2005 hearing, petitioner was present, together with substitute counsel Atty. Roy
Villa and her first witness. Petitioner moved in open court to postpone trial on the ground that there are pending motions that
have to be resolved, and that the substitute lawyer had yet to confer with the witness, since her true counsel, Atty. De la Vega -
who originally interviewed the witness - was not present. This time, the trial court, dismissed the case on the ground of failure
to prosecute on petitioner’s part, pursuant to Section 3, Rule 17. Petitioner filed an MR. Trial court denied. CA denied.

Issue:

W/N COURT OF APPEALS AND THE TRIAL COURT COMMITTED SERIOUS ERROR AND GRAVE ABUSE OF
DISCRETION IN DISMISSING PETITIONER’S COMPLAINT FOR FAILURE TO PROSECUTE.

Held:

Yes. To constitute failure to prosecute, his non-appearance must be equated with unwillingness to proceed with the trial as
when both plaintiff and counsel made no appearance at all, or with the assumption that plaintiff has already lost interest in
prosecuting his action.In the same way that should the ground for dismissal be delay, this delay or failure to proceed must be
for an unreasonable length of time beyond the reasonable allowance which by judicial leniency the litigant is normally
entitled. The Court declares that both courts erred in dismissing the case. It cannot be said that the petitioner refused to
present her evidence. She was 81 years of age and the record indicates that she attended the scheduled hearing of October 12,
2005. It must also be noted that the delay or failure contemplated under Rule 3, Sec 17 must be for an unreasonable length of
time. In petitioner’s case, the continuance she sought was not for an unreasonable time. It was within the period expected by
and made known to the defendant and the trial court during pre-trial. In fact, it was only until the next scheduled setting on
November 23, 2005, which was just over one month away. This may not be characterized as delay, as such scheduled hearing
was expected by respondent and could not have come as a surprise to him.

XVIII. PRE-TRIAL (RULE 18, SEC. 1 TO 10)


43. LCK
FACTS:
Industries Inc vs.
Planters The conduct of pre-trial in civil
Petitioner LCK is a domestic corporation duly organized and existing as such under Philippine laws. Respondent bank is a
Development actions has been mandatory as
banking institution duly authorized to engage in banking business under Philippine laws.
Bank 170606 23 early as 1 January 1964 upon the
petitioner LCK obtained a loan from the respondent bank in the amount of P3M as evidenced by two promissory notes.
November 2007 effectivity of the Revised Rules of
As a security for the loan obligation, petitioners-spouses Chiko and Elizabeth Lim executed 2 Real Estate Mortgage (REM) over
SOMEROS Court. Pre-trial is a procedural
a parcel of land (Quezon City property) and (Baguio City property).
device intended to clarify and limit
Subsequently, petitioner LCK incurred default in its payment. Several and final demands were made by the respondent bank to
the basic issues between the
petitioner LCK who failed or refused to pay its obligation.
parties and to take the trial of cases
Consequently, respondent bank extrajudicially foreclosure of the Baguio City property which was sold at the public auction for
out of the realm of surprise and
P2,625,000.00. Since the proceeds were not enough to satisfy the entire loan obligation respondent bank further extrajudicially
maneuvering.
foreclosure of the Quezon City property. sold at a public auction for P2,231,416.67 Atty. Anigan as one who signed the
certificate. The respondent bank was the highest bidder on both occasions.
Pre-trial seeks to achieve the Civil Case No. Q-98-33835.
following:
Prior to the auction sale of the Quezon City property, petitioners, filed with the RTC of QC, an action for Annulment of the
(a) The possibility of an Foreclosure of Mortgage and Auction Sale of the Quezon City property with Restraining Order/Preliminary Injunction and with
amicable settlement or of a Damages against respondent bank and Atty. Anigan.
submission to alternative the counsels for both petitioners and respondent bank failed to appear in the scheduled hearing for the issuance of TRO, the
modes of dispute resolution; RTC deemed the prayer for TRO abandoned.
(b) The simplification of the Thereafter, the RTC conducted a pre-trial conference. In the Pre-Trial Order dated 8 September 2000, the parties
issues; made admissions and stipulations. the RTC also defined the issues as follows: (1) whether or not the petition was filed with the
(c) The necessity or desirability Office of the Clerk of Court; (2) whether or not the extrajudicial foreclosure of real estate mortgage by defendant bank was made
of amendments to the in accordance with the provisions of Act No. 3135; and (3) whether or not the parties are entitled to their respective claims for
pleadings; attorney's fees and damages.
(d) The possibility of obtaining The parties were given 15 days from receipt of the Pre-Trial Order to make amendments or corrections thereon.
stipulations or admissions of the parties agreed to submit the case for the decision of the RTC based on the stipulations and admissions made at the pre-trial
facts and of documents to avoid conference. On the same day, the RTC required the parties to submit their respective memoranda.
unnecessary proof; In their Memorandum, petitioners, aside from reiterating issues previously raised in their Complaint, further claimed that there
(e) The limitation of the was an overpayment of the loan obligation by P1,856,416.67.
number of witnesses; respondent bank maintained that the complaint filed by petitioners is devoid of merit. that petitioners' claim for overpayment
(f) The advisability of a was not among the issues submitted for the resolution of the RTC. For failure of petitioners to promptly raise the alleged
preliminary reference of issues overpayment, the RTC is now barred from adjudicating this issue.
to a commissioner; RTC declared the foreclosure and the auction sale of the Quezon City property legal and valid, but ordered respondent bank to
(g) The propriety of rendering return the overpayment made by petitioners in the amount of P1,856,416.67.
judgment on the pleadings, or Motion for Reconsideration filed by the respondent bank was denied by the RTC.
summary judgment, or of CA-G.R. CV No. 73944
dismissing the action should a
valid ground therefor be found CA granted the appeal of the respondent bank and partially reversed the RTC Decision insofar as it ordered respondent bank to
to exist; pay the overpaid amount of P1,856,416.67 to petitioners.
(h) The advisability or necessity In deleting the award of overpayment, the appellate court emphasized that the primary purpose of pre-trial is to make certain
of suspending the proceedings; that all issues necessary for the disposition of the case are properly raised in order to prevent the element of surprise. Since the
and alleged overpayment was only raised by the petitioners long after the pre-trial conference, the court a quo cannot dispose of
(i) Such other matters as may such issue without depriving the respondent bank of its right to due process.
aid in the prompt disposition of The Motion for Reconsideration filed by petitioners was denied by CA
the action. Petitioners filed a Petition for Review on Certiorari,
ISSUE: whether or not the issue of overpayment was raised by the parties and included in the pre-trial order.
HELD: YES, Needless to say, the fact of overpayment, though not expressly included in the issues raised in the Pre-Trial Order
dated 8 September 2000, can be evidently inferred from the stipulations and admissions made by the parties therein. Even only
upon plain reading of the said Pre-Trial Order, it can be readily discerned that there was an overpayment.
The purpose of entering into a stipulation of facts is to expedite trial and to relieve the parties and the court as well of the costs of
proving facts which will not be disputed on trial and the truth of which can be ascertained by reasonable inquiry. Its main
objective is to simplify, abbreviate and expedite the trial, or totally dispense with it.
The parties themselves or their representative with written authority from them are required to attend in order to arrive at a
possible amicable settlement, to submit to alternative modes of dispute resolution, and to enter into stipulations or admissions
of facts and documents. All of the matters taken up during the pre-trial, including the stipulation of facts and the admissions
made by the parties, are required to be recorded in a pre-trial order.
Thus, Section 7, Rule 18 of the Revised Rules of Court provides:
SEC. 7. Record of pre-trial. — The proceedings in the pre-trial shall be recorded. Upon the termination thereof, the court
shall issue an order which shall recite in detail the matters taken up in the conference, the action taken thereon, the
amendments allowed to the pleadings, and the agreements or admissions made by the parties as to any of the matters
considered. Should the action proceed to trial, the order shall explicitly define and limit the issues to be tried. The contents
of the order shall control the subsequent course of the action, unless modified before trial to prevent manifest injustice.
Generally, to obviate the element of surprise, parties are expected to disclose at the pre-trial conference all issues of law and fact
they intend to raise at the trial. However, in cases in which the issue may involve privileged or impeaching matters, or if the
issues are impliedly included therein or may be inferable therefrom by necessary implication to be integral parts of the pre-trial
order as much as those that are expressly stipulated, the general rule will not apply.
Thus, in Velasco v. Apostol, this Court highlighted the aforesaid exception and ruled in this wise:
A pre-trial order is not meant to be a detailed catalogue of each and every issue that is to be or may be taken up during
the trial. Issues that are impliedly included therein or may be inferable therefrom by necessary
implication are as much integral parts of the pre-trial order as those that are expressly stipulated.
The case at bar falls under this particular exception. Upon scrupulous examination of the Pre-Trial Order dated 8 September
2000, it can be deduced that the parties stipulated that the remaining sum of petitioner LCK's obligation as of 13 October 1997
was P2,962,500.00. In the same Pre-Trial Order, the parties likewise stipulated that the Baguio City property was sold at the
public auction for P2,625,000.00 and the Quezon City property for P2,231,416.67. On both occasions, respondent bank emerged
as the highest bidder. By applying simple mathematical operation, the mortgaged properties were purchased by the respondent
at the public auctions for P4,856,416.67; thus, after deducting therefrom the balance of petitioner LCK's obligation in the
amount of P2,962,500.00, an excess in the sum of P1,893,916.67 remains.
As the custodian of the proceeds from the foreclosure sale, respondent bank has no legal right whatsoever to retain the excess of
the bid price in the sum of P1,893,916.67, and is under clear obligation to return the same to petitioners.

44. Paranaque Trial court has the discretion on whether FACTS:


Kings Enterprises to grant or deny a motion to postpone
vs. Santos 194638 and/or reschedule the pre-trial Respondent Catalina L. Santos (Santos) entered into a Contract of Lease with Frederick O. Chua (Chua) over eight (8) parcels of
2 July 2014 conference in accordance with the land located in Parañaque City (leased premises), specifically giving the latter the "first option... or priority to buy" the same in
TAN circumstances obtaining in the case. case of sale.[10] Chua then caused the construction of a 6-door commercial complex on the leased premises but, by reason of
business reverses, he was constrained to assign his rights thereon to Lee Ching Bing (Lee), who likewise assumed all obligations
This must be so as it is the trial court under the lease contract with Santos. Lee, in turn, executed a Deed of Assignment over the leased premises, including all
which... is able to witness firsthand the improvements thereon, in... favor of petitioner.
events as they unfold during the trial of a
case. Postponements, while permissible, On March 19, 1991, petitioner filed a Complaint before the RTC against Santos and respondent David A. Raymundo (Raymundo)
must not be countenanced except for to whom Santos allegedly sold the leased premises on September 21, 1988 for a consideration of
clearly meritorious grounds and in light
of the attendant circumstances P5,000,000.00, without giving petitioner the opportunity to exercise its priority to buy the same. Petitioner claimed that, when
it objected to the sale, Santos repurchased the subject properties for the same price, and offered... them to petitioner for
P15,000,000.00. The latter made a counter-offer of P5,000,000.00 but, before replying thereto, Santos sold the subject
properties again to Raymundo on May 15, 1989 for P9,000,000.00. Petitioner argued that the sale was... simulated and that
there was collusion between Santos and Raymundo (respondents).

Respondents respectively moved for the dismissal of the Complaint on the main ground that it stated no cause of action.
Raymundo alleged that there were, in fact, previous offers made to petitioner that the latter simply ignored. Santos, on the other
hand, maintained that petitioner had already recognized and respected Raymundo's status as the new owner-lessor of the
subject properties due to its payment of lease rentals to Raymundo, and, as such, is now estopped from challenging

Raymundo's title

Eventually, the foregoing CA Decision was reversedon petition for review before the Court in a Decision dated February 26, 1997,
upon a finding that the Complaint "sufficiently alleges an... actionable contractual breach" on the part of respondents. The Court
explained that the trial and appellate courts based their decision on the allegation that Santos had actually offered the subject
properties for sale to petitioner prior to the final... sale in favor of Raymundo, but that the offer was rejected. However, the Court
held that in order to have full compliance with the contractual right granting petitioner the first option to purchase, the sale of
the subject properties for the amount of P9,000,000.00, the price... for which it was finally sold to Raymundo, should have
likewise been first offered to petitioner. Necessarily, the Court remanded the case to the trial court for further proceedings.

Petitioner moved for the reconsideration of the said Order, as well as the voluntary inhibition of the presiding judge for alleged
acts of "undue deference for and haste in granting all the motions and wishes of [respondents] and his consistent denial... of the
motions of [petitioner]." The motion was, however, denied by the RTC, in an Order dated June 11, 1998, and the case was set for
pre-trial on July 7, 1998.

On July 2, 1998, petitioner filed a Motion to Cancel Pre-Trial, claiming that it was preparing a petition for certiorari and
prohibition which (a) was to be filed with the CA before the scheduled pre-trial on July 7, 1998, and (b) was intended to
challenge the validity of the RTC's Orders dated May 18, 1998 and June 11, 1998 by raising alleged prejudicial questions that
must be resolved first before the pre-trial and trial on the merits of the case could proceed
Incidentally, the petition for certiorari and prohibition that was actually filed at 2:17in the afternoon of July 7, 1998, (contrary to
petitioner's assertion in its Motion to Cancel Pre-Trial that it was to be filed before the July 7, 1998 pre-trial) was resolved by the
CA in favor of petitioner in a Decision dated December 6, 1999, where it was determined that the Motion to Strike Out was
denied prematurely. On the other hand, the CA declared the petition for voluntary inhibition moot and academic with the
appointment of a regular judge for Branch 57. Thus, the Motion to Strike Out was ordered to be resolved by the regular judge.

Subsequently, the petition for review on certiorari filed by respondents before the Court to question the December 6, 1999 CA
Decision was dismissed by the Court in a Decision dated October 23, 2006.

Meanwhile, on July 7, 1998, the day of the pre-trial sought to be cancelled, the RTC denied petitioner's Motion to Cancel Pre-
Trial in its First Order of even date. Accordingly, the RTC directed the parties to proceed to pre-trial as... scheduled.

The trial court then required petitioner to start the pre-trial with the statement of its cause. However, counsel for petitioner,
Atty. Nelson Santos, refused to do so saying he would just furnish the court the following day with a copy of the petition for
certiorari and... prohibition filed with the CA.[49] Consequently, upon motion of the opposing counsel, the RTC (a) declared
petitioner non-suited, and (b) dismissed the Complaint in its Second Order.

ISSUE: Whether or not the motion to cancel was properly denied.

HELD: Yes, the motion to cancel was properly denied. At the outset, it should be emphasized that the trial court has the
discretion on whether to grant or deny a motion to postpone and/or reschedule the pre-trial conference in accordance with the
circumstances obtaining in the case. This must be so as it is the trial court which... is able to witness firsthand the events as they
unfold during the trial of a case.

Hence, the cancellation of the pre-trial on the ground of the impending filing of a petition for certiorari and prohibition, as there
was no proof at the time of the hearing that said petition was in fact filed, was obviously a... dilatory tactic designed for petitioner
to control the proceedings of the court. More importantly, even with the actual filing of the petition for... certiorari at 2:17 in the
afternoon of July 7, 1998, no restraining order was issued by the CA enjoining the trial court from proceeding with the pre-trial.
The appellate court correctly emphasized, in the... assailed Decision dated September 22, 2010, that the mere elevation of an
interlocutory matter through a petition for certiorari does not by itself merit a suspension of the proceedings before the trial
court, unless a temporary restraining order or a writ of... preliminary injunction has been issued.

45. Alarcon The purpose of entering into a stipulation Facts:


vs. CA 126802 28 of facts or admissions of facts is to Petitioner Roberto Alarcon filed a complaint for annulment of a deed of sale with damages against Bienvenido Juani (private
January 2000 expedite trial and to relieve the parties respondent), Edgardo Sulit, and Virginia Baluyot. Petitioner alleged that before he left for Brunei to work, he left with his father,
TANADA and the court, as well, of the costs of Tomas Alarcon, an SPA to administer, mortgage, or sell his properties in Baliwag, Bulacan. Upon his return from Brunei, he
proving facts which will not be disputed found out that by virtue of the SPA, a portion of his land containing an area of 2500 square meters was sold to Juani, Sulit, and
on trial and the truth of which can be Baluyot pursuant to a document of sale allegedly executed by his father for a nominal consideration of Php 5,000.00. The
ascertained by reasonable inquiry. The vendees were able to register the sale and cancel the title of petitioner and to have new certificates of title issued in their names.
rules have made it mandatory that a pre- Petitioner prayed for the nullification of the sale contending that, his father's signature on the Deed of Sale was forged, there was
trial should first be conducted before no consideration of the sale, his father had no more authority to sell the land since the special power of attorney had already
hearing any case. The parties themselves been earlier revoked, and that the signature of the notary public who allegedly attested said sale was falsified. In their answer,
are required to attend or their Juani and Baluyot averred that Juani had been the tiller-occupant for almost 10 years of the land with an area of 1000 square
representative with written authority meters owned by petitioner for almost 10 years. He alleged that he was lured by Tomas Alarcon as the attorney-in-fact of his son
from them in order to arrive at a possible to give up his right as tiller of the land in exchange of ownership of a parcel of land with an area of 2500 square meters. He then
amicable settlement, to submit to acceded to Tomas Alarcon and bought the land. Out of 2500 square meters, Juani sold 1000 square meters to Baluyot and 500
alternative modes of dispute resolution, square meters to Sulit. Finally, Juani, Baluyot, and Sulit alleged that Juani was never aware of the revocation of the SPA granted
and to enter into stipulations or to Tomas Alarcon. After all the issues were joined, the trial court set the case for pre-trial conference. During the pre-trial
admissions of facts and documents. All of conference, the parties, represented by their counsel, made some admissions of facts. On such basis, the judge rendered a partial
the matters taken up during the pre-trial, decision, declaring the sale void ab initio and the TCTs issued in favor of Juani, Baluyot, and Sulit as null and void. Thereafter,
including the stipulation of facts and the the trial court issued its pre-trial order, closing and terminating the pre-trial. Petitioner moved for the execution of the Partial
admissions made by the parties are Decision since no MR or appeal was filed therefrom. At the same time, for the purpose of ending litigation of the case, petitioner
required to be recorded in a pre-trial offered to drop his claim for damages against the defendants if they were also willing to waive their claims against him and his
order. father. Upon the opposition of defendant Baluyot, the trial court denied petitioner's motion. The trial court then dismissed the
complaint as against Juani and the latter’s counterclaim against petitioner. Petitioner’s motion for execution was granted. Since
there was no appeal, the Partial Decision became final and executory. However, the judgment could not be executed because the
defendants allegedly refused to surrender their respective Owner's Duplicate of Transfer Certificates of Title issued to them by
the Register of Deeds. Private respondent Juani filed with the CA a petition for the annulment of the Partial Judgment. The CA
granted the petition and set aside the Partial Decision, holding that it was vitiated by extrinsic fraud since Juani, being
unlettered, was not apprised of the proceedings held in the trial court. The CA directed that the case be remanded to the court of
origin for further proceedings.

Issue:
Whether or not the Partial Decision in the pre-trial conference was valid.

Held: Yes.
The SC held that it cannot be argued that there was extrinsic fraud since Juani was not deprived from having a trial. Ordinary
laymen may not be knowledgeable about the intricacies of the law which is why lawyers are retained to make the battle in court
fair and square. The SC found no extrinsic fraud in the proceedings, as opposed to the CA’s finding, since the Partial Decision
itself stated that: ". . . during the pre-trial of the case conducted on June 3, 1986 and August 1, 1986, all the parties and their
respective counsels appeared. . . . ." Also, the transcript of the stenographic notes (TSN) of the hearing conducted on June 3,
1986 would show that Juani, together with defendant Baluyot, was represented by counsel, Atty. Venancio Reyes. From the TSN,
the admissions were clearly made during the pre-trial conference and, therefore, conclusive upon the parties making it. The
purpose of entering into a stipulation of facts or admissions of facts is to expedite trial and to relieve the parties and the court, as
well, of the costs of proving facts which will not be disputed on trial and the truth of which can be ascertained by reasonable
inquiry. The rules have made it mandatory that a pre-trial should first be conducted before hearing any case. The parties
themselves are required to attend or their representative with written authority from them in order to arrive at a possible
amicable settlement, to submit to alternative modes of dispute resolution, and to enter into stipulations or admissions of facts
and documents. All of the matters taken up during the pre-trial, including the stipulation of facts and the admissions made by
the parties are required to be recorded in a pre-trial order. On the basis of the clear admissions made by the parties in the case,
the Partial Decision was rendered. Juani cannot now claim that he was denied his day in court when judgment was rendered on
the basis of their (Juani, Baluyot and Sulit) counsels' admissions. Since it was patent that the deed of sale was a forgery, no
parcel of land was transferred to the vendees. Thus, the TCTs which Juani, Baluyot and Sulit obtained pursuant to said deed of
sale were null and void. Respondent court, therefore, committed a reversible error in giving due course to the petition filed
before it, the same not being based on extrinsic fraud and, moreover, it was barred by prescription.

46. Corpuz vs. Section 5 of Rule 18 provides that the


Corpuz 175677 31 dismissal of an action due to the Facts: Azucena Corpuz was a card holder of Citibank Mastercard and VISA card. Each card had a credit limit of ₱40,000.00. In
July 2009 plaintiff’s failure to appear at the pre- view of her then impending official business trip to Europe, Azucena paid in full on December 7, 1998 her monthly charges on
UBAY trial shall be with prejudice, unless both credit cards via checks and also made advance check payments of ₱20,000.00 on December 8, 1998 for her VISA Card, and
otherwise ordered by the court. In this
case, the trial court deemed the another ₱20,000.00 for her Mastercard on December 14, 1998, to cover future transactions.
plaintiffs-herein spouses as non-suited
and ordered the dismissal of their While in Italy on December 9, 1998, Azucena dined at a restaurant. To settle her bill of 46,000 liras, she presented her VISA
Complaint. As the dismissal was a final Card, but to her surprise and embarrassment, the restaurant did not honor it. She then brought out her Mastercard which the
order, the proper remedy was to file an restaurant honored. On even date, Azucena incurred a bill of 378,000 liras at a shop which she intended to charge to her credit
ordinary appeal and not a petition for cards. This time, both her VISA and Mastercard were not honored, drawing her to pay the bill in cash.
certiorari. The spouses’ petition for
certiorari was thus properly dismissed by Upon her return to the country, Azucena wrote Citibank on January 13, 1999 informing it that her credit cards had not been
the appellate court. honored and demanding the refund of her overseas call expenses amounting to 132,000 liras or ₱3,175.00 at the time. Citibank
did not respond to the letter, however, drawing Azucena to write Citibank for the cancellation of the cards.
Procedural infirmities aside, this Court
took a considered look at the spouses’ Citibank still sent billing statements to Azucena, however, charging her interest charges and late payment penalties. Only after
excuse to justify their non-appearance at Azucena’s counsel informed Citibank of imminent legal remedies on her part did Citibank indulge Azucena with a written
the pre-trial but found nothing explanation why her credit cards were not honored in Italy.
exceptional to warrant a reversal of the
lower courts’ disposition thereof. Spouses Corpuz filed a complaint for damages against Citibank at the RTC of Las Pinas City. Citibank files its Answer with
Compulsory Counterclaim. After an exchange of pleadings ─ reply, rejoinder and sur-rejoinder ─ by the parties, and the issues
having been joined, the trial court set the case for pre-trial conference on May 5, 2003 during which the spouses and their
counsel failed to appear, despite notice. On Citibank’s counsel’s motion, the trial court, by Order of even date, dismissed t he
spouses’ Complaint and directed Citibank to present evidence on its Compulsory Counterclaim.

Spouses assailed the Trial Court order via petition for Certiorari. And the Court of Appeals ruled against the spouses.

Issue: Whether or not the non-appearance of Spouses Cruz and their counsel at the pre-trial is excusable

Held: No. Section 5 of Rule 18 provides that the dismissal of an action due to the plaintiff’s failure to appear at the pre-trial shall
be with prejudice, unless otherwise ordered by the court. In this case, the trial court deemed the plaintiffs-herein spouses as
non-suited and ordered the dismissal of their Complaint. As the dismissal was a final order, the proper remedy was to file an
ordinary appeal and not a petition for certiorari. The spouses’ petition for certiorari was thus properly dismissed by the appellate
court.

Procedural infirmities aside, this Court took a considered look at the spouses’ excuse to justify their non-appearance at the pre-
trial but found nothing exceptional to warrant a reversal of the lower courts’ disposition thereof.

Counsel for the spouses admit having failed to inform his clients of the scheduled pre-trial because he forgot to note the same in
his calendar and eventually forgot about it due to "heavy workload." The spouses eventually admitted too having received the
notice of pre-trial. Azucena, who is a lawyer herself, advanced the reason that she forgot about the scheduled pre-trial owing to
her then forthcoming retirement at the Office of the Solicitor General to thus press her to accomplish her assigned work
including winding up all administrative matters in the office prior to her leaving.

While Section 4 of Rule 18 of the Rules of Court allows as an exception a valid cause for the non-appearance of a party at the pre-
trial, the instances cited by the spouses and their counsel hardly constitute compelling exigencies or situations which warrant
occasional flexibility of litigation rules.
As the failure of the spouses to appear at the pre-trial amounted to a failure to comply with the Rules or any order of the court,
the dismissal of their Complaint was essentially due to their fault and the therein defendant Citibank could still prosecute its
Counterclaim in the same or in a separate action.

47. Paredes Absence of counsel for defendants at pre- The protracted legal battle between the parties began with a complaint for the establishment of a right of way. Alleging that
vs. Verano 164375 trial does not ipso facto authorize the petitioners had blocked the passageway in violation of a Compromise Agreement.
12 October 2006 judge to declare the defendant as in
YUMUL default and order the presentation of In their answer, petitioners denied having violated the Compromise Agreement. They alleged that like them, respondents were
evidence ex parte. It bears stressing that not actual residents of Barangay Tagnipa where the "road right of way" was established. Subsequent to the answer, petitioners
nothing in the Rules of Court sanctions filed a motion to dismiss on the ground of lack of cause of action.
the presentation of evidence ex parte The trial court under Judge Kapili and CA denied the motion to dismiss.
upon instances when counsel for
defendant is absent during pre-trial. The The pre-trial was rescheduled multiple times due to the parties’ counsels not appearing. RTC acceded and reset the pre-trial for
Rules do not countenance stringent 23 January 2004.
construction at the expense of justice and
equity.
Shortly before the new pre-trial date, counsel for petitioners filed a Manifestation of Willingness to Settle With Request for
Cancellation dated 5 January 2004.12 Apart from manifesting his willingness to settle the complaint, petitioners' counsel
through the Manifestation suggested to the opposing counsel that he be informed of the terms of the proposed settlement.
Correspondingly, petitioners' counsel requested the cancellation of the 23 January 2004 hearing.

The hearing still pushed through and private respondents and their counsel were present. So were petitioners Baybay and
Paderes, and co-defendant Alago, but not their counsel. RTC allowed respondents to present their evidence ex parte, "for failure
of the defendants counsel to appear before the RTC”. Petitioners filed a motion for reconsideration, but this was denied by the
RTC.

Petitioners filed a petition for certiorari with the Court of Appeals, assailing the orders of the RTC. However, this was denied
outright.

ISSUE/S:
WON the absence of counsel for defendants at the pre-trial is a ground to declare defendants in default and to authorize
plaintiffs to present evidence ex parte.

HELD:
NO. The order of the RTC allowing respondents to present evidence ex parte was undoubtedly to the detriment of petitioners.
Since the RTC would only consider the evidence presented by respondents, and not that of petitioners, the order strikes at the
heart of the case, disallowing as it does any meaningful defense petitioners could have posed.

The Constitution guarantees that no person shall be deprived of property without due process of law. One manner by which due
process is assured is through the faithful adherence to the procedural rules that govern the behavior of the party-litigants.

And under Section 5, Rule 18, the failure of the plaintiff or defendant to appear during pre-trial authorizes the court to either
dismiss the complaint, if the plaintiff were absent; or to allow the plaintiff to present evidence ex parte, if the defendant were
absent.
Simply put, nothing in the Rules of Court authorizes a trial judge to allow the plaintiff to present evidence ex parte on account of
the absence during pre-trial of the counsel for defendant.

WHEREFORE, the instant petition is hereby GRANTED and the resolutions of the Court of Appeals affirming the Orders of
the Regional Trial Court are REVERSED.

48. Sps. The failure of Spouses Salvador to attend FACTS:


Salvador vs. Sps. pre-trial conference warrants the Petitioners Spouses Salvador sold a parcel of land located in Mandaluyong City to respondents Spouses Rabaja, through Rosario
Rabaja 199990 4 presentation of evidence ex parte by Gonzales, the seller’s agent. From 1994-2002, respondents have been renting the property from petitioners. When they learned
February 2015 Spouses Rabaja. The Court reiterates the that petitioners were looking for a buyer, respondents expressed desire to purchase the land and petitioners introduced Gonzales
ZAPANTA rule that the failure to attend the pre-trial to respondents as administrator of the land. Petitioners even handed to Gonzales the owner’s duplicate certificate of title on the
conference does not result in the default property.
of an absent party. Under the 1997 Rules
of Civil Procedure, a defendant is only In 1998, respondents paid 48,000 pesos to Gonzales, who then had an SPA executed by petitioner-Rolando, in the presence of
declared in default if he fails to file his petitioner-Herminia. On the same day, the parties executed a contract to sell, stipulating the amount of the property at 5 million
Answer within the reglementary period. pesos. Respondents made several payments amounting to 950,000 pesos to Gonzales pursuant to the SPA provided earlier as
On the other hand, if a defendant fails to evidenced by the check vouchers signed by Gonzales and the improvised receipts signed by Herminia. But in June 1999,
attend the pre-trial conference, the petitioners complained that they did not receive any payment from respondents, so respondents suspended further payment of
plaintiff can present his evidence ex the purchase price; and as a consequence, they received a notice to vacate the subject property from Spouses Salvador for non-
parte. payment of rentals.

An ejectment suit was filed by petitioners before the MeTC, while a case for rescission of contract was filed by respondents
before the RTC. Petitioners won in the ejectment suit and were able to garnish 593,400 pesos from respondents’ time deposits
representing payment for the back rentals, as ordered by the trial court in the writ of execution.

RTC reversed MeTC and ruled that there was no lease agreement between the parties. This prompted petitioners to appeal the
case before the CA, which reinstated MeTC’s ruling. This CA decision became final and executory.
Meanwhile, RTC, in the rescission case, ordered the rescission of the contract. Petitioners were only able to attend the first of the
many pre-trial conference, leading to the RTC’s issuance of an order of default, allowing respondents to present evidence ex-
parte . The RTC ruled in respondents’ favor and held that t-he contract was one of sale not a contract to sell, which can
appropriately be rescinded, being a contract with reciprocal obligations. In its ruling in favor of respondents, it ordered the
return of the garnished amount and the 950,000 pesos which represents the purchase price.

CA affirmed the RTC’s decision, modifying it on the part that the latter held solidary liability of Gonzales and petitioners. CA
held that since Gonzales did not exceed his authority, he is not solidarily liable with petitioners in the obligation to return the
purchase price.

ISSUES:
1.Whether or not default order must be lifted for existence of reasonable grounds to justify non-attendance of petitioners.
2.Whether or not the receipts given by Gonzales, SPA, and contract of sale are valid

RULING:
1.The failure of Spouses Salvador to attend pre-trial conference warrants the presentation of evidence ex parte by Spouses
Rabaja. The Court reiterates the rule that the failure to attend the pre-trial conference does not result in the default of an absent
party. Under the 1997 Rules of Civil Procedure, a defendant is only declared in default if he fails to file his Answer within the
reglementary period. On the other hand, if a defendant fails to attend the pre-trial conference, the plaintiff can present his
evidence ex parte. Justice Regalado, in his book, clarified that while the order of default no longer obtained when a party fails to
appear at the pre-trial conference, its effects were retained. There is no dispute that Spouses Salvador and their counsel failed to
attend the pre-trial conference set on February 4, 2005 despite proper notice. Spouses Salvador aver that their non-attendance
was due to the fault of their counsel as he forgot to update his calendar. This excuse smacks of carelessness, and indifference to
the pre-trial stage. It simply cannot be considered as a justifiable excuse by the Court.

2.On the SPA. According to Article 1990 of the New Civil Code, insofar as third persons are concerned, an act is deemed to have
been performed within the scope of the agent's authority, if such act is within the terms of the power of attorney, as written.
Respondents did not recklessly enter into a contract to sell with Gonzales. There was a valid SPA so respondents properly made
payments to Gonzales, as agent of petitioners; and it was as if they paid to the petitioners.

On Gonzales’ receipt of payment. It is of no moment, insofar as respondents are concerned, whether or not the payments were
actually remitted to petitioners. Any internal matter, arrangement, grievance or strife between the principal and the agent is
theirs alone and should not affect third persons.

On the contract of sale. The Court agrees with the courts below in finding that the contract entered into by the parties was
essentially a contract of sale which could be validly rescinded.

49. Aguilar vs. The pre-trial cannot be taken for granted. FACTS:
Lightbringers It is not a mere technicality in court
Credit proceedings for it serves a vital objective: This case stemmed from the three (3) complaints for sum of money separately filed by respondent Lightbringers Credit
Cooperative the simplification, abbreviation and Cooperative (respondent) on July 14, 2008 against petitioners Aguilar and Calimbas, and one Perlita Tantiangco (Tantiangco)
209605 12 expedition of the trial, if not indeed its which were consolidated before the First Municipal Circuit Trial Court, Dinalupihan, Bataan (MCTC). The complaints alleged
January 2015 dispensation. that Tantiangco, Aguilar and Calimbas were members of the cooperative who borrowed the following funds:
ABAD
AMENDED VER: 1. In Civil Case No. 1428, Tantiangco allegedly borrowed P206,315.71

Instead of defendant being declared "as 2. In Civil Case No. 1429, petitioner Calimbas allegedly borrowed P202,800.18
in default" by reason of his non-
appearance, this section now spells out 3. In Civil Case No. 1430, petitioner Aguilar allegedly borrowed P126,849.00
that the procedure will be to allow the ex
parte presentation of plaintiff’s evidence Tantiangco, Aguilar and Calimbas filed their respective answers. They uniformly claimed that the discrepancy between the
and the rendition of judgment on the principal amount of the loan evidenced by the cash disbursement voucher and the net amount of loan reflected in the PNB
basis thereof. While actually the checks showed that they never borrowed the amounts being collected. On the scheduled pre-trial conference, only respondent
procedure remains the same, the purpose and its counsel appeared. The MCTC then issued the Ordeallowing respondent to present evidence ex parte. Respondent later
is one of semantical propriety or presented Fernando Manalili (Manalili), its incumbent General Manager, as its sole witness.
terminological accuracy as there were
criticisms on the use of the word Aguilar and Calimbas insisted that they should have the right to cross-examine the witness of respondent, notwithstanding the
"default" in the former provision since fact that these cases were being heard ex parte. In the interest of justice, the MCTC directed the counsels of the parties to submit
that term is identified with the failure to their respective position papers on the issue of whether or not a party who had been declared “as in default” might still
file a required answer, not appearance in participate in the trial of the case. Only respondent, however, complied with the directive.
court.
In its Order, dated April 27, 2011, the MCTC held that since the proceedings were being heard ex parte, the petitioners who had
9

If the absent party is the plaintiff, then been declared “as in default” had no right to participate therein and to cross-examine the witnesses. Thereafter, respondent filed
his case shall be dismissed. If it is the its formal offer of evidence.
defendant who fails to appear, then the
plaintiff is allowed to present his MCTC: The MCTC dismissed the complaint against Tantiangco because there was no showing that she received the amount
evidence ex parte and the court shall being claimed. Moreover, the PNB check was made payable to “cash” and was encashed by a certain Violeta Aguilar. There was,
render judgment on the basis thereof however, no evidence that she gave the proceeds to Tantiangco. However, the MCTC found both Calimbas and Aguilar liable to
respondent for their respective debts. The PNB checks issued to the petitioners proved the existence of the loan transactions.
Their receipts of the loan were proven by their signatures appearing on the dorsal portions of the checks as well as on the cash
disbursement vouchers.

RTC: Affirmed the MCTC decisions. It held that the PNB checks were concrete evidence of the indebtedness of the petitioners to
respondent. The RTC relied on the findings of the MCTC that the checks bore no endorsement to another person or entity. The
checks were issued in the name of the petitioners and, thus, they had the right to encash the same and appropriate the proceeds.

CA: Dismissed petition for review stating that the petition was formally defective because the “verification and disclaimer of
forum shopping” and the “affidavit of service” had a defective jurat for failure of the notary public to indicate his notarial
commission number and office address. Moreover, the entire records of the case, inclusive of the oral and documents evidence,
were not attached to the petition in contravention of Section 2, Rule 42 of the Rules of Court.

ISSUES: Whether or not the CA erred when it dismissed the petition for review filed before it by the petitioners under the Rule
42 of the rules of court.

HELD: The rule is that a court can only consider the evidence presented by respondent in the MCTC because the petitioners
failed to attend the pre-trial conference on August 25, 2009 pursuant to Section 5, Rule 18 of the Rules of Court. The Court,
33

however, clarifies that failure to attend the pre-trial does not result in the “default” of the defendant. Instead, the failure of the
defendant to attend shall be cause to allow the plaintiff to present his evidence ex parte and the court to render judgment on the
basis thereof.

The case of Philippine American Life & General Insurance Company v. Joseph Enario discussed the difference between non-
34

appearance of a defendant in a pre-trial conference and the declaration of a defendant in default in the present Rules of Civil
Procedure. The decision states:c

Prior to the 1997 Revised Rules of Civil Procedure, the phrase "as in default" was initially included in Rule 20 of the old
rules, and which read as follows:

Sec. 2. A party who fails to appear at a pre-trial conference may be non-suited or considered as in default.

It was however amended in the 1997 Revised Rules of Civil Procedure. Justice Regalado, in his book REMEDIAL
LAW COMPENDIUM, explained the rationale for the deletion of the phrase "as in default" in the amended provision, to wit:

1. This is a substantial reproduction of Section 2 of the former Rule 20 with the change that, instead of defendant being declared
"as in default" by reason of his non-appearance, this section now spells out that the procedure will be to allow the ex parte
presentation of plaintiff’s evidence and the rendition of judgment on the basis thereof. While actually the procedure remains the
same, the purpose is one of semantical propriety or terminological accuracy as there were criticisms on the use of the word
"default" in the former provision since that term is identified with the failure to file a required answer, not appearance in court.

If the absent party is the plaintiff, then his case shall be dismissed. If it is the defendant who fails to appear, then the plaintiff is
allowed to present his evidence ex parte and the court shall render judgment on the basis thereof. Thus, the plaintiff is given the
privilege to present his evidence without objection from the defendant, the likelihood being that the court will decide in favor of
the plaintiff, the defendant having forfeited the opportunity to rebut or present his own evidence.

The pre-trial cannot be taken for granted. It is not a mere technicality in court proceedings for it serves a vital objective: the
simplification, abbreviation and expedition of the trial, if not indeed its dispensation. More significantly, the pre-trial has been
36

institutionalized as the answer to the clarion call for the speedy disposition of cases. Hailed as the most important procedural
innovation in Anglo-Saxon justice in the nineteenth century, it paved the way for a less cluttered trial and resolution of the case.
It is, thus, mandatory for the trial court to conduct pre-trial in civil cases in order to realize the paramount objective of
simplifying, abbreviating and expediting trial. chanRoblesvirtualLawlibrary
37

In the case at bench, the petitioners failed to attend the pre-trial conference set on August 25, 2009. They did not even give any
excuse for their non-appearance, manifestly ignoring the importance of the pre-trial stage. Thus, the MCTC properly issued the
August 25, 2009 Order, allowing respondent to present evidence ex parte.
38

50. Tolentino
vs. Laurel 181368 the failure of a party to appear at the pre- Facts: Respondents, in their complaint before the Regional Trial Court, alleged that they are the registered owners of a parcel of
22 February 2012 trial has adverse consequences. If the land covered by Transfer Certificate of Title (TCT) No. T-4392 which they have developed into fishponds. respondents informed
ARIBON absent party is the plaintiff, then his case petitioners, through Gustavo C. Tolentino, Sr. (Gustavo) who was then representing them, that the area they are occupying was
shall be dismissed. If it is the defendant inside the respondents' property and, therefore, they should vacate and leave the same. Respondents filed a suit against
who fails to appear, then the plaintiff is petitioners to recover the property and demand payment for unearned income, damages and attorneys fees. Petitioners averred
allowed to present his evidence ex parte that since the subject property is owned by the Republic, and they are occupying the same by virtue of a Fishpond Licensing
and the court shall render judgment on Agreement, their stay was lawful. Petitioners were declared in default for failure to appear at the pre-trial conference. Despite
the basis thereof several resetting of pre-trial conference of which petitioner was notified, petitioners still failed to appear. Trial court issued an
Order allowing respondents to present their evidence ex parte, instead of declaring petitioners in default. RTC ruled in favor of
respondents. Petitioner elevated the case to the CA claiming that they have been denied of due process. CA denied. Petitioner
filed an MR, but likewise denied. Hence, this petition

ISSUE: W/N the petitioners were denied their day in court

HELD: No. In the case at bar, the trial court gave petitioners every chance to air their side and even reconsidered its first order
declaring petitioners in default. Petitioners were given more than ample opportunity to be heard through counsel. When the
petitioners were first declared in default on August 27, 1996, the pre-trial conference was set and reset for several times, which
prompted the trial court to allow the respondents to present their evidence ex parte. Thereafter, the judgment was rendered. The
Court explained that such allowance was in accordance with Rule 18 of the 1997 Rules of Civil Procedure and with due regard to
the constitutional guarantee of due process. A perusal of Sections 4 and 5 of Rule 13 of the ROC, the failure of a party to a ppear
at a pre-trial has adverse consequences. If the absent party is the plaintiff, then the case shall be dismissed. If it is the defendant
who fails to appear, then the plaintiff is allowed to present his evidence ex parte an the court shall render judgment on the basis
thereon. Hence, petition denied.
51. Saguid vs. Under Section 6, Rule 18 of the Rules of FACTS: 17-year old Gina Rey was married but separated de facto from her husband when she met petitioner Jacinto Saguid.
CA 150611 10 Civil Procedure, the failure of the After 9 years of cohabitation, the couple eventually decided to separate as well.
June 2003 defendant to file a pre-trial brief shall Private respondent Gina Rey filed a Complaint for Partition and Recovery of Personal Property with Receivership against Jacinto
BRIONES have the same effect as failure to appear Saguid with the RTC. RTC declared petitioner in default for failure to file a pre-trial brief as required by SC Circular No.1-89. CA
at the pre-trial, i.e The plaintiff may denied petitioner’s Motion for Reconsideration and also ordered that private respondent was allowed to present evidence ex
present his evidence ex parte and the parte for failure of defendant to file a pre-trial brief. Petitioner’s second Motion for Reconsideration was denied.
court shall render judgment on the basis The petitioner contends that his failure to file a pre-trial brief is justified because he was not represented by counsel.
thereof.
The remedy of the defendant is to file a ISSUE: Whether or not the trial court erred in allowing Gina Rey to present evidence ex parte.
motion for reconsideration showing that
his failure to file a pre-trial brief was due HELD: No, the trial court didn’t err.
to fraud, accident, mistake or excusable In the case at bar, petitioner insists that his failure to file a pre-trial brief is justified because he was not represented by counsel.
neglect. This justification is not, however, sufficient to set aside the order directing private respondent to present evidence ex parte,
inasmuch as the petitioner chose at his own risk not to be represented by counsel. Even without the assistance of a lawyer,
petitioner was able to file a motion for extension to file an answer, the required answer stating therein the special and affirmative
defenses, and several other motions. If it were true that the petitioner didn’t understand the import of the April 23, 1997 order
directing him to file a pretrial brief, he could have inquired from the court or filed a motion for extension of time to file the brief.
Instead, he waited until May 26, 1997, or 14 days from his alleged receipt of the April 23, 1997 order before he filed a motion
asking the court to excuse his failure to file a brief. Pretrial rules are not to be belittled or dismissed because their non-
observance may result in prejudice to a party’s substantive rights. Like all rules, they should be followed except only for the most
persuasive of reasons when they may be relaxed to relieve a litigant of an injustice not commensurate with the degree of his
thoughtlessness in not complying with the procedure prescribed.

52. Tiu vs. Pre-trial is an answer to the clarion call FACTS: The present Petition arose from a Complaint for recovery of ownership and possession of real property, accounting and
Middleton for the speedy disposition of cases. It is damages filed against herein petitioner before the RTC of Oroquieta City.
134998 19 July essential in the simplification and the
1999 speedy disposition of disputes. In light of PROCEDURAL HISTORY: Before the commencement of trial, the court a quo sent a Notice of Pre-trial Conference, stating
CASTRO the objectives of a pre-trial and the role that witnesses whose names and addresses are not submitted at the pre-trial may not be allowed to testify at the trial, and
of the trial court therein, it is evident that documents not marked as exhibits at the pre-trial, except those then available or existing, may be barred admission in evidence.
judges have the discretion to exclude In his Pre-trial brief, petitioner averred that he would be presenting six witnesses, but he did not name them. When trial ensued,
witnesses and other pieces of evidence herein respondents, as plaintiffs in the case, presented their witnesses in dure course. When his turn came, petitioner called his
not listed in the pre-trial brief, provided first witness ,Antonia Tiu, respondents objected, arguing that the witness could not be allowed to testify because petitioner had
the parties are given prior notice to this failed to name her in his Pre-trial Brief. The RTC ruled in favor of respondents, stating that Antonia Tiu could not be presented
effect. as a witness.
In his Pre-trial Order, however, the trial ISSUE: WON a judge may exclude a witness whose name and synopsis of testimony were not included in the pre-trial brief
judge did not exercise his discretion to
exclude the unlisted or unnamed RULING: Yes. Pre-trial is an answer to the clarion call for the speedy disposition of cases. It is essential in the simplification
witnesses. Rather, it simply provided that and the speedy disposition of disputes. In light of the objectives of a pre-trial and the role of the trial court therein, it is evident
[t]he defendant will present six that judges have the discretion to exclude witnesses and other pieces of evidence not listed in the pre-trial brief, provided the
witnesses. It made no mention at all that parties are given prior notice to this effect.
they would be barred from testifying
unless they were named. Significantly, it In his Pre-trial Order, however, the trial judge did not exercise his discretion to exclude the unlisted or unnamed witnesses.
also stated that plaintiffs will offer ten Rather, it simply provided that [t]he defendant will present six witnesses. It made no mention at all that they would be barred
witnesses, without however naming from testifying unless they were named. Significantly, it also stated that plaintiffs will offer ten witnesses, without however
them. Since the Order allowed naming them. Since the Order allowed respondents (as plaintiffs before the trial court) to present witnesses, it necessarily
respondents (as plaintiffs before the trial follows that it should grant the same right to petitioner.
court) to present witnesses, it necessarily
follows that it should grant the same Indeed, the court and the parties must pay attention not only to the pre-trial briefs, but also to the pre-trial order. Section 7 of
right to petitioner. the same Rule states:

SEC. 7. Record of pre-trial. -- The proceedings in the pre-trial shall be recorded. Upon the termination thereof, the
court shall issue an order which shall recite in detail the matters taken up in the conference, the action taken thereon,
the amendments allowed to the pleadings, and the agreements or admissions made by the parties as to any of the
matters considered. Should the action proceed to trial, the order shall explicitly define and limit the issues to be
tried. The contents of the order shall control the subsequent course of action, unless modified before trial to prevent
manifest injustice.
Hence, the provision in the Pre-trial Order allowing petitioner to present six witnesses shall control the subsequent course of
action. The court a quo proceeded with the trial without modifying the Order. In the same vein, respondents did not challenge it
before the trial. Neither did they invoke the power of the trial court to compel the petitioner to submit the names of his witnesses
and summaries of their testimonies. By their silence, respondents acquiesced to the Pre-trial Order allowing the presentation of
petitioner's unnamed witnesses. Modifying a pre-trial order during the trial or, worse, when the defendant is about to present
witnesses will indubitably result in manifest injustice. This could not have been the intention of the Rules.

53. Kent vs. Although the RTC has legal basis to FACTS: This petition draws its origin from a complaint for recovery of real property and annulment of title led by petitioner,
Micarez 185758 9 order the dismissal of the case, the Court through her younger sister and authorized representative, Rosita Micarez-Manalang, before the RTC. Petitioner is of Filipino
March 2011 finds this sanction too severe to be descent who became a naturalized American citizen after marrying an "American national. She is now a permanent resident of
DE GALA imposed on the petitioner where the the United States of America. The petitioner claimed that her parents the herein respondents fraudulently and clandestinely
records of the case is devoid of evidence transferred her property to her brother, one of the respondents.
of willful or flagrant disregard of the
rules on mediation proceedings. There is Aware that it was difficult to register a real property under her name, she being married to an American citizen, she purchased
no clear demonstration that the absence the subject property and registered it under the name of her parents. A deed of absolute sale was executed between her parents
of petitioner’s representative during and the owner of the said property. TCT was issued in the name of her parents. Years later the petitioner learned that the said
mediation proceedings was intended to property was sold by her parents to her brother. Considering that all the respondents are residents of United States summons
perpetuate delay in the litigation of the was
case. Neither is it indicative of lack of pre-trial conference with power to enter into compromise agreement. The RTC ordered the referral of the case to Philippine
interest on the part of petitioner to enter Mediation Center, however respondents failed to appear during the schedule. The Court ordered the petitioner to present her
into a possible amicable settlement of the evidence ex-parte. However, the counsel of the respondents clarified that it was the counsel of the petitioner who did not appear
case. during the scheduled mediation proceedings, the respondents counsel further explained that their counsel had inadvertently
affixed his signature on the space provided for the counsel of the plaintiff in the mediation report
PROCEDURAL HISTORY: Considering that all the respondents are now also permanent residents of the USA, summons was
served upon them by publication. Meanwhile, the respondents executed two special powers of attorney authorizing their counsel
to file their answer and to represent them during the pre-trial conference and all subsequent hearings with power to enter into a
compromise agreement.
After the parties had filed their respective pre-trial briefs, and the issues in the case had been joined, the RTC referred to case to
the Philippine Mediation Center (PMC)/. Mediator Esmeraldo padao, Sr. issued a Mediator’s Report to the RTC allegedly due to
the non-appearance of the respondents on the scheduled conference before him. Acting on said report, the RTC issued an order
allowing petitioner to present her evidence ex parte.
Later, Padao clarified, though a Manifestation, that it was petitioner who did not attend the mediation proceedings. He
explained that respondent’s counsel inadvertently fixed his signature for attendance purposes on the column provided for the
plaintiff’s counsel in the mediator’s report. In light of this development, the RTC issued an order dismissing the case for failure
of plaintiffs and her counsel to appear during the mediation proceeding.
Petitioner filed a motion for reconsideration invoking the relaxation of the rule on non-appearance in the mediation proceedings
in the interest of justice and equity. Petitioner urged the trial court not to dismiss the case based merely on technicalities
contending that litigations should as much as possible be decided on the merits.
ISSUE: WON the dismissal is the proper sanction for failure to attend the mediation process
RULING: No. Although the RTC has legal basis to order the dismissal of the case, the Court finds this sanction too severe to be
imposed on the petitioner where the records of the case is devoid of evidence of willful or flagrant disregard of the rules on
mediation proceedings. There is no clear demonstration that the absence of petitioner’s representative during mediation
proceedings was intended to perpetuate delay in the litigation of the case. Neither is it indicative of lack of interest on the part of
petitioner to enter into a possible amicable settlement of the case.
Manalang was not entirely at fault for the cancellation and resettings of the conferences. Respondents’ representative and
counsel, Atty. Miguel, came late during the January 19 and February 9, 2008 conferences which resulted in their cancellation
and the final resetting of the mediation proceedings to March 1, 2008. Considering the circumstances, it would be most unfair to
penalize petitioner for the neglect of her lawyer.
Assuming arguendo that the trial court correctly construed the absence of Manalang as a deliberate refusal to comply with its
Order or to be dilatory, it cannot be said that the court was powerless and virtually without recourse. Indeed, there are other
available remedies to the court a quo under A.M. No. 01-10-5-SC-PHILJA, apart from immediately ordering the dismissal of the
case. If Manalang’s absence upset the intention of the court a quo to promptly dispose the case, a mere censure or reprimand
would have been sufficient for petitioner’s representative and her counsel so as to be informed of the court’s intolerance of
tardiness and laxity in the observation of its order. By failing to do so and refusing to resuscitate the case, the RTC impetuously
deprived petitioner of the opportunity to recover the land which she allegedly paid for.
Unless the conduct of the party is so negligent, irresponsible, contumacious, or dilatory as for non-appearance to provide
substantial grounds for dismissal, the courts should consider lesser sanctions which would still achieve the desired end. The
Court has written "inconsiderate dismissals, even if without prejudice, do not constitute a panacea nor a solution to the
congestion of court dockets, while they lend a deceptive aura of efficiency to records of the individual judges, they merely
postpone the ultimate reckoning between the parties. In the absence of clear lack of merit or intention to delay, justice is better
served by a brief continuance, trial on the merits, and final disposition of the cases before the court.

54. Citibank We reiterate the previous admonitions of


Facts: Private respondents, spouses Cresencio and Zenaida Velez filed a complaint for specific performance and damages
N.A. vs. Chua this Court against "precipitate orders of
against Citibank in the RTC of Cebu. Thereafter Citibank filed a criminal complaint against private respondents for violation of
102300 17 March default as these have the effect of denying
BP Blg. 22 (Bouncing Checks Law) and estafa. Citibank submitted its answer to the complaint filed by private respondents. On
1993 the litigant the chance to be heard. While
the date of the pre-trial conference, counsel for Citibank appeared, presenting a special power of attorney executed by Citibank
ESPIRITU there are instances, to be sure, when a
officer Florencia Tarriela in favor of petitioner bank's counsel, the J.P. Garcia & Associates, to represent and bind petitioner
party may be properly defaulted, these
bank at the pre-trial conference of the case at bar.
should be the exceptions rather than the
rule and should be allowed only in clear Despite of this special power of attorney, counsel for private respondents orally moved to declare petitioner bank as in default
cases of an obstinate refusal or on the ground that the special power of attorney was not executed by the Board of Directors of Citibank. Petitioner bank was
inordinate neglect to comply with the then required to file a written opposition to this oral motion to declare it as in default. In said opposition petitioner bank
orders of the court. Absent such a attached another special power of attorney made by William W. Ferguson, Vice President of Citibank, constituting and
showing, the party must be given every appointing the J.P. Garcia & Associates to represent and bind the BANK at the pre-trial conference and/or trial of the case of
reasonable opportunity to present his "Cresencio Velez, et al. vs. Citibank, N.A.". In an Order dated April 23, 1990, respondent judge denied private respondents'
side and to refute the evidence of the oral motion to declare petitioner bank as in default and set the continuation of the pre-trial conference for May 2, 1990.
adverse party in deference to due process
On the scheduled pre-trial conference, private respondents reiterated, by way of asking for reconsideration, their oral motion
of law".|||
to declare petitioner bank as in default for its failure to appear through an authorized agent and that the documents presented
are not in accordance with the requirements of the law. Thereafter, petitioner bank filed a manifestation, attaching therewith a
special power of attorney executed by William W. Ferguson in favor of Citibank employees to represent and bind Citibank on
the pre-trial conference of the case at bar.
On August 15, 1990, respondent judge issued an order declaring petitioner bank as in default. It reasoned out that "Defendant-
bank, although a foreign corporation, is bound by Philippine laws when doing and conducting business in the Philippines, and
its corporate powers could only be exercised by its Board of Directors (Sec. 23, B.P. Blg. 68). The Special Power of Attorney
executed by Mr. William W. Ferguson in favor of the alleged Citibank employees to represent the defendant in the pre-trial
conference, made no mention of J.P. Garcia & Associates as one of the employees of the defendant. It stands to reason
therefore, that the defendant-bank has no proper representation during the pre-trial conference.
Petitioner bank then filed a petition for certiorari, prohibition and mandamus with preliminary injunction and/or temporary
restraining order with the Court of Appeals.
Issue: Whether Citibank’s counsel is validly authorized to represent Citibank during the pre-trial

Ruling: YES. Although as a general rule, all corporate powers are to be exercised by the board of directors, exceptions are made
where the Code provides otherwise. Since the by-laws are a source of authority for corporate officers and agents of the
corporation, a resolution of the Board of Directors of Citibank appointing an attorney in fact to represent and bind it during the
pre-trial conference of the case at bar is not necessary because its by-laws allow its officers to execute a power of attorney to a
designated bank officer, William Ferguson in this case, clothing him with authority to direct and manage corporate affairs.

Since the general power of attorney granted to Ferguson allows him to delegate his powers in whole or in part, there can be no
doubt that the special power of attorney in favor, first, of J.P. Garcia & Associates and later, of the bank's employees, constitutes
a valid delegation of Ferguson's express power to represent petitioner bank in the pre-trial conference in the lower court.

From the outset, petitioner bank showed a willingness, if not zeal, in pursuing and defending this case. It even acceded to private
respondent's insistence on the question of proper representation during the pre-trial by presenting not just one, but three,
special powers of attorney. Initially, the special power of attorney was executed by Florencia Tarriela in favor o, petitioner bank's
counsel. Private respondents insisted that this was not proper authority required by law. To avoid further argument, a second
special power of attorney was presented by petitioner bank, executed by William Fersugon, the highest ranking officer of
Citibank, in favor of its counsel. But since the authority to delegate of Fersugon in favor of an agent is limited to bank employees,
another special power of attorney from Wiliam W. Fersugon in favor of the Citibank employees was presented. But the
respondent trial court judge disregarded all these and issued the assailed default order. There is nothing to show that petitioner
bank "miserably failed to oblige"; on the contrary, three special powers of attorney manifest prudence and diligence on
petitioner bank's part.

In fact, there was no need for the third power of attorney because we believe that the second power of attorney was sufficient
under the by-law provision authorizing Fersugon to delegate any of his functions to any one or more employees of the petitioner
bank. A reasonable interpretation of this provision would include an appointment of a legal counsel to represent the bank in
court, for, under the circumstances, such legal counsel can be considered, and in fact was considered by the petitioner bank, an
employee for a special purpose. Furthermore, Fersugon, who heads the Philippine office thousands of miles away from its main
office in the United States, must be understood to have sufficient powers to act promptly in order to protect the interests of his
principal. We reiterate the previous admonitions of this Court against "precipitate orders of default” .. (see doctrine)

Considering further that petitioner bank has a meritorious defense and that the amount in contest is substantial, the litigants
should be allowed to settle their claims on the arena of the court based on a trial on the merits rather than on mere technicalities.

55. Calalang A pre-trial cannot validly be held until Facts: Respondent Filipinas Manufacturers Bank filed a complaint of a sum of money against the petitioner Conrado Calalang
vs. Court of the last pleading has been filed, which and 3 other defendants namely, Hugo M. Arca, Rio Arturi Salceda and the Acropolis Trading Corporation with the CFI of Rizal.
Appeals 103185 last pleading may be the plaintiff's reply, Calalang filed a motion for the dismissal of the case on the ground that the plaintiff has no cause of action against him. This
22 January 1993 except where the period to file the last necessitated the filing of an opposition from the plaintiff, a reply to said opposition from the defendant Calalang, and a rejoinder
LEE pleading has lapsed. to the said reply. The defendant Arca, on the other hand initially sought an extension of time to file a responsive pleading then
filed a motion for a bill of particulars, then later also a motion to dismiss the case. After his motion to dismiss was denied Arca
filed a motion for reconsideration. In all these incidents pleadings and counter-pleadings were filed and hearings held on the
motions, which resulted in the case dragging on for a considerable time. The case was set for pre-trial several times when, as
aforestated, the issues were not yet joined for only Arca had initially filed his answer to the complaint. The case was ordered
dismissed at least two times when the plaintiff’s counsel failed to appear at these pre-trials but the dismissals were considered
and the class set anew. Another factor that contributed to the confusion in the proceedings and the delay in the case is the fact
that the case was assigned from one judge to another due probably to the judicial reorganization that took place. In fact, there
were no less than 4 judges who handled the case. The answer of Arca was filed only on 1985 while Calalang’s was filed on 1987.

Issue: Whether the pre-trial conference was premature

Held: No. The pre-trial conference was not pre-mature. A pre-trial cannot validly be held until the last pleading has been filed,
which last pleading may be the plaintiff's reply, except where the period to file the last pleading has lapsed. The period to appear
and file the necessary pleading having expired on the Acropolis Trading Corporation, the lower court can direct that a pre-trial
conference be held among the answering defendants. however, though it is within the discretion of the trial court to declare a
party non-suited for non appearance in the pre-trial conference, such discretion must not be abused. The precipitate haste of the
lower court in declaring the respondent bank non-suited was uncalled for and deserved a second look. Considering the fact that
the counsel for the plaintiff/respondent bank did arrive for the pre-trial conference, though a bit late and that counsel for the
defendant was himself also late, the trial court should have called the case again. An admonition to both counsels to be more
prompt in appearing before the Court as scheduled would have sufficed, instead of having dismissed the complaint outright.
Unless a party's conduct is so negligent, irresponsible, contumacious, or dilatory as to provide substantial grounds for dismissal
for non-appearance, the courts should consider lesser sanctions which would still amount into achieving the desired end.
"Inconsiderate dismissals, even if without prejudice, do not constitute a panacea nor a solution to the congestion of court
dockets, while they lend a deceptive aura of efficiency to records of individual judges, they merely postpone the ultimate
reckoning between the parties. In the absence of clear lack of merit or intention to delay, justice is better served by a brief
continuance, trial on the merits, and final disposition of the cases before the court." (Ruiz v Enstenzo) And there is authority that
an order dismissing a plaintiff's complaint without prejudice for failure of his counsel to appear at a pre-trial conference must be
reversed as too severe a sanction to visit on a litigant where the record is devoid of evidence reflecting the litigant's willful or
flagrant disregard for the Court's authority.
XIX. INTERVENTION (RULE 19, SEC. 1 TO 4)
56. Mactan Intervention is a remedy by which a third Facts:
Cebu International party, not originally impleaded in the A Complaint for Reconveyance, Cancellation of Defendant's Title, Issuance of New Title to Plaintiffs and Damages was filed by
Airport vs. Heirs of proceedings, becomes a litigant therein Leila for herself and on behalf of the other heirs of the late Estanislao Miñoza. It alleged that Leila's late great grandfather,
Miñoza 186045 2 to enable him, her or it to protect or Estanislao Miñoza, was the registered owner of Cadastral Lot Nos. 986 and 991-A (subject lots) at Cebu City. It was, likewise,
February 2011 preserve a right or interest which may be alleged that the late Estanislao Miñoza had three children, namely, Adriana, Patricio, and Santiago, all surnamed Miñoza. In
LIMIN affected by such proceedings. Under Rule the late 1940s, the National Airports Corporation (NAC) embarked in an expansion project of the Lahug Airport. Among the
19, intervention shall be allowed when a properties that were acquired through a negotiated sale were the subject lots. Leila claimed that their predecessors-in-interest,
person has (1) a legal interest in the Adriana, Patricio, and Santiago, executed a Deed of Sale conveying the subject lots to the NAC on the assurance that they can
matter in litigation; (2) or in the success buy the properties back if the lots are no longer needed. The expansion project eventually did not push through. More than 40
of any of the parties; (3) or an interest after the sale, plaintiffs informed the NAC's successor-in-interest, petitioner MCIAA, that they were exercising the buy-back
against the parties; (4) or when he is so option of the agreement, but the MCIAA refused to allow the repurchase. After the parties filed their respective pleadings, trial
situated as to be adversely affected by a ensued. However, before the MCIAA could present evidence, a Motion for Intervention was filed before the RTC of Cebu City
distribution or disposition of property in by the heirs of Filomeno T. Miñoza, the heirs of Pedro T. Miñoza, and the Heirs of Florencia T. Miñoza (Intervenors), who
the custody of the court or an officer claimed to be the true, legal, and legitimate heirs of the late Estanislao Miñoza. The intervenors alleged in their complaint (1)
thereof. Moreover, the court must take that the plaintiffs in the main case are not related to the late spouses Estanislao Miñoza and Inocencia Togono; (2) that
into consideration whether or not the Adriana, Patricio, and Santiago, executed, in fraud of the intervenors, an Extrajudicial Settlement of the Estate and (3) that the
intervention will unduly delay or same Adriana, Patricio, and Santiago, fraudulently, deceitfully, and in bad faith, sold the subject properties.
prejudice the adjudication of the rights of
the original parties, and whether or not Issue:
the intervenor's right or interest can be Whether or not Motion for Intervention shall be granted.
adequately pursued and protected in a
separate proceeding. Furthermore, the Ruling:
allowance or disallowance of a motion for Intervention is a remedy by which a third party, not originally impleaded in the proceedings, becomes a litigant therein to
intervention rests on the sound enable him, her or it to protect or preserve a right or interest which may be affected by such proceedings. Under Rule 19,
discretion of the court after intervention shall be allowed when a person has (1) a legal interest in the matter in litigation; (2) or in the success of any of
consideration of the appropriate the parties; (3) or an interest against the parties; (4) or when he is so situated as to be adversely affected by a distribution or
circumstances. Consequently, the denial disposition of property in the custody of the court or an officer thereof. Moreover, the court must take into consideration
of the motion to intervene by the RTC whether or not the intervention will unduly delay or prejudice the adjudication of the rights of the original parties, and
was but just and proper. whether or not the intervenor's right or interest can be adequately pursued and protected in a separate proceeding.

In the case at bar, the intervenors are claiming that they are the legitimate heirs of Estanislao Miñoza and Inocencia Togono
and not the original plaintiffs represented by Leila Hermosisima. True, if their allegations were later proven to be valid claims,
the intervenors would surely have a legal interest in the matter in litigation. Nonetheless, this Court has ruled that the interest
contemplated by law must be actual, substantial, material, direct and immediate, and not simply contingent or expectant. It
must be of such direct and immediate character that the intervenor will either gain or lose by the direct legal operation and
effect of the judgment.With regard to the second contention, this issue would unnecessarily complicate and change the nature
of the proceedings. The parties would also present additional evidence in support of this new allegation of fraud, deceit, and
bad faith and resolve issues of conflicting claims of ownership, authenticity of certificates of titles, and regularity in their
acquisition. This would definitely cause unjust delay in the adjudication of the rights claimed by the original parties, which
primarily hinges only on the issue of whether or not the heirs represented by Leila have a right to repurchase the subject
properties from the MCIAA. In general, an independent controversy cannot be injected into a suit by intervention.
Furthermore, the allowance or disallowance of a motion for intervention rests on the sound discretion of the court after
consideration of the appropriate circumstances. Consequently, the denial of the motion to intervene by the RTC was but just
and proper.

57. Pulgar vs. FACTS: The Municipality of Mauban, Quezon assessed the buildings and machinery of the Mauban Plant, a coal-fired electric
RT of Mauban generation facility owned by the Quezon Power Limited (QPL), at a market value of P29, 626,578,291,00 or an annual real
157583 10 estate tax of P500 Million, more or less.
September 2014
NAKAGAWA Quezon Power Limited filed a sworn statement declaring that the properties had a value of only P15, 055,951,378.00 and
tendered to the Municipal Assesor the amount of P60, 233,805.51 as first quarter instalment payment of the real estate taxes.
The Municipal Assessor rejected it, hence QPL filed a complaint for Consignation and Damages before the RTC against the
province of Quezon, the municipal assessor and treasurer, and the provincial assessor and treasurer of Quezon, tendering to
the RTC the payment for the first quarter installment payment on real estate taxes.

The defendants asserted that QPL is estopped from questioning the authority of the municipal assessor since it paid realty
taxes based on the former’s assessment. Flumencio, alleging himself as a taxpayer and resident of Quezon Province, filed his
Motion for Leave to Admit Answer-in-Intervention, since the power plant of QPL was responsible for the mindless disturbance
of the forest and marine environment. His answer in intervention was admitted.

The RTC later dismissed the complaint filed by QPL for lack of jurisdiction in the absence of tax payment under protest which
QPL tried to skirt by alleging that it is the authority of the municipal assessor which it challenges. It ruled that the Local Board
of Assessment Appeals that has jurisdiction to hear the case. It also dismissed Frumencio’s motion for intervention since it had
no leg to stand with the dismissal of the main case. His motion for reconsideration denied, Frumencio filed a petition for
review on certiorari before the Supreme Court to challenge the dismissal of his motion for intervention.

ISSUE: Whether or not the RTC erred in dismissing Pulgar’s motion for intervention as a consequence of the dismissal of the
main case.

HELD: No. Jurisdiction over an intervention is governed by jurisdiction over the main action. Accordingly, an intervention
presupposes the pendency of a suit in a court of competent jurisdiction.

In this case, Pulgar does not contest the RTC’s dismissal of Civil Case No. 0587-M for lack of jurisdiction, but oddly maintains
his intervention by asking in this appeal a review of the correctness of the subject realty tax assessment. This recourse, the
Court, however, finds to be improper since the RTC’s lack of jurisdiction over the main case necessarily resulted in the
dismissal of his intervention. In other words, the cessation of the principal litigation – on jurisdictional grounds at that –
means that Pulgar had, as a matter of course, lost his right to intervene. Verily, it must be borne in mind that:

[I] Intervention is never an independent action, but is ancillary and supplemental to the existing litigation. Its purpose is not
to obstruct nor x xx unnecessarily delay the placid operation of the machinery of trial, but merely to afford one not an original
party, yet having a certain right or interest in the pending case, the opportunity to appear and be joined so he could assert or
protect such right or interests.

Otherwise stated, the right of an intervenor should only be in aid of the right of the original party. Where the right of the latter
has ceased to exist, there is nothing to aid or fight for; hence, the right of intervention ceases.

XX. CALENDAR OF CASES (RULE 20, SEC. 1 TO 2)


58. Ang
The principal issue of alleged grave
Kek Chen vs. Facts:
abuse of discretion in violation of
Bello L-
Circular No. 7 of this Court,
76344-46 30 Petitioner questions the alleged grave abuse of discretion amounting to excess of jurisdiction, committed by respondent Judge
regarding the manner of raffle of
June 1988 Abundio Bello in violating Administrative Circular No. 7 regarding the raffle of Criminal Cases and prays for its outright
cases, not denied or explained by
NATO dismissal.
public respondent, is not a trivial
one. The raffle of cases is of vital
importance to the administration of Petitioner Ang was charged before the then Manila City, with the crimes of "MALTREATMENT," "THREATS," and "SLIGHT
justice because it is intended to PHYSICAL INJURIES," committed against one LE HE CO Y YU DE ANG, as follows:
insure impartial adjudication of
cases. By raffling the cases public 1. by then and there, slapping her and giving her fist/blows on her head- Criminal Case No. 021429 (Maltreatment)
suspicion regarding assignment of
cases to predetermined judges is 2. threatening to kill her- Criminal Case No. 021430 (Threats)
obviated.
3. assault and use personal violence- Criminal Case No. 021431 (Slight Physical Injuries)
A violation or disregard of the
Court's circular on how the raffle of After the prosecution had presented its evidence, Ang filed a Demurrer to Evidence which was denied. Ang elevated the incident
cases should be conducted is not to to the RTC of Manila on certiorari and prohibition with prayer for preliminary injunction and/or temporary restraining orders,
be countenanced. A party has the which was denied.
right to be heard by an impartial
and unbiased tribunal. LLjur On appeal, the Court of Appeals affirmed in toto the RTC.
When the respondent judge
conducted the raffle of the three Meanwhile, the then presiding judge of MTC Branch VIII (where the raises were pending) was promoted to the RTC of Manila.
criminal cases in question,
apparently in violation of the · Respondent judge, as officer-in-charge of the MTC (Manila), directed the return of the case records to the
Court's Circular No. 7, he did not Clerk of Court for "re-raffle.
only arouse the suspicion that he
had some ulterior motive for doing · Petitioner, however, alleged that he received the corresponding order only on August 23, 1984, or AFTER
so, but he violated the cardinal rule the cases had already been actually "re-raffled" and assigned to respondent judge on August 16, 1984.
that all judicial processes must be
done above board. We consider the · On September 27, 1984, Ang filed a motion to re-raffle the cases, which was denied. The subsequent motion
procedure of raffling cases to be an for reconsideration was likewise denied Hence, the present petition.
important element of judicial
proceedings, designed precisely to Issue:
give assurance to the parties that
the court hearing their case would W/N the judge acted in GADLEJ in raffling the criminal cases in violation of Circular No. 7 of this Court (regarding the manner of
be impartial. On this point, we raffle of cases).
found the petition meritorious.
Held:

The cases are remanded to the Executive Judge for re-raffle in accordance with this Courts Circular No. 7, except for the case
regarding threat which was dismissed. ;;

Solicitor General stated that the issue of the alleged non-compliance with the Court's circular regarding the raffle of cases was
trivial, that the Court's guidelines on the matter did not vest any substantive right and a violation thereof did not per se infringe
any constitutional right of the accused, and that the raffling of cases did not involve an exercise of judicial function, but was a
mere administrative matter involving the distribution of cases among the different branches of the court, which could not be the
subject matter of a special civil action for certiorari.

A violation or disregard of the Court's circular on how the raffle of cases should be conducted is not to be countenanced. A party
has the right to be heard by an impartial and unbiased tribunal.

The raffle of cases is of vital importance to the administration of justice because it is intended to insure impartial adjudication of
cases. By raffling the cases public suspicion regarding assignment of cases to predetermined judges is obviated.

When the respondent judge conducted the raffle of the three criminal cases in question, apparently in violation of the Court's
Circular No. 7, he did not only arouse the suspicion that he had some ulterior motive for doing so, but he violated the cardinal
rule that all judicial processes must be done above board. We consider the procedure of raffling cases to be an important element
of judicial proceedings, designed precisely to give assurance to the parties that the court hearing their case would be impartial.
On this point, we found the petition meritorious.

XXI. SUBPOENA (RULE 21, SEC. 1 TO 10)


59. Collado vs. A subpoena is a process directed to FACTS:
Bravo A.M. No. P-99- a person requiring him to attend Complainant Lorena O. Collado charged respondent Teresita G. Bravo, Clerk of Court of the Municipal Trial Court (MTC) of
1307 10 April 2001 and to testify at the hearing or the Naguilian, La Union, with Grave Misconduct and/or Conduct Prejudicial to the Best Interest of the Service.
PEREZ trial of an action, or at any
investigation conducted by Complainant alleged that on July 11, 1997, she received through priority mail, a subpoena from the MTC of Naguilian, La
competent authority, or for the Union, directing her to appear before the said court at 2:00 P.M., July 14, 1997. The subpoena was duly signed by respondent
taking of his deposition. in her capacity as Clerk of Court. Before proceeding to said court, complainant sought assistance from the Office of the
Governor of La Union and Mr. Arthur T. Madayag, Legal Assistant II of the Provincial Legal Office, who was detailed to
Absent any proceedings, suit, or accompany her to court.chanrob1es virtua1 1aw 1ibrary
action commenced or pending
before a court, a subpoena may not Upon arriving at the MTC of Naguilian, complainant talked to Respondent. When complainant asked for copies of the
issue. complaint and other details of the case, respondent replied that no complaint had been filed and her intention in issuing the
subpoena was to allow a certain Perla Baterina, the labor recruiter of complainant’s son, Emmanuel Collado, to talk to
complainant.

Complainant claimed that she felt humiliated, harassed, and experienced extreme nervousness as a result of respondent’s
issuance of the subpoena.chanrob1es virtua1 1aw 1ibrary

In her answer, respondent admitted issuing the subpoena. She claimed, however, that it was done with good intentions since
she only acceded to the urgent request of the spouses Rogelio and Perla Baterina who came to her office on July 7, 1997, airing
their grievances against complainant. Respondent averred that her only purpose in issuing the subpoena was to enable
complainant and the Baterinas to settle their differences. 3

In its Memorandum, the Office of the Court Administrator (OCA) recommended that the complaint be docketed as an
administrative matter and respondent be fined Five Thousand Pesos (P5,000.00) for Grave Misconduct with a Warning that
the commission of a similar act would merit a more serious penalty.

The Court required the parties to manifest whether they were willing to submit this case for decision on the basis of the
pleadings already filed. Respondent agreed. Though complainant had not yet responded, and her compliance is now deemed
waived, we shall now resolve her complaint.

ISSUE:
Whether or not the Clerk of Court committed grave misconduct.

HELD:
Yes.
Respondent’s act of issuing the subpoena to complainant was evidently not directly or remotely connected with respondent’s
judicial or administrative duties. It appears that she merely wanted to act as a mediator or conciliator in the dispute between
complainant and the Baterinas, upon the request of the latter.

Respondent as Clerk of Court is primarily tasked with making out and issuing all writs and processes issuing from the court.
She should have known or ought to know what a subpoena is. "A subpoena is a process directed to a person requiring him to
attend and to testify at the hearing or the trial of an action, or at any investigation conducted by competent authority, or for the
taking of his deposition."
She should have known that a process is "the means whereby a court compels the appearance of the defendant before it; or a
compliance with its demands." Hence, absent any proceedings, suit, or action commenced or pending before a court, a
subpoena may not issue. In this case, respondent knew there was no case filed against complainant. Neither had complainant
commenced any proceeding against the Baterinas for whose benefit the subpoena was issued. Respondent, then, had
absolutely neither the power nor the authority nor the duty to issue a subpoena to the complainant.

Perusal of the subpoena she issued to complainant shows that the form used was the one used in criminal cases, giving
complainant the impression that her failure to appear would subject her to "the penalty of law," and that the subpoena was
issued with the trial court’s sanction. We find, therefore, that respondent was using without authority some element of state
coercion against complainant who was understandably compelled to heed the contents of the subpoena resulting in her
humiliation. Such naked abuse of authority by complainant could not be allowed to pass without appropriate sanction.
Accordingly, this Court has no recourse but to agree with the recommendation of the OCA that respondent be disciplined and
fined.chanrob1es virtua1 1aw 1ibrary

WHEREFORE, respondent Teresita G. Bravo is hereby found GUILTY of Grave Misconduct and Conduct Prejudicial to the
Best Interest of the Service for which she is fined Five Thousand Pesos (P5,000.00) with a WARNING that a repetition of the
same or similar act would be treated more severely.

SO ORDERED.

60. Universal In order to entitle a party to the Facts:


Rubber Products, Inc. issuance of a "subpoena duces Converse Rubber Corporation and Edwardson Manufacturing Co., Inc. (respondents) sued Universal Rubber Products, Inc.
vs. CA L-30266 29 tecum," it must appear, by clear and (petitioner) for unfair competition.
June 1984 unequivocal proof, that the book or
PURIFICACION document sought to be produced
contains evidence relevant and Respondent corporations made a request to the respondent Judge Hon. Pedro C. Navarro to issue a subpoena duces tecum
material to the issue before the against the treasurer of petitioner as regards to its sales invoices, sales books, and ledgers.
court, and that the precise book,
paper or document containing such Petitioner moved to quash the subpoena on the grounds that:
evidence has been so designated or 1. the said subpoena is both unreasonable and oppressive as the books and documents called for are numerous and
described that it may be identified voluminous;
(Arnaldo vs. Locsin, 69 Phil. 113). A 2. there is no good cause shown for the issuance thereof; and
"subpoena duces tecum" once 3. the books and documents are not relevant to the case pending below.
issued by the court may be quashed
upon motion if the issuance thereof Respondent judge denied the motion to quash.
is unreasonable and oppressive, or
the relevancy of the books,
documents or things does not Petitioner filed a motion for reconsideration seeking the said court to reconsider its order denying the motion to quash the
appear, or if the persons in whose subpoena duces tecum. Respondent judge denied the MR.
behalf the subpoena is issued fails
to advance the reasonable cost of Petitioner Universal Rubber Products, Inc. filed its present petition for certiorari with preliminary injunction, alleging that in
production thereof (Sec. 4, Rule 23, so denying its motion to quash the subpoena duces tecum and its subsequent motion for reconsideration, respondent Judge
Revised Rules of Court). acted with grave abuse of discretion amounting to an excess of jurisdiction

Issue: Whether the issuance of the "subpoena duces tecum" is proper in a suit for unfair competition

Held:
Well-settled is our jurisprudence that, in order to entitle a party to the issuance of a "subpoena duces tecum", it must appear,
by clear and unequivocal proof, that the book or document sought to be produced contains evidence relevant and material to
the issue before the court, and that the precise book, paper or document containing such evidence has been so designated or
described that it may be identified. A "subpoena duces tecum" once issued by the court may be quashed upon motion if the
issuance thereof is unreasonable and oppressive, or the relevancy of the books, documents or things does not appear, or if the
persons in whose behalf the subpoena is issued fails to advance the reasonable cost of production thereof.

In the instant case, in determining whether the books subject to the subpoena duces tecum are relevant and reasonable in
relation to the complaint of private respondent for unfair competition, we have to examine Republic Act No. 166, which
provides:

CHAPTER V. — Rights and Remedies


xxx xxx xxx
Sec. 23. Actions, and damages and injunction for infringement. — Any person entitled to the exclusive use of a
registered mark or trade name may recover damages in a civil action from any person who infringes his rights and the
measure of the damages suffered shall be either the reasonable profit which the complaining party would have made,
had the defendant not infringed his said rights, or the profit which the defendant actually made out of the
infringement, or in the event such measure of damages cannot be readily ascertained with reasonable certainty, then
the court may award as damages a reasonable percentage based upon the amount of gross sales of the defendant of the
value of the services in connection with which the mark or trade name was used in the infringement of the rights of the
complaining party. In cases where actual intent to mislead the public or to defraud the complaining party shall be
shown in the discretion of the court, the damages may be doubled.
The complaining party, upon proper showing may also be granted injunction.

In giving life to this remedial statute, we must uphold the order of the court a quo denying the motion of the petitioner to
quash the "subpoena duces tecum" previously issued against the petitioner. In a suit for unfair competition, it is only
through the issuance of the questioned "subpoena duces tecum" that the complaining party is afforded his
full rights of redress.

61. Roco vs. A subpoena is a process directed to FACTS: Cal’s Corp. (respondent) filed a case for violations of BP 22 against Roco with the MTCC of Roxas. The MTCC
Contreras 158275 28 a person requiring him to attend declared the cases submitted for decision on account of Roco’s failure to adduce evidence and Roco was convicted. Roco
June 2005 and to testify at the hearing. Well- appealed to the RTC and they remanded the case back to the MTCC.
SABIO settled is the rule that before a
subpoena duces tecum may issue, During the pendency of the remanded cases, Roco filed with the MTCC a “request for issuance of subpoena ad testificandum
the court must first be satisfied that and subpoena duces tecum”, requiring Cal Corp’s duly authorized representatives to appear and testify in court. The MTCC
the test on relevancy and test on granted the order of Roco’s request for the issuance of the subpoenas.
definiteness be followed.
The private prosecutor manifested that it was improper for the trial court to have directed the issuance of the requested
subpoenas. Cal’s Corp. maintained that the production of the documents was inappropriate because they are immaterial and
irrelevant to the crimes for which Roco was being prosecuted. The issuance of the subpoenas was then denied.

The MTCC ruled in favor of Cal’s Corp. Roco went to the RTC but was dismissed. Roco then went to the CA on appeal but was
also dismissed.

ISSUE: Whether or not the denial of the request for the issuance of the subpoenas was proper.

HELD: YES. A subpoena is a process directed to a person requiring him to attend and to testify at the hearing. Well-settled is
the rule that before a subpoena duces tecum may issue, the court must first be satisfied that the test on relevancy and test on
definiteness be followed.

In the case at bar, while Roco passed the test on definiteness, he fails in the test of relevancy. Roco would want it to appear that
the books and documents subject of his request for subpoena duces tecum are indispensable, or, at least, relevant to prove his
innocence. The issuance of the subpoena serves no purpose but to further delay proceedings in the pending criminal cases.

62. People of the Section 9, Rule 23 applies only


Philippines vs. to civil cases. Facts:
Montejo L-24154 31 A criminal case (Criminal Case No. 3225) was filed in the CFI of Zamboanga City against a certain Felix Wee Sit for double
October 1967 homicide and serious physical injuries, causing the death of 2 young girls and injuries to 4. Trial commenced, after which it
ARIBON was stated that a certain Ernesto Uaje Salvador, a permanent resident of Montalban, Rizal was considered a material and
important witness in the case. At the time the case against the accused was called for trial, then presided by the respondent
Judge, the witness had returned to Montalban, Rizal. Repondent Judge issued a subpoena to Uaje addressed in Montalban,
Rizal, ordering him to appear at the trial of the case set for continuation. Although Uaje had received the subpoena, he failed to
appear. Hence, the City Fiscal moved for an order of arrest. The respondent Judge sought to annul the petition. MR was filed
and was likewise denied. When the petition was given due course, a preliminary injunction was issued. Respondent judge and
accused filed an answer, denying the allegation that Uaje was a material witness in the case. In a special affirmative defense,
the respondent alleged that in applying Section 9, Rule 23 in the case at bar, the accused cannot be held to constitute contempt
because his residence was not less than 50 km from the place of trial. Thus, there was no grave abuse of discretion on
respondent judge’s part.

Issue:

W/N the lower court erred in holding that Section 9, Rule 23 applies to both criminal and civil cases.

Held:

No. While the Court states that the counsel of respondent’s arguments that the application of Section 9, Rule 23 makes no
distinction between a civil and criminal case and must thus be applied to the case at bar does not lack plausibility, the
contention of t he respondents failed to enlist the assent of a majority of the Court. Thus, the Court interprets that Rule 23, Sec
9 is solely applied to civil cases. Under the circumstances, in view of the serious handicap to which the prosecution would thus
be subjected in proving its case, the respondent judge’s denial of an order of arrest based on a clear misapprehension of the
Rules of Court could be viewed as amounting to grave abuse of discretion.

63. Genorga vs. It was argued that under the FACTS: Dr. Gil Geñorga was a former municipal health officer in Masbate and was later assigned to Negros Oriental. A
Quintain Adm. Matter Rules of Court, a witness is warrant of arrest was issued against him for his failure to appear as government medico-legal witness in a pending murder
No. 981-CFI 29 July not bound to attend a hearing case before respondent Judge Pedro C. Quitain of the CFI of Masbate.
1977 if held outside the province he
Dr. Geñorga averred that he was able to testify but his nonappearance was by reason of the fact that he had previously
SOMEROS resides unless the distance be
asked respondent Judge thru a telegram if he would be reimbursed for the traveling expenses to be incurred and that he
less than 50 kilometers from
had received no reply. As a result he was humiliated for having been arrested and confined in the headquarters of PH
his residence to the place of
Constabulary as if he were a criminal. But the Provincial Commander of Negros Oriental, allowed him to proceed to
trial. Such contention did not
Masbate alone at his own expense." Respondent Judge then dictated in open court the order for his release.
command the assent of this
Now, Dr. Geñorga filed an administrative complaint for grave abuse of authority and conduct against judge Quitain for
Court. Section 9 of Rule 23
ignoring him and treating him with discourtesy.
(Sec 10, Rule 21 of the 1997
Respondent Judge in his comment. he averred that order of arrest issued for his failure to appear in Court, in spite of a
ROC and 2019 Amendment) is
subpoena duly served upon him as a government witness in the case and did not deny the other allegations because it was
thus interpreted to apply solely
the honest conviction of the undersigned that he is not duty bound to make any inquiry for the complainant, much less
to civil cases.
advice the complainant in any manner. That the telegram lacked the element of courtesy since did not sound as a request.
He also denies the allegation of discourtesy because he sympathizes with the complainant for the latter's experiences, but
there was no way out of the predicament except to obey the subpoena. Thereafter he issued the order of immediate release
of the complainant.
The matter was elevated to CA with Acting Assistant Judicial Consultant Relova for study, report and recommendation to
which it said that the charges be dismissed.
He explained that to be administratively liable would be allowing a disregard of the coercive power of the courts to compel
attendance in court of cited witnesses. (Section 5 (e) of Rule 135 of ROC)
This court accepts such recommendations.
ISSUE: whether a CFI hearing a criminal case may compel by subpoena the attendance of a witness in his sala in
Zamboanga City, when the known address of such witness is at Montalban, Rizal.
HELD: YES, It is loathe to clip what undoubtedly is the inherent power of the Court to compel the attendance of persons to
testify in a case pending therein. What was done by Judge Quitain was, therefore, within his discretion. There was no grave
abuse of authority. Nor can the accusation of conduct unbecoming a judge be taken seriously.
It was argued that under the Rules of Court, a witness is not bound to attend a hearing if held outside the province he
resides unless the distance be less than 50 kilometers from his residence to the place of trial. Such contention did not
command the assent of this Court. Section 9 of Rule 23 (Sec 10, Rule 21 of the 1997 ROC and 2019 Amendment) is
thus interpreted to apply solely to civil cases.

XXII. COMPUTATION OF TIME (RULE 22, SEC. 1 TO 2)


64. Neypes vs. CA The fresh period of 15 days becomes significant only when a party opts to file a FACTS:
141524 14 September 2005 motion for new trial or motion for reconsideration. In this manner, the trial Petitioners Domingo Neypes, Luz Faustino, Rogelio Faustino, Lolito Victoriano,
TAN court which rendered the assailed decision is given another opportunity to Jacob Obania and Domingo Cabacungan filed an action for annulment of
review the case and, in the process, minimize and/or rectify any error of judgment and titles of land and/or reconveyance and/or reversion with
judgment. preliminary injunction before the Regional Trial Court, Branch 43, of Roxas,
Oriental Mindoro, against the Bureau of Forest Development, Bureau of Lands,
In this case, the new period of 15 days eradicates the confusion as to when the Land Bank of the Philippines and the heirs of Bernardo del Mundo, namely, Fe,
15-day appeal period should be counted ' from receipt of notice of judgment Corazon, Josefa, Salvador and Carmen.
(March 3, 1998) or from receipt of notice of 'final order appealed from (July 22,
1998). The parties (both petitioners and respondents) filed various motions with the
trial court. Among these were: (1) the motion filed by petitioners to declare the
respondent heirs, the Bureau of Lands and the Bureau of Forest Development in
default and (2) the motions to dismiss filed by the respondent heirs and the
Land Bank of the Philippines, respectively.

In an order dated May 16, 1997, the trial court, presided by public respondent
Judge Antonio N. Rosales, resolved the foregoing motions as follows: (1) the
petitioners' motion to declare respondents Bureau of Lands and Bureau of
Forest Development in default was granted for their failure to file an answer, but
denied as against the respondent heirs of del Mundo because the substituted
service of summons on them was improper; (2) the Land Bank's motion to
dismiss for lack of cause of action was denied because there were hypothetical
admissions and matters that could be determined only after trial, and (3) the
motion to dismiss filed by respondent heirs of del Mundo, based on
prescription, was also denied because there were factual matters that could be
determined only after trial.

The respondent heirs filed a motion for reconsideration of the order denying
their motion to dismiss on the ground that the trial court could very well resolve
the issue of prescription.

February 12, 1998, the trial court dismissed petitioners' complaint on the
ground that the action had already prescribed. Petitioners allegedly received a
copy of the order of dismissal on March 3, 1998 and, on the 15th day thereafter
or on March 18, 1998, filed a motion for reconsideration. On July 1, 1998, the
trial court issued another order dismissing the motion for reconsideration [3]
which petitioners received on July 22, 1998. Five days later, on July 27, 1998,
petitioners filed a notice of appeal [4] and paid the appeal fees on August 3,
1998.

On August 4, 1998, the court a quo denied the notice of appeal, holding that it
was filed eight days late. [5] This was received by petitioners on July 31, 1998.
Petitioners filed a motion for reconsideration but this too was denied in an order
dated September 3, 1998.

Via a petition for certiorari and mandamus under Rule 65 of the 1997 Rules of
Civil Procedure, petitioners assailed the dismissal of the notice of appeal before
the Court of Appeals.

ISSUE: Whether the court of appeals erred in dismissing the petitioners'


petition for certiorari and mandamus and in affirming the order of the RTC
which dismissed the petitioners' appeal in the RTC, even after the petitioners
had paid the appeal docket fees.

HELD:
Yes, the CA erred in this case.

The SC states that petitioners seasonably filed their notice of appeal within the
fresh period of 15 days, counted from July 22, 1998 (the date of receipt of notice
denying their motion for reconsideration). This pronouncement is not
inconsistent with Rule 41, Section 3 of the Rules which states that the appeal
shall be taken within 15 days from notice of judgment or final order appealed
from.
Neither does this new rule run counter to the spirit of Section 39 of BP 129
which shortened the appeal period from 30 days to 15 days to hasten the
disposition of cases. The original period of appeal (in this case March 3-18,
1998) remains and the requirement for strict compliance still applies.

65. Luz vs. National In explaining Rule 22, Section 1, the SC, in its Resolution in A.M. No. 00-2-14- Facts:
Amnesty Commission SC, provided, among others that: “Any extension of time to file the required Petitioner Luz was charged with violation of PD No. 1866 (illegal possession of
159708 24 September pleading should therefore be counted from the expiration of the period firearms) in the RTC of Makati City. Thereafter, he filed an application for
2004 regardless of the fact that said due date is a Saturday, Sunday or legal holiday.” amnesty with the Local Amnesty Board for Metro Manila. The Board denied his
TANADA The extension granted by the Court of Appeals should be tacked to the original application. The National Amnesty Commission affirmed the denial of the Local
period and commences immediately after the expiration of such period. Amnesty Board. The MR was likewise denied. Under Rule III, Section 4 of NAC
Administrative Order No. 2, Series of 1999, the petitioner had until December 7,
2002, a Saturday, within which to file a petition for review of the resolution with
the CA. On Dec. 9, 2002, petitioner filed a motion in the CA for an extension of
15 days from Dec. 9, 2002 or until Dec. 24, 2002 within which to file his
petitioner, alleging therein that he had just engaged the services of counsel who
needed additional time to study the case and draft the petition. Petitioner,
however, failed to file his petition for review. December 24 and 25, 2002 were
declared as a national holiday. On Dec. 26, 2002, petitioner filed a second
motion for extension of 15 days from December 26, 2002 or until January 10,
2002, within which to file his petition. He filed his petition for review with the
CA on January 10, 2003. On January 13, 2003, the CA granted petitioner’s first
motion for a 15-day extension, to be counted form Dec. 7, 2002 or until Dec. 22,
2002, within which to file said petition. On February 20, 2003, the CA denied
petitioner’s second motion for having been filed out of time. Petitioner filed a
motion for reconsideration of the February 20, 2003 Resolution claiming that,
since the last day to file his petition was a Saturday, December 7, 2002, and the
next day, December 8, 2002 was a Sunday, the last day for filing the petition
was December 9, 2002. The CA denied the motion, relying on A.M. No. 00-2-14-
SC issued on February 29, 2000, which provides that any extension of time to
file the required pleading should be counted from the expiration of the period
regardless of the fact that the said due date is a Saturday, Sunday, or legal
holiday.
Issue:
Whether or not petitioner timely filed his second motion for extension of time to
file his petition for review.
Held: No.
Petitioner claimed that the CA resolutions are contrary to Rule 22, Section 1 of
the Rules of Court and previous SC ruling. He avers that the rule in A.M. No. 00-
2-14-SC should apply only where a motion for extension does not specifically
state the reckoning date of the extension prayed for, such that the extension will
be reckoned from the last day of the period, even if it was a Saturday, Sunday, or
a legal holiday. He contends that even assuming that his second motion for
extension was filed out of time, the Court of Appeals should have admitted his
petition for review in the interest of justice. The SC held that petitioner’s motion
for a second extension of time to file his petition for review was filed out of time.
In explaining Rule 22, Section 1, the SC, in its Resolution in A.M. No. 00-2-14-
SC, provided, among others that: “Any extension of time to file the required
pleading should therefore be counted from the expiration of the period
regardless of the fact that said due date is a Saturday, Sunday or legal holiday.”
The extension granted by the Court of Appeals should be tacked to the original
period and commences immediately after the expiration of such period. Under
the Resolution of this Court in A.M. No. 00-2-14-SC, the CA has no discretion to
reckon the commencement of the extension it granted from a date later than the
expiration of such period, regardless of the fact that said due date is a Saturday,
Sunday, or a legal holiday. The CA cannot be faulted for granting petitioner’s
first motion, reckoned from December 7, 2002, and not December 9, 2002. In
so doing, it merely applied Rule 22, Section 1, as clarified via the said
Resolution. Had Had the CA granted the petitioner's first motion for extension
and reckoned the 15-day period from December 9, 2002, instead of from
December 7, 2002, the appellate court would have acted with grave abuse of its
discretion. However, in this case, the SC applied a liberal interpretation of the
said Resolution to the petitioner, in light of the peculiar factual background of
the case.

You might also like